You are on page 1of 392

Solution Booklet

PHYSICS
[XI TOPIC]
Copyright © reserved with Motion IIT JEE and Publications
All rights reserved. No part of this work herein should be reproduced or used either
graphically, electronically, mechanically or by recording, photocopying, taping, web
distributing or by storing in any form and retrieving without the prior written permission of
the publisher. Anybody violating this is liable to be legally prosecuted.

Corporate Head Office


394 - Rajeev Gandhi Nagar Kota, (Raj.)
Ph. No . : 93141-87482, 0744-2209671
IVRS No : 0744-2439051, 52, 53,
www. motioniitjee.com , info@motioniitjee.com
Solution (XI Topic)
TOPICS COVERED

1. Units & Dimensions

2. vector & calculus

3. Kinematics

4. N.L.M Friction

5. Circular, W.P.E

6. Center of Mass

7. Rotational Dynamics

8. SHM

9. Waves

10. Sound Waves

11. Heat-1

12. Heat-2

13. Elasticity & Thermal Expansion

14. Gravitation

15. Fluid

16. Surface

17. Errors

394 - Rajeev Gandhi Nagar Kota,Ph. No. 0744-2209671, 93141-87482,


93527-21564, IVRS No. 0744-2439051, 0744-2439052, 0744-2439053
www.motioniitjee.com, email-info@motioniitjee.com
Page # 4 CONTENTS Solution Slot – 2 (Mathematics)

S.NO. TOPIC ..................... PAGE NO.


 Units & Dimensions ...................................................................................................................................................... 5 to 15
Exercise - I ........................................................................................................................................................................ 5 – 7
Exercise - II ...................................................................................................................................................................... 8 – 10
Exercise - III .................................................................................................................................................................... 11 – 13
Exercise - IV .................................................................................................................................................................. 14 – 15
 Vector & Calculus ........................................................................................................................................................ 16 to 35
Exercise - I ..................................................................................................................................................................... 16 – 21
Exercise - II .................................................................................................................................................................... 22 – 28
Exercise - III ................................................................................................................................................................... 29 – 31
Exercise - IV .................................................................................................................................................................. 32 – 35
 Kinematics .................................................................................................................................................................... 36 to 72
Exercise - I ..................................................................................................................................................................... 36 – 47
Exercise - II .................................................................................................................................................................... 48 – 61
Exercise - III ................................................................................................................................................................... 62 – 68
Exercise - IV .................................................................................................................................................................. 69 – 72
 N.L.M Friction .............................................................................................................................................................. 73 to 105
Exercise - I ..................................................................................................................................................................... 73 – 82
Exercise - II .................................................................................................................................................................... 83 – 93
Exercise - III .................................................................................................................................................................. 94 – 101
Exercise - IV ................................................................................................................................................................ 102 – 105
 Circular, W.P.E ............................................................................................................................................................ 106 to 143
Exercise - I ................................................................................................................................................................... 106 – 117
Exercise - II .................................................................................................................................................................. 118 – 127
Exercise - III ................................................................................................................................................................. 128 – 136
Exercise - IV ................................................................................................................................................................ 137 – 143
 Center of Mass .......................................................................................................................................................... 144 to 175
Exercise - I ................................................................................................................................................................... 144 – 152
Exercise - II .................................................................................................................................................................. 153 – 162
Exercise - III ................................................................................................................................................................. 163 – 170
Exercise - IV ................................................................................................................................................................ 171 – 175
 Rotational Dynamics ................................................................................................................................................. 176 to 216
Exercise - I ................................................................................................................................................................... 176 – 186
Exercise - II .................................................................................................................................................................. 187 – 198
Exercise - III ................................................................................................................................................................. 199 – 206
Exercise - IV ................................................................................................................................................................ 207 – 216
 SHM ............................................................................................................................................................................. 217 to 238
Exercise - I ................................................................................................................................................................... 217 – 220
Exercise - II .................................................................................................................................................................. 221 – 226
Exercise - III ................................................................................................................................................................. 227 – 232
Exercise - IV ................................................................................................................................................................ 233 – 238
 Wav es ......................................................................................................................................................................... 239 to 257
Exercise - I ................................................................................................................................................................... 239 – 242
Exercise - II .................................................................................................................................................................. 243 – 248
Exercise - III ................................................................................................................................................................. 249 – 255
Exercise - IV ................................................................................................................................................................ 256 – 257
 Sound Waves ............................................................................................................................................................. 258 – 276
Exercise - I ................................................................................................................................................................... 258 – 260
Exercise - II .................................................................................................................................................................. 261 – 265
Exercise - III ................................................................................................................................................................. 266 – 271
Exercise - IV ................................................................................................................................................................ 272 – 276
 Heat-1 .......................................................................................................................................................................... 277 to 291
Exercise - I ................................................................................................................................................................... 277 – 279
Exercise - II .................................................................................................................................................................. 280 – 282
Exercise - III ................................................................................................................................................................. 283 – 286
Exercise - IV ................................................................................................................................................................ 287 – 291
 Heat-2 .......................................................................................................................................................................... 292 to 314
Exercise - I ................................................................................................................................................................... 292 – 295
Exercise - II .................................................................................................................................................................. 296 – 298
Exercise - III ................................................................................................................................................................. 299 – 303
Exercise - IV ................................................................................................................................................................ 304 – 314
 Elasticity & Thermal Expansion ............................................................................................................................. 315 – 325
Exercise - I ................................................................................................................................................................... 315 – 316
Exercise - II .................................................................................................................................................................. 317 – 318
Exercise - III ................................................................................................................................................................. 319 – 322
Exercise - IV ................................................................................................................................................................ 323 – 325
 Gravitation .................................................................................................................................................................. 326 – 340
Exercise - I ................................................................................................................................................................... 326 – 327
Exercise - II .................................................................................................................................................................. 328 – 331
Exercise - III ................................................................................................................................................................. 332 – 336
Exercise - IV ................................................................................................................................................................ 337 – 340
 Fluid ............................................................................................................................................................................. 341 – 364
Exercise - I ................................................................................................................................................................... 341 – 345
Exercise - II .................................................................................................................................................................. 346 – 351
Exercise - III ................................................................................................................................................................. 352 – 359
Exercise - IV ................................................................................................................................................................ 360 – 364
 Surface ........................................................................................................................................................................ 365 – 378
Exercise - I ................................................................................................................................................................... 365 – 367
Exercise - II .................................................................................................................................................................. 368 – 370
Exercise - III ................................................................................................................................................................. 371 – 374
Exercise - IV ................................................................................................................................................................ 375 – 378
 Errors .......................................................................................................................................................................... 379 – 392
Exercise - I ................................................................................................................................................................... 379 – 381
Exercise - II .................................................................................................................................................................. 382 – 385
Exercise - III ................................................................................................................................................................. 386 – 387
Exercise - IV ................................................................................................................................................................ 388 – 392

394 - Rajeev Gandhi Nagar Kota, Ph. No. 0744-2209671, 93141-87482, 93527-21564
www. motioniitjee.com , email-info@motioniitjee.com
Units & Dimensions 5

UNITS 10. D
1. A
L Henery
Kilogram is not a physical quantity, its a   M0L0 T 0 A 1
unit. RCV ohmFarad Amp.
V=IR
2. C
Light year is a unit of distance, which is 11. C
cover by light in a year. Because same physical quantities are added
and subtracted.
3. C [at] = M0L0T0
PARSEC is a unit of distance. [a] T = M0L0T0, [a] = M0L0T-1
It is used in astronomical science.
12. D
4. C
Find dimension in all options.
S.I. unit of energy is Joule.
Here stress = Force/Area
5. B M1L1T 2

SI unit of universal gravitational constant L2
G is - stress = [M1L–1T–2]
GM1M2
We know F  USE OF DIMENSIONS
R2
Here M1 and M2 are mass 13. D
R = Distance between them M1 and M2 n1u1 = n2u2
F = Force
1m2 = n(xm)2
2 2
FR Nm 1
G  n 2
M1M2 kg2 x
So, Unit of G = N–m2 kg–2
14. B
6. B  = 2g/cm3
Surface Tension (T) :- n1u1 = n2u2
W J n1 M11L13   2 M12L23 
T 
A m2
So S.I. unit of surface tension is joule/ 1 3 1 3
m+2 M  L  10 3  10 2 
n1  2 2   2   2   
 M1   L l   1   1 
DIMENSIONS
= 2×10-3×106
7. C = 2×103 Kg/m3
1
Angular Frequency (f)   MoLo T 1
T 15. D
By checking each option.
So, here dimension of length is zero
V2 L1T 1
 1 1 2
 
2

8. A rg [L ][L T ]
Dimension of universal gravitational const.
G M1M2 L2 T 2
F  = [MoLoTo]
R2 L2 T 2

FR 2 M1L1T 2  L2 16. C
G   M1L3 T 2
M1M2 M2 Theory

9. D
Stefan-Constant()
Unit  w/m2-k4 = wm-2k-4

www.motion.ac.in
6 Solution Physics (XI) Topic

17. A 22. C
1 3 2 By Similar triangle concept In ABC and DEC
 M1   L1   T1  A
n2 = 6.67 × 10-11      
 M2   L 2   T2  D

1 3 2 4 F
1000  100  1  x
= 6.67 × 10-11       2
 1   1  1  C
B E G y
= 6.67 × 10-11 × 103 2
n2 = 6.67 × 10-8 3
AB BC
18. A 
DE EC
1 3 4 3 8
 M1   L1    x 
n2 = 13600     x 2 3
 M2  L2  In ABC and FGC
1000  100 
1 3
AB BC 4 3 3
    y
= 13600     FG GC 2 y 2
 1   1 
n2 = 13.6 gcm -3
23. ABC
19. A (a) (99)1/2
1/2
Given  1 
P = 106 dyne/cm2 = (100–1) = 101 
1/2 
 100 
n1u1 = n2u2
 1 1 
n1 M11L11 T12   106 M12L21 T22  = 101  200  80000  = 9.9498
 
1 1 2 (b) (120)1/2 = (100+20)1/2
M   L 2   T2 
n1  106  2     
 M1   L1   T1   20 
1/2
 1
1/2

1   1 
1 1 = 101   = 101  
6   100   5
= 10   100 
1000     1 1 
102 = 101  10  200  = 10.95
 10  3  10 N / m2
6 5  
10 (c) (126)1/3 = (125+1)1/3
1/3
 1 
20. C = 51  
 125 
W
P  1 1 
t = 51  125  3  
Watt = Joule/sec.  9  1252 
Joule = Watt-sec. = 5.01329
One watt-hour = 1 watt×60×60 sec
1 Hour=60×60sec. = 3600 watt-sec 24. B
= 3600 Joule
Take small angle approximation
= 3.6 × 103 Joule

BASIC MATHEMATICS D
21. A rm
D
h Sin  = r
m

1.8
0 D
1m Sin 0.50° = r
m
h
tan 1.8°   D
1 0.50  
Small Angle Approximation 180 384000
tan 1.8°  1.8° 
D  0.50   384000
180
z D = 3349.33  D  3350 km.
h = 0.01 m
=  cm

394-Rajeev Gandhi Nagar, Kota


Units & Dimensions 7

25. (a) 9.95 (b) 0.99 28. C


(1 + x)n  1+nx, |x| <<1 (Area)1 = r12  r2
(a) 99 = (99)1/2 = (100–1)1/2 (Perimeter)1 = 2r1 = 2r
1/2
 1   1  (Area)2 = r22  (4r)2 = 16r2
= 101   = 101   = 9.95
 100   200  (Perimeter)2 = 2r2 = 2(4r) = 8r
1 1 A1 1
(b)  1.01 = (1+0.01)-1 
1.01 A2 16
1
1  P1 1
 1   
= 1   = 1  100  = 0.99 P2 4
 100   
29. B
26. (a) 0.14 (b) 0.09
(a) sin 8° 148
Using small angle approximation. 3
sin 8°  8° 4 3
External Volume  r
8 3
 = 0.139  0.14
180 4 256
 (4)3 
(b) tan 5° 3 3
Using small Angle Approximation
tan 5°   5° 4 3
Internal volume  r
3
5
= = 0.087   0.09 4 108
180  (3)3 
3 3
x
256 108
v  Volume of Material =  
3 3
v0 148
o
27. (i) (ii) t =
3
t
o
–3
30. C
cos(120°) = cos (90+30)
z v = – sin 30
= -1/2
o
t
(iii) (iv)
t
o

(i) v = v0–at (ii) x = 4t–3

x
v
v0
o
t
t
o
–3

(iii) x = 4at2 (iv) v = –gt

z
v

o
t

t
o

www.motion.ac.in
8 Solution Physics (XI) Topic

Units 7. C
1. B vo
Fundamental quantities does not depends x(t)  [1  e t ]

each other So, In length, time and velocity
Dimension of vo and 
here velocity is derived quantities.
Here e–t is dimensionless so,
2. B [] [t] = MoLoTo
Solar day  Time far Earth to make a
complete rotation on its axis MoLo T o
[]   T 1
Parallactic second [1 Parsec]  It is a T1
distance corresponding to a parallex of one [] = MoLoT-1
second of arc. Here 1-e–t is a number
Leap year  A leap year is year (time) Vo
Containing one extra day. [x(t)] 

Lunar Month A lunar month is the time [Vo] = [L1] [T–1]
between two identical view moons of full [Vo] = MoL1T–1
moons.
1 Lunar month = 29.53059 days.
8. B
3. A P = Po Exp (–t2)
A unitless quantity never has a nonzero Here Exp (–t2) is a dimensionless
dimension or unit less quantity never has So, dimension of [t2] = MoLoTo
a dimension MoLo T o
So, [] 
Change in length T2
Ex. Strain  [] = MoLoT–2
Original length
It has no unit and dimension  MoLoTo
9. C
4. C
Dimensionless quantity may have a unit
Ex. Angle Unit Radian USE OF DIMENSIONS
Dimension MoLoTo 10. B
F  Aavbc
DIMENSIONS F = KAa vb c
5. C = K[L2]a [L1T–1]b [M1L–3]c
Only same physical quantities can be added F = K[McL2a+b–3c T–b]
or substracted, M1L1T–2 = K[Mc L2a+b–3c T–b]
It’s only multiply and divided only. c = 1
So, a/b denote a new physical quantity. –2 = –b  b = 2
and
6. C 2a + b – 3c = 1
b 2a + 2 – 3 = 1  a = 1
v = at  So F = A1 v2 q1
tc
Same physical quantity can be added or  F = Av2
substracted.
Dimension of a 11. A
[v] = [at] n1u1 = n1u1

[a] =
[v] L1 T 1
  L1T 2
  
n1 M11L21T13  1 M12L22 T23 
[t] T1 1 2 3
M  L   T 
Here t + c is also a Time (t) n1   2   2   2 
b   M1   L1   T1 
[v]    1 2 3
t  20  10   5 
     
[b] = [v] [t] = L1T-1 × T1  1   1  1 
[b] = L1 20  100
  16
555
n1 = 16
Unit of power in new system = 16 Watt.

394-Rajeev Gandhi Nagar, Kota


Units & Dimensions 9

12. C 17. B
2k
Here 1 is a number..
ma
It’s a dimensionless quantity.
 2k  o o o
 ma   [M L T ]
 
[m][a]
[K] 
[] Slope = tan 
1 1 2
ML T = tan (127°)
  M1Lo T 2 = tan (90+37°)
L1
So dimension of [b] is = – cot 37°
2
 ma   MLT  4
[b]     2 
= –
 3
 K   MT 
[b] = L
unit of b is metre 18. A
sin (1) < cos [0°]
13. B  180 
sin   1
EJ2   
J=mvr, J = [ML2T–1]
M5G2 sin (57.32) < 1
= [M0L0T0]
so, Ture.
Dimension of Angle = [M0L0T0]
19. D
14. B
tan (405°)
F = KAdvx
tan (360+45°)
M1L1T–2 = K[L2] [M1L-3] [L1T-1]x
tan 45°
M1L1T–2 = K[M 1L-1+xT–x]
1
By comparison of power
–1 + x = 1  x = 2
20. C
sin(15°) = sin (45°–30°)
= SIN 45 . COS 30 - COS 45. SIN 30°
BASIS MATHEMATICS
1 3 1 1
 .  .
15. C 2 2 2 2
Arc 3 1
Angle = 
Radius 2 2

0.9° =
2
21. D

0.9× ×R =  4x + 3y + 1 = 0
180 3y = –4x – 1
 4 1
= R y x ....(i)
200 3 3
Comparing euqestion (i) with [y = mx + c] –
16. D
1/2 4 1
 1  m , C
(100–1) = 101 
1/2  3 3
 100 
 1 1  y
= 101  200  80000  = 9.9498
 

1

3

www.motion.ac.in
10 Solution Physics (XI) Topic

1. B,D 8. A,B
sin (90 + ) = cos  = cos (–)
2. B,D & sin (180 – ) = cos (90 – )
sin  = sin 
3. A,B,C
9. A,B,C
4. B, C Given, y = kx2
[A] = [Y] k  positive non–zero constant
Trignometric function must be dimension less. This is the equation of parabola, so

5. A, D y
2
Dimension of at & bt & F must be same.
so,
a [T] = M1L1T-2
a = M1L1T-3
b [T2] = M1L1T-2 x
O
b = M1L1T-4
we can also use this equation as straight
6. A, B line.
By theory
y  x2 or y x
7. A,B y y
1
given, a =
2
so, a = sin 30, a = cos 60
2
1 1 O
x
O
x
a= , a=
2 2

394-Rajeev Gandhi Nagar, Kota


Units & Dimensions 11

Units 6. A
1. D Dimension of b = MoLoT–1
Unit of universal gas constant (R) Dimension of c
PV = nRT P  Pressure [cx] = MoLoTo [Dimension less]
V  Volume MoLo T o
[c]   MoL1T o
PV L1
R T  Temperature
nT  b  MoLo T 1
So, Dimension of  c   o 1 o
  ML T
N / m2  m3
 R Univ. Gas. Const. = MoL1T–1
mol.  K
n  No. of mole 7. D
F
Nm  sin  t 
  Joule K 1mol1 V2
mol.  K Here sin (t) is dimensionless.
{n-m = joule} [t] = MoLoTo
MoLo T o
2. B 
T1
 
 T 1
Unit of impulse =  Impulse = Force ×
time  F  M1L1T 2 M1L1 T 2
[]   2   1 1 2  2 2
m m V  [L T ] LT
= kg sec = kg = mv
sec 2 sec [] = [M L T ]
1 –1 o

The unit is same as the unit of linear


momentum. 8. C
In new system
DIMENSIONS Length  m 2m
3. D Velocity  m/sec. 2m/sec
F = Pt–1 + t Force  kgm/sec2 2kgm/sec2
Here F and Pt–1 is a same   Momentum (P) = mv = kg m/sec.
Physical quantity m m sec
P  kg  
[F] = [Pt–1] sec m sec
[F] m m
[P]    [F  t] = ML T-2×T P  kg 
[t ] sec 2
m / sec
= MLT-1 In new system
We find it is same as dimension of momentum  m  2m
= MLT–1 P1   2kg 
 sec 2
 2m / sec 
4. D P1  2kg m / sec  2P
Action = Energy × Time So, Here unit of momentum is doubled.
= M1L2T–2 × T1
= M1L2T–1 9. B
It is same as dimension of Impulse × distance F × apqfr
= MLT–1 × L1= M1L2T–1 F = K apqfr
[M1L1T-2] = K[L1T-2]p [M1L-3]q [T-1]r
USE OF DIMENSIONS
5. D
M L T   KM L
1 1 2 q p 3q
T 2pr 
Y = a sin (bt – cx) Comparing both sides
Dimension of b q=1
Here bt is dimensionless p  3q = 1
[bt] = MoLoTo p 3(1) = 1 p = 4
2p r = 2
MoLo T o
[b]   MoLo T 1 2(4) r = 2
[T1 ]
r = 6
It is a dimension of wave frequency.  F = Ka41f-6

www.motion.ac.in
12 Solution Physics (XI) Topic

10. D 14. B
K= s2 f = cmx Ky
2 2
(ML T ) Here f is frequency of oscillation, m is mass
[] = and K is force.
(L2 )
[] = M1 L0 T–2 Taking Dimensions –
(M T –2) [T–1] = [M1]x [M1L1T–2]y
[T–1] = [Mx+yLyT–2y]
on comparing –
11. D
 –1 = –2y
[] = L
[L]1 / 2 1
ML2 T 2  y
[L] 2
 = [M1][L5/2][T–2] 1
 x+y=0  x
2
12. A
 = M1 L–1 T–1 BASIS MATHEMATICS
1 CGS units = g cm–1 s–1
1 SI units = kg m–1 s–1 15. B
= 1000 g (100 cm)–1 s–1
= 10 g cm–1 s–1 B
Thus, 1 Poiseuilli = 10 poise

13. D
x M

4m

A O
3d N d

By similar triangles BAO & MNO


x 4d

4 d
x = 16 m

394-Rajeev Gandhi Nagar, Kota


Units & Dimensions 13

Units 1
1. D –1 = –2c  c 
2
Planck Const. (h)  1 = a – 3b + c
1 = a – 0 + 1/2
ML2 T 2
E = hf h 1
1 a , V  K1 /2og1 /2
2
T squaring both sides
Unit  J-S V2 = kg
Dimension = M1L2T-2 × T 1
= M1L2T-1 BASIC MATHEMATICS
This is also a dimension of Angular momentum. 6. A
= mvr By sine Rule :
= MLT-1 L=M1L2T-1
sin53 sin 90 sin 37
 
DIMENSIONS 2x2  1 5 3
x x
2. B 4 4
 a   x=y
 P  2 v  b   RT
 v 
7. C
Dimension of a
a Resultant = F12  F22  2F1F2 cos 
[P]   2  Same Physical quantity can be
v  Given, F1 = F2 = 5N
added or substracted  Angl e i s sm al l , so smal l angl e
[a] = [P] [V2] approximation
[a] = PV2
cos   1
3. C  cos1.8  1
F = A sin ct + B Cos Dx
2 2
Here Sin ct and Cos Dx is dimensionless  5  5   2 5 5 1
quantity
So, [c] [t] = MoLoT o  25  25  50
MoLo T o  100  10
[c]   MoLo T 1
T1
and dimension of A and B is same as dimension 8. B
of force [F] = M1L1T-2
A M1L1T 2
So,   MoLo T o
B M1L1 T 2
C MoLo T 1
  MoL1 T 1
D MoL1 T o

h
USE OF DIMENSIONS
60

30°
°

4. C
By checking the dimension in all options
[Pressure] = M1L–1T–2 If h is the height of tower, then
d
5. B h
cot 2  cot 1
v  abgc
v = ka b gc
100 3
[MoL1 T 1 ]  K[MoL1 T o ]a [M1L3 T o ]b [MoL1 T 2 ]c h  50  3  150
cot 30  cot 60
[MoL1T 1 ]  K[MbLa3b  c T 2c ]
Comparing both sides
b = 0

www.motion.ac.in
14 Solution Physics (XI) Topic

1. C 7. A
I = mr2 5, 1, 2
 [I] = [ML2]
   8. B
and  = moment of force = r  F
 1 q1q2
 []  [L][MLT2 ]  [ML2 T 2 ] From Coulomb's law, F 
40 R2
2. C q1q2
'rad' is used to measure biological effect of 0 
4FR 2
radiation.
Substituting the units we have,
3. B
2
1 2 Lq2
Li  2 0 
C2
 AT   M1L3 T 4 A2 
Magnetic energy 
2 2t N  m2 MLT 2  L2  
[L inductance, i  current]
Energy has the dimension = [ML2T-2]
9. AS;BQ;CP;DR
Equate the dimensions, we have
[Q2 ] 1
[ML2 T 2 ]  (henry)  (A) U kT
[T 2 ] 2
 [ML2T–2] = [k] K
[ML2 ]
 [henry]   [K] = [ML2T–2K–1]
[Q2 ]
dv
(B) F  A
4. A dx
m = 3.513 kg and v = 5.00 ms-1
So momentum, p = mv = 17.565 MLT 2 
    ML1T 1 
As the number of significant digits in m is 4  
L2LT 1L1   
and in v is 3, so p must have 3 (minimum)  
significant digits. Hence,
(C) E = hv
p = 17.6 kg ms-1
 [ML2T2] = [h][T–1]
 [h] = [ML2T–1]
5. C
F = qvB dQ kA
(D) 
B = F/qv = MC–1T–1 dt l

ML2 T 3L 
   MLT 3K 1 
6. A  k  
L2K   
Value of main scale division  
Least count 
No. of divisions on vernier scale
1 1 1o 10
  MSD     1 min
30 30 2 60

394-Rajeev Gandhi Nagar, Kota


Units & Dimensions 15

1. D 3. C
By checking dimension all option Dimension of N = L–3
[Dipole moment] = [M0L1T1A1] Dimension of M = M1
[Electric Flux] = [M1L3T-3I-1] Dimension of 0 = M–1 L–3 T2 Q2
[Electric field] = [M1L1T-3I-1] Dimension of e2 = Q2

2. A  P,Q ; B  R,S ; C  R,S ; D  R,S Ne2 L–3


so dimension of =  ML3 T –2
m 0 M
GMeMs
(A) F 
r2 1 1
= Gravitational force between sun and earth = =
T2 T
 GMe Ms = Fr2 4. B
 [GMeMs] = [Fr2] = [MLT-2][L2] = [ML3T-2] Use formula

3RT Ne2
(B) Vrms= = rms speed of gas molecules p =
M m 0
3RT 2
 and V= f 
  vrms c=f
M

 3RT  2 5. 3
or  M   [vrms ] =[LT
T-1]2 = [L2T-2]
  d  raS1/nfb .....(1)
(C) F = Bqv = magnetic force on a charged putting dimension in equation (1) –
particle
1 /n
a  M1L2 T 3 
F  F2  L1  M1L3     T 1 b
 = v or  B2q2  =[v]2=[L2T-2]  L2 
Bq  
1
GMe a
(D) v0  L1  M n L-3a T–3/n–b
R e = orb i t al v e l oci ty of
earth's satellite
6. A,C,D
GMe  GMe  h
  v20 or   =[ v20 ]=[L2T-2] P=
Re  Re  
(p) W = qV  (Coulomb) (Volt) = Joule h
M1L1T–1 =
or [(Volt)(Coulomb)(Metre)]= [(Joule)(Metre)] L1
= [ML2T-2][L]=[ML3T-2] L2  h
(q) [(kilogram)(metre)3(second)-2]=[ML3T-2] L h
(r) [(metre)2(second)-2]=[L2T-2]
7. B,D
1 () = M–1 L–3 T4 A2
(s) U = CV2  (farad)(volt)2 = Joule
2 [KBT] = [E] = [M1L2T–2]
or [(farad)(volt)(kg)-1] =[(Joule)(kg)-1] n = [L–3]
q = [A1 T1]
= [ML2T-2][M-1]=[L2T-2]

www.motion.ac.in
16 - Solution Physics (XI) Topic

1. A 7. B
N
Resultant lies in b /w min. & maximum Values
30m
G
A A B  R  A B
40m
So 10  6  R  10  6
O C E
w
4  R  16
S

Displacement= 30m in East 8. D

2. B E D
F2
F

F C

F1
A B
 = 90°
      
So | F | F12  F22 AB  BC  CD  DE  EF  FA  0
Resultant of these vectores is null vector
3. C
40

|R | 302  402 = 50 R 9. D

40 4 | R | P2  Q2  2PQ cos 
tan  =  
30 3 when  = 
 = 53° with East 30 
cos = –1  | R |min  P  Q

4. B
  10. A
| F2  F1 | (250)2  (500)2 F2
  
= 250 5 N W of N R  A B

F1 500 | R | A2  B2  2AB cos 
tan  = F  250
2
F1 when  = 0
a = tan–1(2) 
cos = 1  | R |max  A  B
5. (i) A (ii) B (iii) A
  C

(i) A and B = 105° 11. C


  60°   
(ii) A and C = 150° x C  A B
30° 45°
  45° 
(iii) B and C =105° 60°
| C | A2  B2  2AB cos 
B

A
132 = 122 + 52 + 2×12×5 cos   = /2

6. B
12. B
R=A+B
   
B PQ PQ

A P2  Q2  2PQ cos  = P2  Q2  2PQ cos 


C 4PQ cos = 0   = /2
 
| C  R | 0

394-Rajeev Gandhi Nagar, Kota


Vector & Calculus Solutions - 17

13. A 18. C
 
A  3iˆ  2j ˆ
ˆk A  0.5 î  0.8 ĵ  ck̂
 
| A | 32  22  12  | A | 14 | Â | (0.5)2  (0.8)2  C 2
C2 = 1 – 0.89  C = 0.11
14. A
y 19. A
 
Let A  2 î  2 ĵ  B  î  3 ĵ
25m
 
30°  be the angle between A & B
x' x  
A.B 22 3
cos     
| A |.|B | 8.2
y'
3 1
x= –25 cos 30° y= 25 sin 30° cos  
2 2
 = 15°
15. C
y 20. A
 
60 km/h Fx  2 î Fy  3 ĵ
y   
F  Fx  Fy = 2 î  3 ĵ

30 km/hr x 21. D

60 cos = 30  cos  = 1/2 B
 = 60°  y = 60 sin 60°
3
= 60 = 30 3 km / hr
2 
A
16. (a) B (b) C

(a) 3 (b) ˆi  2j ˆ
ˆk   C    
A.B  0  A.C  0  but B  C  0
    
A  î  ĵ  k̂  B  2 î  ĵ B  C have direction along A
    
ˆ . (2 î  ĵ)
A.B = (iˆ  ˆj  k) so A parallel with B  C
=2+1=3
22. B
î ĵ k̂ AB cos = 8 ...(1)
 
(b) A  B  1 1 1 AB sin  = 8 3 ...(2)
2 1 0
(2)   (1)  tan  = 3   = 60°
ˆ  2)
 ˆi(0 – 1) – ˆj(0 – 2)  k(1
= – î  2 ĵ – k̂ 23. B

R  A2  B2  2AB cos 
17. (a) A (b) C
2 2
(a) 6 (b) 6 3 6 5  1  13  2 1 13  cos 
  180 = 1 + 169 + 26 cos 
| A | 4 | B | 3 , = 60° 10 = 26 cos 
   
(a) A.B | A || B |cos   5 
  cos1  
1  13 
=4×3× 6
2
   
(b) A  B = | A || B | sin 
3
= 43 = 6 3
2

www.motion.ac.in
18 - Solution Physics (XI) Topic

24. B 32. B
  y = (x - 1) (x2 + x + 1)
a b 3
   dy
a b 1
dx
 
 1 x2  x  1   x  1 2x  1
   
a  b  3 a 3 b dy
   3x2
2a 4b dx
 
a 2b 33. D
25. B sin x
 y
cos x
A  2iˆ  2j
ˆ
y
dy cos x cosx  sin x   sin x 
2 2î  3 ĵ 
tan   dx cos2 x
3
3 dy cos2 x +sin2x

 dx cos2 x
2
  tan1  
3 x dy 1
2   sec2 x
dx cos2 x
26. C
  
AB  B  A  2iˆ  3j ˆ  ˆi  ˆj  k
ˆ  4k ˆ 34. A
2x  5
 ˆi  4j ˆ
ˆ  5k y
3x  2
27. D dy 3x  2  2   2x  5 3
y = x2 + x + 8  2
dx 3x  2
dy
 2x  1
dx dy 19

28. C dx 3x  2 2
S = 5t3 - 3t5
ds 35. A
 15t2  15t 4
dt
nx
ds y
dt

 15t2 1  t 2  x
1
29. A x  nx
dy x
y = 5 sin x  2
dx x
dy d
dx
5
dx
 sin x  dy 1  nx

dx x2
dy
 5 cos x
dx 36. ( 2x + 1) 4
y = (2x + 1)5
30. C y = u5 u = 2x + 1
y  x sin x dy du
 5u4 2
dy du dx
 x cos x  sin x
dx dy dy du
 .
dx du dx
31. D = 10u4  10(2x + 1)4
y = ex nx
dy ex
 ex  n x 
dx x

394-Rajeev Gandhi Nagar, Kota


Vector & Calculus Solutions - 19

37. (4 - 3x)8 42. D


y = (4 - 3x)9 x = 2t2 + t
y = u9 u = 4 – 3x
dx d  2
dy 8 du   2t  t 
 9u  3 dt dt
du dx
dy dy du  t  2 sec  9
 .
dx du dx
43. D
dy
 27u8 = –27 (4 - 3x)8  = 2t2 – 2
dx
d d  2
a  2t  2 
8 7
dt dt
 x  x a = 4t
38. 1  7  y  1  
7
   at t = 2sec
a = 4(2)
x
y = u–7 u  1 a=8
7
dy du 1 44. C
 7u8 
du dx 7 y = 2x2 – x + 1
8
dy dy du  x dy
 .  u8   1    4x  1
dx du dx  7 dx
dy
Now, 0
39. 8 dx
x = -t2 + 4t + 4  4x – 1 = 0
dx  4x = 1/4
 2t  4
dt 2
1 1
dx Now, y x 1 / 4  2     1
 0  2t  4  t = 2 4 4
dt
2
dx  1  1 1 1 7
 2 Maxima  2   1   1 
dt2  16  4 8 4 8
So x = -4 + 8 + 4 = 8
45. B
40. ymax = 39 ; ymin = 38
y = 2x3 - 15x2 + 36x + 11 t3
S  2t2  3t  4
dy 3
 6x2  30x  36 ds
dx  t2  4t  3
dy dt
 0  6x2  30x  36  0 ds
dx  0
On solving dt
x = 2, 3 t – 4t + 3 = 0
2

t = 3,1
d2y
 12x  30
dx2 d2 s
Now,  2t  4 ....(i)
2 dt2
dy
At x=2  ve Maxima put t = 1 in eq. (i) Put t = 3 in eq. (i)
dx2
= 2(1) – 4 = 2(3) – 4
d2y = –2 < 0 =2>0
At x=3   ve Minima
dx2 Maximum Minimum
ymax = 39 ymin = 38
46. A
41. A y = 4x2 – 4x + 7 ....(i)
V = 2t – 1 dy
 8x  4
dV d dx
 2t  1
dt dt Now, 8x – 4 = 0
= 2(1) 1
a=2 x
2

www.motion.ac.in
20 - Solution Physics (XI) Topic

(c) y = x2 –2x + 1
d2 y
 8  0 Minima I   ydx
dx2
Put value of x in eq. (i) –
 x 
2
  2x  1 dx
2
1 1
y  4   4   7 = 6 x3
2 2   x2  x
3
47. D 1 3 1
y = x3 – 3x ....(i) 50. (a) x–3 (b)  x (c)  x 3  x2  3x
3 3
dy (a) y = –3x–4
 3x2  3
dx I   ydx
Now,  3x2 – 3 = 0 I   3x 4dx
 3x = 3
2

 x2 = 1 I = x–3
 x=±1 (b) y = x–4
I   ydx
d2 t
 3 2x  = 6x
dx2 I   x4dx
Putting values – 1 3
I x
d2 y d2 t 3
2
60;  6  0 (c) y = x–4 + 2x + 3
dx dx2
Minima Maxima I   ydx
Put x = –1 in equation (1)
y = (–1)3 – 3(–1)

I   x 4  2x  3 dx 
y = –1 + 3 = 2 1 3
I  x  x2  3x
3
48. B
y = x3 – 3x 1
51. (a)  (b) –5x–1 (c) 2x + 5x–1
dy x
 3x2  3 1
dx (a) y 2
x
Now,  3x2 – 3 = 0
I   ydx
 3x = 3
2

x=±1 1
I  x2 dx
d2 y
 6x 1
dx2 I
x
After putting value of x = 1 5
(b) y 2
d2 y x
 6 1  6  0 (Minimum) 5
dx2 I   2 dx
x
I   5x 1
x3 x3
49. ( a) x 2 (b) (c)  x2  x
3 3
5
(a) y = 2x (c) y 2
x2
I =  ydx
I   ydx
=  2xdx
= x2  5 
I    2  2  dx
 x 
(b) y = x2 I  2x  5x 1
3
2 x
I =  x dx  3

394-Rajeev Gandhi Nagar, Kota


Vector & Calculus Solutions - 21

3 1 3
1 1 2
2 (b) y x
52. (a) x 3/2
(b) 3x 2 (c) x 2  2x 2 2
3 3 3
3 1  1 
(a) y x I    x 2 dx    x 2 dx
2 2 2
1
I   ydx 
I x 2

3 3 2
5
I  x dx (c) y x
2 2
I  x3 / 2 3 
5
I    x 2 dx
3 2
(b) y 5
2 x 3 2
I    x dx
3 2
I  dx 
3
2 x Ix 2
1
I  3x 2 x3 x6
55. x 
1 3 2
(c) y x y = (1 – x2 - 3x5)
x 
I   ydx   1  x2  3x5 dx 
 1  3 6
I   ydx    x   dx x x
 x Ix 
3 2
3 1
2 2 56. –3 cos x
I x  2x 2
3 y  3 sin x
2 4 2 I   ydx   8 sin x dx
1 3 3 3
53. (a) x4/3 (b) x (c) x 3  x 3
2 4 2 I  3 sin x dx  3 cos x
43
(a) y x 1
3 57. nx
1 3
43 4 1
I  x  x3 dx
3 3 y
3x
4
1
I  x3 I   ydx  
dx
3x
1 1 1 1
(b) y I   dx   n x
33 x 3 x 3
1 1 1 58. 15
I   3 dx   1 dx 1
3 x 3
x3 I  5dx
2 2
1
I x3 1
I  5 x 2  5 1  2  15
2
3 1
(c) y x 3 3
x 59.
 1  2
I   ydx    3 x  3  dx 1

 x I  2 d
4 2 4
3 3 3 3  1  3
I x  x I   1  4 
4 2 2  4 2  2
1 60. 21
 3
54. (a) 2
x (b) x (c)
x

2
4
x 
I   2  3  dx
1 2
1 2 4
(a) y x  x2 
2 I    3x 
1 4  2
1 
I   ydx   x 2 dx 16  4 
2 I  12    6 
 4   4 
I x I=16 + 5 = 21

www.motion.ac.in
22 - Solution Physics (XI) Topic

1. B Let OR be the Resultant of A & C



So | OR |= O A 2   O C 2  2  O A   O C  c o s 
(a) 
R  2r
 
 Angle of OR with OC is 450
A = 3kg m/s N - E
 
b = A (B)

b = -4 × 3 Kg m N - E

(b) b = 12 kg m S - W

 Angle b/w OB & OR is 00


b Represent different Physical quantity..
  
let OX be the resullant of OB & OR
2. D  2 2
so OX =  OB    OR   2 OB  OR  cos 0
AC2 = AB2 + BC2
AC2 = (10)2 + (12)2  2

AD = AC + CD
2 2 2
OX =
2
r    2r  2  2r  r 
AD2 = (10)2+(12)2+(14)2 
OX = r (1+ 2 )
2 2 2
AD = 10  12   14 
AD  21 m 6. A
If   ses then cos 
3. A 
Vector can be paralley shift. So A vector is so from R  A2  B2  2AB cos 
not changed if it is displaced parallel to it 
self. R  se s.

4. C
7. D
Here  = 900

Given A  3iˆ  4j
ˆ
B sin 
tan   
A  B cos  ˆ
B  ˆi  ˆj  2k
6 3   
tan  =     370 R  A  B  4iˆ  5j ˆ
ˆ  2k
8 4

R 4iˆ  5j ˆ
ˆ  2k
5 C R̂   
R 16  25  4

4iˆ  5j ˆ
ˆ  2k
R̂ 
45

394-Rajeev Gandhi Nagar, Kota


Vector & Calculus Solutions - 23

8. B  
A  B  AB sin  ˆ
n
 
n̂ is perpendicular to A & B

12. C

A  2iˆ  3j
ˆ

Same magnitude & Angle b/W 1200 B  5iˆ  6j
ˆ
    
then R  A  B A  B  7iˆ  3j
ˆ
  
R = 2A cos /2 A  B  72  32  58

R =2
13. A
So Resultant lies on + x axis & Magnitude is 2 
cm. ˆ
A  2iˆ  ˆj  2k

ˆ
B  ˆi  ˆj  k
9. A
  
ˆ
R  A  B  3iˆ  k

R ˆ 3iˆ  k
3iˆ  k ˆ
R̂    
R 91 10

11
x 
14. 5   1 
Resultant of all the forces is zero.  2 

10
10. A x 
y    1
   2 
a  x1ˆi  y1ˆj, b  x2ˆi  y2ˆj
  x
y = u–10 u 1
b  a 2

 
x2ˆi  y2ˆj   x1ˆi  y1ˆj dy
  10u1
du 1

du dx 2
on comparing
du dy du
x2  x1 , y2  y1  .
dx du dx
x2 y
 2 11
x1 y1 x 
= –5 u–11  5   1 
2 
x2 y2 x1 y1
So x  y  x  y
1 1 2 2 15. 5 cos 5x
y = sin 5x
11. D y = sin u u = 5x

dy du
 cos u 5
du dx

dy dy du
 .
dx du dx
= 5 cos u  5 cos 5x

www.motion.ac.in
24 - Solution Physics (XI) Topic

2 2 1
16. cos x   2e2x 22. 1  2x  
3
x x x2
y = sin (x) + n(x2) + e2x
 1  1 
dy 2 y   x   x   1
 cos x   2e2x  x  x 
dx x
dy  1  1   1  1
 dx  1  2  x  x  1   x  x 1  2 
17. 2 cos (x + )  x     x 
y = 2 sin (x + )
dy 2 1
y = 2sin u u = x +   1  2x  3  2
dx x x
dy du
 2 cos u 
dx dx cot x
23. y
1  cot x
dy
 2 cos u
dx
 2
 2
dy 1  cot x   Cosec x  cot x(cosec x)

= 2 cos (x + ) 
dx (1  cot x)2

2 dy  cos ec2 x
18.  sin x
x2  dx 
(1  cot x)2
y = n x2 + sin x y = 2nx + sin x

dy 2 24. y = 2u3, u = 8x – 1
   cos x
dx x
dy dy du
 ·
2
d y 2 dx du dx
also  2  sin x
dx x = (6u2)(8)
= 48 u2
6 13 / 7 = 48 (8x – 1)2
19. x  2 sec2 x tan x
49

y  7 x  tan x = x1/7 + tan x 25. y  sin u, u  3x  1

dy 1 6 / 7 dy du
  x  sec2 x  cos u , 3
dx 7 du dx

d2y  6 13 / 7 dy dy du
 x  2 sec2 x tan x  ·
also dx du dx
dx2 49
= cos u. 3
= 3 cos u
20. ex tan x  ex sec2 x = 3 cos (3x + 1)
dy
y = ex tanx   e x tan x  e x sec2 x
dx 26. A
S = 4 + 2t4
21. 2xsin4x+4x2 sin3x cosx+cos-2x+2x sinx cos-3x
ds
y = x2sin4x + xcos-2x   8t3
dt
dy
  2x sin4 x  4x 2 sin3 x cos x  d
dx a  24t2
cos-2x + 2x sinx cos-3x dt
Now, t = 1 sec.
a = 24 (1)2
a = 24 m/sec2

394-Rajeev Gandhi Nagar, Kota


Vector & Calculus Solutions - 25

27. A 32. A
P = t2 + 2t + 1
a
S  bt2
dP t
F  2t  2
dt
ds a
at t = 2 sec v  2  2bt
dt t
F = 2(2) + 2
F=6N dv 2a
a  3  2b
dt t
28. D
2a
P  2t3  t2  2t  1   2b
t3
dP
F  6t2  2t  2
dt
33. A
at t = 2 sec v = 3t2 – 2t + 1
F = 6(2)2 + 2(2) + 2
dv
F = 30 N = 6t – 2
dt

29. A dv
= 0  6t – 2 = 0
q = 2t2 + 3t + 1 dt

dq 1
i  4t  3 t=
dt 3
at t = 5 sec
i = 4(5) + 3 34. x + y = 60  z  xy
i = 23 A y = 60 – x  z = 60x – x2

30. A dz
z = 60x – x2   60  2x
S = 1 + t + t2 dx

ds dz d2z
v  1  2t   0 x = 30   ve maximum
dt dx dx2

1 x = 30 y = 30
K.E.  mv2
2

1 2 40  a2
  3  103 1  2t  35. a2 + 4ah = 40  h 
2 4a

1 
2 48  a
2

  3  103  1  4t2  4t  v = a2h = a  4a 
2  
at t = 5 sec
a3
1 v = 10a –
  3  103 1  4 25  20  4
2
K.E. = 1.815 × 105 erg. dv 3a2 40
  10  0 a
da 4 3
31. C
 = t2 + 3t – 5
 2x 
3
36. I  5x  7 dx
d
  2t  3
dt x 4 5x2
I   7x  c
at t = 5 sec 2 2
 = 13 Rad/sec.

www.motion.ac.in
26 - Solution Physics (XI) Topic

1 2  42. I   cos   tan   sec   d


37. I     3  2x  dx
5 x 
 sin  1 
I   cos     d
x 1  cos  cos  
I   2  x2  c
5 x
I   sin   1 d
I   cos     c
38. I  x  3 x dx
 1 1
 2
I    x 2  x 3  dx
43. I  d
 

4
2 32 3 3 2
I x  x c  2  1
3 4 I      42  2 
2
  2

39. I   x 3  x  1 dx 3 2
I 
2
 x 
2
I  x 3 dx

3 7
1 1
I  c 44. I  x2dx
x 2x2 0

3 7
 x3  1 7
t t t I   7  0 
40. I dt  3 0 3 3
t2

 1 3
 
I    t 2  t 2  dt 
  45. I   cos x dx
0
2
I2 t c 
t I  sin x0  sin   sin0  0

1
 t dx
41. I  t3 dt 46. I  3x  2
0

4 t 1 1 1
I    3  3  dt I   n 3x  2    n5   n2
t
 t  3 0 3

1 5
 5
  I n
I    4t 3  t 2  dt 3 2
 

5
 1 47. I   ydx
 t 3 1  t 2
I  4  5
 3  1    1 b
b
2 I   3x2dx   x3   b3
0
0

2 3
I  2t 2  t 2 c
3

394-Rajeev Gandhi Nagar, Kota


Vector & Calculus Solutions - 27

1. A,B,C 5. A,B
  
| R |max | A |  | B |
(i) (ii)     
| R |min | A |  | B | if | A || B |
    
| R |min | B |  | A | if | B || A |
(iii) (iv)  
Case-I If | A || B |
Right hand rule   
| R |max | A |  | B |
   
| R |min | A |  |B|
2. A,C  
 3 | A |  | B|
B   
 1 | A |  | B|
A
– 
B    
3 | A | 3 | B|| A |  | B|
 

2 | A | 4 | B|
A 
|A| 4 2
      
A  B  A   B  |B| 2 1
     
A B  A B Case-II If
    | B || A |
A  B is resultant of A & –B .   
| R |max | A |  | B |
   
3. A,D | R |min |B|  | A |
      
Given, a  b  a  b  2 a 3 | A |  | B|
    
& a  b 1 | B|  | A |
   
3 | B | 3 | A|| A |  | B|
a2  a2  2a2 cos   a2  a2  2a2 cos   2a  
2 | B | 4 | A|
2a2 1  cos    2a2 1  cos    2a 
|A| 1
2a2 2 cos2  / 2   2a2 2 sin2  / 2   2a  
|B| 2
2a cos /2 + 2a sin /2 = 2a
cos /2 + sin /2 = 1
6. A,B
Squaring both side :
    Vm  8iˆ
cos2  sin2  2 sin ·cos  1
2 2 2 2 Vw  V cos 45ˆi  V sin 45ˆj
  Relative velocity,
2 sin ·cos  0
2 2
Vwm   V cos 45  8  ˆi  V sin 45ˆj ....(i)
 
sin  0 cos  0
2 2 V sin 45
Now,  tan 60 
  V cos 45  8
 0  90
2 2 V/ 2
 = 0°  = 180°  3
V/ 2 8
4. B,C 8 6
V km/h
3 1
Now, putting value of V in equation (i) –
 8 6 1   8 6 1 ˆ
Vwm   ·  8  ˆi   · j
 3 1 2   3 1 2 
www.motion.ac.in
28 - Solution Physics (XI) Topic

2 2 11. A,C
 8   8 3 
Vwm      
 3 1  3  1 Displacement S vector =  x – 4 ˆi   y  3 ˆj
2 y
 8   (x,y)
   1  3
 3  1

8 16
 2  km/h
3 1 3 1 (4,3)
x
7. B,C,D
  
Vector product of A & B may be less of equal S   x  4 2   y  32
to scalor product and It may be also zero.
10   x  4 2   y  3 2
8. A,B,D
2
   2
100   x  4    y  3 ....(1)
C  A B
 Given (y – 3) = 6
C  A2  B2  2AB cos  y=9
 Put in Equation (i)
B 
C (x – 4)2 = (8)2
x–4=±8
x–4=8 x – 4 = –8
x = 12 x = –4
 (12, 9) (–4, 9)
A

if  = 90° 12. A,C


C2 = A2 + B2 
x – component of d1 i s posi ti ve and
if  > 90°
cos  = Negative y–component is also positve.

C2 = A2 + B2 – 2AB x – component of d2 is Negative and
C2 < A2 + B2
y–component is positive.
if  > 90°  
cos  = Positive x and y – components of d1  d2 are positive.
C2 = A2 + B2 + 2AB
C2 > A2 + B2 13. A,D
  
9. ABD A and B is in a plane vector C is in different
By theory plane.
  
So A  B  C can not be zero.
10. A,D   
A  B  C Lies in a plane different from that
Magnitude of ˆi  xj ˆ
ˆ  3k of any of the three vectors.

magnitude of  4ˆi+  4x-2  ˆj  2k


ˆ

2
2
16   4x  2   4
1  x2  9 
2
4 + 4x2 + 36 = 16 + 16x2 + 4 – 16x + 4
3x2 – 4x – 4 = 0
3x2 – 6x + 2x – 4 = 0
3x(x – 2) + 2 (x – 2) = 0
(x – 2) (3x + 2) = 0
x = 2, –2/3

394-Rajeev Gandhi Nagar, Kota


Vector & Calculus Solutions - 29

1. 2(F12 + F22)  
F2 4. F1 =–(12 3 –1) j & F2

=(12–5 3 ) i +(12 3 – 15) j

In x-direction
 10cos30° + F2sin30° = 15cos37°
F1 
F2  2(12  5 3)N  F2  F2 sin30iˆ  F2 cos 30j
ˆ
P2 = F12  F22 + 2F1F2 cos
Q2 = F12  F22 – 2F1F2 cos  (12  5 3)iˆ  (12  5 3) 3j
ˆ

P2  Q2  2(F12  F22 ) In y -direction


F2 cos 30° + 10sin30° + 15 sin37° = F1
F1
90° (12  5 3) 3  5  9  F1
90-Q
F1 = 14 + 12 3 – 15 = (12 3  1)
F2 F2 = 180–Q 
ˆ
F1 = (12 3 –1)  F1  (12 3  1)j

2. 0 
N 5. 6AO
4m
A B     
37°
AB  AC  AD  AE  AF
3m
5m
E
w
E
O
F D

O
S
Displacement = 0 A C

  B
3. (a) |F3 | = 8 N, q = 90° (b) F2 = –6 i      
y   AO  OB    AO  OC    AO  OD 
   
F1
  AO  OE    AO  OF 
F2
37°

x OA
O OF

F3 OE

(a) x - direction  F1 sin37° = F2 sin 


OB
3
10   6 sin    = 90°
5 OD
  OC
y direction  | F 3 || F1 | cos 37 + F2 cos      
  5AO  OB  OC  OD  OE  OF 
| F 3 | 10  4 / 5  0  8  
 5AO  AO
 
(b) | F 2 | 6 sin (ˆi)  6 cos (j)
ˆ
 6AO
As  = 90° then F2 = 6iˆ

www.motion.ac.in
30 - Solution Physics (XI) Topic


6. 2FA dq 1 du
       du   2  2r
 AB  BC  CD  DE  EF   AF 2 u dr
  dq dq du 1
  .   .(2  2r)
 AF  AF dr du dr 2 u

 2AF 1r
 
 2FA 2r  r 2

  x2 
7. 2OC  1
      12. With u =  8  + x –   ,
  x
 AO  BO  CO  DO  EO  OF
  dy dy du
 OF  OF 3 x 1
y = u4 : dx  du dx  4u .  4  1  2   4 ,
 x
 2OF 3
   x2 1  x 1
  OF   OC  4
 8
 x –    1 2 

x 4
  x 
  2OC 4
 x2 1
 y    x   y = u4
8. 8  8 x
2
  x 1
ˆ ;
r1  2iˆ  ˆj  k r2  3iˆ  2j ˆ
ˆ  2k  u  x
8 x

ˆ
F  2iˆ  ˆj  k dy du x 1
  4u3  1 2
   du dx 4 x
w  F · r2  r1 
dy dy du
  .
ˆ  ˆi  3j
w  2iˆ  ˆj  K ˆ
ˆ  3k dx du dx
3
 2+3+3  x2 1  x 1 
|w| = 8 J 4
   x    1 2 
 8 x   4 x 

dy – x sin x – cos x x sin x  cos x 13. 24


9.  2 +
dx x cos 2 x 5 2
I  rdr
cos x x 2
 y  5 2
x cos x r2  1
I   25  2  2
dy x    sin x   cos x cos x  x sin x 2 2
   2
dx x2 cos2 x I = 24
dy  x sin x  cos x cos x  x sin x
   14. 0
dx x2 cos2 x
2

10. 3sin x cosx + 3cos3x


2 I  sin  d
0
tan q
p 2 2
1  tan q I   cos 0   cos 0
dp (1  tan q) sec2 q  tan q sec2 q I   cos2  cos0  0
 
dq (1  tan q)2
dp sec 2 q 15. e–1
 
dq (1  tan q)2 1
I   exdx
1– r 0
11.
2r – r 2 1
I  ex   e1  e0 
0
 q  2r  r 2
I  e 1
 q  u u = 2r – r2

394-Rajeev Gandhi Nagar, Kota


Vector & Calculus Solutions - 31

1. 2
40
R2 = P2 + Q2 + 2PQ cos60° 6. x m
3
7Q2 = P2 + Q2 + PQ  P2 – 6Q2 + PQ = 0
v = b2  – (1)
2 40 = b2 + 4  b
P  Q
   6 0 40  b2
Q
  Q   –(2)
4b
P  P  P from (2) to (1)
   3   2   0  2
Q Q  Q
v = b2
 40  b  2

4b
b
2. a = – 7, b = – 3, c = – 4 40b b 3

  v=
4

4
b
F1  2iˆ  aj ˆ  F  5iˆ  cj
ˆ  3k ˆ
ˆ  bk
2

  b3
ˆ  F  cˆi  6j ˆ v = 10b –
F3  bˆi  5j
ˆ  7k
4
ˆ  ak 4
    dv 3b
F1  F2  F3  F4  0  In x-direction = 10 – =0
db 4
2+5+b+c= 0  b + c = –7 ....(1)
in y-direction 40
b= m
a + c + 5 + 6 = 0  a + c = –11......(2) 3
in z–direction
–3 –b –7 –a = 0  a + b = –10 ...(3)
On solving a = –7, b = –3, c = –4 b
7. Using n subintervals of length x  and
n
dy b
3.  – x 2 sin x 2
dx right- endpoint values : Area =  2x dx  b
dy 0
=–x2sinx + 2x cos x – 2[x cosx+sinx] + 2sinx
dx y = 2x
= – x2 sin x + 2x cos x – 2 x cos x b
– 2 sin x + 2 sin x = – x2 sin x  ydx   2x dx
0
dr
4.  cos   sec 2   x2 
b
d  2 
dr  2 0
= [1 + sec ] cos  + sin  [sec tan]
d  b2 
2 3  2  0
sin  cos   1 2 
= cos  + 1 = = cos + sec2
cos2  cos2  = b2

5. x = 30 & y = 30
b2 b(4  b)
Let m = xy 8. b 
m = 60 y - y2 4 4
x
dm y  1
= 60 – 2y = 0 2
dy b
x 
y = 30  ydx    2  1 dx
0
x = 30
b
 x2  b2
  x  b
4 0 4

b  b  4
 b   1  b  
4   4 

www.motion.ac.in
32 - Solution Physics (XI) Topic

1. (A)  Q, (B)  R, (C)  P, (D)  S 5. B


(A) R = 2A cos / 2  A = 2A cos /2 S-1
 = 120°
8N
(B) A – B  R  A + B  4  R  12
R = 12 N
   
(C) A.B | A | . | B | cos 
  6N
A.B 0 
cos  =   = | R | 10 N
| A || B | 8 9
S-2
cos = 0   = 90°  
   | A || B | 1
(D) R  A  B    
 | A  B |2  | A.B |2  1
| R | A2  B2  4  1  9  14 LHS
(AB sin )2  (AB cos )2
2. S = t3 – 2t2 + 5t + 4
(A) at t = 1 sin2   cos2   1  S-2 is true
S=1–2+5+4  S=8
6. A
ds S-1
(B) = 3t2 – 4t + 5  at t = 1  
dt A.B  0
 
ds then A & B are perpendicular to
 3– 4 5  4  
dt each other  A.C  0
 
d2s then A & C are perpendicular to each
(c)  6t – 4
dt 2    
other  B  C | B || C | sin n̂
d2 s
at t = 1   6–4  2 n̂ shows the direction
 
which is along to n̂ .
dt 2 
so A is parallel to B  C
  
3. (a) FR  F1  F2  2i  5 j  4k S-2
   
  A B & A C
F1  2 î  2 ĵ  F2  3 î  4k̂ 
so A is perpendicular to plane formed by
    
(A) FR  F1  F2 = 5 î  2 ĵ  4k̂ B & C true
 
F.F 7. A
(B) cos  =  
1 2

|F1 ||F2 | S-1 C

6 3 
cos    cos  = C 
2 2 5 5 2 B

 
(c) F1.F2  F1F2 cos  A  B
A
    
F1.F2 6 A BC  0
F1 cos   =
F2 5 statement - 1 is true.
S-2
4. C C
S-1
 
Vector quantity obeys the triangle law of C B
addition
S-2 A  B
A–BR A+B A
  
S-1 is true & S-2 is false A BC  0
Statement - 2 is true.

394-Rajeev Gandhi Nagar, Kota


Vector & Calculus Solutions - 33

8. A  
  A B
A  î  ĵ  B  k̂
    
A.B | A || B | cos  B
 = cos    = /2
S-2 
  
A
    A.B
A.B | A || B | cos   cos    
| A || B | 
 3iˆ  2j  ˆ  6j
ˆ  2iˆ  2k ˆ 
9. B
Statement - 1  6j ˆ  4iˆ
ˆ  4k
Distance is a scalar quantity because of it
has only Magnitude  68 unit
Statement - 2
Distance is the length of path traversed

(V) F  2 î  3 ĵ  6k̂
10. (i) True (ii) False (iii) True (iv) True
        
(i) A.B | A || B |cos   B.A | A || B |cos  | F | 4  9  36 = 49 = 7
    
(ii) A  B  B  A  R î  ĵ  k̂
Because in this magnitude is same but (vi) R  î  ĵ  k̂  R̂   
|R | 3
direction is different.
 
(iii) let A & B be two non- zero vector î ĵ k̂
  R̂   
A  B = A B sin n ˆ 3 3 3
      
if  = 0 then A  B  0 (vii) A  B  then A.B  0
   = 90°
then A & B are collinear

(viii) A  î  ĵ
P
y 1
tan    1
x 1
(iv)   = 45°



(y) A
tan  = m = slope = 0

  45 
î (x)
 
11. (i) A.B 
 2iˆ  5k
ˆ · 3j ˆ
ˆ  5k  (ix)
  
A BC  0
= 0 + 25 = 25 unit   
  given A.(B  C)
(ii) c  B A    
  –(B  C).(B  C)
3iˆ  4j
ˆ      
c  25  =– (B).(B  C) – (C).(B  C)
5
 =–0–0=0

c  3iˆ  4j
ˆ5


c  15iˆ  20jˆ
   
(iii) A  B then A  B = zero
 
(iv) A  3 î  2 ĵ B  2î  2k̂
Area of parallelogram =

www.motion.ac.in
34 - Solution Physics (XI) Topic

1. D 6. A
 dx 2
x  t3  6t2  3t  7  v  dt  3t  12t  3
dy
a = 6t –12 = 0 ; t = 22 sec
dt
v = 3(2)2 – 24 + 3
v = 12 – 24 + 3
P Q R v = –9m/s
 
sin120 sin90 sin150
P Q R 7. A
 
3 1 1  
2 2 F1  F2  10 dyne
P:Q R= 3:2:1  
F  2 F1 cos  / 2  10 dyne
2. A
  = 120°

8. C
   
A B  A B

A2 + B2 + 2AB cos = A2 + B2 – 2AB cos


projection on Shore line = 18 cos60° 4ABcos = 0
= 9 km/h

2
3. D
 9. A
A  ˆi  2j ˆ
ˆ  3k
  Displacement BC = AB2  AC2
B  4iˆ  2j ˆ  AB  3iˆ  4j
ˆ  3k ˆ

  3iˆ  4j
ˆ (r  r cos )2  r2 sin2 

3iˆ  4j
ˆ ˆ ˆ
AB   V  10  5   6i  8j
5   BC = 2rsin/2
C
4. D r r
  F
F1  2 F 2 B
O A
F2
F2 sin 
tan  
F1  F2 cos 
  = 90° F1 10. A
1
then F1 + F2 cos = 0  cos = 
2
 = 120°

5. B b


abˆ | a2  b2  2ab cos   
Angle b/w P & Q is 0
 = 60°  | ˆ
ab ˆ | 1 60°
a
so Resultant = P2  Q2  2PQ cos 

R PQ

394-Rajeev Gandhi Nagar, Kota


Vector & Calculus Solutions - 35

11. B 13. A

A  a1ˆi  a2ˆj

B  4iˆ  3j
ˆ

|Aˆ | 1  a12  a22  1 ...(i)


 
A.B  0
4a1 – 3a2 = 0
4a1 = 3a2 ...(ii)

OD2  OE2  ED2  OD  32  42  5miles 2 16 2


(i) (ii)  a1  a1  1
9
9 3
a12  , a1 = = 0.6, a2 = 0.8
25 5

12. B

A  3iˆ  2j
ˆ  8kˆ 14. B
 
 B  xa
B  2iˆ  xj
ˆ kˆ 
    if x is –ve direction of B change
A.B | A || B | cos   = 90°  
  if x is +ve direction of B same as a
A.B  0  
B & a are colinear vector
6 + 2x + 8 = 0 , x = –7

www.motion.ac.in
36 - Solution Physics (XI) Topic

Section A - Distance, Displacement, Velocity 5. B


and Acceleration, Equation of t  62.8 sec
Motion
in each lap car travel a distance =  2R
1. B  2  3.14  100  628 m
In each lap displacement of the car = 0
Average speed
r r Total Dis tan ce 628
A B =   10 m / s
Total Time 62.8
Total Displacement
Average Velocity 0
Total Time
Displacement = 2r
distance = r 6. C
let the acceleration of the body is a and u  0
2. B 10 m
B 0,10) 1 2 1 2
( 10
,1 then x1  at  a10 
2 2
1 2 1 2 1 2
x 2  a 20   x1   a 20   a 10 
C
2 2 2
1 1 2 1 2
,0)
(0,0 A 10
m  a 10 30   x 3  a 30   a 20 
10m 2 2 2
Fly start from A and reaches at B. 1
 a 10 50    x1 : x 2 : x 3  1 : 3 : 5
B 2

7. D
Let constant acceleration = a
10m
1
S= at2
2
C 1
A 10 2 S1  a  102  50a
2
 AB  10 2   10
2 2
 10 3m 1 1
S2  a  202  a  102  150a
2 2
3. B S2 = 3S1
d d
From A to B t1= hr  From B to A t2= 8. B
20 30
hr In inclined initial u = 0
d 1
t1
S= at2 and a = gsin
A B 2
d 1 2
l = g sin   (4) ...(i)
A t2 B 2
 1
Total Dis tan ce  g sin  t2 ...(ii)
 Average Speed = 4 2
Total Time From (i) and (ii)
2d 2d t = 2 sec
 
t1  t 2 d d  v  24 km/hr

20 30 9. A
u = 10m/sec a = - 2m/sec2
4. B Total time taken when final is zero.
Total Dis tan ce v=0
Average velocity =
Total time 0 = 10-2t
t = 5 sec
2000 80V
48    V  60 km/hr 1
1000 1000 40  V S = ut + at2
 2
40 V

394-Rajeev Gandhi Nagar, Kota


Kinematics - 37

1 14. C
St=5 = 10 × 5 - × 2 × 25 = 25 1
2 S = ut + at2
1 2
St=4 sec. = 10 × 4 – × 2 × 16 = 24
2 1
St=5 - St=4 = 25 – 24 = 1 m O = u(8) – (10)(8)2
2
a u = 40 m/s
S  u  2n  1
2 Now, v = u – gt1 ....(i)
2 –v = u – g (t1 + 4) ....(ii)
S = 10 + ×9 (1) = (2)
2
=1m u – g(t1 + 4) = –u + gt1
2u – 4g = 2gt1
10. B
t1 = 2 sec
Initial velocity u = 0, Let acceleration = a
1 1
St=2 = a×4=2a H  40 2    10  2 
2 2
1 9 H = 60 m
St=3 = a×9= a
2 2
9 5a 15. D
St=3 - St=2 = a - 2a = u = 6 m/s
2 2
9a 7a 1
t sec
St=4 - St=3 = 8a - = 2
2 2
V = u + at
S3 5 1
 V = 6 – (10) ×
S4 7 2
V=1

Section B - Motion under Gravity 16. A


11. C
V = u + at
h 1 2 O = –u – 16t1
 gt1 ....(1)
2 2
u
1   t1 ....(1)
h  g(t1  t2 )2 ....(2) 16
2
From equation (1) and (2) V = –4t2
2t12  (t1  t2 )2 V
t2  ....(2)
2t1  t1  t 2 u
V2 = u2 + 2as
( 2  1)t1  t2 O = u2 – 2 × 16 × H
t2

2 1 u2
t1 = H ....(3)
2 1 2 1 32
t1 =  2  1 t 2
V2 = 2 × 4 × H
V2
12. C H ....(4)
Acceleration for both is g 8
1 –a,–g +a,–g
a  gt 2a
2 t1 t2
1
b  gt2b
2 from (3) & (4)
t a : tb  a : b u = 2v ....(5)
2 t2 4V
Now,  
13. D 1 t1 u
At Hmax , v = 0 4V

Acceleration constant & it is due to gravity 2V
a =g  2

www.motion.ac.in
38 - Solution Physics (XI) Topic

17. B
O = u – (g + 2)t1 v = (g – 2)t2 20. A
v = u + at
u u O = 50 – 10 × t
t1   v = 8t2
 g  2  12 t = 5 sec.
a
v Sn  u  2n  1
t2  2
8
10  
–g–2 –g+2 S1  50  2 1  1
2
S1 = 45 m
ascent descent
v2 = u2 + 2as v2 = 2(g – 2)H 10
S5  50  2 5  1
u2 = –2(g + 2)H 2
8 2 t22 S5 = 50 – 45
u2
H= H S5 = 5 m
 g  2 2 g  2
S1 45 9
t12 82  12 8 2  
   S5 5 1
t22 122  8 12 3
 9:1

18. C Section C - Graphs


1
S  ut  at2 ....(1) 21. C
2
dx
1 2 Vinst = (slope of x-t graph)
–40 = ut –  10  t dt
2
At C tan  =+ve At E   90º  ve slope 
–40 = ut – 5t2 ....(2)
Now, v2 = u2 + 2as At D   0º At F   90º  ve slope 
(3u)2 = u2 + 2(–10)(–40)  At E vinst is negative
9u2 = u2 + 800
8u2 = 800 22. C
u = 10 m/s From graph it is clear that velocity is always positive
Put value in equation (2) – during its motion
–40 = 10t – 5t2 so displacement = distance
5t2 – 10t – 40 = 0 displacement = Area under V-t curve
t = 4 sec.
1 1
  20  1  20  1   1 10
19. C 2 2
v2 = u2 + 2as
1 10  1 10  55 m
–(a+g) v (a–g)
v1 23. D
m
S B
O = v2 – 2(a + g)H
A
v2
H ....(1)
2  a  g 60
º
30º
v' = –2(a – g)H
2 t
VA tan 30º VA 1/ 3 1
v '2    
H ....(2) VB tan 60º VB 3 3
2  g  a
Now, from (1) & (2) 24. A
2 2 Total Distance = Area
v v'
 under the curve (Positive + Negative)
2  a  g 2  g  a
1 1 1 1
  4  1  4  2  1 4    2  1  2  2   1 2
ga mg  F 2 2 2 2
v'  v v =2+8+2+1+4+1
g a mg  F
= 18 meter

394-Rajeev Gandhi Nagar, Kota


Kinematics - 39

25. A 31. A
v dx
(acceleration) = Slope  v  dx  vdt
t dt
10 x 2
OA  1
10  dx    t  2dt
AB  0  0 3 0
2
10  t2 
BC   0.5 x  3    2t 
20 2 0
26. B 4
x 3  4
Distance = Total Area under curve 2
= 105 m x–3=6
Displacement = 90 - 15 = 75m x = 9 m.

27. C 32. (a) B


Equation of given sin curve is v
ds
x   A sin t slope  tan   v
dt
dx t
V   A cos t (b) D
dt
dv
tan   a
28. B dt
1
d 
dv
 tan 45   v   v 2 33. (a) A
dt dt Displacement
dv 1
 v2 t = 3,  3 1 1 1
 4  20  2  20   20  4   4  10
dt v =
2 2 2
dv 1 = 100
 m/sec2
dt 3 Displacement
Ave. velocity =
time
29. C 100 50
  m/s
a.s = Area of graph 14 7
1 (b) C
a.s =  10  30
2 Total distance
avg. speed =
dv total time
V . S  150
ds Total distance =
dv 150 1 1 1
V   4  20  2  20   20  4   4  10
ds 30 2 2 2
v 30 = 40 + 40 + 40 + 20
10 vdv  0 5 ds = 140
v 140
 v2  30 avg. speed =  10 m/s
   5  s 0 14
 2 10
v2 100 v2 34. A
  5 30  0  ;  150  50
2 2 2
v2 = 400 v2f v2
 i  Area under (a – s) curve
v = 20 m/s 2 2
1 1
30. A  11 
2 2
 dx 
vinst    v2f 1
 dt 12.5 0 
80 8 2 2
   2 m/s vf = 1 m/sec
16  12 4

www.motion.ac.in
40 - Solution Physics (XI) Topic

Section D - Variable Acceleration 40. C


a = t3 – 3t2 + 5
35. B
dv
Vx  2at  t3  3t2  5
dt
Vy  2bt v 2

 t  3t2  5  dt
3
 dv 
2 2
V  2t a  b 6.25 1
2
 t4 
36. A v  6.25    t3  5t 
4 1
1 2 v = 8 m/s
2s  gt2  s  gt
2 t4
ds v  t3  5t
v  gt 4
dt
dx t 4
  t3  5t
37. D dt 4
ds x 2 4 
v  3t 2  12t  3 t 3
dt  dx    4  t  5t  dt
8.30 1
dv
a  6t  12 2
dt  t5 t 4 5t2 
x  8.30     
a  0  t  2 sec  20 4 2 1
V2 sec  3(2)2  12(2)  3 x = 15.6 m
= + 12 -24+3 = -9 m/s
41.
38. D (a) D
dv x = 2t3 – 3t2 + 1
 t2
dt dx
v  6t2  6t
v 4 dt
 dv   t  2 dt Now, v = 0
0 0 6t2 – 6t = 0
2 4
6t (t – 1) = 0
   t  2  dt +  t  2  dt t = 0, 1 sec
0 0
41.
2 4
  t  2 2    t  2 2  (b) A
   
 2 0  2 2 at origin t = 0
= 2 + 2 = 4 m/s v = 6t2 – 6t
v=0
39. C
a = 3t2 + 2t + 2 42. A
dv 
 3t2  2t  2 v  aiˆ  6tj
ˆ
dt vx = a, vy = bt
v 2
dvx
 dv   3t  2t  2  dt
2
ax  0
dt
2 0

2
dvy
v  2  t3  t2  2t 0 ay  b
dt
v–2=8+4+4–0–0–0 
v = 18 m/s a  axˆi  ayj
ˆ  oˆi  bj
ˆ
 2
a  0   b2

a b

394-Rajeev Gandhi Nagar, Kota


Kinematics - 41

43. B v
 1 Ln  A Bv  
v 5 1x     t
Initially, x = 0  B  0
v 5 10  A  Bv 
v = 5 m/s Ln  Bt
A
dv 5 A – Bv = Ae–Bt
a  .5 1  x
dt 2 1  x Bv = A (1 – e–Bt)
a = 12.5 m/s2 A
v 1  eBt 
B
44. C
v = 20 + 0.1 t2 48. A
dv t = (a – bx)
a  0.1 2t 
dt Q x

a = 0.2 t  vdv    a  bx dx


Non – uniform acceleration v 0

45. B bx2
0  ax 
2
0t5
dv 
1 bx 2a
 25  t2 2 a  x
dt 2 b
1
v 5 2 49. B
 dv   25  t 
2
dt
0 0 ds
v
 t5 dt
v 7
3 5 7 10
 dv   dt
8
vt  5 5  ds   2t  13 dt   3t  8 dt    4t  1 dt
0 5 7

5 7 10
46. D  2t2   3t2   4t2 
  13t     8t     t
t2  2 0  2 5  2 7
x
2
 3  49 3 
dx 2t  25  65    56   25  40 
vx    2 2 
dt 2
2 2
2 2  2 10   10  2 7   7
vx  2
2  90  36  56  40  2  51  10  7
= 247
x2
y
2 50. C
dy 2x  dx   t
vy   .  f  f0 1  
dt 2  dt   T
 f=0
2  t2 
 .2 = t2 = (2)2 = 4  t
22 0  f0 1  
 T
 t=T
v  vxˆi  vyˆj  2iˆ  4j
ˆ
dv  t
 f0  1  
47. A dt  T
vx T T
dv  t
 A  Bv  dv   f0 1  T 0 dt
dt 0 0

v t
f0 T
1 vx 
dv  2
 A  Bv  dt
0 0

www.motion.ac.in
42 - Solution Physics (XI) Topic

Section E - Ground-to Ground projectile 57. D


Motion, Equation of Trajectory, (i)
Projectile from tower, Projectile For  and 90-
Motion in Inclined plane
Range is same
51. B
 = 150
y = 8t – 5t2 x = 6t
90- = 750
1 2
y  u sin  t  .gt x = u cos  · t u sin  .u cos 
2 (ii) R =
g
After comparing the above equation
sin (90 - ) = cos 
u sin  = 8 ....(1)
u cos  = 6 ....(2) cos (90 - ) = sin 
4
tan  
3 58. B
4 1
sin   5 E mv2
5 4 2
From equation (1) – At Highest Point
4  vel = vcos 
u·  8
5 3 1 E
KE  mv2 cos2    E cos2   (  = 45°)
u = 10 m/s 2 2

52. D 59. D
2u sin  R = u2 sin 2 / g
T
g Rmax = u2/g
2  9.8  sin 30 u2
T  22 
9.8 g
T = 1 sec
for  = 15°
u2 sin 30 1
53. B R  22   11m
g 2
ge
gm 
6 60. D
2
u sin2  x
R  y  x tan  1  
6 gm  R
x2
y  x tan   tan 
u2 sin2 R
Rm 
gm Compare from eqn.
Rm = 6 R tan = 16
tan  5

R 4
54. C
64
In a projectil e motion the vel oci ty i s R  12.8 m tan = 16
5
perpendicular to the acceleration for one 1 g 5

instant only. 2 u2 cos2  4.2
u2 sin 2 2g 2  16
R  
55. D g 5 g
R  12.8 m
The information is insufficient to decide the
relation of RA with RB.
61. C
Range of  and (90-) is same
56. D
Horizontal Component of velocity If  = 30°
Because there is no acceleration in horizon- So (90 - )= 60°
tal Direction

394-Rajeev Gandhi Nagar, Kota


Kinematics - 43

62. B 67. A
at max height 2h 2r
t t' 
u g 
 Vx  ux   ucos  ,  = 60°
2
2h
2 22 t g
u sin2 33uu n 
So range  =
t' 2r
g 22 gg

 2h
63. C n
2r g
For both particles uy = 0 and ay = -g
1 2 68. A
h gt  h  same  t  same
2 d   cos 60  t
 cos 60t
64. B cos 30 
AB
Vel. of Bomb is same as the vel. of aeroplane. AB   cos 60t / cos 30
ux = 360 km/h & uy = 0.
1 2
1 AB    
Sy = uyt + a t2 , Here u = 0 2 3
2 y
1 2 69. A
1960 =  9.8t
2
2u sin 
t  20 sec T
g

65. B when Angle increase, time of flight increase.

70. A
/ s
15m
u 
30°
37º o °
n30
A B 0° gsi
3
p 9m sin gcos30°
20
° 30° g
30 20cos
30°
In this process both time taken is same.
2 u sin 
T
g 2g 2  sin30  20
T  
2u sin 37 gy g cos 30
T
g
401 40
2  15  3 T =
  1.8 Sec 3 2 3g
10  5 g
2
9
Minimum Velocity   5m/ s
1.8  4/ 3

66. D Section F - Relative Motion


u2 sin 2 71. B
R 
g Total Length of 2 trains = 50 + 50 = 100
Velocity V1 = 10
 R2 > R1 V2 = 15
So, u2 > u1 V1 + V2 = 25
100
time   4 sec
25

www.motion.ac.in
44 - Solution Physics (XI) Topic

72. A 75. A
Let they meet on height h from the top.
N and it take time t.
80 Km/h
A

(i) W E
h
B
60 Km/h 50 m
S
50–h
(ii) Here VA & VB are '' to each other.
VAB  VA  VB
N R
60 Km/h 80 Km/h 1 2
A h   gt
B 2
45° h = 5t2 ....(i)
W E
(iii) 1 2
50  h  25t 
.gt
2
S 50 – h = 25 t – 5t2
60 3 from equation (1) –
tan = = 50 – 5t2 = 25 t – 5 t2
80 4
= 37° 50 = 25 t
t = 2 sec.
(iv) 37+=450
tan 37  tan  76. C
1
3
1  tan 
4
3 3
 tan   1  tan 
4 4
30 m/sec
7 1
tan  
4 4 80 m
VB  60km / hr
80 8
t  sec
1 30 3
tan  
7 4
Halftime = sec
VAB  VA  VB 3


73. C t1    t1 g
2
g 2
VAB  10  5  5m / s 10m/s 5m/s
A B 4 40
100   10  m/sec
t  20 sec 100m 3 3
5
77. D
74. C
Let they meet at S distance from turning point.
20 25

S S  250
20 S + 20 × 250 = 25 S
Parabolic path
20  250
S
5
S  1000 m
S = 1 km

394-Rajeev Gandhi Nagar, Kota


Kinematics - 45

78. C 82. B
Q

3 m/s 
v12 vb = 10 m/s
monkey

4 m/s dog P vs = 4 m/s


v
   sin   s
v12  v1  v2 vb
  4 
v12  v12  v22  16  9  5   sin1  
 10 
2
79. B   sin1   with PQ up the stream
5

83. C
15 m/s
given, Vbw = nVw
Vbw = velocity boat w.r.t. water
Vw = velocity of water
Vw
8 m/s sin  
Vbw
2 2
velocity  8   15 1 vbw
sin   
= 17 m/s n
1 P vw
80. B   sin1  
|a| = |cos 37 + 2 cos 53| n

4 3  1
 2 Angle with stream =  sin1  
5 5 2 n
10
  2 m/sec2 84. A
5 vbr = vb-vr
given v r  v br  v b
Section G - River Boat Problems
Let swimmer speed in still water = x
Let velocity of water = y
81. B
x + y = 16 , x - y = 8
x = 12 and y = 4
40 km/h
85. A
0.5 km
w
10  60 
vn vm
w

30 km/h 120 1 vr
vr   m / sec
10  60 5
0.5
t w
40 12.5  60 
vm cos 
t = 0.125 × 60
t = 0.75 min cos  = 37° vm cos 
vm
vm sin  = vr 
3 1
vm  vm sin  vr
5 5
1
vm  m/sec
3

www.motion.ac.in
46 - Solution Physics (XI) Topic

86. C
v
v 10
v m
tan   m sin  
vr m 15
w v m cos 
vm 10 2
 sin   
15 3
vm sin  vr
 vm 2  2
   tan  
 vr 5 5
vm+vr
w
89. A
Both reach
w W
vm cos   ....(1) t
t vm cos 
vm sin   vr ....(2)
w
vm  vr  ....(3)
T w v m cos 
vm vm
by solving (i), (ii), (iii)  
vm t2  T 2
 2 vm sin 
vr t  T2
So statement are is correct
and statement (2) is correct and explanation.
87. C
60 90. C
V
6 vsw v
V = 10 m/s 60m
  
Vsw  Vw  V 45° 1 1km
 hr vm
2 2 vw=5 m/s 2
Vsw  5  10 

 25  100
vr
 125
 5 5 m/s

88. C
1 1  vm  vr  1  1
t t  2
10vr
vr 2 vm  vr
vr

vm
1 1  v  vr   2
  m
w vr 2 2  vm  vr 

vm 2  v  vr   2
1  m
vr
vr vm  vr

vm 
w 2  vr 

  vm  vr   2 
10 vr  vm  vr 
w
vm sin   2  2vr ; vr  1 km/h
15
w
tan  
10vr

394-Rajeev Gandhi Nagar, Kota


Kinematics - 47

Section H - Rain Problems, Aircraft wind 1 2


pr ob le ms , Re la ti ve M otion SC  vt  gt
2
between two projectiles
10 = 2vt
91. C 2

Vm  3î dAB   vt 2   sA  1 gt2 


 2 
VR  10 ĵ
2 2
VRM  VR  VM   vt    vt 

2
 10 ĵ  3 î  VRM  109 m/sec.  2  vt   2  vt 

dAB  5 2
92. A
vx = v, ax = 0 97. B
vy = 0
Without wind A reaches to C and with wind it
ay = g reaches to D in same time so wind must deflect
from C to D so wind blow in the direciton of CD
x=vt
VAG  VAW  VWG
Because horizontal velocity of plane and bomb
is always same.  VAG t  VAW t  VWG t

93. D AC  VAW t
N
CD  VWG t
vHT=vH-vT

-vT vT = 50 South - West 98. B


W E
V HT 2v0
S vH = 50 with respect to ground T 
g
94. D
   2v0 T
v r  v1  v 2 with respect to lift T '  
2g 2

vr  v12  v 2 2  2v1v 2 cos 
 99. A
vr max when cos   1 = Relative acceleration is zero, between any
 v r  v1  v 2 two particle.
So relative velocity cloase nat change with
95. D time in verticle direction.

vSB  5 m / sec  ˆj  100. A

aSB  15 m / sec2  ˆj 
v =  + at
v = –5 – 15 × 2 = –35 m/sec2
2

96. C 1

A
v

IA
v V1y  1  gt
1 2 B
gt c V2y  2  gt
2
v
vt V21y   2  1 

1 2
SA  vt  gt
2

www.motion.ac.in
48 - Solution Physics (XI) Topic

 
Section A - Distance, Displacement, Velocity V & a are perpendicular to each other so
and Acceleration, Equation of dot product = 0
Motion  
1. D V ·a  0
 2(2t – u) + ut = 0
2. A 8t = 8
V = u + at t = 1 sec.
V – u = 10 × 0.5
VB – VT = 5 ....(i) 6. A
V2 = u2 + 2as 
P1 : x(t) = 3.5 – 2.7 t3, a  t   ve
(V2 – u2) = 2 × 10 × 3 = 60 
P2 : x(t) = 3.5 + 2.7 t3, a  t   ve
VB2  VT2  60 
P3 : x(t) = 3.5 + 2.7 t2, a  t   ve
from equation (1) 
(VB – VT) (VB + VT) = 60 P4 : x(t) = 2.5 – 3.4t–2.7 t2, a  t   ve
P1, P2 & P3, P4  All speeds up. Same will
60 speed up in +ve direction, some in –ve
VB  VT   12
5 direction.

3. D
C Section B - Motion under Gravity

7. B
4 5
5

Stone is dropped
2

u1 u2 so time taken by stone to reach the bottom


of the wall t 1
A O
2m B 3m 1 2
h gt1
2
4 2 5 2h
t t  t1   i
u1 u1 g
u1 2 5 5 time taken by sound to comes from bottom
 
u2 4 4 h
to upper end t 2  ...(ii)
v
4. A 2h h
1 sec 1 sec  Total time  t1  t 2  g  v


1m 2m 8. A
distance Travelled by (first ball)
1  2
1  u1  a 1 ....(1) 1
2 S = ut + at2
2
1 4 1 2
3  u 2  a ....(2) = 5×2+  10  2
2 2
(1) × 4 – 2 = 30 m B
1=2u Relative Method t=2 sec
Velocity of first ball v1
1 after 2 sec.
u unit 30m
2 V = u + at
V = 5 + 10 × 2 = 25
25 m/s
5. A 30 A
t  2 sec
 v1  25
r   t2  4t  6  ˆi   t2  ˆj
 30  2v1  50  v1  40 m / s
V  2t  4  ˆi  2tjˆ

a  2iˆ  2j
ˆ

394-Rajeev Gandhi Nagar, Kota


Kinematics - 49

9. D 13. B
1
H = ut1 - gt 2 ....(i) u=0
2 1 H2
v = u + at
H
 t1  t2 
u=g   ....(ii) H1
 2  t2
u=20m/s
From (i) and (ii)
B B
g 1 Hmax H = H1 + H2
H= (t 2 + t1t2)- gt12 t1
2 1 2
1
H 1 2
H2  0  g 3  45 m
H = gt1t2 A 2
2
1 2
H1  20  3  g 3  15 m
10. C 2
u2 Height of Building = H1 + H2 = 45 + 15 = 60 m
 Hmax   u  2hg
2g
Given
Section C - Graphs
Hmax = 5 m
u 2gh 2H 14. C y
t   Point C
g g g P C
Average Vel. vector is B D
25 al ong the x-axis at
  1sec A
10 point 'c' instanteous
in 1 min = 60 Balls. vel. vector is along the x
x-axis.
11. B
Length of groove is L
15. B
(geff)AB =g
Area = = 0.4  0.2  0.4  0.2  0.4  0.2
2L 2L
t1   1
geff g   0.4  0.2  0.6  0.2
2
 t2 
2L
g sin30
(geff)CD = g sin30° Area = 0.4  a.dx  vdv 
 t1 : t 2  1 : 2 v - v = 2ax
f
2
i
2

then v 2f  v i2  2 Area
12. D
2
v 2f  0.8   0.8 
T  3T 
Distance travelled in   to   Vf = 1.2 m/s
2  4 

1 T
2
gT2 T
16. B
d1  g   x  tt  2T 
x Parabola
2 4 32 2
T
distance travelling 4 x  t 2  2Tt
3T
T dx
to T second 2  2t  2T
2 dt
T O T 2T t
2 2
d2 x
1 T gT t=0 and  2
d2  g   dt 2
2 2 8
v
d1 8 1 T 2T
  0
d2 32 4 t
d2 = 4d1

www.motion.ac.in
50 - Solution Physics (XI) Topic

17. D 23. A
a
10
4
 3
6 8 t
O 2 4 x= 2 
3
t
–20
-2m/s2
Total Distance = Upper area + Lower area
1  2 1
18. C  33.3    2    20   2  20
2  3 2
Speed  33 .3  33.3  66.6 m

24. C
v - t Displacement is zero
O x
T 2T t
(m)
19. B
20  10 30
V = u + at, a=  0 2 4 6 8 t(s)
64 2
30 2 25. C
0  10  t t=  0.66 sec .
2 3
Particle comes to rest when v= 0 12  v
Area =  m
on observing graphs V=0 at t = 0, 4.66 sec, 2 2 1m/s
8 sec  
Av velocity =  m/ s 0 t
Incorrect t = 5 sec 22 4 2sec

20. C 26. D
Rate of change of velocity is maximum Slope of v-t curve gives aceleration
t = 4 to 6 sec Here slope of P1 > slope of P2 (ap1  a p2 )
–20 – 10 –30 Relative velocity in their motion continously
a= = = – 15 m/sec2
6–4 2 increases.

21. A Section D - Variable Acceleration


(Area of vt graph gives the displacement. 27. B
Here 0 to 2 sec.
dx
Distance = Displacement x  5 sin 10t  v x   50 cos 10t
dt
1
Area in 0 to 2 sec. = 2×10 = 10 dy
2 y  5 cos 10t  v y   50 sin 10 t
Position = -15 + 10 = -5 m dt
2 2
V 2net  Vx  v y
22. A
v net  (50) 2 (sin 2 10t  cos 2 10t ) = 50 m/sec
t = 4 to 6 sec
t = 4 to 6 sec
–20 – 10 –30 28. D
a= = = – 15 m/sec2 v = lnx m/s (Given)
6–4 2
vdv 1
v (m/s) a=  ln x
dx x
 Fnet = 0
a=0  x=1m
10

2 4 4.66
t 29. C
2 2 0.66=2/3 
F  2sin 3tiˆ  3cos3tjˆ
Maximum Displacement dv
a=  2 sin 3 tiˆ  3 cos3 tjˆ
1 1 2 dt
  2  10  2  10    10 v t
2 2 3 t

 dv  2 sin3t dt ˆi  3 cos3t.dtjˆ
 
 33.3 m o
0 0

394-Rajeev Gandhi Nagar, Kota


Kinematics - 51

2 3 u dt
v cos3 t  ot ˆi   sin3 t  ot ˆj  dx 
3 3  1  aut
r t
 2 1 
 dx   cos3t  1 ˆi  sin3tjˆ  . dt
 s t
u dt
 3  
o o integrating between proper limits  dx   1  aut
 2 
t t
 1
t
0 0
r –   cos 3tdt   dt  ˆi   sin3t ˆj dt
3  0  0
0 1
2 t 2 1 t  S ln (1  aut )
  sin3t  o ˆi 
2 
tiˆ  2 cos 3t  o ˆj a
(3 ) 3 3
For t = 1 sec
 2 ˆ 2 33. A
r  i  2 ˆj
3 3 u = 10 m/s, S = 10.5 m
a = – tan 45° , a = – 1 m/s2
30. B 1 2
S  ut  at
F  Be  ct 2
v t –10.5 = 10t – 0.5 t2
B ct B ct t2 – 20t – 21 = 0
a
m
e  dv 
0 o
m 
e dt 
20  400  4  21
B  –ct B t
v e  1  At t   v 2
mc   mc
20  22
t  21 sec
31. B 2
v  t2  t = t (t-1)
dv 34. D
 a  2t  1 
dt  r
Motion is consider as Retards Vavg 
when V & a are in opposite Direction t
 
Case - 1 if r  0 Vavg  0
If v > 0 then a < 0
But t2 – t > 0, t > 1  uV
Vavg 
and a > 0 for t > 1 2
so not Possible
Case - 2 35. B
v < 0, a > 0 V = t2 – 6t + 8
t2 – t < 0, 2t – 1 > 0
dV
1  2t  6  0
t  (0,1) , t > dt
2
t = 3 sec.
1 Speed V = (3)2 – 6(3) + 8 = 0
 t 1
2 at t = 3 sec
Also at t = 2 sec,
V = (2)2 – 6(2) + 8 = 0
S–1 is True, S–2 is True, S–2 is Not a correct
32. C explanation for statement–1

dv 36. A
The retardation is given by   av 2 a = 2 sin (t)
dt
dV
 2 sin  t 
integrating between proper limits dt
v t
dv 1 1 v t
   v 2   a dt or  at   dV   2 sin t .dt
u 0 v 4 0 0
dt 1
  at  2
t
dx u V cos t
 0

www.motion.ac.in
52 - Solution Physics (XI) Topic

42. D
2
V   cos t  1 1
 H  40t   10  t 2
2
40 50
dX 2 5t 2  40 t  H  0
  cos t  1 53º
dt  40
Now, t1  t 2   8 sec 30
x t 5
2 
 dx   cos t  1 .dt
0 0
 43. A
By Equation of trajectory
t
 2 2t  gx 2
x   2 sin  t    y  x tan   2
  0 2u cos2 
 = 53°
Put t = 0, t = 1
2 4x 10 x 2
x  y 
Distance 3 1800

180y  240x  x 2
37. B
44. B
2 2t 
Distance travelled x  sin  t  
2  V  aiˆ  b  ct  ˆj  ux ˆi  uy  gt ˆj
 
2uxuy 2ab
38. B R 
Displacement in t = 0 to t = t g c
2 2t
S 2
sin  t   45. C
 
U2y
39. A h
2g
s v   s U y  2gh
v=0
Touches at
T U. 2gh highest point
R  Ux T 
s s g
Vav  aav  h hmax
T T
2h
R U
 s  s g
  
T T 
=1 46. C
Because horizontal
Section E - Ground-to Ground projectile component of the vel.
Motion, Equation of Trajectory, is never change in
Projectile from tower, Projectile projectile motion.
Motion in Inclined plane Horizontal Component
ucos   v cos 
40. D  v  ucos  sec 
By theory

41. A 47. B
uy = 50 sin 53° = 40 m/s
/s
50 m 1
50 2
sy= uyt- a y t ux  50cos53º = 30m/s
40
2
53º
53º Here Sy = 75, uy = 40, ay = 10
30
1
75  40.t   10  t 2
2
 t 2  8t  15  0
 t 2  5t  3t  15  0 , t1 = 3sec, t2 = 5sec

394-Rajeev Gandhi Nagar, Kota


Kinematics - 53

x = uxt 52. B
x 2  30  5  150m As   , H and T both increases
x1  30  3  90m
But R  from 0º to 45º & at  = 45° Max then
 x 2  x1  150  90  60m
decreases
48. A Ans (B) R  then  [  from from 30º to 60º]
In t = 2 sec , x = uxt ( ux = 30)
x  30  2  60m while H  and T  .
1 2
y  40  2   10  2
2
 80  20  60m
Section F - Relative Motion
Distance = x2  y2
Distance  60 2m 53. B
H H
VE  m / s and VM  m/s
49. C 60 180
2uy H H 180
T t  
gcos  VE  VM H H 4

60 180
15
10 3

25 3 30º  t  45 sec


T
10  cos 30º 60º
5
3 30º g
T  2 sec 54. B
Initial vel. = u
50. C & Final vel. = -u
a
a = acceleration of lift
ay = –g cos  u
u = velocity relative to
ax = g sin  lift According to problem
uy = v – u = u – (g + a) × t u
ux = 0
2u
1 2 t
Range = a x T ga
2
2.v  at  gt  2u
T
g cos  2u  gt
2 a 
1  2v  t
 g sin   
2  g cos  
v2 55. C
R 2 tan  sec 
g a
51. D 2
g
2 
v a
90º p g

1 2
R  ux t  ax t straight line path
2

Q
56. C
 ux  0. uy  v  V1 V2
 
2uy 2v
 T   16 8
gcos  gcos  V1  V2   m/sec ....(1)
10 5
1
R= g sin T2 3
2 V1  V2  ....(2)
2
5
1  2V  (1) + (2)
R= g sin  
2  g cos   11 11 3
R = Tv tan 2V1   V2 
5 10 5

www.motion.ac.in
54 - Solution Physics (XI) Topic

11 11 6 5 1 Section G - River Boat Problems


V1  V2    
10 10 10 10 2
60. B
11 1 given : Vbr= 5 km/hr
V1  , V2 
10 2 vr=4 km/hr
4km/hr
d = 0.2 km 5km/hr
57. B 37º
d=0.2km
10m/sec 4km/hr
B C 0.2
(8m) t
4 3km/hr

(8m) (8m) = 0.05 hr = 3 min


2m/sec BC = 1 Km/hr × 0.05 hr.
B C
0.05
A D t2   60
(8m) 3 4km/hr
x = 1 min
t
2 t1 + t2 = 4 min A 1km/hr
y
t
10
61. B
x y vr=5m/s

2 10
5x = y
If x = 6 m ; y = 30 m (Both will be at same
vmr=vm-vr 60m vm
place)
x 6
t   3 sec vr
2 2 vmr
60
vm   12m / s
5
58. B
Let velocity of the aeroplane be 
vr =5m/s
v P  u cos 30î  u sin 30 ĵ and velocity of the
wind be v, then vmr= 122  5 2  13m / s

3 ˆ u  ˆ 62. B
u t i   t  5t 2  ˆj  vtk
2  2 
B 0.05 km
 400 3iˆ  80j ˆ
ˆ  200k vbr
vbr
4

3 u
 u t  400 3, t  5t 2  80, vt  200 37º
2 2
u 2 3 A vr=4 km/hr
ut  800 and t  5t  80
2
 400  5t 2  80 Vbr = 5 km/hr
v
sin  = r
 t 2  64  t  8 sec . 5
d 4
t=  cos      37º
59. A v br cos  5
Each porticle move v
perpendicular with the neighbour particle so sin 37º = r  v r  3km / hr
5
no component of v along the line of motion
of neighbour
a
velocity so vel. of approach = v  t 
v

394-Rajeev Gandhi Nagar, Kota


Kinematics - 55

63. B l l 3l  2l
2Vm   
8 12 24
l
B C
Vm 5

l 48
l 48
w
t   9.6
Vm Vm
Vm 5

 t = 9.6 hr

A 66. C
l
l = 120 m Vm  Vr 
Case (1) t1

W l
t Vm  Vr 
  10  60 t2
 Vm 
t 1 + t2 = 4
120 1
Vr   m/sec l
10  60 5 20  ....(i)
t1
Case (2)
Vm sin     Vr l
10  ....(ii)
t2
W
Vm cos  
12.5  60 Eq. (1)
Eq. (2)
10  60 100 4
cos    
12.5  60 125 5 20 t2

 = 37° 10 t1
3 1 1  t2 = 2t1
Vm   , W  10  60 3t1 = 4
5 5 3
4 8
 1 t1 = hr t2 = hr
 Vm   , W = 200 m 3 3
 3
l
20 
64. B 4 /3

x 4 80
Vm  Vr  Vm = nVr l  20   km
t 3 3

y 67. A
Vm  Vr 
t
l
Vm  Vr x
 m
Vm  Vr y

n 1 m 300 m
 3m/sec
n1 1 5m/sec

m> 1
37°
4m/sec 2m/sec
65. A
l d 300
Vm  Vr  t   100 sec
8 v 3
l l = vt = 2 × 100
Vm  Vr  l = 200 m
12

www.motion.ac.in
56 - Solution Physics (XI) Topic

Section H - Rain Problems, Aircraft wind 72. C


pr ob le ms , Re la ti ve M otion
N V2 N
between two projectiles v s2 s1
s2 v s2
30º 30º
Question No. 68 to 71 (4 questions) 10km/hr
W E W E
s1 v s1
68. B
a AB  0 S S
 Straight line
East component of both ship must be same.
from fig :
69. C
VS2 sin 30º = vs1
Given v1 cos 1  v 2 cos 2  v xA  v xB
 
v A  v XA ˆi  v yA ˆj ; v B  v XB ˆi  v yB ˆj VS2 = 10 = 20 km/hr
 /2
 v AB  v yA ˆj  v yB ˆj
73. A
Drops of rain move parallel to the walls if vrp
70. D makes angle with the horizontal.
2uy   
T  Same VRC  VR  VC -v
 
1
g 
 VR ĵ  VC î
uy 2 v re v xr v2
H v2 6
2g tan   v  2
 1 v2 v1
VAB  VxA î  VxB î  = tan -1(3)

71. B
2ux uy
u2 sin 2 74. B
R 
g g
when  and 90   range is same
75. B
v  v ˆi  v ˆj  v ˆi  v ˆj y
21 2x 2y 1x 1y
V u2 sin 2 2u2 sin  cos 
 v 2y  v1y  v 2y  v1y V R  
tan     g g
 v 2x  v1x  v 2x  v1x
  1
tan = –ve
2
x u2 sin2  u2 cos2 
h1  , h2 
2g 2g

u2 sin2   u2 cos2 
R2  4  4
2g  2g
R2 = 16 h1 h2
R  4 h1h2

394-Rajeev Gandhi Nagar, Kota


Kinematics - 57

Section A - Distance, Displacement, Velocity Section B - Motion under Gravity


and Acceleration, Equation of Motion
1. A 5. A,D A
d/3, t1 d/3,t2 d/3,t3 
AB=l=2Rcos  R
B
d a=g cos  R

d/ 3 d v2=2×g cos  ×2Rcos


Now t1  v 
6 1
1 v cos and 2R cos = g cos t2
d/3 d d/3 d 2
t2    t 3  
3 9 6 18 4R
t2 
g
d 18
Average Velocity    3 m/s
d d d 6
  6. A,C
6 9 18
(A) At the top of the motion v = 0 but a = –
g.
2. A,B,C,D
X=T2–t3 v=0
a=-g
 dv
(A) 0= t2–t3  t  (A, B) a 
 dt
dx 2 O
(B) v=  2t  3 t 2  v  0  t  (C) If particle is moving with costant
dt 3
velocity
d2 x (D) No
(C) a=  2  6 t
dt 2
when t=0  a=2 ; v=0 7. A,C
 
 | V |, V 
(D) Acceleration at t = ; a=0 
3  dV
a
 net force = 0 dt
In circular motion speed may be constant
3. A,C but velocity will not be constant and particle
t=2 have some acceleration.
v=10-5t v=0
When v = 0 at t = 2 sec. Section C - Graphs
2
5t
Max displacement = 10t 
2 8. A,B,C,D
put t=2  20-10=10m
Distance traveled in first 3 seconds 9. A,B,C
 1 2
|Displacement|  Distance. A particle revolving in
= 10+  0   5  1   = 12.5 m
 2  a circle has zero average velocity everytime it
reaches the starting point.
4. B,C,D
Average speed of a particle in a given time is never
dv
(B)  a  less than the magnitude of the average velocity.
dt
v dv
(C) Object is slowing down. It is possible to have a situation in which 0
a dt

V d
(D) origin but v  0.
dt
the particle is moving towards origin. The average velocity of a particle is zero in a time

www.motion.ac.in
58 - Solution Physics (XI) Topic

interval. It is possible that the instantaneoous 2 1


velocity is never zero in the interval.
(A) Hmax =
u sin22  20  
4  0.25m

2g 2  10
10. (A), (C), (D)
(B) Minimum Velocity ucos 
dx
v x   x
dt  20  cos 30º
dx
  dt  2 x  t  C 3
x1/ 2  20   15m / s
2
but given at t = 0; x = 4  c = 4 2u sin 
( t  4) 2 (6) 2 36 (C) T
x x   9 m (Putting g
4 4 4 1
t = 2 sec.) = 2  20  2
10
dv 1 1 1
av  x  m/s2  sec
dx 2 x 2 5
1
Section E - Ground-to Ground projectile (D) mgh = 1 × 10 × = 2.5 Joule
4
Motion, Equation of Trajectory, Projectile
from tower, Projectile Motion in Inclined 13. A,B,C,D
plane
u2
h=  u  2gh
2g
11. (A), (B)
u2
(a) Rmax =  2h
dv g
(i) V   V (ii) a =  V (b) R= nHmax
dx
u2 sin 2 u2 sin2 
dv n
dv =   dx   V g 2g
dt 4 = n tan 
0 x
4
 dv    dx  =tan-1  n 
v0 0  
v t
dv 2 4u sin2  2  u2 sin 20
2
gT  g    tan 
v v   0 dt (c)
g2 g
0
gT2 = 2 R tan
2uy uy 2
 v 0   x (d) T  ,Hmax 
g 2g
 v   Ratio 1:1
ln     t
 V0  14. A,B
v Put the value of T, R, H, in the given equation
x 0 V  V0 e t
 and solve each option.

V0 15. A,B,C,D
V at t   V  0
et gx 2
y=x tan  
 A&B are correct answer 2u2 cos2 
Given y= ax – bx2
12. C,D g
on comparing tan   a b
u2 sin 2 2u2 cos 2 
R=
g  1  tan2   sec 2   1  a2
2
20 sin  3 1/2
3  sin2  g  g 
10 2 b=
2u2  
1  a2  u  
 2b
1  a2 

 
   30 º
394-Rajeev Gandhi Nagar, Kota
Kinematics - 59

1 18. A,B,C
 ux  u cosθ  u.
2
1 a O
vav  0
and   tan 1
a 2u
g
16. A,C,D
2h
2H 2H Sav 
T  0.4  2u u  2gh
g g
g
 H  0 .8m
R= 0.4 × 4 = 1.6m
and Uy= 2gH  2  10  0.8  4m / s
  45º

17. A,B,C,D u
u
2u sin 
(A) T
g
u2 u
2  10  sin  sav   
T   1 sec 2u  2 
10 g
g
2  10 3  3
(B) T  3 sec
2  10 Section F - Relative Motion

19. B,C,D
V = u + at
60° Vcos30°  –4 = 2 + a × 4
3 30°
10
(C) 5
V 3
Vsin30° a = m/s2
60 2
5 3
4 m/s 2 m/s
V cos 30  5 3
V = 10 Belt
1 4 m/s
Vy  v sin30  10 
2
1 3 2
=5 (B) Now s = 2×4    4 
2 2
v = u + gt
–5 = 5 – 10 t =8 – 12 = –4m (w.r.t. ground)
t = 1 sec w.r.t. Belt
(C) Relative velocity ui = 6m/s and v=0
1 3 2
s= 6×4    4 
2 2
(D) = 24-12=12 m
(D) Displacement w.rt. ground is zero
30° 30°
30° 1 3 2
0 = 2×t   t
2 2
Max angle between intial velocity vector and 8
velocity is 60°. t= sec
3

www.motion.ac.in
60 - Solution Physics (XI) Topic

20. A,C,D
E 25 5
D VB  Vr  VBx   m/sec
120 24
2a

a 3
F 3 C
a 10 1
VBy  = m/sec
30º 30º 120 12
A a B

(A) A to F 23. B,C,D


Total Desplaceme nt Theory
Average velocity =
Total time
a v
 
5a / v 5 d V
2a 2
(B) A to D =  v
3a / v 3

a 3 v 3
(C) A to C  
2a / v 2
V
a tmin 
(D) A to B  v d
a/v

21. C,D
From theory. d
V
V cos 

u
V sin 
Section G - River Boat Problems
u
sin  
22. B,D V
d 30 1 V sin  = m
Vr    m/sec
t 120 4 l V cos 

t d
P
30m d d
25 t 
V cos  V  u2
2

 d

t  2
 2
10m  V u 
VB

Vr
Section H - Rain Problems, Aircraft wind
A VB B
problems, Relative Motion between two
5m projectiles
Question No. 46 to 49 (4 questions)

d 10 1 24. A,C
VA    m/sec
t 120 2
2u sin 
T
   1 1 g
VAr  VA  VB  ˆj  ˆi
2 4
2  24
T   4.8 sec
1 1 5 5 10
VAr    
4 16 16 4 Vsin = 24 – 10 × 2 = 4
For (B) V cos  = 32
1
Tan  , V  32.25 m/sec
8

394-Rajeev Gandhi Nagar, Kota


Kinematics - 61

25. C,D
26. B,D
Particle will collide
y Because
Vy1  u sin  ; Vy2  u sin 

Vy1  Vy2
2 3
1
2usin
and time of collision 
T1 T2 T3
gcos

x
R
1.5R
2R

Vy1  Vy2  Vy2


T1 = T2 = T3
T1  T3
 T2 
2

www.motion.ac.in
62 - Solution Physics (XI) Topic

Section A - Distance, Displacement, Velocity 4. V = max


and Acceleration, Equation of When a = 0
Motion dv
1. x = t2 - 4  a  2bt  0
dt
y=t-4 a
t
t=y+4 2b
x = (y+4)2 - 4 a2 a2 a2
v  
= (y+4-2) (y+6) 2b 4b 4b
x = y2+8y+12
Section B - Motion under Gravity
(b) crosses x axis  y = 0
A ua= 0
t = 4 sec 5. Max height of B
crossess y axis  x = 0 u2
 4h  uB  8gh
2g
t =  2 sec h
Relative velocity
VAB = 0  8gh   8gh
2. (a)
B ub 
dr h h
velocity =  (2t  1)î  3 ĵ  (6t 2  8t)k̂ t= 
dt 8gh 8g
= 5î  3 ĵ  8k̂
(b) 6. 50 m
dv Given a = 0.2 m/s
acceleration = aeff = g +a
dt
 2 î  (12t  8)k̂ =  2 î  16k̂  7 2  3 2 
1
(c) S = ×10×  2    2   = 50 m
2     

Speed = V  25  9  64  98  7 2
Section C - Graphs
(d)
 7.
a  4  256  260  2 65
V
(a) tan 30 =
1
d2r
3. a(t)   6tĵ  cos tk̂ 1
dt2 V m / s.
3

r  6t î  t 3 ĵ  cos tk̂

dr
 6 î  3t 2 ĵ  sin tk̂ V
dt V
(b) tan 60 = = 1
F(t)  ma(t)   18tĵ  3 cos tk̂ t
2
 3
a   3ĵ  0  3  V 
2 2
v(t)  6 î  3t 2 ĵ  sin tk̂ V
average acc = = 3 /2
t
v()  6 î  32 ĵ

speed = 36  94 = 3 4  4

394-Rajeev Gandhi Nagar, Kota


Kinematics - 63

 dv x î d2 y ĵ
a 
dt dt 2

v   30 î  40 ĵ

(c) (c) a   16 î  8 ĵ

Section E - Ground-to Ground projectile


Motion, Equation of Trajectory,
1 a (m/s2)
0  1 a  Projectile from tower, Projectile
3 3 Motion in Inclined plane
1
1  2a  0 3

t (s) 11. At t= 2 sec    45 º


2  2.5  3 1 2 2.5
 vy = vx
uy
8. V = u + at for t = 4 sec, uy = 0  T = u = 40 m/s
V g  y
V = 0 + 5t
At t= 2 sec vy = 40 – 20 = 20
v max
tan   5 5t  vy = vx= 20
t
v max  5t v= 202  402  20 5
T
Displacement t (25-t) 25

1 1
=  t  5t  25  2t   5t   t  5t 12. ux=50 3 m/s
2 2
= 125t – 5t2 uy =50 m/s
2
125t  5t Vx =ux=50 3
Average velocity 20 =
25
Vy = uy–gt= 50–10t y
125 t – 5t2 = 500 u
t = 20, 5 50  10t
tan  60   u=100 90º
50 60º
t  20 50 3 60º
30º
x
50 3
t = 20 sec
Section D - Variable Acceleration

gx 2
9. a = 3t2 - 4t +1 13. y= 3x 
2
v = t3-2t2+t+C
C=0 gx 2
y = x tan  
t 4 2t 3 t 2 2u cos2 
2
x   C
4 3 2
C=0 on comparing tan   3  = 60°

at t = 2 sec. and u2 cos2 = 1


2 2 u  2m / s
x=4- (8) + 2 =
3 3

1
14. s = 100×3+ ×30×32 = 435 m
2
10. y=0,t=5 V = 100 + 30 × 3 = 190 m/s
Vy = -8t
 Hmax =
190 2  sin 2 53º = 1155.2
at t = 5 2  10
vy = -40 A Total = 348 + Hmax
vx = -30 = 348 + 1155.2
www.motion.ac.in
64 - Solution Physics (XI) Topic

= 1503.2 m = maximum altitude


B u=190 sin 53º; – ay =10 : sy = – 348 Y B
16.
after t = 3 sec. uy = 152 m/s

u=152

A 37º
53° X

Vb  10 î  12 ĵ  Vw  u î
     
v bw  v b  v w , v b  v bw  v w

1  10iˆ  12ˆj  uiˆ


–348 = 152t   10  t 2
2
 10  u ˆi  12ˆj
5t2 – 152t – 348=0, t = 32.54 sec
12 3 12
Total = 35.54 sec tan 37º   
10  u 4 10  u
R = 435cos 53º + 190 cos 53 × 32.54 10  u  16 , u  6m / s
3 2
= 435 × + 190 × × 32.54 = 3970.56m
5 5
Section G - River Boat Problems

Section F - Relative Motion 200


17. t  50 sec
4
/s 
0.
2m Vy = 4 sin  100m
A Vx = 4 cos  - 2 200m 
3m
tan = 2 50m
0 60º 4m/s
4 sin 
15. 2
4 cos   2 2m/s
4m

0. B
2m 4 cos - 2 = 2 sin
/s
2 cos - 1 = sin
Now, we solve the problem w.r.t. B then
4 cos2 + 1 - 4 cos = 1-cos2
0.1m/s /s A
0. m 5 cos2 = 4 cos   = 37°
2m VBY
0.2
/s = 1
v
A

vB = tan  
0.2
3m m/ 2
0.2 3/2 30º s
60º vb 
0.2 3/2 30º vAB=0.2m/s 4 sin  = 2 sin (90+  )

3m 4 m/s vr
/s 60º 4 sin  = 2 cos 
.2m
0 VAY 60º
0.1m/s C B 2 cos  2 2 1
shortest
sin  =   
distance 4 4 5 5
shortest Distance BC = 1 sin 60º
100 100

3
m  50 3cm t = 4 cos       4  cos  cos   sin  sin  
2

100 125
t= 
3 3
4
5

394-Rajeev Gandhi Nagar, Kota


Kinematics - 65

Section H - Rain Problems, Aircraft wind


d 120m
18. tmin= v =10×60 sec pr ob le ms , Re la ti ve M otion
br between two projectiles
tmin = 600 sec v br
120 =vr×600 19. v rw  20ˆj
1 Vm  5î
vr= m/s
5
Vw  15 î
  
v rw  v r  v w
  
v r  vrw  v w  20 ĵ  15 î
vr 1
sin      
vbr v br  5  v rm  vr  v m  20ˆj  15iˆ  5iˆ  10iˆ  20ˆj

10  1
tan      tan 1 
d 20 2
12.5×60 = v cos 
br
 d 
  10  60
 v br  2

6
10  60 1
12.5×60=
cos 
4 20.
cos      37º W
5 in
d
sc
re
3 1 en
Now, 5  v  5
br
  
1 10  60 v rc  v r  v c
v br  m/s 12.5×60 =
3 cos  6
tan      tan 13 
4 2
cos      37º
5
3 1
Now, 5  v  5
br

1
v br  m/s
3

www.motion.ac.in
66 - Solution Physics (XI) Topic

Section A - Distance, Displacement, Velocity 1


and Acceleration, Equation of srel = urelt  arelt2
2
Motion
1. 1 1  3g  2
srel = 0    8 
Bill board t = 2sec t = 14 sec
2 2 
t=0
(A falls off 8 sec)
2U 12 U Ut srel = sA/ballon = –48g = –480m
1 150  5 1 
    122
150  5
t
For B urel = 0
2  18 12  18
3g
1500 m arel = 
2
14 U + Ut = 1500m ...(1)
1
srel = urelt  arelt2
1 150  5 1 150  5 2
   12 2   t  1500 m
2 18 12 18
1 3g 2
 t  30 sec  srel = sB/ballon = 0 
2 2
 6
U(30 + 14) = 1500 m  U  122.7 km / hr sB/ballon = –27g = –270m
Separation distance between 'A' and 'B'
2. vi = 54km/hr = 15 m/s = |sA/ballon| – |sB/ballon|
vf=0  0=15-0.3t  t = 50 sec = 480 – 270 = 210 m
Distance travelled by the locomotive
4. Let us choose the x and y directions along OB and
1 2
s=ut-  at OA respectively. Then,
2
ux = u = 10 3 m/s, uy = 0
1 ax = – g sin 60° = – 5 3 m/s2
s = 15(50)  (0.3) (50)2 =375 m
2
and ay = – g cos 60° = – 5 m/s2
Position of the locomotive = 400-375 = 25m
(a) At point Q, x-component of velocity is zero.
Hence, substituting in
vx= ux + axt
Section B - Motion under Gravity
10 3
3. 210 m 0 = 10 3 – 5 3 t  t= = 2s Ans.
5 3
Since particles are being dropped from a
moving body i.e., a moving reference frame, (b) At point Q, v = vy = uy + ayt
we used reference frame of balloon itself for  v = 0 – (5) (2) = –10 m/s Ans.
both the particles.
Motions of particles 'A' and 'B' are w.r.t. ballon, Here, negative sign implies that velocity of particle
so ballon is reference point therefore it is at Q is along negative y direction.
assumed to be at rest.
(c) Distance PO = |displacement of particle along y-
direction| = |sy|
1 1
Here, sy = uyt + a y t 2 = 0 – (5)(2) 2 = – 10 m
2 2
 PO = 10 m
 1
Therefore, h = POsin30° = (10)  2  or h = 5m Ans.

(d) Distance OQ = displaement of particle along


We denote ballon by b x-direction = sx
For A urel = 0
1 1
3g Here, sx = uxt + a x t 2 = (10 3 )(2) – (5 3 )(2)2
arel =  2 2
2 = 10 3 m or OQ = 10 3 m

394-Rajeev Gandhi Nagar, Kota


Kinematics - 67

PQ = (PO) 2  (OQ)2 = (10) 2  (10 3 ) 2 d  dx 


 sec2    sin x  
= 100  300  400 dt  dt 
PQ = 20 m d  dx 
   cos2 sin x  
dt  dt 
Section C - Graphs
5. Area of V–T curve give displacement. dx
Now, at x = /2,  = 0°, u
Distance travelled by the particle dt
= 50 + 50
d
= 100 m   u
Av. velocity = zero dt
Putting this in (1), we get
6. Particle return to 
 a  u2
starting point
it means displacement = 0
 upper area = Lower area
Section E - Ground-to Ground projectile
Motion, Equation of Trajectory,
v Projectile from tower, Projectile
20 Motion in Inclined plane
9. Bullets will spread in a area of radius equal to the
10 range of bullets. Therefore for area to be maximum.
 (t-25) t 4(t-25)
0  t v2 v4
10 20 25
Range should be maximum. i.e.  A 2
g g
x
 
x 10. v 2  u2  2 a . S y
20  x  4t  25
tan  =  tan  =
5 t  25  2 2 2gH
v  u 2  2gH  v 2  u 2  –
1 1 1 5 2
Now,  20  20   5  20   t  25   4  t  25 
2 2 2
On solving t = 36.2 sec 3 2 (U sin )2 6
v  g  U sin   v
5 2g 5
2
Section D - Variable Acceleration v 5
v

7. 1.5 m/s 
H
dv u H/2
= – v2 + 2v – 1
dt 
Terminal velocity is attained when a = 0
2
dv U cos   v
 =0 5
dt
the speed is 1.5 m/s when acceleration is tan   3    60º
one fourth of its initial value
2 2
8. u2 v cos  v    cos 1
5 5
Say at any instant, the velocity makes an
angle  with the x-axis.
 10 t
 
u  u cos ˆi  sin ˆj  11. tan 53 º 
U
 4 10t
 dv  d ˆ d ˆ  
 a  u   sin  i  cos  j ....(i) ....(1)
dt  dt dt  3 U
Ut
dy tan 53º  .....(2)
Now, tan    cos x 1 .7  5 t 2
dx

www.motion.ac.in
68 - Solution Physics (XI) Topic

also R – B cos = 0 for shortest path ...(iii)


U.ms–1
Thus, sin = 1/3 from eqn. (i) and (ii)
5t2 or R = B cos
Ut
U.ms–1 d 2 2d
1.7m 53º 1  1 / 9  
T0 3T0
1.7-5t2
10t ms–1 Case-II : If B < R
37º  d 
x     R  B cos  

 B sin  
2
from (1) & (2) : t  sec d
5 x  R cos ec  B cot 
3 2 B
from (1) : U   10 
4 5
U = 3 ms–1 dx
For min. x 0
d
or B(–cosec2) + Rcosec cot =0
5 2 cos = B/R
45º 45º Time taken in this case is given by
12. 45º
R d
3T0 
45º 45º B R2  B2

2  5 2 sin 45º also B  d / T0


T  1sec
g
3 d
On solving, we get R 
2 T0
Section F - Relative Motion

y axis
–1
Section H - Rain Problems, Aircraft wind
5 ms
13. pr ob le ms , Re la ti ve M otion
x
–1
between two projectiles
10 ms
30º 
1
ms
5 3 
10 3 ms  2 / 2 V RC
20 ms–2

VR
30º 90  
15. 2  /2

 VC
1 1
O  5T   10 3 T 2  T
2 3

Section G - River Boat Problems | VR | 2
tan  / 2       2 tan11/ 3
3 d |  VC | 6
14.
2 T0
D
Concept: Case (i) : If R < B, boat can
cross river along a path perpendicular to flow
Case (ii) : If B < R drift can not be zero
apply calculus in this case.
Case-I : If R < B
d
Shortest Path :  3T0 ...(i)
B sin 

d
Quickest Path :  T0 ...(ii)
B

394-Rajeev Gandhi Nagar, Kota


Kinematics - 69

1. C Area of rectangle ABED gives distance travelled


Parachute bails out at height H from ground. in time t.
Velocity at A s2 = (ft1)t
v  2gh Distance travelled in time t2,
1 f
 2  9.8  50  980ms1 s3  (2t1 )2
Velocity at ground, v1 = 3ms-1 (given) 22
Acceleration = - 2ms-2 (given) Thus, s1 + s2 + s3 = 15s
 s  (ft1 )  ft12  15s
v2  v12
 H h or
22
h=50m  1 2
980 9 971 s  (ft1 )t  2s  15s  s  ft1 
   242.75 A  2 
4 4 H
v  2gh
 H  242.75  h or (ft1 )t  12s ...(ii)
From Eqs. (i) and (ii), we have
 242.75  50  293 m  Ground
12s (ft1 )t
 t
s 1 or t1 
2. D (ft1 )t1 6
2
 N y From Eq. (i), we get
v1   5î v 2
 x 1 1 t 1 2
v2   5î v2 s f(t1 )2  s f   ft
   2 2 6 72
 v  v2  v1  5jˆ  5iˆ
W E
  v1 v1
v  5 2 4. C
 Given, t = ax2+bx
v 5 2 1
    ms 2 S Differentiating w.r.t t
t 10 2 dt dx dx
For direction,  2ax b
dt dt dt
5 dx 1
tan  =   1 v 
5 dt (2ax  b)
1 Again diferentiating w.r.t. t
Average acceleration is ms2 towards north-
2 d2x  2a dx
west. 2
 2
.
dt (2ax  b) dt
3. A d2 x
 f
The velocity time graph for the given situation dt2
can be drawn as below. Magnitudes of slope of 1 2a 2a
 . or f 
OA = f (2ax  b)2 (2ax  b) (2ax  b)3
Y
 f  2av3
v(m/s)
A B
5. D
A Projectile can have same range if angle of
X
O t1
D
t
E
t2 C projection are complementary ie,  and (900-)
t(S) Thus, in both cases
f 2u sin 
and slope of BC  t1  ...(i)
2 g
f
v  ft1  t2 2u sin(900  ) 2u cos 
2 t2   ...(ii)
 t2=2t1 g g
From Eqs. (i) and (ii)
In graph area of OAD gives distance
4u2 sin  cos 
1 2 t1t2 
s ft1 ....(i) g2
2

www.motion.ac.in
70 - Solution Physics (XI) Topic

2u2 sin 2 10. C


t1t2  u
g2 1 2
h gt (Parabolic)
2
2 u sin 2 2
 u
g g v= - gt and after the collision, v = gt (straight
2R line)
 t1t 2  Collision is perfectly elastic then ball reaches to
g
same height again and again with same velocity
 2
u sin 2  O y
 R  
 g  +v1
  h
or t1t2  R
t
t1 2t1 3t1
t
6. A
-v1
v x

dx dx
or  x or  dt 11. B
dt x
 
Perform integration u  3î  4 ĵ ; a  0.4 î  0.3 ĵ
  
x dx t
u  u  at
x 
00

  dt  3î  4 ĵ  ( 0.4 î  0.3 ĵ)10
[at t = 0,x=0 and let at any time t, particle is  3î  ĵ  4î  3 ĵ  7 î  7 ĵ
at x]
x Speed is 72  72  7 2 unit
x1 / 2
  t
1/2 0 12. D
 
or x1 / 2  t v  ky î  kxĵ
2
2 dx dy
or x  t2 or x  t2  ky,  kx
4 dt dt
dy dy dt kx
7. C   
dx dt dx ky
Kinetic energy at highest point,
ydy = xdx ; y2 = x2 +c
1
(KE)H  mv2 cos2 
2 13. A
Maximum range of water coming out of the foun-
0 2 K tain,
=K cos2  =K(cos 60 ) 
4
v2
Rm 
g
8. B
 Total area around fountain,
v = v0 + gt + ft2
dx v4
or  vo  gt  ft 2 A  R 2m  
dt g2
 dx  (vo  gt  ft2 )dt
x 1 14. A
So,  dx   (v o  gt  ft 2 )dt
0 0 dv
 2.5 v
g f dt
 x  vo  
2 3 dv 0 t
  2.5 dt  v 1 / 2dv   2.5 dt
9. B
v 
6 .25  0

t 1/2 0
at 2 (x1-x2)   2.5[t]  [2v 0 ]
6.25
 t 2s
Here, x2 = vt and x1 
2
 at2 
x1  x 2   vt  
 2 
So, the graph would t
O
be like
394-Rajeev Gandhi Nagar, Kota
Kinematics - 71

15. D 19. A
–H = ut – 1/2 g t2
sin2 
2
Hv ,   30 t' = u/g
g Given t = nt'
ux = v cos  2
angular momentum = muxh  nu   nu 
 –H = u  g  – 1/2 g  
   g  u
m(v cos ) v2 sin2 
g
nu2 n2u2
 –H = – 1/2
g g
3 mv3
16 g H
nu2 1
 –H = (1 – n)
g 2
16. D
 –2Hg = nu2 (2 – n)
Maximum speed with which the boy can throw
stone is  2Hg = nu2 (n – 2)

u  2gh  2  10  10  10 2m / s.
Range, is maximum when projectile is thrown at 20. C
an angle of 450 thus, Ist stone
2 2
u (10 2 ) 0  t  8 sec
R max    20 m
g 10 vr = 40 – 10
= 30 m/s
17. C ar = 0
As the force is exponentially decreasing, so it's
acceleration, i.e., rate of increase of velocity
t=2sec
will decrease with time. Thus, the graph of ve-
locity will be an increasing curve with decreas-
ing slope with time.
t=6sec
F F dv
a  o e bt 
m m dt
v t Fo Fo  1  bt t
bt  v  e sr = vr × t = 30 × 8 = 240 m
  dv   m e
0 0
dt
mb 0

(y2–y1)m t=8sec
Fo bt t Fo
 e  (eo  e bt ) 240
mb 0 mb
Fo
 (1  e bt ) ` t=1sec
mb
t(sec)
Fo 8
with vmax =
mb
8 sec < t  12 sec
18. D vr increases in magnitude and relative
acceleration is g downwards
u  22  12  5
 = tan-12 (y2–y1)m
240
gx2
y = x tan  -
2u cos2 
2

10x2
y = 2x – t(sec)
1 8 12
25
5
y = 2x – 5x2

www.motion.ac.in
72 - Solution Physics (XI) Topic

1. B 4. 0005

2. B


VA = 100 3 cos 30 î + 100 3 sin 30° ĵ
y = Distance in between observer & object = 100 î  50 2 ĵ
d = relative displacement between observer 
& object VB = x cos 60° î + x sin 60° ĵ
d x x 3
tan     = + ĵ
y 2 î 2
If y is very large , then angle subtended by
  x  x 3 
displacement d in a given time is very small as  
compare to nearer object. VB  VA =  2  150  î +  2  50 3  ĵ
   
so statement (I) is correct.
Now, observer velocity w,r.t laboratory frame = V1 As A sees B at 90° to its line of motion hence
& object velocity w,r.t laboratory frame = V2 
the angle between – x axis and VBA = 60°
 Velocity of the object w.r.t observer
  
V21  V2  V1 x 3
 50 3
so statement (II) is also correct. 2
tan 60° = x
150 
2
3. 5 m/s2
x x
2u sin  150 – = – 50
t= 2 2
g
x = 200

2  10  3 / 2 Hence VBA =  50 î  50 3 ĵ
 t= = 3 sec
10 
Now | VBA | = (50)2  (50 3)2
1 = 50 × 2 = 100 m/s
S = ut + at2
2 500
Thus time to collide = = 5 sec
1 100
 1.15 = 5 × 3 – ×a×3
2
5. 2 or 8
3a 4 = 0.2 × t × 1/2 × 2 × t2
or 1.15 = 5 3 
2  t = 1.9
or a = 5 m/s2 ~
— 2 sec.

394-Rajeev Gandhi Nagar, Kota


NLM & FRICTION - 73

1. A v cos  = u sin 
From constrained v = u tan 
+2 – vB – vB + 1 = 0

vB  3 / 2m / s  5. D


s6
2. A co
V B
30°
VB
B
20m/s
0 v'
20m/s 37° VA=1m/s
B
v
60°
A

A 10 m/s 10
VBcos60°

v'  0 VA cos 60° = VB cos 60°  VA = VB


 10
2
Here Resultant vel. of block 'B' is v 6. B

So component of resultant in the direction Component of force


of v' is in y direction is NA

v cos 37° = v', v cos 37° = 20 NA sin 60° = 500 A


60°
1000 NB B
20  5 60°
v  25 m / s NA 
4 3 30°
30°
Component of force
3. B in x direction is

–6  6 NA cos 60° = NB
v p1  0
2
500
 NB 
 | v p1 || v p2 | 0 3
P1 P2
v D = – vC
6 m/s

4 m/s
6m/s

 velocity of C is 7. D
A B C D
= 4 m/s

v1 v2
4. C 1 2v1 2
A
90°– 2v1 v2

u 2v1+v2
=v3
V
2
B
3 2v1+v2 = 2v3
v3
From constrained Motion - (along the rod
vel of each particle is same so component
of the velocity in the direction rod is)

www.motion.ac.in
74 - Solution Physics (XI) Topic

8. A part tends to continue in the state of motion


because of inertia of motion. Therefore, they
dx1 dx2 fall in forward direction. Conversely, when
x1 + x2 = l   0
dt dt the bus is in the state of rest, the passengers
v P – vB + vP = 0 in it have inertia of rest. When the bus
accelerates suddenly, the passengers tend
to continue in their state of rest and hence
x2 are left behind, relative to the position of
aP
aB m x 1 P bus. Therefore, they fall in the backward
direction.

aP 1
2vP = vB. It gives 2aP = aB  a  2 13. B
B
By Theory

9. B
14. C
F1  F2  .....Fn  0 By Theory

F2  F3  .....Fn   F1


15. A
F1 By Theory
a
m
16. A
10. B =  sin  
Action and Reaction are equal and opposite mg sin 

11. C 17. B

 dP  d
F , F (2 cos tˆi  2 sin tj)
ˆ
dt dt

F  2 sin tˆi  2 cos tj
ˆ


P  2 cos t î  2 sin t ĵ

F.P  FP cos 

4 cos t sin t  4 cos t sin t  FP cos  T – 2×9.8 = 2×0.2


T = 19.6 + 0.4 = 20N
cos   0

  90 18. C
V = Constant a = 0

12. A Fnet = 0

The bus and the persons in it are in the state P - 300 = 0


of motion before the application of the P = 300N
brakes. When the brakes are suddenly 2000-2S = 0
applied, the bus comes to the state of rest,  S = 1000N
the lower part of passengers which is in
contact with bus comes to rest, but the upper

394-Rajeev Gandhi Nagar, Kota


NLM & FRICTION - 75

19. B sec , T 
From horizontal equilibirium If tension is more then string may be break-
T2sin60° T1sin30°
T2 T 3
 1 23. A
2 2
5Kg. Given M is the mass of the aeroplane.
T2cos60° T1cos30°
T2  3T1 Let R be the upthrust acting on it. Since it is
50 N falling down with an acceleration a,
From vertical equilibirium
Mg  R = Ma ... (i)
T2 3 T1 Let a mass m kg be thrown out. The
  50
2 2 remaining mass is (M  m) kg and now the
plane begins to rise up with an acceleration
 T1  25N , T2  25 3N a m/s2.
Now R  (M  m) g = (M  m)a ... (ii)

20. B Adding equations (i) and (ii),

F = T sin  A Tcos mg = (2M  m)a


T
 B or, m (g + a) = 2Ma
F
F Tsin
 T=
sin  M 2Ma 2  9600  5
 m  a g kg = = 6400 kg
5  10
Mg
T cos  = Mg , T=
cos 
24. C
Mass less spring
21. C

25. D
T1 T2 m3 T3
m2 60N masin =mgcos
m1

a = g cot
Take a system (m1 + m2 + m3)
T3 = (m1 + m2 + m3) a 26. A
60 = 60 a
pseudo force
60 mg sin 
a  1m / s2
60

mg
For body m3 mg sin 

T3-T2=m3a From trolley frame


60-T2=30 mg sin  = mg sin 
T2=30N =

22. C 27. A
(A) 2T = W, T = W/2 F.B.D. of block N

(B) W = 2T cos  T   T
N2 = F2 + (mg)2
F=10N
W
{as there is no accel-
T= eration in the direction
2 cos  mg=10
of N}
In (C) option  is greater so
N = 10 2 N

www.motion.ac.in
76 - Solution Physics (XI) Topic

28. C T = mg ...(i)
F.B.D. of block A N 2T cos  = Mg...(ii)
Applying Newton’s From equation (i) and (ii)
secondlaw for block g  2mg cos  = Mg

A in vertical direction.  always > 0 so M < 2 m


mAg
m Ag – N cos  = mA g
32. C
Where  is the angle of the wedge
 N cos  = mA g T T

as < 90° m1 a m2 a
 N=0 m1 g m2g

T – m1g = m1a ....(i)


29. A
m2g – T = m2a ....(ii)
P On solving equation (i) and (ii)
Acceleration of the system : a  (1)
Mm
 m  m1 
The FBD of mass m is shown. a 2 g
 m1  m2 
R sing  = ma (2)
R
R cos  = mg (3)
From (2) and (3) a 33. A

A = g tan  Mg
asystem =
Putting the value of a in mn  M
(1), we get mg
mMg
P = (M+m) g tan  T
mn  M

30. B
34. A
The block (m) will move in vertical direction
as seen from ground. If we observe the block mg
T
with respect to the wedge (M), its motion 2
will be horizontal towards right.
mg 1
F.B.D. of m as seen from wedge 2T cos    cos  
2 2
 = 60°

amM mA 35. C
Case (i)
T – mg = ma
2mg – T = 2ma a T T a
N mg
On solving m
a = g/3
case (ii) here F = T
31. B
T – mg = ma
F=2mg
T T = 2 mg , a = g
 T m
T T On comparing a of case of (i) < case of (ii)

m A B M C m

394-Rajeev Gandhi Nagar, Kota


NLM & FRICTION - 77

36. C 40. A 30º


T – mg = ma ...(1) 2kx cos 60º = mg
Mg – T = Ma ...(2) kx = mg
60º
from (1) and (2) Net force kx = mg
= 2mg cos 60º
M>>m 2T
Mm = mg = ma
a g
Mm
mg
a a  g  10 m / s 2
Put M >> m a = g T T
m a
 T = 2 mg,
M 41. C
2T = 4mg
At 11th second lift is moving upward with
acceleration
37. B
0  3.6
Acceleration of B and C is sance so they can a  1.8m / s2
2
g
be treated as a system. a = (2m - m) g = Tension in rope, T = m(g-a)
3
1500(9.8-1.8) = 12000 N
mg - T = mg
T = 2mg/3 = 40/3  13N 42. B
T = N + 30 g (i)
38. B T + N = 50 g (ii)
Suppose T be the tension in each part of the T
string passing over the pulley. T is given by: T

2m1m2g 2  1  5 5 30 g
T   g
m1  m2 15 3 N

For equilibrium of the pulley there is an 50g


upward force of 2T acting on the string joining N
the pulley to the spring. Therefore, reading of  N = 10g = 100 N.
spring balance will be 2T = 10g/3. So reading
will be 10/3 kg which is less than 6 kg. 43. C
FBD of M : If M exerts force F = 7 Mg/4 on
39. C floor, then from third law floor also exerts
force F on box in upward direction.
Initially, F = m1g = k1x1
F – Ma = Ma
F  m 2g a
x1 = ....(1) 7Mg
k   Mg  Ma
4 M
m 2g Mg
Finally x2 = .... (2) 3g
k  a F
4
From conservation of energy,

1 44. B
m1g(x1 + x2) =
2

k x12 – x 22  .... (3)

From (1), (2) & (3), F = (m1 + m2)g

www.motion.ac.in
78 - Solution Physics (XI) Topic

ma cos 30 = mg sin30
mg  ma
a = 5.6m/s2 N=
2

ma cos 45° = mg sin 45° + N ...(2)


45. C
Put the value of N
Consider the situation in gravity free space.
ma mg (mg  ma)
 
46. D 2 2 2
Reading of spring balance
 1  
2m1m2g a  g 
T  N  1  
m1  m2

51. B
ma
47. C sin
A  h = r – r cos 
Mass m falls freely
m ma  mg cos = mg sin 
N=0  m
m

gs tan  = u
gc

a in
mg cos  = ma sin  
os

mg

m

 1
ac

a = g cot  cos =
os

B C

1 2
48. A
Weighing Machine  1 
h = r(1 – cos) = r 1 
2k

g
always Measure 2k W g
1 2 
M M  
W
Normal froce 30º 30º

N = 20 cos30° = 10 3
52. A
F < fsmax F
f
49. A friction=F
2T = m1g For F > fmax
m1 2T
T = m2 g friction constant
T
2m2g = m1g
m2
53. A
m1 m2g
m2 
2 By theory

54. B
50. B
fmax > mg sin
macos45°
sin
N
at this condition block remains rest when
ma mg sin > fmax
sliping slant
mgsin45° m N FC
gs
45°
in4 For  < angle of repose f

mgcos45° Fc = mg m
mg
For  > angle of repose
mg
N = mg cos 45° + ma sin 45° as   f = mg cos  

N = mg cos  

394-Rajeev Gandhi Nagar, Kota


NLM & FRICTION - 79

55. B 59. A
fmax =  mg cos
f = R = ma
3
fsmax  0.7  2  9.8  =7 3
2 a
ma R
mg sin = 9.8
As mgsin <fsmax so friction requird is mgsin. mg

to balance the block ma = mg


56. A
a = g/
move with a constant velocity
So ma = m g (in negative direction)
60. A
a=g
 v2 – u2 = 2as vf2 = vi2 + 2as 2g
a= =4 3
32
v  2gs here vf = 0, vi = v
v = u + at
v = 10 – 4 × 1 2
57. C v=6
Floor will provide the normal force and fric-
tion force the net reaction is provide by the 61. C
floor is R.
Acceleration a = (g sin  + g cos )
N R
g 1 3g
V = A V = + 0.5 × g × =
fr A 2 2 2 2
floor

62. D
58. C
a F 2kg
Fsin 
N
 1kg f=2N

m Fcos 
amax  2m / s2

fr mg  2m / sh2 3kg T

F sin  + N = mg T = 6N
or N = mg – F sin  ...(1) 10M–6 = 2m
fr = N ...(2) 8m=6
F cos  – fr = ma ...(3) M=6/8 = 3/4 kg
on solving (1), (2) & (3)
63. A
F cos   (mg  F sin )
a
m fmin  (m1  m2 )g

F
a= (cos + sin) – g
m

www.motion.ac.in
80 - Solution Physics (XI) Topic

64. B 66. C
Since M1 g sin 30° = M2 g
 Net pulling force = 0
A or acceleration of system = 0
f1 T Friction coefficient is not required in this
B
question.
f2 T
C
F
f3 67. C

R2 = 7 g y
f1 = 3 g N
B T
f2 = 7 g 37°
x
5×5
R 3 = 15 g
f 2 = R2
F C T 50
f 3 = R 3
N = 50 cos 37° + 25 sin 37° = 55

The free body diagram of B and C are


separately shown in Figures. 68. C
T = f1 + f2 = 3g + 7g
= 10g
Net pulling force mg - 0.5  5  g g
= 10  0.25  10 = 25 N a= = =
Total mass 10 4
Now F = f2 + f3 + T
equation of motion for the hanging mass will
=   7g +   15g + 10g be,
= 80 N
5g
5g – T = 5a =
4
65. C
15
T= g
4
T
T
15  9.8
a = = 36.75 N
4
M
2

M1 fixed
m1g sin    m2g sin 

//////////////////////// 69. C

 m g sin  
Let M1 > M2  m2g  1 
a1 =  2 
M1 g sin  – T = M1 a...(i)
m1  m2
T – M2 g sin  = M2 a ...(ii)

M1M2 (sin   sin )g  m 


On solving T =  m2  1 g
M1  M2 2 
a1 = 
m1  m2

394-Rajeev Gandhi Nagar, Kota


NLM & FRICTION - 81

The given angle of inclination is less is less


 m  than the angle of repose. So, the 1 kg block
 m1  2 g
a2 =  2  has no tendency to move. [Note that mg sin
m1  m2 is exactly balanced by the force of friction.
So, T = 0.]
s = ut + 1/2at2
u=0
73. D
s  same
For D Mg – T = ma .....(1)
a1t12 = a2t22
for A T – f = ma

 m   m  T – 2mg = ma .....(2)
 m2  1 g  m1  2 
 2   2  gt2 assuming (B + C) a single blade
gt =
2

m1  m2 m1  m2 So f = 2 ma
2mg = 2ma
m1 19 a = g
m2 = 11 By eq (2) T – 2mg = mg
T = 3mg
70. B By (1) Mg – 3mg = Ma
Net pulling force on the s ystem should be
3mg
zero, as velocity is constant. Hence, M=
1 
mA g sin 30° = mA g cos 30° + mB g

 mA   mA 3  74. D
 
 mB =  2  –  2 
    T

a m1
1 3
= 10  2  0.2  2  = 3.3 kg
 
0
20

71. C 200  150


200- T = 40a a 
70

T 5 50 5
a T 200  T  40  a 
7 70 7
100 m

37° 200 6
mg T  200   200 
7 7
T = mg
T = 200 × 0.85
T = 100 mg sin 37° + 0.3 × 100 g cos 37°
T  168 N
[Put g = 9.8]
T = 588 + 235.2
75. A
mg = 823.2  m = 82.33 = 83 kg
a1kg = a2kg = a3kg = a
60 – (30 + 18) = 69
72. D
12 = 6a
If  represents angle of repose, then, tan 
a = 2m/s2
= 0.8
 = tan-1 (0.8) = 39°

www.motion.ac.in
82 - Solution Physics (XI) Topic

76. C 78. A

f
T/ 2

F F 45°
T
f  =30°  =60° kg
15 T/ 2
F = mg sin 30° +  mg cos 30° 150 / 2
150 / 2
45°150
mg
 [1   3 ] ...(1)
2
T = 50 N
F + f = mg sin 60°
Component of force (in y direction)
mg T 150
F= [ 3 – ] ...(2) 
2 N=  N = 200 / 2
2 2
Now (1) = (2) 200
f 
2
( 3  1)
1  3  3     Component of force in x direction
( 3  1)
150 T
  fr
2 2
77. B 150 50   200 1
   =
T – mg sin 45° = ma 2 2 2 2

mg mg g
T– = Given a  79. D
2 5 2 5 2

6 mg
T
5 2

1 > 2 so a2 > a1 both will move in com-


bined form with same acceleration
6g sin  – f1 – f2 = 6a
a = 2.4

80. A

3 3 4 1 4
6a  6  10   10  4    10  2  
5  4 5 4 5

a =1.3 m/s2
2g sin 37 – T – f = 2a
2g × 3/5 – T – 4 = 2 × 1.3
T = 5.3

394-Rajeev Gandhi Nagar, Kota


NLM & FRICTION - 83

1. A 5. A
6 5
From constrained Let
4
A 7 3 B
a1 + a2 + a3 + a4 = 0 C  c x î  c y ĵ C
1 2

– a – aB – aB + f = 0 A1 4 C Cx = a 
From constrained a b
2 3
 f a 1 a1 + a2 + a3 + a4 + a5 + a6 + a7 = 0
aB     = f  a  B
2 2 2 –a – b + 0 + 0 – b – a + c = 0
cy = (2a+2b)  (By constrain Motion)
In ground frame
2. A
A 3 4 B
 C  a î – ( 2a  2b) ĵ
From constrained 2 5
a1+a2+a3+a4+a5+a6 = 0 1 6
6. A
–aC+2+2–1–1–aC = 0 C b
In hori zontal
aC =1 m/s2  di re ct i on ne t 1 2
ac ce l e rati on i s 1
zero.
3. A
So, b cos 2 = a cos 1 a
0.6 m/s 3 a cos 1
A
2 b
cos  2
1 4
B v
7. C
From constrained
For B : mg – T = ma
v1 + v2 + v3 + v4 = 0
ma
v – 0.6 – 0.6 – 0.6 = 0 For A : 2T – mg 
2
V= 1.8 m/s 2T a/2
m A
5
 mg  ma mg
2 T
4. B m B
a
2g mg
v0 a
vA 5
2 1
4
A 3 B
5 8. B
Conceptual
From constrained
9. B
v1 + v2 + v3 + v4 + v5 = 0
V0 – VA – VA + V0 + V0 = 0 F
2m m 2m m
F
3 V0
VA  F F
2 a a
3m 3m
3V0
 VAB = VA – VB = – V0 F F 2F
2 N1  m  N2 
3m 3 3
V0 1 2
 ( towards Right ) N1 : N2  :  1: 2
2 3 3

www.motion.ac.in
84 - Solution Physics (XI) Topic

10. B N1 and N2
N1 = 96.59 N, N2 = 136.6 N
11. C
(a) T = mg + ma 15. B
(b) T = mg – ma
A
T = mg
T mg 45°
mg
T
12. C B
C 45°
When a body is stationary its acceleration is
mg
zero. It means net force acting on the body mg

is zero. i.e.,  F  0 . Or we can say that all
Force along the rod is same
the forces acting balance each other.
mg
= mg cos 45° =
2
13. A
Conceptual
16. A
T’
14. A 2T
 mg
N2
2 45°

T T M
mg 45° 45°
 T= ....(i) Mg
2
45° m
N1
30° mg
T mg
T' cos   ....(ii)
2
N2
30° 45° T
45° N1 T' sin    Mg ....(iii)

30° 2
30° 45°
from eqn (ii) and (iii)
N2
T
(tan   1)  Mg ....(iv)
30° 2

from eqn (i) and (iv) we get


45°
N1
2M
 tan   1 
mg
m

In vertical direction
17. C
N1
N 3
50   2 ...(1)
2 2 Tsin
A B
In horizontal direction 
T


N1 N Tcos 
 2 ...(2) W
2 2
T’ W/2
On solving eqn (1) and (2) we get

394-Rajeev Gandhi Nagar, Kota


NLM & FRICTION - 85

T cos = T ....(i)
NAB
W
T sin  = ....(ii)
2 30°
NBC Fpseudo=ma
From equation (i) and (ii) we get

W
 T = cot 
2

mg
F.B.D of Cylinder w.r.t. to carriage
18. B
In upward motion u and NBC = ma + NAB sin 30° (2)
He nc e N AB rem ai ns c onst ant and N BC
as v  mg + bv
increases with increases in a.
Force 
acceleration 
22. C
and takes less time to reach at top. 2T = m1 g ...(1)
m2g – T = m2a ...(2)
2T
19. C T – m3g = m3 a ...(3) 2T

on solving
(A) 40 cos 30° = 20 3 N
T T

(B) weight = 5 kg 4 1 1 m1 m2
  m3
m1 m 2 m3 a
(C) Net = zero

20. B 23. B
The masses A and B of m and 2m
respectively are initially along the
ma
horizontal position through the line AB.
When the masses are left free, B comes
mg a
down, A moves up with acceleration a.

( 2m m)g g
Let the mass of a block is m. It will remain Now, a = 
2m  m 3
stationary if forces acting on it are in
equilibrium i.e., ma cos   ma sin   a = The initial velocities of both of them is zero.
g tan  When the lighter mass A moves up through
Here mg sin  = Pseudo force on block, mg a height 15 m, its velocity v is given by
= Weight.
10
v = 2 a  S  2  15 = 10 m/s
3
21. C
The free body diagram of cylinder w.r.t.
24. B
carriage is as shown.
Si nce net acc el erati on of cy l i nder i s
horizontal,
NAB cos 30° = mg
2mg T
T T
2
or NAB = mg (1) m m m
3 F1=2mg F2=mg
A B C
2m
mg
mg

www.motion.ac.in
86 - Solution Physics (XI) Topic

a1 = g 2mg – T = 2 ma a3 = 0 29. (i) A (ii) A (iii) C


T – mg = ma a2 = g/3 (iv) D (v) B (vi) D
 a1 > a2 > a3 (vii) B (viii) B

25. C a=0
Pulley is fixed from the ceiling (a) V=0
N=mg
If pulley is fricitonless then there is no effect w.m
of mass of pulley.

26. B a
(b) N = mg + ma
12 m/s2
kx kx
w.m
10kg 20kg
200 N

Force on 10 kg block Kx = ma
a
= 12 × 10 = 120 N (c) N = mg – ma
So w.m

20kg 200 N Independent of the direction of velocity.


Kx = 120N

30. B
80
a  4 m / s2 Readings of spring balance equals to tension
20
at its hook.

27. B
31. B
T Since, downward force along the inclined
2m T=0 plane = mg sin = 5×10×sin30° = 25 N
kx
2mg 2m
kx
32. B
m kx Let A applies a force R on B,
2mg
mg Then B also applies an opposite force R on A
as shown.
T = Kx + 2 mg ....(i)
Kx = mg ....(ii)
R
T = 3 mg
After cutting T = 0 A
a = 2 m/s2
downwards net force B mg
R
3mg 3g
a  =
2m 2
For A : mg – R = ma
 R = m(g–a) = 0.5 [10–2] = 4 N
28. (i) A (ii) A (iii) A
(iv) C (v) B (vi) C
33. A
(vii) C (viii) B
N – mg cos 60° = ma cos 60°
N = m cos 60° (a+g) = 400 N

394-Rajeev Gandhi Nagar, Kota


NLM & FRICTION - 87

g sin 37° – (0.3) xg cos 37° = 0

a cos 60° 3 60 20
6–  x8 = 0  x    2.5 m
10 38 8

N 38. C
60° FBD
a 60°
1N 2kg T 3kg
fmax=2N fmax=6N

Net force without friction on system is ‘7N’


Reading will be 40 kg in right side so first maximum friction will
come on 3 kg block
34. D 1N 2N
2kg 2N 3kg 8N
N = applied force = 12 N 1N 6N

 fmax = N = 72N So f1 = 1N, f2 = 6N, T = 2N


Since weight w < fmax
Force of friction f = 5 N 39. A
N
 Net contact force = N2  f 2 

(M  m)g sin 
= (12)2  (5)2 = 13 N (M  m)g cos 
(M+m)g

For equilibrium condition


35. A (M+m)g sin  =  ( M + m) g cos
fms = 0.3×2×10 = 6N tan  = 
Applied force < fms Here   coefficient of friction between
So, body would not move. board & log.

36. C 40. A

  0.2 B A
P 4 kg T
T=0 m m F
Q 5 kg F
  0.1 F
frA  s  k  smg  s   k
f1 = 0.2 × 40 = 8 N
f2 = 0.1 ×90 = 9 N Initially

8 F  fr A  0  t – s mg = 0  t = s mg
Max. acceleration for system a = = 2 m/s2
4 [till or frB = smg t – smg = s mg
Minimum force needed to cause system to t = 2 s mg]
move = 9 N
T  F  frA  frB

37. D T  t   smg  frB


a = g sin -  g cos t = smg block be will not move
At the x increases, u  a  smg < t smg block be will not move,
so when a = 0 instant give maximum speed static friction will work

www.motion.ac.in
88 - Solution Physics (XI) Topic

after t > smg kinetic friction will work to m1 and m2.


So no relative motion between them
F   s mg   k mg
a  x = 0 (Always)
m
f
So T = F – s mg – ma
43. A
after t = 2s mg t

Q
N N
41. B 
P (P  Q sin )
m
1000/3 /////////////////////////////////////// fr
N mg
(Q cos   mg )
T
37° amax
fr =  N =  (mg + Q cos )
f fr = P + Q sin 
mg
250
(P  Q sin )
 = (mg  Q cos )
T cos 37° = f
N + T sin 37° = mg
44. C
3T
 N = 100 g – T sin 37° = 100 g –
5

and T cos 37° =  N

3T
T cos 37° = (100 g – )
5

1000 1
on solving T = ( = )
3 3

T – Mg = mamax

1000 t>1 3=a


 250  25  a max
3

g 10
amax   m / s2
3 3

42. D

m1 m2
a

m 1a m 2a
m1 m2

anet = a – g anet = a – g

 fr static and fr kinetic


both provide same acceleration

394-Rajeev Gandhi Nagar, Kota


NLM & FRICTION - 89

 N+ F sin 30° = mg cos 37°


or N = mg cos 37°– F sin 30°

4 1
= (4)(10)  5  -(10)  2 
   

or N = 27 N (i)
fmax = N = 0.5 × 27 = 13.5 N

4
mg sin 37° = (4)(10)   =32N
5

aB / T  3
 3
  and F cos 30° = (10)  2  = 8.66 N
aB  aT  3  
 
aB  3  aT  2 left side Now since mg sin 37° > fmax + F cos 30°
Therefore block will slide down and friction
VB  0  2  1  2 m / s
will be kinetic.
0 = 3 – 2 t = 1.5 sec – t

47. D
45. A The system is at rest (Fnet = 0)
For maximum M/m; Limiting friction will be
N acting on both blocks (at contact surfaces).
40 N
F = Net pulling force on the whole system
0N mg + Mg cos  = Ng sin 
°=6
37 Mg(sin - cos ) = mg
sin 100 cos 37°=80N
0
10
37° M 

m (sin    cos )

Net driving force = 60 – 40 = 20 N(down


the plane) As resisting force is greater than 48. C
net driving force, the friction will be static Mg sin  – T = Ma ...(1)
of nature and friction force is 20 N (up the T = Ma ...(2)
plane)
Now eq. (1) – eq. (2)
a T
Mg sin  – 2T = 0
46. C M
a
 T
Mg sin  in fixed
gs
M
Drawing free body diagram of block,  Fy  0 T= M
2

y
x 49. B
°
30 m1g  m2g sin 300  a1(m1  m2 )
sin °
+
F 30
s
N co
°
F m2g  m1g sin 300  a2 (m1  m2 )
37 f+
sin
g 4a2  a1
m
mg cos 37°
37°

www.motion.ac.in
90 - Solution Physics (XI) Topic

6 g sin 60  f1  T = 6a … (i)


 m g  m g where a is the acceleration down the plane.
 m1g  2   m2g  1 
 2  = 2  4
 m1  m2   m1  m2  N1 = 6g cos 60 … (ii)
   
    f1 = N1 = 0.1  6g cos 60 … (iii)
 6g sin 60  0.1  6g cos 60  T = 6a
m1 3
m2 = 2 6 10  3
 0.1
6 10
T  6a … (iv)
2 2
The equations of motion for 4 kg mass can
50. D
be written similarly considering the motion
of 4 kg mass separately.
T + 4g sin 60  f2 = 4a … (v)
N2 = 4g cos 60 … (vi)
f2 =  N2 = 0.6  4g cos 60 … (vii)
T + 4g sin 60  0.6  4g cos 60
= 4a … (viii)

m1g sin  – T = m1a 4 10  3 4 10


T 0.6 4a
T + m2g sin  = m2a 2 2
m2g – T = m2a Solving the above simultaneous equations
T – m1g sin 30° = m1a in T and a, we get T = 6 N

51. B 54. A
If the plane makes and angle  wi th
52. A horizontal

Relative slipping present tan  = 8/15 If R is the normal reaction

 = .4  15 
f = mg = 28 R = 170g cos  = 170×10×   =1500N
 17 
35a = 28  a = 0.8 m/s2
Force of friction on A = 1500×0.2=300 N
Force of friction on B = 1500×0.4=600 N
53. D
Considering the two blocks as a system, the
The force diagram of the masses placed on net force parallel to the plane.
the inclined plane is shown in Figure.
= 2× 170g sin  –300–600 = 1600 – 900 =
Considering the motion of 6 kg mass the
700
equation of motion can be written as
700 35
 Acceleration   m / s2
F2 340 17

T Consider the motion of A alone.


T N2
35
4g 170 g sin  – 300 – P = 170×
17
N1
(where P is pull on the bar)
6g
P = 500 – 350 = 150 N

394-Rajeev Gandhi Nagar, Kota


NLM & FRICTION - 91

1. B,D 5. A,C
From constraint relation, T = mg …(1),
x B = xP + xA 2T cos 37º + Mg cos 37º = Mg sin 
vB = vA + u
acceleration will be same as Pulley is moving 6. A,C
with constant velocity N sin  = ma
N cos  = mg
2. B,C
Monkey & block both N = m a 2  g 2 and tan  = a/g
reach the pulley at the
same time.
7. A,B,C
Upward acceleration
F  mg N3
= 
m 
N1
Time taken by monkey N4 N3
 m1g m2 g
t o re ac h pul l ey i n
case-2 is more as
N2
compared to case-1.
Block will reach in the N3 = m2 g cos 
s am e ti me , but N3 sin  = N1
m onke y have l es s N2 = ml g + N3 cos
acceleration in upward  N1 = m2 g sin  cos 
direction, as only a  N2 = m1 g + m2 g cos2
component of 'F' helps
monk ey t o re ac h 8. A,C
pulley. NA = mg cos and NB = mg cos 53º
t1 < t2 and in case-2, block reaches the pulley
NA 4 cos  4
earlier than monkey. as given N =  =
B 3 cos 53º 3
3. A,C 4 3 4
a1 cos  = a2 …(1)  cos = ×  cos =
3 5 5
100 – T = 10 a2 …(2)
T cos  = 5a1 …(3) mg[sin 53º– sin ] g
Now, a = =
2m 10
4. A,B,C
F=t 9. A,B,D
ma = t
t
a  at ....(1)
m

dv  t
St. line 
dt m

 t2
 v=  v  t2 ...(2) N = m(g + a) cos30°
m 2
Parabola on solving (1) & (2) 3
N = 12. =6 3 N
v  a2 Parabola. 2
 block is at rest

www.motion.ac.in
92 - Solution Physics (XI) Topic

 f = m(g + a) sin 30° = 6 N 16. BC


Net reaction force a net = g +a
F = m (g + a) = 12 N a11= (g+a) sin
 Bl oc k i s acc el erat ed v erti cal l y along the plane
upwards
1
 Net force on the bl ock is = ma S = ut + at2
2
(upward) = 1 × 2 = 2 N
17. C,D
10. A,C Stati c fri cti on i s al ways equal to the
a1 cos  = a2 …(1) unbalanced force parallel to the contact
100 – T = 10 a2 …(2) surface, while kinetic friction is always equal
T cos  = 5a1 …(3) to  × N.

11. A,C 18. A,B


4g – T – F = 4a fstaticmax  15   sN
T = 2g = 2a
15 15 15
s     0.6
12. B N mg 25

m2 3 Now let k then


a g  10  3m / s2
m1  m2 73 15 – fr = ma
 15 – k 25 = 2.5 a

13. AC 15  2.5a
k  ...(1)
2 .5
kx kx 1
Now x = ut + at2
2
m1 m2
spring B m2 1
k  10 = 0 + × a × (5)2
m1g m2g
2
m1 A
10  2 4
C  a= 
55 5
kx  m 2 g 4
m1g = kx a= m2  a= m / s2
5
a=0 15  2.5  4 / 5
 k   0.52
Before Burnt 2 .5
T = kx = m1g
Just after burning just at 1 sec 19. B,D
(A) m2 will be upwards. fc = N (Given)
(B) m1 will be = 0
 fc = N2  f 2
14. A,B,D Acceleration to condition f = 0
The block is in free fall condition  fc = N
It acceleration along the incline is
g sin  and N=0 20. A,B

15. ABCD

g
tan  =
g
 = 45° (fmax)AB = 0.3 × 60g (fmax)B/g = 0.2 (100 g) f

394-Rajeev Gandhi Nagar, Kota


NLM & FRICTION - 93

21. A,D 25. A,B,D


Never lose contact As there are no external forces acting and
B
Along the string,  A + B system, its total momentum is
acc and vel is conserved. If the masses A and B are 2m
same Au and m and v is common velocity.

22. B,D m
B
a
A
A 2m
T
T
T mu = (m + 2m) v
fixed
30° B
mg u
or v 
3
T + mg sin  = ma ....(1)
Work done against friction = loss in KE =
mg – T = ma ....(2)
1 1
3g mu2  3m  v2
on solving (1) & (2) a = 2 2
4
1 1 u2 2 1
3g  mu2  3m.   mu2
T= 2 2 g 3 2
4
The force of friction between the blocks is
23. AB mg

mg g
Acceleration of A to right = 
m 2
T a
mg
b Acceleration of B to left =  g
T T g m
1k
T Ac ce l e rati on of A re l ati ve to
8
6
1 A 2 kg g 3g
kg B=   g 
2 2
6 20
°

37°
8
37

6 - T = a ...(1) a - 2b + c = 0 26. A,B


T
2T - 6 = b ...(2) (6-T)-2(2T-6)+(10- )=0
2

T
20 - T = 2c ..(3) 6-T-4T+12+10- =0
2

T 11T
28  5T  
2 2

10 56
a m / sec2 T  Newton
11 11

24. A,B,C
If the tendency of relative motion along the
common tangent does not exist, then
component of contact force along common
tangent will be zero.

www.motion.ac.in
94 - Solution Physics (XI) Topic

1. 6 metre/sec2 Where mT  Total mass = 10 + 15 + 18=33


From pulley (3) Kg.
ac = 0 m/s2
1 O O 2 O

5. 2 sec
O
3 er
ind A
cyl 60° 30°
30°
O x
g
= 4 g g
2a1  a3 0° 2
2
a2 = s6
2 g co
2
2a1 = 2a1 – a3
2a3 – 2a1 + a3 = 0 g
length of oA = 5, a =
4

1 1 g 2
2. 10 m/sec2 s= at2  5 =  .t  t = 2 sec
2 2 4
Acceleration of pulley
P

aA  aB 1  7 6. 10/3 m/s2
aP =   4 m/s2 (upward)
2 2 For movable pulley
Acce l erati on pul l e y Q wi l l be 4 m/s 2
O  aA
downwards. aB 
2
aD  aC Q
aQ 
2 aB
aC aD
aD  2
4 C D
2
Pulley Q
O
 aD = 10 m/s () 2
A aA

3. 10 N
F = ma aA = 2aB (i)
S = ut + 1/2 at 2 Free body diagrams of A and B:

4. 332.5 N
2T
T – mT g = mT acm = [mAaA + mBaB + mcac] aA T
T = mTg + mA aA + mBaB+ mCaC B aB
30
° A
= 330 + 10 × (–2) + 15 × 1.5 + 8 × 0 sin
mA
= 330 + 22.5 – 20 mBg

= 332.5 N

394-Rajeev Gandhi Nagar, Kota


NLM & FRICTION - 95

(i) Equation of motion of block A and B 11. 0


Pseudo force doesn’t exist in Inertial ref.
mA g
(ii) For A : T  =mAaA = mA(2aB) (ii) frame.
2
For B : mBg – 2T = mBaB (iii)
12. 12 N
From (ii) and (iii) [2 (ii) + (iii)]
mBg – mA g = [4mA + mB] aB

[mB  mA ]g [4  2]  10
 aB  
[4mA  mB ] [4  2  4] F
N
20 5
  m / s2
12 3
N = mg cos = 2.5 g cos 37°
10 N sin 37° = F
Hence aA = 2aB = m/s2
3
F = 2.5 × 10 × cos 37° × sin 37°
=12 Newton
7. 4
By constraint relation 13. 2
VA sin 60º = VP2 (1 + cos 60º) + VB 
f = – m a î = – 1 × 2 î = 2 î Newton.

 VP2 = 4 m/s.
14. =2

8. 8 m
N g  f = m
T = m2 a 3
m/3
m1g – 4T = m1(a/4) If friction coefficient is

m 2
9. x2 > x1 > x3 x1 : x2 : x3 : 15 : 18 : 10  then  g  mg
3 3
(a) T1 = 20 N = kx1
(b) T – 20 = 2a  30 – T = 3a =2
2/3mg
On solving a = 2m/s 2

T = 24 N = kx2 15. 0

(c) T – 10 = a 2kg
20 – T = 2a A f1
f1 F = 25N
B = 8kg
40 f2
On solving a = 10/3 m/s2 & T = N = kx3
3
f1max = 0.2 × 2 × 10 = 4
20 24 13.3 f2max = 0.5 × 10 × 10 = 50
So x1 = , x2 = , x3 =
K K K firstly applied f2 then f1
x2 > x1 > x3 x1 : x2 : x3 = 15 : 18 : 10 Here f2max > 25
 So f1 =0
10. 2
kx cos  = (Reading) A 16. 2.8 N
kx sin  = (Reading) B Conceptual of

www.motion.ac.in
96 - Solution Physics (XI) Topic

17. 5 19. mAg sin + F – T – f = mAa (i)


Denote the common magnitude of the
mA g cos  N
maximum acceleration as a. For block A to
T f T
remain at rest with respect to block B, a F A f A
N mA g cos 
 Sg . Let us assume a = mSg for mass of C mA g sin  f' N''
mAg cos 
to be largest. The tension in the cord is then
T = (mA + mB)a + kg(mA + mB) = (mA + f = kmAgcos (ii)
mB)(a + kg). T –mBg sin  – f – f’ = mBa (iii)
This tension is related to the mass m C f’ = k.N (iv)
(largest) by f’ = k(mAgcos + mBg cos )
T = mC (g –a). Solving for mC yields Solving above equations, aA = 5.2 ms–2,
mA  mB   S  k   1.5  0.5 0.6  0.4  5kg T = 215 N
mC = 1  S 1  0.6

20. 1 kg

mg sin  T
18. a=
M  2m(1  cos ) 1k
g


in3
gs

T = g sin 37° +  g cos 37°, T = mg


m = sin37° + cos37° = 1 Kg

394-Rajeev Gandhi Nagar, Kota


NLM & FRICTION - 97

3. 5
 g^ g^  g^
1. aA  i  j , aB  i T2sin30º = T1sin60º
4 2 4

T2 3
= T1 T1cos60º T2cos30º
2. 1 2 2
T1 60º 30º T2
L e t
T2 = 3 T1
acceleration of
M
blocks A and B T1sin60º T2sin30º
b e a and b T1 3T2
2d  = 20 20
vertically 2 2
up w a rd s , a 2d
d T1 = 3 T2 = 40
respectively.
b
da T1 = 3 × 3 T1 = 40
b
2 T1 = 10 Newton
a
 d = 2b + a c
A B C
c = 2d = 2a + 4b
4. 556.8 N , 1.47 sec
So according to geometry of the given figure,
downward acceleration of block C will be for man of mass m1 a m1 G  a m 2R  aR G
equal to (2a + 4b). Now considering FBDs,
we get the following : am1 G  (1.2  a)

T1 T1 T2 T2
T1 T2
for man of mass m2 a m 2 G  a m 2R  a R G

= (2 – a)
So now
m1a A C m (2a+4b)
m 2b F 3
T  mg  m1 (1.2  a) ...(1)

B B m3g

m2g T1 a
a
T
For block A, T1 – m1g = m1a ....(i)
T m2=60kg
For block B, 2T1 – m2g = m2b ....(ii) am1R  1.2m / s2 am 2R  2m / s 2
m1
For pulley F, T1 = 2T2 ....(iii) =40kg
For block C, m3 g – T 2
T  mg  m 2 ( 2  a) ...(2)
= m3 (2a + 4b) ....(iv)
Solve eq. (1) & (2) and put m1 = 40 kg
Solving above equations, T1 = 22 N, T2=11N
m2 = 60 kg
a = 1 ms–2, b = 1 ms–2
you get a = 2.72 m/s2
Hence, acceleration of block A,
T = 556.8 N
a = 1 ms–2 ()
Acceleration of block B,
5. 2N
b = 1 ms–2 ()
First find out acceleration of A so for this
Acceleration of block
C = (2a + 4b) = 6 ms–2 ()  a  20  2 F2  20  2  8

aA = 4 m/s2

www.motion.ac.in
98 - Solution Physics (XI) Topic

Now use pseudo concept (in which A is non We geta = 2m/sec2, T = 2.4 N
inertial frame) When B touch the ground at this time
F2 velocity of partical A is
 mBaA
v = 2(0.5) = 1 m/s2
It move upward untill the velocity of A is
zero.
50cm  0  1  gt , t = 0.1 sec

2F2 F2 = 8 N B remain at rest on ground for t   2t


F1 = 20 N
B 1kg t  2  0.1  0.2 sec

A=1kg

 8  4  4 m / s2  m1  2m 2 
 g
7.
 2m 2 
50 1
Now   4  t2 Initial m1 > 2m2
100 2

1
t  0.5 sec
2

m1g m1g
6. (a) 2 ms–2, (b) 2.4 N 0.3 (c) 0.2 s m1g
2 2
2
T – 0.2 g = 0.2 a ...(1) m2
mg – T = ma ...(2) T=m1g m2g
T'
adding (1) and (2) mg – 2 = (m + 0.2) a m1

mg  2 m1g
a ...(3)
m  0 .2
after cutting
Part i c l e B m ov es d ow nw ards w i t h a
acceleration so

m1g m1g
2 2
T m1g
a T 2
a
m2 a
A 0.2 kg
B
m
m2g
0.2 g
mg 0.25 m m1

m1g
1 2
0.25  at
2  m2a  m1g / 2  m2g

1  mg  2  2  m  2 m2 
0.25    0.5 [Given t = 0.2 sec ] a 1  g m/s 
2
2  m  0 .2   2 m 2 

 m  0.3 kg
Now put value m = 0.3 kg is eq. (2) & (1)

394-Rajeev Gandhi Nagar, Kota


NLM & FRICTION - 99

after wards a = 0 so at t = 3 sec v = 8


m sin  cos 
8. (a) a = g cot, (b)  min  m/sec
m cos 2   M
a   k g (sliding), v = u + at
(a) Using pseudo concept

ma sin   N  mg cos  6
 8  2  10 k (2)   k ,
10  2

N  k  0.3 sec

ma m

(pseudo) M mMv20
10.
mg a 2F m  M

The same frictional force is effective on A
When N = 0 and B. This force produces retardationon A
and acceleration on B till they acquire a
 a  g cot  common velocity.
(b)  N1  N cos   Mg  f   N1 F = ma = Ma' where a is the absolute
ret ardati on of m, a' i s t he abs ol ut e
  (N cos   Mg)  N  mg cos  acceleration of M.
Relative retardation of m = a – (–a') = a + a'
Initial relative velocity = v0
Final relative velocity = 0
m N sin 
 v20  2  a  a s

N N1 
N cos  where s is the distance covered by m relative
to M

F F 2F m  M
 f   (mgcos2   Mg) or v20  2    s  s
m M mM
Wedge not move when

f  N sin   mg cos  sin  mMv20


or s
2F m  M
 (mgcos2   Mg)  Mg cos  sin 

Mg cos  sin  1/2


  g  1m  2M 
2
Mg cos   Mg 11.  
 Mr2  mr1 

Ev i d entl y, t he l arge r bl oc k of m as s
9. s = 0.4 , k = 0.3
experiences more centrifugal force radially
at t = 1 sec it start slipping so. outwards, compared to the block of smaller
at this moment acceleration of block = sg block m,
t = 1 sec a = 4(t) = 4(1) = 4 m/s2 [M > m and r2 > r1]
  = sg s = 0.4 Figure shows there F.B.D.

2
v = 2t
T mw2r T
m m
after that at t = 1 sec v = 2m/sec. Mw2r2
f1 f2
at t = 2sec v = 8 m/sec

www.motion.ac.in
100 - Solution Physics (XI) Topic

Owing to the larger force experienced by = 1 + 2 + 3 +4


block of mass M, it ends to fly off radially. where 1, 2, 3,  4 are the instantaneous
In the situation of limiting equilibrium, we lengths of the segments of the string.
have    

T = m2r1 + f1  2x – y  2x  y
 T + f2 = M2r2  2ax = ay
(where f1 and f2 are frictional forces for the N = max and mg – mN – T = may and 2T – N
two blocks and the surface)
= MAx = Max
f1 = 1mg
Eliminating T, A and N
 f2 = 2Mg
The above two equations get reduced to
2mg 4mg
T = m2r1 + 1mg ....(i) ax  and
M  5m  2m M+5m+2m
T + 2Mg = M2r2 ....(ii)
Subtracting Eq. (i) from Eq. (ii)
2 5mg
 a a2x  a2y 
2Mg = M2r2 – 2r1 – 1mg m+5m+2m

g 1m  2M
2 
Mr2  mr1 13. i. 2.5 ms–1 ; ii. 2.5 ln 2
Let at any time t, vel oci ty of M be v
 g  1m  2M  downward. Then
  
 Mr2  mr1  vrel = vB + v and f = 0.4 (vB + v)
i. If finally it comes to rest, then f = 0.4 vB
= mg si 30°

2 5 mg  vB  2.5 ms–1
12.
m  5m  2m
ii. At any time, mg sin 30° – f = ma
Let X be the leftward displacement of M and
mdv
x and y be the leftward and downward  5  0.4 12.5  v  
dt
displacement of m as shown in the figure.
Then by constrain relation, we have
dv
 5  5  0.4v  1 
dt

1 t
T dv 2
     dt  t=2.5 ln 2
T mN Mg 2 v 0 5
T

N M 14. 3
N
N
mN
mg T
N1

   
x=X  xX
 ax = Ax
More friction will out on block of small block
and 1 + x + 2 + 3 – x + 4 + y
'm' No block kept in front will now front.

394-Rajeev Gandhi Nagar, Kota


NLM & FRICTION - 101

15. 0
N1 = mg cos 
N2
and f1 = mg cos 
T
a f2
M
N1

sin
Mg
f1
a m
 Mg cos 

T si n
mg
Mg

mg cos 
Equations of motion are
mg
T – f1 + mg sin  = ma ....(i)
Mg sin  – T – f2 = Ma ....(ii)
N2 = mg cos  Solving Eqs. (i) and (ii), we get T = 0
and f2 = mg cos 

www.motion.ac.in
102 - Solution Physics (XI) Topic

1. C
15
According to work - energy theorem, a  2   10 ms2
3
W = K = 0
 Initial acceleration, a = 10 ms-2
=> work done by friction
+ work done by gravity = 0
4. B
l
  (mg cos )  mglsin  0 v2 100  100
2 s  100  100
  1000 m
2k g 2  0.5  10 52

or cos   sin 
2
5. D
or   2 tan  In the frame of wedge, the force diagram of
block is shown in figure. From free body dia-
2. A gram of wedge.
When friction is absent
a1 = gsin 

1
 s1  a1t12 ...(i)
2
When friction is present
a2 = g sin  - kgcos

1
 s2  a2t 22 ...(ii)
2 For block to remain stationary.
From Eqs. (i) and (ii) ma cos  = mg sin 
or a = g tan 
1 1
a1t12  a2t 22
2 2
M  m
a1t12  a2 (nt1 )2 (t2  nt1 ) 6. f   g cot 
 2 
or a1 = n2a2
Drawing force diagrams of the rod, we have
a g sin   k g cos  1
 2   2
a1 g sin  n N2
N1 O
g sin 450  k g cos 450 1 mg
 2
g sin 450 n2 N

 Mg
1 1 f
or 1  k  or k  1 
n2 n2 y

3. D
x
Given, m = 0.3 kg, x = 20 cm
and k = 15 N/m N1

F = - kx ...(i)
mg
and F = ma ...(ii) N2  N
2
 ma = - kx
Mg

f
15
a    20  102
0.3

394-Rajeev Gandhi Nagar, Kota


NLM & FRICTION - 103

This is the equilibrium of coplanar force, ence 9. D


using the equation  Fx  0,  Fy  0 and net F1 = mg(sin  + cos)
moment about point O = 0 F2 = mg(sin -  cos)
we have the equations,
F1 sin    cos 
  m
mg F2 sin    cos 
N  N2  Mg  ....(i)
2
tan    2  
N1 = f ....(ii)   3
tan    2  
mg
N  N2  Mg  ....(iii)
2
10. C
L
and Mg cos   fL sin  x3 f
2 y=
6
= ML cos  ....(iv)
Solving these four equations, dy 3x2 x2
= =
dx 6 2 m
M  m
We have f   g cot
 2  f =  mg cos 
= mg sin 
 = tan 
7. C
Acceleration of system, x2
0.5 = x=1
2
F
a
mM
x3 1
y= = m
k 6 6
m M F

So, force acting on mass,

mF
11. A
F = ma  Assume the system is
mM A B
F
i n equi l ibri um. Net
grav i t at i onal forc e
8. D must be balanced by
mg sin = ma friction force from the
 a = g sin  wall.
where a is along the inclined plane Force of friction = 120 N
 Vertical component of acceleration is
gsin2
 Relative vertical acceleration of A with re-
spect to B is
g (sin2600-sin2300)

g
  4.9 ms2 (In vertical direction)
2

www.motion.ac.in
104 - Solution Physics (XI) Topic

1. 11.313 m T cos  = ma F = 2T sin 


1 F cos  F
aAB = m / s2 w.r.t B a .  cot 
2 2 sin  m 2m

1 1 2 A F x
2  t a .
2 2 2m a 2  x 2
0.1×10×cos45°
t = 2 sec
6. B
2. D
This is the equilibrium oif coplanar forces.
Hence, 7. B
Fx = 0  F=N Due to inertia particles left at their places
 Fy = 0, f = mg c = 0 when we pull the clock suddenly.
 N + f = 0  Since, f  0
 N  0  Answer is (D). 8. (B)
for sliding
3. B tan  =   = 60º N cm
15 f
N shifted downwards
to stop the toppling for 10cm
3mg r mgsin
limiting conditions
N . (5) = f . (7.5) 
2m a m
2m A
mg cos (5) = mg sin  (7.5)
2mg T
T 2mg T mg 5  10 2
m B tan     33.69º
75 3
when string cut T = 0
 ma2 = mg
a2 = g 9. B
3mg – 2 mg = 2ma
a = g/2 10. A

4. 10 m/s2 11. 5N
4
 mg sin+ mg cos = 3
m
g 15 (mg sin – mg cos)
15

sin = cos at 45°


×2
/5

1 × 25 1 +  = 3 (1 – )
=6

37°
20
4 = 2  = 0.5
N=5 = 5

37° 12. A,C


20 – 6 – 4 = 1 × a Components of 1N force : 1cos  along the
a = 10 m/s2 incline opposite to mgsin  and 1sin 
perpendicular to the incline.
5. B
If = 45 the cos= sin.
F If > 45 then cos< sin so frictional force
a acts towards Q.
T T If < 45 then cos> sin so frictional force
 
x acts towards P.
T

394-Rajeev Gandhi Nagar, Kota


NLM & FRICTION - 105

13. D 15. D
There will be no slipping if friction
l
balances the net force acting downwards mg cos 60  R1 l  x 
2
along the incline
fsmax =  m2 g cos  l 1
 mg ·  R1 l  x 
2 2
Force i n the downward d i rec ti on i s
(m1 + m2) g sin  thus angle at which slipping mgl 2 
  l  x
starts 4 3
(m1 + m2) g sin  =  m2 g cos 

0.3  2
tan  = = 0.2 R1
R1sin30°
3
given tan 11.5° = 0.2 R1cos30° 30° x
Thus  = 11.5° l
2
x

Thus for angles less than 11.5° there won’t °


30
be any slipping hence friction is static and R2
l/2 h
equal to (M1 + M2) g sin  for greater than mg
11.5° the friction is dynamic and is equal to
 m2 g cos . f

14. C, D
 3l  8l  8x

5l
 x
8

h
cos 30 
lx

3 h
 h 3 3
2 5l  
l l 16
8
R1 = R2 (given)
N1 = 2N2
R1cos30°=f
N2 = mg + 1N1
By torque balance: R1
 R 2  mg
2
l
mg cos  = (1N1 cos  + N1 sin ) l
2 3R1
 mg
2
mg
cos  = 1N1 cos  + N1 sin 
2 2
R1  mg  R 2
1 = 0 3

mg
cos  = N1 sin  16
2 f = R1cos30° f 
3
mg
 N1 tan  = Note : Considering the reaction force
2
to be Normal reaction only.
2 = 0
 N1 = 0

www.motion.ac.in
106 - Solution Physics (XI) Topic

Circular Motion
OP  PQ  QO
1. C  km min–1
10
i = 0 ; f = 80 rad/sec
t = 5 sec 
1 1 1
 2 km min-1 = 3(+4) km/hr
80 10
f = i + t   = = 16 rad/sec2
5

1 2 6. C
= t = 200 rad
2
According to cosine formula

2. C r 2  r 2  x2
cos 60°=
2r 2
Given, 0 = 0, t = 2 sec. B
 = 0, next 2 sec.,  = O2 2r2 cos 60° = 2r2 – x2
r x
x2 = 2r2 – 2r2 cos 60° 60°
1 2 1 A
1 =  t   22  2 O r
= 2r2 [1–cos 60°]
2 2
= 2r2 [2 sin2 30°] = r2
1 2 1 x =r
2 =  2  2   22  6 
2 2
Displacement AB = x = r
2 6
 3
1 2 7. B
Here, r = 12 cm

3. D 7
Frequency v = Hz
Given 100

a = 10m/sec2    5rad / sec2 The angular speed of the insect is

a=r 7
 = 2v = 2× =0.44 rads-1
100
10
r =2m The linear speed of the insect is
5
v = r = 0.44×12 = 5.3 cms-1

4. A
8. B
Angul ar ve l oci ty i s al ways d i rec te d
In time t particle has Q
perpendicular to the plane of the circular t=t
path. Hence, required change in angle = 0° rotated an angle a
t=0
 = t. Displacement O R P
5. D
s = PQ = QR   PR 2
Net displacement of the cyclist = zero
Since the initial position coincides with the s= (a sin t2 )  (a  a cos t)2
final position.
Total distance travelled t
Average speed of the cyclist=
Total time taken
s = 2a sin
2

394-Rajeev Gandhi Nagar, Kota


CIRCULAR MOTION & WPE - 107

9. A 12. D
Particle is moving in a circle of radius 'a'
and center (a,0) with constant angular
velocity ''. At time t = 0 particle is at origin
and it s tart s
y
rotating clockwise. t=t
At ti me t i t has
a
rotated an angle  t=0 xy x
(a,0)
given by :
 = t
y = a sin  = a sin t 30
cos 30 
and x = a – a cos  = a (1–cos t) d

r = a (1–cos t) + a sin t ĵ
v 20 3  3
  = 1rad/s
R 60
10. A
13. B
dx Angular velocity of particle P about point A.
x = 2t  vx = =2
dt
v v
Y = 2t2 A  
rAB 2r
V
dy A n g u l a r
 vy = = 4t
dt ve l oci ty of
C r B
particle P about A
2r P
vy
4t point C.
 tan  = v  2 =2t
x
v v
Differentiating with respect to time we get, C  
rBC r

d
(sec2) =2 A v / 2r 1
2  
Ratio A v /r 2
d d
or (1+tan2) = 2; or (1+4r2) =2
2 dt
14. B
d 2 d In a uniform circular v
or  ; at t = 2 s is
2
dt 1  4t dt moti on, the
a v
ac ce l e rati on is
d 2 2 disrected towards
r
 
dt 1  4(2)2
17
rad/s t he c entre whi l e O r a
velocity is acting
tangentially.
11. D

v Re lative 7 15. C


PQ    0.7rad / s
R 10 r = 25 × 10-2 m. f = 2/sec
 = 2f = 4 rad/sec
Acceleration = 2r = (4)2 × 25 × 10-2
= 16 × 25 × 10-2 2 m/s2
= 42 m/s2

www.motion.ac.in
108 - Solution Physics (XI) Topic

16. D 21. B
For a particle performing uniform circular Given  = 1m
motion. magnitude of the acceleration u = 4 m/s
remains constant.
Tension provides necessary
l
5
centripeters force so
17. C
0.5 Kg.
Tangential accelerations, at = r = 4 m/s2 mv2 0.5  42
T  m2l    8N
Radial acceleration l 1 5N

v2 60  60
at = 2r =   3m / s2 22. A
r 1200
In I case mg = m2R ....(1)
Hence, resultant acceleration of the car
In I case mg = m(2)2R’ ....(2)
2 2 2 2
a  a a  4 3
t r
= 5 m/s
2
From (1) & (2) m2R = m42R’

R
R’ =
18. B 4
Net acceleration : Given R = 40 cm, R’ = 10 cm

2
 v2 
a a a 2 2  2
   at 23. B
c t R
 

As v increases, a also increases. T  2
g
So size of arrow should be increasing and
angle between velocity and acceleration
T 1 
should be acute.  = 0.5%
T 2 

19. D
24. C
Given
m1 R 1
a = 10m/sec2    5rad / sec2 Given  1; 1 
m2 R2 2
a=r
If centripetal force is same
10
r =2m
5 m1v 12 m 2 v 22 v1 R1 1
   
R1 R2 v2 R2 2

20. A
25. D
constant speed and variable velocity

26. C
When s tri ng breaks , onl y t ange nt i al
component of acceleration will survive.
Acceleration vector's Hence, path followed is tangential to circular
path.

394-Rajeev Gandhi Nagar, Kota


CIRCULAR MOTION & WPE - 109

27. B 30. C
We know the Tension provides necessary As car is moving in anticlockwise direction
centripetal force and have tangential acceleration as well as
So T = m2 radial acceleration.
 Friction component should be along
19 tangential and radial acceleration.
Given m = 0.1,  = 2 

=1  T = m2 31. C

2
 10  F f 2  (mg)2
T = 0.1 ×  2  1
  
Now when the angular speed of the rod is
increasing at const. rate the resultant force
2 100 
= 0.1 × 4   1  40 N will be more inclined towards f .
2

28. A f(f  m  2r)

The situation is shown in figure. For the


massm to be stationary, the tension in the
string should provide the necessary F

centripetal force on
M
the rotating mass M.
T mg
Now
T = mg and T = ml2 
Hence the angle between F and horizontal
T
or mg = m l2 plane decreases so as with the rod.
m
ml2
or m mg 32. C
g
KE = Ks2

29. C 1
mv2  ks2
2
The different forces are shown in figure.
From figure
mv2 2ks2
Fc  
mv2 R R
T sin  = = m 2r = m2l sin  (i)
r
2ks2
and T cos  = mg v2 
m
From equation (i)
T = m2l = m2 ( l = 1 m) 2k
v s
From equation (ii), m
m2 cos  = mg
dv 2k
aT   v
g g 10 dt m
 cos  = 2  
 42n2 42 (5 / 2 )

or cos  = 0.5 i.e.,  = 60° 2k 2k


aT =  s
m m

aT  2k  s

www.motion.ac.in
110 - Solution Physics (XI) Topic

FT = mat = 2ks 36. C

2 2 v= Rg tan 
2
 2ks2  s
Fnet  (2ks)     2ks 1   
 R  R  R = 10 3 m,  = 30°

1
33. C = 10 3  10  = 10 m/sec = 36 km/hr
3

37. D
To balance the torque of the centripetal force
he bend inwards.

38. A

v = Rgtan (when no friction)

 If v >
 Rgtan
R
Then friction is in downward direction
 628
R   2  100 If v < Rgtan
 314
Then friction is in upward direction
mv2 1000  16  16
F  =640 N
R 400
39. A
fs = fmaC
34. C
Given that mv2
µmg = 
v = 72 km/h., r = 80 m r

We know that
v rg

v2 20  20 1
tan    
rg 80  10 2 40. B
F = (mg)
1
  tan1   Centripetal force F = mv2/r
2
 mg = (mv2/r) or r = v2/g

(12)2
35. C or r  36m
0.4  10
Car will not slip when moving with speed v

394-Rajeev Gandhi Nagar, Kota


CIRCULAR MOTION & WPE - 111

WORK, POWER, ENERGY 47. B

 (î  ĵ  k̂)
41. C
F = 30 3
When a man tr4ies to displace a wall by
applying a force and it does not move, then 
  r = 2 î  k̂
the work done is zero because S = 0.
  
w  F. r  30 3 J

42. C
Whe a weightlifter lifts a weight by height 48. C
h(say), then Work done by the lifting force
F: W  Fx dx   Fy dy   ydx   xdy
W1 = Fs cos 0° = +Fs
But work done in holding it up is zero Also, x2 + y2 = a2 (eq. of circle)
because the displacement is zero. 0 a
W  a2  x2 dx   a2  y2 dy
43. C a 0

Tension in the string is along the radius of Integrating by proper subsitution, we get
circular path adopted by the bob, while
displacement of the bob i s along the a2
W J
circumference of the path; hence angle 2
 
between F and s is always 90° and so W=0.
49. C
44. C Given F = 5N, d = 10 m,
In circular motion centripetal force is always we know w = Fd cos 
p erpe nd i c ul ar to i nst antane ous 25 = 5 × 10 cos 
displacement, whereas tangential force is
 = 60º
along the displacement.

50. B
45. B
w = mgh,
Work done = area under F-x graph
5 3
cos  = 4/5
= are a of rec tangl e ABCD + area of
rectangle LCEF + area of rectangle GFIH + = 10 × 9.8 × 3
area of triangle IJK = 294 joule 4

= (2-1)×(10-0)+(3-2)(5-0)+(4-2)(-5-0)
51. C
1
+ (5-4)(10-0) = 15 J
2 mg
w=
2n2
46. A n=6
Work done = Area under F-x graph with
mg
proper algebraic sign =
72
1 1
= ×20×4– ×20×4=0J
2 2

www.motion.ac.in
112 - Solution Physics (XI) Topic

52. B 57. B
f = k N (Tangentially)
1
 W = – 2r k N kEi  mv2
2
–ve sign indicate that f & ds is opposite
1 2
kEf  m v  2
2
53. A
Given kEf = 2kEi
1
w = k (x22 – x12)
2 1 2 1
m  v  2   2  mv2
2 2
1
= 10 (62 – 42) = 100 N cm = 1 joule
2 v2  2v

54. B
v  
2 1  2

Displacement of block relative to wedge,


2 2 1
v  2 2 2
1 2 1 2 1
S  0  (g sin37)t2
2

1 3 58. B
 10   (4) = 12 m
2 5 Work done by conservative force = -U

Relative to ground
 59. D
S  (12 cos 37  20)iˆ  12 sin37ˆj
1 20 2000
Force, N = mgcos37° K(0.3) 2  10  K 
2 0.09 9
N = 80 N
 1 2000
N  48iˆ  64jN
ˆ work done = . [(0.45)2  (0.3) 2 ] = 12.5 J
2 9
Now, work done
 
W  N.S = 48(12c os 37° + 20) – 64 60. D

(12sin37°) 2 K.Eman = K.E.boy

= 960 J 1 1 M 2
2
2  M  v man  . v boy
2 2 2
55. D
v boy
Work done by friction Vman = ...(i)
2
may be (-), (+) or zero
1 1 M
 M(vman  1)2  . v 2boy
56. B 2 2 2

1 V 2boy
Work done = k [(3x)2 – (2x)2] = 2.5 kx2  (v man  1)2 
2 2

 v ma n  ( 2  1)m / sec

394-Rajeev Gandhi Nagar, Kota


CIRCULAR MOTION & WPE - 113

61. A
2gR
v2 =
1 5
mv2  4t2
2
O'
4g
8 a = gcos37º=
v t 5 R
53º

37
m

º
v2 GPE=0
dv 8 R v
m m RC = = A
a1 2 B
dt m O

F  8m  cos tan t
66. B
F  dr
62. A
 W.D. = 0
 du du du Force and displacement are perpendicular
F î  ĵ  k̂
dx dy dz to each other.


F  U [U = sin (x + y)] 67. B

 cos( x  y) î  cos( x  y ) ĵ

  
F( 0, / 4 )  cos î  cos ĵ
4 4

 2
 1   1 
2

| F |     1
 2  2

63. A Mg
Ui = M/2 g /2 + M/2 g /2 =
U = x2 – 3x, x = 0, x = 2 2

(Ui)x=0 = 0, (Uf)x=2 = 4 – 6 = – 2 Uf = 0
k = – U = 2 joule wc = K

Mg
64. B – 0 = 1/2 MV2 = 0
2
By conceptual
V = g m/s

65. C
Ufinal = – (mgR cos 37º – mgR cos 53º) 68. B
along the string
mgR
= F=T
5
In scond case given
1 (K = 20 J)
kfinal = mv
v2
2
wmg + wT = K
From energy conservation
wmg + wT = 20 ĵ
1 mgR
mv
v2 = wT = 20 – wmg
2 5

www.motion.ac.in
114 - Solution Physics (XI) Topic

69. D
k2
(W.D)by friction + (W.D) by spring a
2
= k = kf – ki=0–ki
mk2
1 F  ma 
– 0.25 × 1 × 10 × 4 – × 2.75 ×4 2 2
2
(Distanced covered in t seconds)
1 2
= –  1 v t
2 ds
x 1 t2k 2
v = 8 m/s dt
 v, 
0 v
.ds   dt  x 
0
4

70. C x t2k 2
4
mk 2 mk 4t2
kx = mg 
dw  F.ds
0

 2
.ds =
8
0

mg
x=
k
73. A
1 2
kx  mgx
2

M
m
2

71. D

By energy conservation,

1 1 x
kx02  k(x0  x)2  Mg  
2 2 2

Solving, we get

Mg
Motion will not SHM x  x0
2k
Motion will be periodic
F = –kx
74. B
another case
kx = mg sin 
K=10N/M

mg sin 
x= KX v=?
k 1kg T

T T

72. A
mg A

2T x/2
v k s
Fsp
Mg
dv k ds
 .
dt 2 s dt

394-Rajeev Gandhi Nagar, Kota


CIRCULAR MOTION & WPE - 115

K = 10 N/m Wg = mgh = 10-3 × 10 × 103 = 10 J


2T + Fs = Mg According to work-energy theorem
2T + Kx/2 = Mg ....(1) K = Wg + Wr
Kx = T ....(2) Where is the work done by the resistive force
Putting (1) in (2) on the raindrop?
Wr = K – Wg = 1.25 J – 10 J = – 8.75 J
Mg  Kx / 2
T=
2
77. C
2Mg  Kx
KX = mv 2
4 mg <
R
5KX = 2 × 1 × 10
5 × 10x = 2 × 10
78. B
x = 2/5 = 0.4 m
1 2
mgh 
2
m  5gD / 2 
75. B
If the body strikes the sand floor with a
1 5gD
mgh  m
1 2 2
velocity V, then mgh = mV2
2
5D
With this velocity V, when body passes h
4
through the sand floor it comes to rest after
traveling a distance x. Let F be the resisting
force acting on the body. Net force in 79. B
downward direction = Mg – F Now,
We know that work done by all the forces is
equal to change in KE :

1
(Mg–F) x = 0 – MV2
2
or (MG–F) x = –Mgh or Fx= Mgh + Mgx

 h
or F = Mg 1  
 x

76. D
Here, m = 1 g = 10-3 kg N a
 m 2 ... (i)
h = 1 kg = 1000 m = 10 m 3 2 2
The change in kinetic energy of the drop is
3N
&  mg ... (ii)
1 2
K = mv
v2 – 0 ( u = 0)
2
From (i) & (ii)
1 2g
= ×10-3×50×50= 1.25 J 2g
2 3  2 
a2 3a
The work done by the gravitational force is

www.motion.ac.in
116 - Solution Physics (XI) Topic

80. B 83. A
For light rod
vtop = 0
Using energy conservation

1
v2 + 0 = 0 + mg
mv
2

v= 2g

FH = mg + T2
81. C
FL = mg– T1

84. B

mv2
T  mg cos gq 

mv2
mg(1  cos ) 

v '2  2g(1  cos )


1 v20 1
M  Mv20  Mgl  Mg2 1 1
2 4 2 mgl  mv '2  mg(l  s cos )
2 2
1 MV02
MV02 + = 3 Mg 1 1
2 8 mg  mg(1  cos )  mg(1  cos )
2 2

12g 1 3 1
v0   1  cos    1  cos 
5 2 2 3

1 2
82. C cos   1  
3 3
mv2
Mgsin = 2
 0  cos1  
3
v2   gsin 
 2  1  g
v '2  gl 1   v '2  g   
1 7g  3 2 3
m
2 2

g
1 v' 
 mv2  mg(   sin ) 3
2

1
sin   ,   30
2
Total angle = 90 + 30 = 120°

394-Rajeev Gandhi Nagar, Kota


CIRCULAR MOTION & WPE - 117

85. D 89. B

ac  k2rt2 P = Fv

v2
 k 2rt2 v  krt Ft = mkr
r

2

Fnet  (mk 2rt2 )2  2mk2r2t2 

Tmax mg  mv12



Tmin mv22  mg

4 v12  g 1 1
 , mv12  mg2  mv22
1 v22  g 2 2

4v22  v12  5g v12  4g  v22 P = Fv cos = mk2r2 t

4v22  (4g  v22 )5g 90. D

3v22  9g v2  3  10  10  10m / s Fv

k.d1 k.d2
F1 = F2 
86. A t1 , t2

mv2 w F.d
Mg  N  ,N  0 Power =   F.v
R t t

V= Rg P  v2

P2  kv22
87. C
Given m = 12000 kg, v = 4 m/sec & t = 40 P1 v2
 12
sec P2 v2

1 1 v22
mv2  12000  42 P2  .P1
Pavg =2 = 2 v12
t 40
P2 = 96 h.p
= 2400 W = 2.4 kW

88. C

P = F.V = (R + ma) V

www.motion.ac.in
118 - Solution Physics (XI) Topic

CIRCULAR MOTION 6. A
1. B w = w0 + t
By theory
210 7
2 × = 0 +  × 5  a =  = 1.4
60 5
2. B
Average velocity is defined as the ratio of
di spl acement to ti me taken whil e the 7. D
average speed of a particle in a given interval w = w0 + t
of time is defined as the ratio of distance 100 =10 +  × 15   = 6
travelled to the time taken.
On a circular path in completing one turn, 8. A
t he d i s tanc e trav el l e d i s 2r w hi l e
angular velocity of earth about axis rotation
displacement is zero.
2
displacement is w = rad/sec
Hence, average velocity = 60  60  24
time interval

0 9. C
 0
t
2
wM = .....(1)
Distance 60  60
Average speed =
time interval
2
2r 2  3.14  100 wH = .....(2)
   10 m/s 60  60  12
t 62.8
equation (1) and (2)

3. A wM 12
=
Angular velocity about center = 2 (angular wH 1
ve l oci ty about any p oi nt on i t s
circumference)
10. A
or  = 2 (2) rad/s

or  = 4 rad/s a  6iˆ  8j
ˆ
now v = r  = (1)(4) = 4 m/s  ar = 8 and at = 6
randr

4. B
11. A
From observer point of
at v = r
view a t decreases v a If r  r/2
and an makes the path
of object curved an v 20
turning to the right.  v = = = 10 cm/sec
2 2
v
Turn table rotating uniformly at = 0

5. C v2 v '2 20
ar  ; a 'r  = =10 cm/s2
w = w0 t t w2 = w02 + 2 R R/2 2
20 = 0 t × 5 20 5 20 = 0+2×4×
 = 4 rad/sec2  = 50 = 2r
n = 25/

394-Rajeev Gandhi Nagar, Kota


CIRCULAR MOTION & WPE - 119

12. A
 7 
40  30   t
V wall  4 
=  
R v

10  4 40
v sin  3m R v sin  t  sec.
  7 7
3 / sin 

v sin2  16. D
   v =at a = 0.5 m/s2
3
v = 0.5t
v = 0.6 m/s
(Given = 45°  = 0.1 rad/s) 
aT = R  
5
13. C
aT = 0.5 m/s2
at  3 t
0.5  0.5 2
v2/R ac = t
R
 dV   3tdt 30°
 1 0.5 2
  t
3t 2 at 5 2 R
v
2
ar  0.25  0.4  0.8m / s2
3 t.R 1 4t
tan 30  2
 
 3t  2 3 3t 4
  17. A
 2 
 
 t = 4t  t = (2)2
4 3

 t = 22/3 sec

14. B
Given v  a s We wi l l s ol ve i t by usi ng hori zontal
component of velocity.
vdv a a2 (VH)Q = V (constant)
at   a s. 
ds 2 s 2 & (VH)P increases from initial value v and
then decreases to final value v.
at
v 2 a2s so, tP < tQ (as horizontal displacement of
ar  
R R both is same).

ar 18. D
a 2s
tan   r  Given : T
at R
R mv2/R
MV 2
T m = 16 kg
R R = 144 m
15. A
2 = 02 + 2 Tmax = 16 N
1600 = 900 + 2(2 × 100)
RT
v max 
700 7 m
 
4  100 4
16  144
Now,  = 0 + t  v max   12 m / s
16

www.motion.ac.in
120 - Solution Physics (XI) Topic

19. D 23. D
T3 T2 T1 r
 R cos 
O A B C 2
r = 2R cos 
v sin 
After differentiable A
T1 = m R = m (3l)
2 2
..(1)  v
T2 = T1 + m2 (2l) ...(2) dr d r R
 2R sin 
T3 = T2 + m2 (l) ...(3) dt dt
 R
T 1 : T2 : T3 = 3 : 5 : 6 O
dr
  v rad  v sin 
dt
20. B
kx = m2 r kx
d
m 2r   (– ve because  decreasing)
kx = m (l + x)
2
dt

m2  x
v sin  = 2R sin 
x l
k  m2 r=l+x v = 2R = 0.4 m/s

a  a 2t  ar2   = constant
21. C
v2
d  a  ar   at = 0
2T sin  Rd2R R
2
If d is small V2
 ar = = 0.32 m/s2
 2R R
d d
sin ~

2 2
d /2
2

d /2 d /2
24. D
d

d T
2T  Rd2R
2 mv02
Fcp   N1
R
T   2R 2
N2 = mg

22. D Ft   N12  N22

m 2
(T+dT) - T = w x dx
 25. C

m 2 vmax = rg = 0.75  60  9.8 =21 m/s


dT = . xdx

Integrate with limit x to  26. A


N = mg v
m 2 N
T 
x

 xdx 
dx 
mv2
=mg
T
r N
T+dT
 x
m 2  x2  gr
T   v=
  2 x
l 
 mg

1 m 2 2 10  2
= [  x2 ] = =10m/s
2  0.2

394-Rajeev Gandhi Nagar, Kota


CIRCULAR MOTION & WPE - 121

27. A WORK, POWER, ENGERGY


The object will slip if centripetal force  force 31. C
of friction  

mr  mg
2 W.D. =  F . ds
r mg
 K [(yiˆ  xj).(dxi
 ˆ ˆ  dyj)]
ˆ
r2  constant

2
 r1   2  
 K ( ydx  xdy )
or   
 r2   1  3,5
K  1,5
d(xy)  20K

2
4 cm  2 
   r2 = 1 cm 32. B
r2   
dw = F.dx
2
28. A  dw = k  x dx

1
v2 v2  x3  k
tan  = . Here = constant w = k 3  =
rg r  0 3

v12 v22 v22 33. D


or  or r2 = r1 ×
r1 r2 v12 w = Nd cos 
= Nd cos 90º = 0
(2v1 )2
or r2 = 50 × =50×4 = 200 m
v12 34. C
 
ˆ .(2iˆ  1j)
W = F . s = (5iˆ  3j) ˆ = 10–3 = 7J
29. C
F.B.D. of the particle N
35. B
in truck travel. mv2
 
R Displacement w.r.t. ground = S + S0
 mg
mv2 mg
Here =mg Since train is moving with constant velocity
R 
net force acting on block is = F .
v2 =  g R = 0.3 × 0.3 × 10 × 27   
v = 9 m/s Therefore work done = F.(S  S0 )

30. B 36. D

v  Rg  F.dx   A(y2ˆi  2x2ˆj).(dxˆi  dyj)


ˆ

= A  (y2 dx + 2x2dy)
w0A = 0 + 0, WAB = A[0+2d2d]
y
C B(d,d)

0 x
A
WBC = A[d2(–d) + 0], WCD = A[0+0]
W = 0 + 2Ad3 – Ad3 + 0 = Ad3

www.motion.ac.in
122 - Solution Physics (XI) Topic

37. C 43. C
For 0 to t 1, slope is constant, speed is
constant and so work done is zero.
For t1 to t2, slope is zero, speed is zero and k m
so work doen is zero. M

For t2 to t3, slope is increasing, speed is
increasing and so work done is positive. –
90°
For t3 to t4, slope is decreasing, speed is
decreasing and so work done is negative. kx cos  = mg sin 
x = m g tan /k kx cos 
38. A
1 2
P.E. = kx kx mg sin 
2
39. B
Maximum velocity will be at Mean Position m 2 g2 tan2 
=
Where Fnet = 0  mg = Kx 2k
1 × 10 = 2 × 100 × x  x = 5 cm
 h = 20 – 5 = 15 cm 44. D
Let m be the mass of the disc. Then
40. D translational kinetic energy of the disc is :

L  1
The weight of hanging part  3  from the KT = mv
v2 ... (1)
  2
table. When it ascends on a smooth track its
As work done = force × distance rotational kinetic energy will remain same
while translational kinetic energy will go
Mg L mgL on decreasing. At highest point.
 W= × =
3 6 18 KT = mgh

1
41. A or mv
v2 = mgh
2
Total energy = E = K.E + P.E.
When speed of the particle is zero. v2 (6)2
i.e., K.E = 0 or h= = = 1.8 m
2g 2  10
 U(x) = E

45. D
42. C
In case of first spring F = k1x1 dU dF
F= – , for min 0
dx dx
F
x1 = K ...(1) and then Ki + Ui = Kf + Uf
1

In case of second spring F = K2 x 2


46. C
F v = at = 10 3 m/s
x2 = K ...(2)
2
In ground frame
 K1 > K2  x 2 > x1 W.D. by gravity + W.D. by normal = k
 More work is done by this force in case of
second spring. 0 + W.D.N = 1/2 × 1 × (10 3 ) 2 = 150J

394-Rajeev Gandhi Nagar, Kota


CIRCULAR MOTION & WPE - 123

47. D
1 1
  mv 2  mga  mga
F S 2 2
v2 = ga
48. A
1 mga
Energy lost by the block due to friction is K.E. = mv 2 
2 2
mgcos where  is length of incline on
increasing  cos decreases hence loss of
energy decreases. 52. A

mv2
49. C N - mg cos  =
R

F 2F 3F  = Angle with vertical

M M M 1m 2/3m 53. A
2m 1m
u2 = 5gR v
 v2 = u2 – 2gR B
Apply work energy theorem
WF + Wmg = K = Kf - Ki (Ki = 0) = 5gR – 2gR = 3gR u
Case I : F(2) – mg × 2 = K.E. Tangential acceleration
Case II : 2F(1) – mg × 1 = K.E. at B is
at = g (downwards)
2 2 Centripetal acceleration at B is
Case III : 3F   – mg ×   = K.E.
3 3
v2
In case III K.E. is maximum. aC = = 3g
R

50. C  Total acceleration will be


Work done by force F a = a 2C  a 2t = g 10
= 100 ×11 ×1/2 = 550 J
Work done by the gravity = mgh
mgh = 550 54. A

550 mv2C
h=  11m NA = mg + mvA2 NC – mg =
5  10 R

mv2B
51. B mg – NB =
R

55. D
N.L. Only Conservative force (mg) is acting So
mg/k
M.P. E.C. is done only for two points (1 and 2)

mg
K= (Given ) 56. B
a B
m N
2
2 mv C
1 1  mg   mg  mg cos  – N = D
 m  v2  k    mg   R 
2 2  k   k 
O mg
2 2 2 2 v2
1 1 mg m g mg N = m(g cos  – )
 m  v2     a2  a R
2 2 a m 2 g2 mg
...(i) A

www.motion.ac.in
124 - Solution Physics (XI) Topic

N=0 59. C

v2 1 2
 cos  = ...(ii) 2MgR = Mv 4Mg
Rg 2

By energy conservation  2 gR  V

1 2
mv  mg(R  Rcos )  v2  2Rg(1 cos ) mv 2
2  mg  N  N = 3 mg Mg N
R
2
Using (i) & (ii) cos  =
3 60. B
height from highest Point = BD = R (1–cos ) On comparing
FV
 2 R
h = R 1   = Ans. F = kV
 3 3
P = F.V = kV2  Now 2P = KV2
2 × kv2 = kV2  V2 = 2V2
57. C
V '  2V
5Rg  5  2.5  10  5 5  10 m / s
61. C
 N2 will be zero in part A, D, C
 
at some point P  F.v

58. A 62. A
Power = Energy/time = mgh/t
 T
(v)

Mg Mv2/R

Mv 2 1
T=  Mg cos   MgR cos  = Mv 2
R 2

1 2 2Mgh  Mgh
 Mgh = Mv  T 
2 R
Straight line

394-Rajeev Gandhi Nagar, Kota


CIRCULAR MOTION & WPE - 125

CIRCULAR MOTION Extension in the spring is


1. A,C
(A) magnitude of velocity is changing hence 3R R
x=R– =
acceleration is present 4 4
(C) Velocity is changing hence it can happen Spring force (F)
by change in direction also as in a uniform
circular motion. Hence acceleration is mg R mg
= Kx = . =
present. R 4 4
So choice (A) is correct.
2. A,B,C,D
Normal reaction N + Fcos60°
By theory
= mg cos60°
3. B,D N = 3 mg/8
As v = 2t and let radius of circular path is r So, choice (C) is correct.
then,

dv v2 4t 2 8. B,C,D
aT = =2 ar = =
dt r r T1 sin 30° = T2 sin 30° + mg
Therefore, ... (1)
(T1 + T2) cos 30° = m2 ( cos 30°)
16 t 4
a= a T2  a 2r a= 4
r2 T1
T1sin30º
T1cos30º
 (B) and (D) are correct.
T2cos30º
T2sin30º
T2
4. A,B

dv v2  T1 + T2 = m2
at  ocp   
dt R T1 – T2 = 2mg
5. B,D ______________________
(B) There are other forces on the particle 2T2 = m – 2mg
(D) The resultant of the other forces varies for T2  0 m2 – 2mg  0
in magnitude as well as in direction.
2g
6. A,C,D 

By theory

7. A,B,C
9. B,D
Particle takes speed
tangentially and act as
a 'Projectile' (curved
path)

10. B,D

v  Rg

www.motion.ac.in
126 - Solution Physics (XI) Topic

WORK, POWER, ENGERGY Etotal = P.E = 3 × 6 + 4 × 4 = 34 J


(–3) (6,4)
U ˆ U ˆ
F = y– i  j –4
11. A,C,D y
(A) F  V
= – 3 î  4 ĵ
(C) Object is at Rest But point of application
of the force moves on the object. a  –3 î – 4 ĵ  | a | 5 m / s 2
(D) The object moves in such a way that Let us assume particle crosses y axis after
point of application of the force remains time t
fixed.
1
x–6= –  3  t2
2
12. A,B,C
at y axis
Conceptual  x=0  t = 2 sec
1
So y–4= –  4  (2)2  8
13. A,B,C,D 2
2g – T = 2a y = – 4m
(P.E.) at y = – 4 and x = 0
 is U(y = –4, x = 0) = – 16 J
2T – 1g = 1  
2 So. K.E. = T.E. – U
1
MV 2  34  ( 16)  50
14. A,B 2
At maximum extension K.E. = 0 V2 = 100  V = 10 m/s

20. A,B,D
15. A,B,C (A) Velocity is not constant
(B) A net contrepetal force atc on in towards
mg 
wmg = mg (l-l cos 37°) = centre
5
(D) A centrepetal acceleration is produced
wT = T.d cos 90° = 0 on it
W centrepetal force = 0
21. A,B,C,D
By theory
16. C,D
Work done by gravity is independent of path 22. A,C,D
by theory
1 1
w mv2   2  16  16 Joule
2 2
17. A,B D
U
1
U  Kx2
18. A,B 2 2
x
(A) The spring initially compressed and A
finally in its N.L. V
u=0 v2
(B) Initially stretched and then in its N.L. a
s 2s

19. A,B,C 2v2  2a d


2v a
u=0 2v2
Given U = 3x + 4y 2v2  d
d
Initially particle at rest at (6,4) 2s

So K.E = 0 d = 2s

394-Rajeev Gandhi Nagar, Kota


CIRCULAR MOTION & WPE - 127

23. A,B,C At D :
Conceptual
mv 2
ND = mg +
rD
24. B,C,D
From figure rB < rD
mg
M.P. x1 = N.L. hence NB > ND
m
k
Kx1 Hence NB is greatest
But block further M.P. m
rC < rA
move downward mg
N C < NA
due to inertia. So m
Hence NC is least
descending through distance
At A & C ; NA < mg
2mg NC < mg
x
k At B & D ; NB > mg
kx
ND > mg
x
at M.P. at  Fnet = 0 ; a
2 m 27. B,C
so a = 0 As T = mg
at lower most point mg T v
mv 2
B  
 mg – mg cos  = A
 2mg  r u
k   mg  ma  a=g
 k  v2 = gr (1 – cos ) ….(1)
Also v2 = u2 – 2gr (1 – cos ) ….(2)
25. A,B,D  3gr (1 – cos ) = u2
T = F = 40
From W - E theorem,
u2
Wnet = (KE) or cos  = 1 –
2 kg 3gr
(40 - 20)S = 40
S = 20m  At A and B tension are same to mg
Wg = –20 × 2 = – 40 J20 N and speed are also equal.
WT = 40 × 2 = 80 J
28. A,B,C,D
26. A,B,C
A
At A :
NA W = mgh
mv 2 = 1 × 1 = 1 Joule
A
mg – NA = B
rA
mg 1
K.E.  mv2
mv 2 2
NA = mg –
rA
1 2

At B : 2
1  2v  1 J
NB
mv2 C
NB – mg = B P = F.V.
rB
D
mg
w = Fd cos (180°)
mv 2
NB = mg + w = -Fd work is (-ve)
rB

At C :

mv 2
NC = mg –
rC

www.motion.ac.in
128 - Solution Physics (XI) Topic

CIRCULAR MOTION
1  2
1. 9 =0+  t  t = 2 sec
2 2
d
=
dt
=  d =  dt 
from v = u + at  v = .2 = 3.14 m/s
2

12 12
0
4.  (1 - e-k t)
37º 53º t k
(0,0)
16sec 25sec
 = 0 –k t = 0  = 0
41sec
50sec t = 0

 =  dt = Area under (-t) d


 0  k
dt
3
 = tan37º =  1d t
4 dt
0 (0  k)


0

3
t = t = × 16 = 12
4  1   0  k 
 k  log   t
   0 
4
 = tan53º =
3
0  k  0ekt
4
12 = t  t = 9 sec 0
3  (1  ek t)
k
1 1
total = × 16 × 12 + 25 × 12 + × 12 × 9
2 2
5R 11π 17v 289v 2
= 96 + 300 + 54 = 300 + 150 5. (i) sec, (ii) (iii) (iv)
6v 6 5R 25R 2
total = 450 rad

450
av = rad/s
50 v2
6. 2 2
R

2. (a) 4.0 cm/s2 (b) 2.0 cm/s2


7. 2
(c) 20 cm/s2
wf2 = wi2 = 2

3. (a) t = 2s, 1m
8. 2 sec
(b) 3.14 m/s (1,0)
v2
 aN 
at = m / s
2 r
2
at  5cm / s2
1 2
 from S = ut + at
2
394-Rajeev Gandhi Nagar, Kota
CIRCULAR MOTION & WPE - 129

and T cos  = mg
v2
Thus 5 
20 mg
T=
cos 
v = 10 cm/s
If u = 0, a = 5 cm/s, v=10 cm/s then
g g
10 = 0 + 5 × t Now,  2l  
cos  l cos 
10
t=  2 sec.
5 2  cos 
T= = 2 g

9. 2g rad/s Tconical Pendulum  g 1


 T  2 cos   
T = 2mg simple Pendulum g  2
T = m2(1)
Ratio = cos 
2mg = m2

  2g rad/sec
12. 10–2 m

kx  k2(  x)
30 35
10. (a) per min., (b) 150 N 104 x  90  103  104 (0.1  x)
 2
x = 0.09(0.1 +x)
T1 sin  x = 0.09 ×0.1 + 0.09x
5
4 T1 x(1–0.09) = 0.09×0.1

m 2r
T1 cos  T cos mg x(0.91) = 0.09×0.1
2

T2 x = 9.89×10–3
T2 sin  x = 10–2 m

 T1 cos  + T2 cos  = m 2 r ...(1)


T1 sin  = mg + T2 sin  ...(2) 2
13. (i) 5 m/s2 (ii) km
3 3

11. cos  1
aT = g cons60 = 10 × = 5 m/s2
2
T cos 
v2
an = = gsin60
R

T sin  m2  sin  50 = vcos30

mg
sec
50 3m / sec m/
100 60º
 v= 30º
T for simple pendulum = 2 60º an g 60
aT
º
g
t=0 50m/sec

For conical pendulum


T sin  = m 2 l sin  50 100
v= ×2=
 T = m2 l 3 3

www.motion.ac.in
130 - Solution Physics (XI) Topic

17. 3J
v2 100  100  2
R= =
g sin 60 3  10  3 t2 t
s=  ds = dt
4 2
2000 2
= m = km
3 3 3 3 md2s 6d2  t2 
F = ma = dt2  dt2    3N
4

14. 5 Now

v2 5 5 2 2 t
2
3  t2 
R= = =5m W  F ds =  3 dt   
g 0.5  10 0 0 2 2  2 0

3
 [(2)2  (0)2 ]  3J
2g 4
15. 2 =
a 3

a 18. (a) Zero, (b) Zero, (c) –mgvt (d) mgvt


As ; cos  =
2a (a) Net force on Block is zero
F
 = 60° V= constant

 N sin 60° = mg F = f = mg f


so, work done by force.
2a
N cos 60° = m  
2
 F . dr  0
2a 2g
 tan 60° =  2 = r
2a a 3
(b) W  N.dr 
0

dr  N
WORK, POWER, ENGERGY N.dr = N dr cos 90° = 0
W=0
16. 20 Ergs (c) W = f.r
= f r cos 180° = – fr
20 = –  mg(vt) = –  mg vt
(Force dyne)

10 
(d) work done by F = F.r = Fr
0
1 2 3 4 5 6 = ( mg) (vt) =  mg vt
–10 x(cm)
–20

W=10×(2–1)+20(3–2)+ 8gh
19.
15
(–20)(4–3)+10×(5–4)
= 20 dyne cm = 20 ergs When block B moves up by 1 cm, block A
moves down by 2 cm and the separation
kg m2 1 becomes 3 cm. We then choose the final
 20   = 0.2 × 10–5 J
sec 2 10 3  10 4
h
point to be when B has moved up by and
3

394-Rajeev Gandhi Nagar, Kota


CIRCULAR MOTION & WPE - 131

24. 3
v 2h
has speed A . Then A has moved down Kx = P
2 3 l1
and has speed vA : P P P
1 = K= 
K + U = 0 K 1

(KA + KB + Ug)f – (KA + KB + Ug)i = 0


1 2 1 2
(KA + KB + Ug)i = (KA + KB + Ug)f P(  1   2 )  k 2  k  1
2 2
2
1 2 1  vA  mgh mg2h 1
0+0+0= mv A  m    P( 1   2 )  k( 2   1 )( 2  1 )
2 2  2  3 3 2

mgh 5 8gh 1 1
 mv2A  v A  P = k ( 2   1 )  P= P(  2 /  1  1)
3 8 15 2 2

2
3
20. B 1
Wspring + W100 N = k (on A)
25. 17.32 m/s
 10  1
Wspring + (100)  100   2 (2)(2)2 W = Rx F cos 0°
  60°

R(1  cos )
(by the force)
Wspring = 4 – 10 = – 6 J

= 10 × × 200
3

2m12 g2
21. gRM
k Work done by g = MgR (1 – cos 60°) =
2
k
T 1 = m1 g
gRM
kx = 2m1g K.E. = RF –
2
m1 m2
1 4m12 g2 2 2
= k. = 2 m1 g 1  10  10  10
2 k 2
k MV 2  10   200 –
2 3 2


dU v2  2  200 – 50
 3
22. (a) F=–
dy
V = 17.32 m/s

dU
(b) F = – =–3ay2 + 2by
dy
100 5 3
26. J,
3 8
dU
( c) F = – dy = – U0 cos y
27. 10 J

23. (a) 875 Joule (b) –250 joule u  3iˆ  4j
ˆ m/s
(c) 625 joule.
(d) Change in kinetic energy of the body is 3 dv x 3
ax  t  t
equal to the work done by the net force in 4 dt 4
10 second. This is in accordance with work-
energy theorem vx 4
3
 
3
dv x 
4 
tdt
0

www.motion.ac.in
132 - Solution Physics (XI) Topic

3 16  m 2     sin   
vx + 3 =  vx + 3 = 6 29. gR    
4  2    R   R 

 vx = 3 m/s

3 dv y  3 reference
ay   t 1 

   t  1 R  d
4 dt  4 
O R
vy
3  16  m 
  dv
4
y 
4  2 
4 dUi = –  Rd  × g × R[1 – cos ]
 

vy –4 = –6–4  vy = –6 m/s


mgR 2
dUi =  [1 – cos]d
vy  3iˆ  6j
ˆ m/s 

1 mgR 2     sin  


Work done, W = K = m(45  25) = 10 J..  Ui =   
2   R   R 

and Uf = 0  Wext = U

5
28. gR, xmin  2R
2 30. 2(u 2  gL)

2.2R
x= v 31. 46 J
g
P = 3t2 – 2t + 1
g 1 1 4
vx  mu 2  mg.2R  mv 2 2
4R 2 2 dW =  (3t
2
 2t  1) dt

1 1 (3R) 2 g W.D. = [t3]24 – [t2]24 + [t]24


mu 2  mg.2R  m
2 2 4R
= (64 – 8) – (16 – 4) + 2
1 2 9Rg 5 = 46 J = change in K.E.
u  2gR   u gR
2 8R 2
32. 50 W
For xmin v should be min.
Total work done
 umin = 5gR  v  gR Pav =
total time

2.2 R 100  1 6  9.8


x= gR . = 2R  = 49 w
2 2  60

394-Rajeev Gandhi Nagar, Kota


CIRCULAR MOTION & WPE - 133

CIRCULAR MOTION
v2
1. 80 : 1 a  2 ar = 2.
R
A  t2 B  t
A = k 1 t2 B = k 2 t
5. 40 m / s 2
From given conditon calculate k1 and k2
Given
2 =k1 ×   = k2 × 4
k1 =2 k2 = 1/4 20
R= m ; v = 80 m/sec
A = 2t2 B = t/4 

d A d B 1 v2 = u2 – 2ats
wA=  4t wB = 
dt dt 4 u=0 ; s = 2(2R)

20
 d A   dB  1 (80)2 = 2at (4. )  at = 40 m/s2
   20    
 dt  t 5 sec  dt t  5 sec 4

2. Slope should be decreasing vR R( vt  R)v1/ 2


6. aN  2
, at 
( 2Rt  vt ) (2Rt  vt 2 ) 3 / 2
d t
= = tan, if , 
dt
7. 2
kx = T = k ( 2r – )
3. 3

VBA  d  v2
WBA = 2Tsin   = (dm)
distance between A & B  2  R

 m 0  v2
v2 Td =   r · (2d) ·
4. (i) v = v0e–s/R (ii) a = 2  2r  r
R
(i) At any moment at = ar
 d   d 
T cos  T cos 
2  2   2 
v
at  
R 2/2 2/2
T T
 d 
dv v2 dv 1  d   T sin 
T sin   2 

v     ds  2 
ds R v R
d d
S
After integration log v =  C ...(i)
R
at t = 0, s = 0, v = v0
C = log v0
m 0 v 2
k (2r –) =
 v  S r
from eq. (1) log v 
 R
 0 
kr (2r   )
v = v0 e–S/R  v2 =
m 0
(ii) At any moment at = av

www.motion.ac.in
134 - Solution Physics (XI) Topic

8. 3 WORK, POWER, ENGERGY


12m/s 5m/s 9. 10 J
18m Initial K.E. of the body
Given a = – Kx  1 1
 mv2   25  4  50 J
vdu 2 2
 Kx
dx Work done against resistive force
5 18
= Area between F–x graph
  v dv  K  x dx  119  K(324)
12 0 1
=  4  20  40 J
119 2
K
324 Final K.E. = Initial K.E. – Work done against
Acceleration of particel at point resistive force
119 119 = 50 – 40 = 10 J
A  18 
324 18
119 10. 6
at  F=2+x
18
2 2 x 2
anet  10 m / s 2  a t  aN
 dw   F dx   2  x  dx
x0
2
 119  2
 10     aN
 10   x2 
2

w   2x  
V2  2 0
 aN  7.5 m / s2   7.5
R
(5)2 25  4
 R   R = 3.3 m w   2  2   = 6 Joule
7.5 7.5  2

11. mgR / 2, 2 gR

5 R
Extension is string x  R  2R 
2 2
Now from energy conservation between point
A & B.
2
 3R  1 4mg R 1
mg  .  mv 2
 2  2 R 4 2

5 R A
2  2
9R
2  4 3R
4R 2

G.P.E. = 0
2R B

v  2 gR

Wf  Wmg  K

1 3
Wf  m (4gR )  mgR
2 2

1
 Wf  mg R
2

394-Rajeev Gandhi Nagar, Kota


CIRCULAR MOTION & WPE - 135

12. B
work done by friction is – µmgL 2
1 1  3g 
k (.01  x)2  k    2g (.01  x  3g / k )
2 2  k 
4 2 3 2
13. , 3a1 = 4a2 + 20, T = 12 N
5 5  1000 (.01  x)2 
1  90 
 0.2  20 x 
6  100
2  2000  2000
2mg
(a) T = ky  T   a cos ec
a 25
after solving x2 
T = 2mg cosec  40  1000
At equilibrium x = 2.5 cm
T cos  = mg
2 mg cot  = mg
a 7
cot  = 1/2 15. m
6
x By work energy theorem,
By fig cot  = x
a y  a cos ec
 T 1
mv02  mgh  mgd  0
x 1 1 2
  x=
a 2 2 18 – 11 = 6d
(b) dFTension = Kydy
7
a a
d m
y  2
2mg a 2 6
a2
0

FTension = kydy = k  2  = k
 0 2
=
a

2

FTension = mga C 15 3
Wtotal = KE y 16. N
 2
WTension + Wgravity = KF dy T
mga + mga = 1/2 mv2 mv 2
N  mg cos 60 º ...(1)
2mga = 1/2 mv2 r
O
v = 2 ag Ans. 1
from E.C. mg cos 60º  mv 2
2
1 2 1
For maximum path kx  mv 2
2 2 v2  5 ...(2)

14. 2.5 cm
at equilibrium kx = mg × 2 × 10
N 60º
2  10
k  2000 N / m 60º
.01 Nsin60º

To just lift the 3kg block force on the 3 kg mv 2


mg
r
block is
upward direction kx = 3g  x = 3kg ///////////////////////////

n. 
 from (1) & (2)
2kg .01m 2kg 3g/k
N = 15 N
Now force on the wedge due to wall

K i.e. 3
= N sin 60º  15  N
2
3kg 3kg

from energy conservation

www.motion.ac.in
136 - Solution Physics (XI) Topic

19 r 3 U2
17. (a) 2 2 r (b) h  , (c) g rg cos   rg  0
27 2 18

u u0/3
mv 2
r
r v

s  mg
 co
mg

Particle leave the surface at top when


2u0 19
U  rg put U0  rg  cos  
27

r
Height from the ground at which it leaves
2r
Now T
g 19
the hamisphere = r cos   r
27
2r 2r
R  2 u0  R  2. rg  2 2r
g g 18. 15 KJ
w = mgh
Now when U = U0/3
= vgh = 1000 × 10 × 1.5 = 15 kJ
from energy conservation

1 U20 1
m  mg R (1  cos  )  mv 2 ...(1)
2 9 2

mv 2
force balance  mg cos  ...(2)
R
from equation (1) & (2)

394-Rajeev Gandhi Nagar, Kota


CIRCULAR MOTION & WPE - 137

(Circular Motion) 4. C
1. D As their period of revolation is same, so is
v2
R 2 2 their angular speed. Centripetal acceleration
a1  1
  2R 1
1
is circular path, a = 2r.
R1 R1
v 22 Thus,
a2   2R 2
R2 a1 2r r
a2  21  1
R2 a2  r2 r2
Taking particle of
a1
mass equal R1

F1 ma1 2R 1 R 1 (Work Power, Energy)


  
F2 ma2 2R 2 R 2 5. A
Momentum would be maximum when KE
2. D would be maximum and this is the case
when total elastic PE is converted into KE.
s = t3 + 5
According to conservation of energy,
ds
 Speed, v   3t 2 1 2 1 (Mv2 )
dt kL  Mv2  kL2 
2 2 M
dv
and rate of change of speed, at   6t
or MkL2  p2 (p  Mv)
dt  P  L Mk
 Tangential acceleration at t = 2s,
at = 6 × 2 = 12ms-2 6. A
and at t = 2s, v = 3(2)2 = 12ms-1 According to conservation of energy,
 Centripetal acceleration, 1
mgH  mv2  mgh2
2
v2 144
ac   ms 2
R 20
H= 100 m
h1= 30 m h 2= 20 m

2 2 2
 Net acceleration = a  a  14 ms
t i
1
 mg(H  h2 )  mv2
2
3. C or v  2g(100  20)
For a particle in uniform circular motion, or v  2  10  80  40 m / s
 v 2
a towards centre of circle
R 7. A
y
mv  v  0
ac F  ma  a  
T  T 

ac x Instantaneous poer = FV
= mav

 v2
mv mv v mv 2
= .at = . .t = t
 a ( cos  î  sin ĵ) T T T T2
R

 v2 v2
or a   cos  î  sin ĵ
R R

www.motion.ac.in
138 - Solution Physics (XI) Topic

F
8. D For AB, 0 = v2 - 2×g×2
According to work - energy theorem,  v2 = 2g × 2
W = K From above equations, a=10g
2
1  vo  1 a = 10g = 100ms-2
Case I - F × 3= m   mv20
2  2  2 Then for PA, FBD of
mg
Where F, is resistive force and v0 is initial ball is F - mg = ma
speed. [F is the force exted by hand on ball]
Case II Let, the further distance travelled => F = m(g+a) = 0.2(11g) = 22 N
by the bullet before coming to rest is s.
Alternate using work energy theorem
1
  F (3  s)  K f  Ki   mv20 Wmg +WF = 0
2
1 1 => -mg × 2.2 +F × 0.2 = 0
  mv0 (3  s)   mv20
2

8 2 or F = 22N
1 3 s
or (3  s)  1 or   1 or s = 1 cm
4 4 4
11. A
9. D
 x4 x2 
Here, the constant horizontal force required V(x)    
 4 2 
to take the body from position 1 to position
2 can be calculated by using work-energy dv
For minimum value of V, 0
theorem. Let us assume that body is taken dx
slowly so that its speed does not change, 4x3 2x
  0
then 4 4
K = 0  x  0, x  1
=WF + WMg + Wtension 1 1 1
450 l
So, Vmin (x  1)    J
[symbols have 4 2 4
their usual meanings] Now, Kmax + Vmin = Total mechanical energy
m
1 1
Fl  Kmax     2
WF= F × l sin 45 =0 M F 4
2
9 mv2 9
Mg or Kmax  or 
WMg = Mg (l-l cos 450), 4 2 4
3
Wtension = 0  F = Mg( 2  1) or v ms1
2

10. D 12. A
The situation is shown in WET
figure. At initial time, the B
Wnet = E
ball is at P, then under the
2m –fSx = Ef – Ei
action of a force (exerted
by hand) from P to A and 1 2 1
 15 x  kx  mv2
then from A to B let ac- 2 2
celeration of ball during PA 1 2
–15x – 5000 x2 =   2  4  16
is a ms-2 [assumed to be A 2
constant] in upward direc- 16
x2   x  5.5 cm
tion and velocity of ball at 0.2m 500
A is v m/s.
Then for PA, P
v2 = 02 +2a × 0.2

394-Rajeev Gandhi Nagar, Kota


CIRCULAR MOTION & WPE - 139

13. B 17. 3
Question is somewhat based on approxima- W = 1/2kx2
tions. Let mass of athlete is 65 Kg. if F is constant
Approx velocity from the given data is 10m/s. W  1/k
65  100 If x is fixed w  k
So, KE   3250 J
2
So, option (d) is the most probable answer.
18. A

14. C W=  Fdx
U(x) 
a

b W=  Fdx
(ax + bx ) dx 2

x12 x L
 ax2 bx3 
U (x = ) = 0 W=  2  3 
 0

dU 12a 6b   aL2 bL3 


as, F = – = –  13  7 
dx x x  W=  2  3 
 
at equilibrium, F = 0

2a 19. C
 x6 =
b
m × 3.8 × 10 7 × 0.2 = W
= (10 × 9.8 × 1) × 1000
a b  b2
 Uat equilibrium = 2 – = m = 12.89 × 10 –3 kg
 2a   2a  4a
   
 b  b 
20. B

b2 h
 D = [U (x = ) – Uat equilibrium] = tan30 
4a 

 h 3 2 3 m

15. D Wf = –mg  or Wf = –mg x

dk mg  mgx ; x


Given,  cons tan t
dt
 k t x  2 3m ; Wall = K

 v t mgh – mg  – mgx = 0


dk h    x  0
Also, P = Fv =  cons tan t
dt
1 1 2
 F  F 2     x   
v t x

2 1
16. C  
4 3 2 3

www.motion.ac.in
140 - Solution Physics (XI) Topic

1. C 4. A  s; B  s,q; C  s ; D  q

(A) U = mgh
(reference at
1 lowest point)
mv
v2 = -mg x1
2

U – U0 = mgh
(reference at
any point)

(B) (i) a > 0, v > 0


1
-mg cos x2 = mv2 -(0+mgx2sin) (iii)  q,r (y = y0 + vt)
2
(ii) a < 0, v > 0
1
-mg cosx2 = mv
v2-mgx2 sin  1 2
2
(i)  s  y  y0  vt  2 at 
 
from the 2 equation , it can be seen
that the  se in mechanical energy in the II (iii) a = 0, v > 0

situation is smaller than that in the I


situation.

(ii)
Stat2 : Co-efficient of friction is a property
of the material and is independent of any
angle of inclination.

2. C
From energy conservation,
(C) R  u2
1 2 1
kx   4k  y2
2 2

y 1
 
x 2
 Correct option is (C).
(D) T2  L

3. D
1 1 5g
5mg  m  mg(1  cos )
2 2 4
cos = –7/8 5. 8
Hence, 3/4 <  <  a = g/3, T = 4.8 N, S = 1/2 at2 = 5/3 m
W = TS = 8 (in joule)

394-Rajeev Gandhi Nagar, Kota


CIRCULAR MOTION & WPE - 141

6. C 9. A
V2 – V1 = VRel
4t
F(t) =  4 [From graph]
3   2
 V2 – V1  2R sin
2
F t
a(t) =
m

dv R R
a=
dt
v v
v t
 dv   adt .
o o
= 2 R sin = 2R sin t

1  4 
v= t dt+4dt 
m  3  10. D

1  2 2 
r  OP  xˆi  yj
ˆ
v=  t  4t 
23 
k
tt t= 4.5 sec.
F 3/2  xˆi  yjˆ  rk r 
3
x
2
 y2 
1  2 81 9
v = 2  3 4  4  2
 
P
r
27 y
=  9 =2.25
4 O x

1 2
KE =  2 2.25
2
= 5.06J. Since, F is along r or in radial direction.
Therefore, work done is zero.
7. D
T = m2 11. 5
324 = 0.5  (0.5) 2

1
 = 36 Radian/S W mv2
2

8. 4 1
 Pt  mv2
Applying energy conservation 2

1 2 1 2Pt 2  0.5  5
kx  Nx  mv2  v   5 m/s
2 2 m 0.2

1
  2  (0.06)2  0.1  1.8  0.06
2 12. B
Height fallen up to point Q
2
1  N
  0.18    or N=4 1
2  10  h  R sin30  40   20 m
2

www.motion.ac.in
142 - Solution Physics (XI) Topic

Work done agai nst fri ct i on = Ini t i al 16. 5


mechanical energy – Final mechanical
energy

1
 mgh  mv2
2
putting the values, we get

1
150  1  10  20   1  v2
2
 v = 10 m/s wmg + wF =  KE

1
13. A -1×4×10+18×5 = ×1×v2
2
At point Q, component of weight along PQ

mg V2
(radially outwards) is mg cos 60° or . -40+90 =  KE = 50
2 2

Normal reaction is radially inwards  n = 0005

mg mv2
N 
2 R
17. C
2
mg mv
or N 
2 R

2
1  10 1  10 
   7.5 N
2 40

14. 5
Velocity of first bob at highest point.
2h
d = vt = 2gh = 2h whi ch i s
v1  gR  gl1 g

(to just complete the vertical circle) independent of g.


= velocity of second bob just after elastic But when the lift falls freely no water leaks
collision. out of the jar as geff = 0.
= velocity of second bob at the bottommost
point

 5gl2 18. A(P),(Q),(R),(T) ; B(Q),(S) ;


C(P),(Q),(R),(S) ; D(P),(R),(T)
l1
 5 2
2
l2 
U0 1   x  
(A) U1 =
2   a  

15. D
2
By theory x
Umin at 1 –   =0
 a

394-Rajeev Gandhi Nagar, Kota


CIRCULAR MOTION & WPE - 143

U
U

X
–a +a
U=0 x
x = ± a, F = 0 at x = ± a
–U 0/3
2
X 
U0  
(B) U2 =  A2 
2  

U
19. A
dv
mx2 = mv
dx
2xdx = vdv
x
x=0 v2 = 2x2 + C
R
At x = , v=0
2
2
U0  x  x2 2R 2
(C) U3 =   2 O= +C
2  a ea 4
2R 2
C=–
4
2R 2
 v2 = 2x2 –
4
R2
v =  x2 –
4

dx R2
=  x2 –
 3 dt 4
U0  x  1  x   U
(D) U4 = a 3 a =
2     3
dx
AT x = – a
 x – R2 / 4
2 =  dt 
4 U0 U On solving, we get option (A).
U4 = =– 0
3 2 3
At x = a, 20. B
Net reac n of the disc on the block i s
2 U U obtained by using
U4 = × 0 = 0
3 2 3    
Frot = Fin + 2m( Vrot ×  ) +
1 x3   
– 3 =0 m(  × r ) × 
a a
1
1 x2 as m2R(et – e–t) ĵ + mg k̂
= 3 =x=±a 2
a a

www.motion.ac.in
144 - Solution Physics (XI) Topic

1. C 4. D
Centre of mass of two particle system lies
on the line joining the two particles

2. C
For square plate ABCD centre of mass is at
O but when two point masses of 3 kg placed
 CM will be at line A.
at C & D then centre of mass shifts on the
line OY because centre of mass is in that
part which has higher mass.

3. C
y

 CM will be at line C.
C
O
5. B
(-1,-4) (2,-3)
4kg 2kg

R
Side of square = R cos 45° =
2

(1,6) (-1,5)
R2 5kg 3kg
Area of square =
2
5  1  3  1  2  2  4x  1 1
xcon  
532 4 7
R 2 R R2 R
R     0   (–  )   σ.
X COM  4 2 2 2 6  5  3  5  2  3  4  (4)
R 2 R2 ycon 
2
R   (–  )   14
4 2
30  15  6  16 45  22 23
  
14 14 14
R[2 – ]
=
2[3  2]
6. D
 The centre of mass of the system is
A

R[2 – ] l
at a distance of from the centre O 3l
D 2
2[3  2] B
10

2m 3m
l
towards the plate as shown in the figure.
C

2
 3l  10 9
CD  l2    l
 10  100

394-Rajeev Gandhi Nagar, Kota


Centre of Mass - 145

7. A
5
   xcm  mt,
 m r  m2r2  m3r3 6
rcm  1 1
m1  m2  m3
1 3
2m   m
2 2  5 mt


1  ˆi  4j
ˆk 
ˆ  2  ˆi  ˆj  k

ˆ  3  2iˆ  ˆj  2k
ˆ
   ycm 
3m 6
12 3

9iˆ  3j ˆ 9iˆ  ˆj  k
ˆ  3k ˆ 10. B
  1.5iˆ  0.5j ˆ
ˆ  0.5k
6 2
P2
K
2m
8. C
y
42
From graph, 4 
2m
(-m 1) m2 m3
x
 m = 2 kg
(R/2,0) (0,0) (R/2,0)
 (B)

11. A

2r
 R R m  r
m1      m2  0  m3 
 2 2
xcm  r
m1  m2  m3
2r
2r
R 2 R R 2 R  m  4r

  
xcm  4 2 4 2
R 2 R 2 m  r 0
2
  R  
4 4

r
R R R 
   ,0
8 8  4  2r

m  r
9. A
y 4r  0  r  2r / 
xcom 
1m 3r  4r
 1,3
 
 2 2
(m) 2r2

2m
3r  4r
 ,1
1 
 2 2m
1m

x
2r
(0,0) 1m 1m 
3  4

1
2m  1  m 
xcm  2
3m

www.motion.ac.in
146 - Solution Physics (XI) Topic

12. A 16. C
Centre of mass will not move in horizontal
(-a,a) direction. Let x be the displacement of boat.
m(a,a)
m
80 (8 – x) = 200x
640 – 80x = 200x
6m
20m
(+m) x = 2.3 m
m Now, Required
(0,-a) x
distance from the shore. 8–x
= 20 – (8 – x) = 20 – (8 – 2.3)
= 20 – 5.7 = 14.3 m
ma  ma  ma a
ycom  
10 m 10
17. C
If mass of block A is m and the mass of block
13. B B is nm.
Let x be the displacement of man. Then nmg – T = nma
displacement of plank is L – x. T – mg = ma
For centre of mass to remain stationary After solving T T
M (n  1)g a A B a
(L – x) = M . x M a= m nm
3 n1
M/3
acceleration of the centre
L
 x= L–x x of mass of system.
4
 
m1a1  m2 a2
aCOM =
14. A m1  m2

Fnet = 0 100g 250g 2
ma  nma a  na n 1
A B = = =  g
 (n  1)m
so acom = 0 2
n1 n  1
10cm/sec
 
m1 a1  m2 a2 = 0
18. B
100 × a1 + 250 (–10) = 0
When internal force acts.
a1 = 25 cm/sec2 east
Net force is zero.

15. C dP
 F= So momentum is conserved.
dt
Centre of mass hits the ground at the position
where original projectile would have landed. Therefore internal force will not change the
linear momentum.
But due to force, K.E. increases.
m COM 2m
R/2 x1 19. D
Speed is constant so K.E.  Constant
m.R R Gravitational potential energy change.
= 2mx1  x1 = 
2 4  Momentum = mv

R 5R  Direction of v changes
 Distance = R + =
4 4  Momentum changes

394-Rajeev Gandhi Nagar, Kota


Centre of Mass - 147

20. D 25. A

P2 Fnet = 0
= K.E.
2m 
then p = conserved
P2
ln = ln K.E. p i = pf
2m
m1v = m2(0) + (m1 – m2) v1
2ln P – ln (2m) = ln K.E.
So the graph between lnp & lnk is straight m1v
v1 = m
line with intercept. 1
 m2 

21. D 26. A
The acceleration of both balls = – g As fnet = 0 from momentum conservation

m1(g)  m2 (g) 4v
acom = =–g  A  4 v  4v  v1 
m1  m2 1  A  4

22. A
27. C
According to Newton's second law of motion.
C1 will move but C2 will be stationary with

 dp respect to the ground.
F = dt

If Fnet = 0 28. (a) B
 It could be non-zero, but it must be constant.
then p = conserved (b) C
It could be non-zero and it might not be
23. A constant.

wall 29. C
m v
(Fnet)x = 0
 Momentum component parallel to the track
is conserved.
but in y direction Fnet is not equal to zero.
Initial momentum of body = mv
So momentum is not conserved in y direction.
& final momentum of body = – mv
Change in momentum = 2mv 30. A

24. C Nimp

 Nimp
Fnet = 0 Nimp
Nimp

then p = conserved
  
p1  p 2  p 3 = 0 Fig. A Nimp Fig. B
  
p 3 = – p1  p2  Net impulse is zero in fig. A but net impulse is
   not zero for the system in fig. B.
mv 3 = mv1  v 2 

   
v 3 = – 3î  2 ĵ   î  4 ĵ 

v 3 =  2î  2 ĵ

www.motion.ac.in
148 - Solution Physics (XI) Topic

31. C
(m)(2 m) 2
a = = m
m  2m 3
A B
and vr = relative velocity of the two.
Substituting in Equation (1), we get
a kx2 = mv 2r
a

 3k 
 x
C  v r =  2m 
 
a
Torque about A : mg = I
2
35. A
mga
 =
2I 36. C
Maximum expansion in spring is given by
2
a mga
 acceleration = = 1 2 1
3 2 3I kx max = v 20
2 2
[ = Reduced mass]
32. D
0 = mx1G + 2mx2G + 3mx3G  2m
 xmax = . v0 = v
k 3k 0
= (x13 + x3G ) + 2 (x23 + x3G ) + 3x3G
= –2L + 2(2L) + 6 x3G
 x3G = L/3 m
37. x1 = (g + a)
k
33. D
m
v v v=0 x2 = (g – a)
k
m 2m m
m
2mv + 0 = 3mv'  v' = 2v/3 x3 = g2  a 2
k
v 2v/3 [m = mass of pendulum
m 3m
k = spring constant]
Pf = Pf
m2
mv + 3m2v/3 = 4mv'  x12  x 22 = .2(g2 + a2) = 2 x 32
k2
3v
3mv  4mv '  v' m / sec
4 x12  x 22
 x3 =
2

34. A
From conservation of mechanical energy 38. D
Let N be the normal reaction between m
1 2 1
kx = v 2r ... (1) and M,
2 2
Equilibrium of M
Here,  = reduced mass of the blocks
N sin 45º = kx … (i)
Equilibrium of m in vertical direction gives

394-Rajeev Gandhi Nagar, Kota


Centre of Mass - 149

N cos 45º = mg … (ii)


1 1
From Eqs. (i) and (ii), we get mV2 = kA2
2 2
mg
x= m 4
k A= V= × 3 = 6 cm.
k 1000
39. B
Let x be the maximum displacement of block
downwards. Then from conservation of me- 43. B
chanical energy: Consider an observer moving with speed v
decrease in potential energy of 2 kg block = with point A in the same direction.
increase in elastic potential energy of both ob server
the springs
v
1 m
A
 mg x = (k1 + k2) x2
2
In the frame of observer, block will have ini-
2mg (2)(2)(10) tial velocity v towards left.
or x=k k = = 0.1 m
1 2 100  300
v
m A
Acceleration of block in this position is –

(k1  k 2 ) x  mg During maximum extension, the block will


a= (upwards)
m come to rest with respect to the observer.
Now, by energy conservation,
(400)(0.1)  (2)(10)
=
2 1 1
mv 2  kx 2max
2 2
= 10 m/s2 (upwards)
mv 2
 xmax =
40. D k

By theory
44. B
41. A

42. C
In the frame (inertial w.r.t. earth) of free
end of spring, the initial velocity of block is Total travelled distance = 2d
3m/s to left and the spring unstreched. then

2d
3 m/s Time between two collisions = v
4kg initial state
0

maximu m extension v0
4kg state So no. of collision/sec =
2d

Impulse in one collision = mv0 – (–mv0)


A
= 2mv0
Applying conservation of energy between
initial and maximum extension state. v0 mv20
F = 2mv0 × =
2d d

www.motion.ac.in
150 - Solution Physics (XI) Topic

45. B 50. A
Impulse = change in momentum
51. C
–I = –m2u – mu
I = 3mu 52. B
If e = 1 then angle = 45°
W.D. = change in K.E.
If 0 < e < 1 then angle is less than
u
45° with the horizontal. So 30º is not possible.
2u I
53. A
1 1
W.D. = m(2u)2 – mu2 In inelestic collision, due to collision some
2 2
fraction of mechanical energy is retained in
form of deformation potential energy.
3 Iu
= mu2  W.D. =  thus K.E. of particle is not conserved.
2 2
In absence of external forces momentum is
conserved.
46. C
Impulse = change in momentum 54. C
e=1
 F.dt = P
As collision is elastic therefore vi = vf

1
So K = 0 k f = ki = m u12  u 22 
Given  F.dt  J 
2
Now, Contact time is twice than the earlier.
 55. C
 F.2dt  J'  J' = 2J In elastic collision e = 1, Energy is conserved
because colliding particles regain their shape
and size completely after collision. Due to
47. D Fnet on the system is zero, momentum is
conserved.

(at the time 56. C


T of collision)
In absence of external force. Momentum of
v A B v the system is conserved.
3m 2m
Impulse = change in momentum
So, –Tt = 2mv – mu (for bullet) 57. C
I = Tt = 3mv (for mass 3m) If e = 1 and m1 = m2 then after collision
3mv = 2mv – mu velocity interchange

3mu
v = u/5  I= 58. B
5
from energy conservation
1
48. B mql  mv2  v  2gl
2
 4 from momentum conservation
I  P  m  v2  v1   0.1  0     0.2 kg m / s
 2 m 2gl  mv '  v '  2gl

1
KE  m  2gl  mgl
49. B 2
Area under the graph gives impulse

394-Rajeev Gandhi Nagar, Kota


Centre of Mass - 151

59. B After collision of A and B.


21m/sec 4m/sec 1m/sec v2
 m m
1kg 2kg 1kg 2kg 3v/5
A Before B A After B A B
21  1  4  2  1  2v2
63. B
21  8  1  2v2
Let mass of ball 2 is m and mass of ball 1 is 2
2v2 = 12  v2 = 6m/sec m.

v2  v1 6 1 5 1 m1u1  m2u2  m2 e u2  u1 


e    v1 
u1  u2 21  4 25 5 m1  m2
v
e = 0.2 v 2mv  em(0  v) 2m m
=
3 3m
1 2
 e=1
60. A
20 m/s So elastic collision.
Let v1 is the velocity
of wall after collision. 25 m/s
64. C
V1  20
e= (e = 1) Just before collision, speed of ball v  2gh
20  (25)
and just after collision
v1 = 65 m/s V1
80 4
v'  2gh = 2gh
61. A 100 5

Before Collision After Collision v2 – u2 = 2aS


u v1 v2 Let h' is the maximum height after collision.
u=0
A B  A B 2
4 
0 – 2gh  = 2x (– g) × h'
v2  v1 5 
e=  eu = v2 – v1 ....... (1)
u
16
Now from momentum conservation × 2gh = 2gh
25
mu = mv1 + mv2 h
4
v'  2gh
u = v1 + v2 ....... (2) 16 5
h' = h v 2gh
25
v1 1e
from (1) and (2) v =
2 1e
65. A
From energy conservation
62. A 2gh
0 v 1 1
1 Collision
st
m m 4m 2
m  2gh 2
+ mgh = mv2
2
2nd Collision A B C h

v = 2 gh
mv  4m(0  v) 3m
Velocity of B v = =
5m 5
2gh
e
3v/5 2 gh
m m
1 v  2gh
A B  e ,
2

www.motion.ac.in
152 - Solution Physics (XI) Topic

66. C 70. D


After the top end of chain falls down by ,
8
the speed of chain is

 g
v = 2g = .
8 2
The mass of chain above
vdn
Ft = = v
7 dt
table is M.
8
F – Ft = ma
 momentum of chain is
dv
F – v = (M0+t)
7 g 7 dt
M = M g
8 2 16
t t
dt dv
67. A

0
M0  t
=  F  v
0

Thrust force acts opposite to v r on the


container. Therefore acceleration will be less 1  M0  t  F  v 
ln  M  = (1) ln  
than g/2 initially. 
  0    F 

68. C M0  t F
m1v1  m2v2   m1v1  m2v2  = |change in M0 =
F  v

momentum|
F  v M0
= |External force on the system|  time = M  t
F 0
interval
= (m1 + m2)g  (2to – 0) = 2(m1 + m2)gto Ft
v = M  t
0

69. D
By theory dv
a=
dt

71. A
d
dn
Ft = vr. = 0 () m
dt
Fr = 10
 Fext. = 0

394-Rajeev Gandhi Nagar, Kota


Centre of Mass - 153

1. C 3. C
Centre of mass of uniform semi-circular disc COM of circle is at O. Let M1 is mass of circle
and M2 is mass of triangle
4R
is at
3

Centre of mass of uniform semi-circular ring a

2R COM of triangle
is at a/3

Centre of mass of solid hemi-sphere is at


M1 M2
3R
8
a/3

R Distance of COM from centre of circle


Centre of mass of hemi-sphere shell is at
2
M2   a2 a
C T H R S D r1 = M  M = ×
1 2 a2  a2 3
h h R 2R 3R 4R
4 3 2  8 3 a2  a a
= =
3a2 (  1) 3(  1)
2. D

4. B

a' C' r 4r  C''


  
(COM of  2 3  (COM of
rectangular semi-cicular)
C' C'' plate)

4  6
= 8cm
3

 r 4r  C
(2r 2 )  
 2 3 
a' (COM of system)
= r 2
2r 2  4R
2 COM of semic circular disc =
3

a' = 2(3r  8r) So from point C distance of COM is 8 cm.


3(4  ) Center of mass coincides

4r
Required Ans (COM from O) = a' + 5. D
3

2(3r  8r) 4r 2r COM of rod


= + = along y-axis
3(4  ) 3 3(4  )
COM lie on this line

COM of rod
along x-axis

www.motion.ac.in
154 - Solution Physics (XI) Topic

6. B u u

Take sector element on disc v'


–m1u + m2u + Mv = 0

R 2d  2R 

0
( sin )
2  3 sin  (m1  m2 )
Ycm  v' = u

R 2d M
 0
( sin )
2

10. A
2 
2 m1 m2
 3
 R sin d  R
0 urel+v'

6
 sin d
0 urel–v' v'
m2(Urel + v') + Mv' = m1(urel – v')
Xcm  0
| m1  m2 | Urel
v' = m1  m2  M
7. D
Positive Negative
11. D
urel–v'
v'

m1 m2 (urel –v')
M1 M M2
A B
v' m(urel – v') = (M + m)v'
By momentum conservation
murel
O = m1 (urel – v') – (m2v' + Mv') v' =
(M  2m)
m1(urel – v') = m2v' + Mv'
(Urel + v'')
m1urel
v' = m  m  M
1 2
v''

from momentum conservation


8. D
A B (u–v')
rel (M  m)murel
m(urel + v'') + Mv'' =
(M  2m)
v'

from momentum conservation COM remains


12. A
stationary
As Fnet in x direction = 0
m2(urel – v') = (m1 + M)v'
mx1 = mx2 [ Fx = 0]
m2 urel – m2v' = m1v' + Mv'
x1 = x2
v'(m1 + M + m2) = m2 urel
Now x1 + x2 = L sin 
m2urel
v' = L sin 
m1  M  m2  CMf =
2

9. A
 
Fnet  0 Vcom  0

 COM is at rest.

394-Rajeev Gandhi Nagar, Kota


Centre of Mass - 155

L sin 

v2
L –x 2x x
 x1 L/2 cos  L+ x a= g= g
L /2
CMi 2L L
x2

L
CMf

13. D 17. C
VCMx = 0 and Fx = 0 from energy conservation
from momentum conservation
mv1 = mv2  v1 = v2 = v(let)
Initial  1
Now energy conservation 2L mg = mv2
2 2
1 2
mg (1 – cos ) = 2  mv 
2 
 u= g
v2  = g (1 – cos )

 18. A
Distance from centre of mass = R =
2
m(L-x) = Mx
mg(1  cos ) mL - mx = Mx
mv2
So T = =
R  /2
mL
mL = x(M×m)  x
T = 2mg (1 – cos ) (m  m)

14. A 19. B
from previous question
2F 2M M F
vmax = V = g(1  cos )1 / 2
F
aCOM =
3M
15. B
w.r. to COM
Only in vertical direction
[ fx = 0 always]
4F/3 2M M 4F/3

L L x2 x1
So displacement = – cos 
2 2

4F 4F 1
L x1 + x2 = k (x1 + x2)2
= [1 – cos ] 3 3 2
2

8F
= (x1 + x2)
16. A 3K

m1  m2
a g
m1  m2 20. D

Let m1 = (L + x) and m2 = (L - x)


21. A
where  is mass per unit length
Momentum is conserved

www.motion.ac.in
156 - Solution Physics (XI) Topic

22. D 25. B
As no external force is acting on system, so (i) From M.C. mv = 2mv'
momentum of system remains conserved. At v' = v/2
the ti me of mi ni m um separati on the
(ii) from M.C. mv = 2mv'
compression in the spring is maximum & the
v' = v/2
velocity of both the block at this moment
would be equal. As initial momentum is non- (iii) Impulse = mv = 3mv'
zero, so at maximum compression the velocity
v
of A & B would be non zero but having different v' =
3
momentum as masses are different. At this
moment the spring energy is maximum SO KE
of system is at its minimum. 26. D
When velocity is same means maximum
23. C compression.
Initially, F = m1g = k1x1  Maximum loss
MR × 8 = MR × 0.4 + Ms × 1
F  m 2g
x1 = ....(1) 0.4MR = MS
k
 MR > MS
m 2g
Finally x2 = .... (2)
k
27. D
From conservation of energy, Infinite
1
m1g(x1 + x2) =
2

k x12 – x 22  .... (3)
28. B
From (1), (2) & (3), F = (m1 + m2)g
2v cos 
g
24. D

–1 before
v(ms) collision collision after collision 29. C
1.0
0.8
 2N sin  .dt  Mv 0 ....(i)

0.2
 N cos  . dt  Mu'
1 2 t(s) mv0
(i)  v is +ve for both.
 N.dt =
2 5
.3

(ii) Yes (when maximum compression) N N N



(iii)  S have greater velocity after collision
then R have before collision and K.E. of S will 3R/2 3R/2
u'
be less then initial K.E. of R

2R
1 1 N N
m V 2 < mR (VR)2
2 s s 2
but VS > VR So ms < mR (3 / 2)R 2  R 2
sin  =
3 / 2R

394-Rajeev Gandhi Nagar, Kota


Centre of Mass - 157

34. D
5 2
sin  = ; cos p = 0.1 (6+4) = 0.1 × 10 = 10 NS
3 3

mv 0 3 2 v0 35. A
= mv'  v' =
2 5 3 5
1
mgh = mv2
2
30. C
v= 2gh
Impulse = change in momentum
By momentum conservation
mv0
 2N sin dt =
2
....(i)
m 2gh + 0 = 2mv'

 N. cos dt = mv' ....(ii)


v' =
2gh h
2
from (i) and (ii)
By energy conservation
5 mv0
 2N 
3
dt 
2
1
(2m)v'2 = 2mgh', m
(2gh)
= 2mgh'
2 4
2N
 dt  mv' h
3 h' =
4
On dividing

2N  5 3 v 36. B
× = 0
3 2N 2v' Let mass of ball 2 is m and mass of ball 1 is 2
m.
v0
v' = m1u1  m2u2  m2e u2  u1 
2 5 v1 
m1  m2

31. D v

When collision is oblique the particles after 2m m v = 2mv  em(0  v)  e = 1


3 3m
collision move so that the momentum after 1 2
the collision is conserved.
So elastic collision.

32. B
37. C
Rubber bal l suffers greater change in
momentum as vf = -vi. initial momentum of FT = v
both balls is same as they are moving with F – v = (M0 + t) a
same velocity and have same mass. F – at = (M0 + t) a v = at
F = at + aM0 + at
33. D F = aM0 + 2at
When the ball hits the floor and it is in
contact with the floor its KE converts into 38. A
its potential energy due to compression and
F = v
conservation of energy is valid during this
period.

www.motion.ac.in
158 - Solution Physics (XI) Topic

39. A 41. D
Ft = V x Ft – Fext. = Ma
N = (x)g + Ft v  2gh
dv
rv – (M0 – t)g = (M0 – t)
dn dt
N = (x) g + v. N
dt
v Ft
N = x g + v (v)
N = xg + v2
N = xg + ().2g(x)
N = 3 gx M0

40. B

vr

t v
 vr .m 
 
M  t
0  0
 g dt =

 dv
u

(1)vr .  M0  t 
v–u= ln  M  – gt

  0 

 M0 
v = u + vr. ln  M  t  – gt
 0 

If we discuss in gravity free space, g = 0

 M0 
v = u + vr.ln  

 M0  t 

42. C

dm
Ft = v r . = v (Av) =  Av
v2
dt
Ft = A. 2gh

394-Rajeev Gandhi Nagar, Kota


Centre of Mass - 159

1. C,D 6. A,B
In case C & D centre of mass of rod me be at
centre but in A & B centre of mass is not at
centre because of non-uniform distribution.
(0,0) (Density continously changes.)

7. B
As net force in x direction is zero. So from
2. B,D
momentum conservation.
Center of mass of ring is at centre and centre
mV0 = (M + m)V2
of mass of chord AB is at its mid point so h
centre of mass of this combination lie at MV0 m v0
the line which makes 45° with x axis. V2 = M
Mm
Y
B
8. B,D
Ring COM line
Velocity of center of mass
A X
MV  mV
VCOM = = V
Mm
Possible combination So both are at rest with respect to centre of
mass. And kinetic energy is converted into
R R R R 
 , ; ,  potential energy.
3 3 4 4

9. C
3. A,B By Energy conservation
In case C & D centre of mass of rod me be
2
at centre but in A & B centre of mass is not 1 1  mv 0 
mv02 = (M + m)   + mgh
at ce nt re b ec ause of non-uni form 2 2 M  m
distribution. (Density continously changes.)
After solving

4. B,C  M  V02
 h = 
Fnet = 0  M  m  2g
 macom = 0
It means acom = 0
10. C
 Vcom = constant.
V1 is the velocity V1 V2
of particel and V2
O
5. C,D is the velocity of
wedge.
(V1 + V2) = vel. of particle w.r.t. wedge

 mV0  M( V0 )   mV0  mV0 


 –   +   = V
(0,0)  M  m   Mm  0

www.motion.ac.in
160 - Solution Physics (XI) Topic

11. B,C
2mV0
As net force in x direction is zero. (d)  vel. of wedge V2 =
Mm
So by momentum conservation
Mv2 – mv1 = mV0 V2 M  m
m Vel. of particle V1 = V0  
and V1 + V2 = V0 V1 M
M  m

MV2  (mV1 ) mv 0
12. B VCOM = =
Mm Mm
As net force in x direction is zero.
So by momentum conservation
14. A,C
MV2 – mV1 = mV0 .......(1)
V1 + V2 = V0 .......(2) V
V
By solving V V
(L – Vt)

M  m
V1 =V0  
(Vt)2  (L  vt)2  L
M  m
2V2t2 + L2 – 2LVt  L2
13. A,B,C,D Vt – L  0
(a) From Q. 9 L
t
 V1 + V2 = V0 V

M  m
V2 = V0 – V0  
15. B,C
M  m
Maximum extension when they have same
(M  m)V0  V0M  V0m 2mV0 velocity.
= =
Mm Mm
3m/s 10m/s
A B
1 4m2 V02
K.E. = × M ×
2 (M  m)2 Momemtum conservation
5 × 3 + 2 × 10 = 7V
M V02  V = 5 m/s
[ h= ]
(m  M) 2g From energy conservation

1 1
4m2 × 5 (3)2 + × 2 × 102
 K.E. = gh 2 2
(m  M)

1 1
2mv0 = × (1120) × x2 + × 7 × 52
(b) V2 = 2 2
Mm
45 + 200 = 1120x2
(c) K.E. = kf – ki
x = 25 cm
2 2
1  4m V0  52
= M –0 T = 2
2  (M  m) 
2 = 0.071 
7  1120

4mM 1 
= 2 mV02  16. B,C
(m  M)  2 

394-Rajeev Gandhi Nagar, Kota


Centre of Mass - 161

17. B,D 22. (A) p always remains conserved as Fext = 0

18. A,B,D
(B) Equal and opposite J =  F dt
m1v1  m 2 v 2
p Before
v1 m 2 p2 K1 m 2
 , K  or  (C)
v 2 m1 2m K 2 m1 p –J J After

19. A,B,C
J p–J
The KE measured from ground frame may be – 
m  m  2J
equal to or greater then that of measured in e= p = –1
p
COM frame. Because KE is frame dependent. m

20. A,B,C
23. A,B,D
2 m/sec 4 m/sec 1 m/sec v'
A B A B
24. B,C,D
Momentum conservation
Impulse = change in momentum
1 × 21 – 2 × 4 = 1 × 1 + 2 × V'
= Area under F – t curve
V' = 6 m/s
1 to
6 1 1 = ×Fo ×
e= = 2 2
21  4 5

Loss of kinetic energy = kf – ki


25. A,B,C
1 1 Angular impulse = J = angular moment
= × 1 × (1)2 + × 2 × (6)2
2 2
J
 Angular velocity =
1 2 1 2 I
–   1  (21)   2  (4)  = 200 J
 2 2 
1 2
K.E. = I
2
21. AB

1 2 
and velocity of mid-point = 
(A) K = µv rel 2
2

( M  m) 26. B,C
(B) amM = am – aM = –F
mM e = 1  Given V

F Before collision
 = am  aM u1 = –v Wall u
m
u2 = u
F(M  m) After collision
(C) 02 = u2 – 2d ×
mM v1 = ?
v2 = u
mMu 2
 F= V2  V1
2d ( M  m )
On solving e =  v1 = v + 2u
u1  u2

www.motion.ac.in
162 - Solution Physics (XI) Topic

27. A,B,D
 F · t = P
V V
Average elastic force M 2M

P m(V  2u)  mu  (mv  mu) For minimum kinetic energy


=
t t
MV0 = 3MV
 V = V0/3
2m(u  v)
=
t 2
1  V0  1 
2
(ii) Kinetic energy of the ball increases by  
K = – 2 3m    mv0

 3 2 
= Kf – Ki
= 2 Joule
1 1
= m(2u + v)2 – mv2
2 2
28. A,B,C,D
= 2mu (u + v)
Inelastic collision
0<e<1

394-Rajeev Gandhi Nagar, Kota


Centre of Mass - 163

x 5 3
1. (a) ( x)    , (b) L 3. a
L 9 4

dx y = ± kx2

x a

 = Ax + B  x dm
0
when x = 0,  = 0  B = 0 xCOM = a

and when x = ,  = 20  dm


0
 20 = A + 0

0  dm = (2y dx) = +2kx2dx


A=

a
0 2k x . x 2dx

 = x + 0
 0
So xCOM = a
2k x 2 dx

 0


0
 . dx . x
5
xCOM =  = 
9 a4  3 3a

0
 . dx =
4a 3 =
4

Now, YCOM = 0 [ Symmetric for x-axis]

4. 16
4  b3  a3 
2. y   If m is mass of one plate, then
3  b 2 – a 2 
m.20  m.20  m.20  m.20  m.0
So C.M. from 0 Zcm = = 16 cm
5m

a 5. At R/5 from the centre of the bigger disc


b
O towards the centre of the smaller disc.

+ b + – a 6. (5a/6, 5a/6)

 b2   4b   a2  4a
     
 2   3   2  3 y
y=
b2  a2 
   
2  2 

7. x
4  b3  a3 
y=  2 2
3  b  a 

www.motion.ac.in
164 - Solution Physics (XI) Topic

(v) Distance moved by B with respect to ground


2 1  R 
(R )(0) –  (2R )(R ) –  = (2 – x) = 2 – 0.2 = 1.8 m
= 2  3 
2 2
R – R
11. g/9 downwards
R 2g – T = 2a
= ]
3( – 1) T–g=a
g = 3a
a = g/3 2kg 1kg
13
8.
4 g g
2  g
aCOM = 3 3 =  (downwards)
If centre of mass is at A (consider it as ori- 9
3
gin) then

 a  ab sin  b sin  
a2     t  t   0
 2  2 3  2gR
12. v
m
1
b 13 M
 
a 4
By momentum conservation
m
9. 100 m
v2 R
m  50  m  30 M
VCOM = = 40 m/s v1
2m
mv1 = Mv2
VCOM = –g
By energy conservation
(40)2
HCOM = = 80 m 1 1
2  10 mgR = mv12 – Mv22
2 2

2
10. (i) zero; (ii) right ; (iii) 20 cm ; 1 1  mv1 
mgR = mv1 + M  
(iv) 2.2 m ; (v) 1.8 m 2 2  M 

A B 2gR
v= m
50 kg 70 kg 1
M
M = 80 m
2m
13. 9 m/s, 9 m
(i) zero Given : Trolley + child = 200 kg ;
(ii) Right mass of child = 20 kg
(iii) 50 (2 + x) + 80(x) = 70 (2 – x) u = 36 km/hr
100 + 50x + 80 x = 140 – 70x Let the new velocity of trolley = V
x(50 + 80 + 70) = 140 – 100 New velocity of boy = (V + 10) m/s
x = 0.2 m or x = 20 cm By momentum conservation
(iv) Distance moved by A with respect to
1000
ground is = 2 + x = 2.2 m 200 × 36 ×
3600

394-Rajeev Gandhi Nagar, Kota


Centre of Mass - 165

= 20 (v + 10) + (200 – 20)v


2000 = 20v + 200 + 200v – 20v kx 20 Kx20
17. (a) , (b)
2m 4
1800
v= = 9 m/s m 2m
200 K
A B
Length of Trolley
Time taken = (a) By momentum conservation
Re lative speed of boy
mv = 2mv'
10
= = 1 sec. v
10 v' =  
2
Required distance = 9 m
By energy conservation

2
m Bv0  m B v 0  v 20 1 1 1 v 1 x2
14. v = m  m , h =  m  m  2g kx02 = mv2 + 2m   + k 0
A B  A B
2 2 2 2 2 4

mv2 kx 20
15. At highest point combined mass velocity is kx02 = mv2 + +
2 4

20
H kx20
v=
2 2m

(b) Work done = K


20 cm
2
1  kx 0  kx 20
6 W.D. = m  2m  – 0 =
v0 2   4
= 2 m/s
3

20 1 m/s
H = 2g = 1m 2
1 m/s 18. m
5 3

1
1.2 = 1.T + g T2 2m/s 4m/s
2
T = 0.4 sec 2 4
urel = 1 m/s
  = urel T = 0.4 m = 40 cm rel. = 4 – 2 = 2

1  2  4  1
16. 2 (4) = kx2
2 4  2 2
2mx  m{x  (R  r ) sin 30º }
0
3m 32
= x2
6  100
(x : displacement of hemisphere)

R r 2
x= = 2 cm x = 0.4 3 = m
6 5 3

www.motion.ac.in
166 - Solution Physics (XI) Topic

19. (i) 20Ns,


F p1  p 2
(ii) 50 N 23. (a)  12.3 m / sec ,
100N mp
(i) By graph
Area = P 2 2
0 t p1  p 2
0.2 0.4
(b) =9.4 m/s
1 1 mp
= × 0.2 × 100 + × 0.2 × 100
2 2
= 20 Ns. Electron

(ii) Favg. t = P Proton


Anti-Neutrino
Favg t = 20
(a) By momentum conservation
200 mpvp = meve + mAvA
Favg = = 50 N
4 mPvP = P1 + P2

P1  P2 1.41028  651027
20. P(N-sec) P(N-s) vp = mP = = 12.3 m/s
1.671027
0.25
0.25
0.125
0.125
t P12  P22
50 100 (ms) (b) = 9.4 m/s
mP
50 100 t(ms)

21. 187.5 N, 625 kPa


24. (a) 4 5 N , (b) 2000 5 N
dm dm Speed at time of collision
FTh = Vr [ m = Ax = Av]
dt dt
v = 2gh = 80
= Av2 = 1000 × 300 × 10–6 × (25)2
= 187.5 N (a) So I = P = mv – m(–v) = 2mv

187.5 = 4 5 m/s
Force
Pressure = =
Area 300  (10  3 )2
(b) I = Aavg t = 4 5
= 625 kPa
4 5
Favg = = 2000 5 N
0.002
22. m  u2  uv  v 2

v v 25. 0.2
180°
60°  dm
u –u Ft = v r . = v (Av) =  Av
v2
dt

|P| = m m2  v2  uv Ft = A. 2gh

394-Rajeev Gandhi Nagar, Kota


Centre of Mass - 167

1. 0ba
If mass is non-uniformly distributed, then L2 L2 5L2

centre of mass of ring may lie from origin to
=
3 2 = 6  5L
circumference. hence o  b  a. L 3L 9
L
2 2
2. L/3, L/6
 CM of system line on L,
4. (a) 360 m, (b) 10800 J
 L  – L/2
L1  y –  =   (x – 0) y
 2   L/2  2m
m v=0
y
R/2 R' m
m
x
R C.M.

C1
 Explosion is an internal process so it
(0, ½ ) will not effect the position of C.M.
x
and because no force in horizontal
C2 L direction so
(½,0 )
x C.M exp losion  x C.M without exp losion
L
y=–x+ m R / 2  (R  R)
2 R
2m

L L
  ,  satisfie it ]  R  R 
3R
3 6 2
so striking point for second particle

0x 5
3. (a) (x) =  + , (b) L 3 R 3  U2 sin 2 
L 9  R  R   = 368
2 2  g 

0
A B
0 20
(a)  = Ax + B mA 2gl
5. vB 
A = A(0) + B = B = 0 [x = 0] mB 1+ m A / mB ; v2

0
B = AL + B = 20  A = 2m 2A g 
L T = 3mAg +
mB
v1
0 ( v cm ) x  0  m A v1  mB v 2
 = x + 0
L
mB
v1  v2 ...(1)
L mA
 
( x)  dx  ( x )  0 x   0  dx
 0 L  E. conservation
(b) xCM = = L
  dx   1 1
m A v 12  mB v 22
0  L0 x   0  dx mAg 
2 2
...(2)

www.motion.ac.in
168 - Solution Physics (XI) Topic

(1) and (2) (a) Given m1  m2  m3  m


2 when m1 and m2 stickes then
1 m  1
mAg  m A  B v 2   mB ( v 2 )2 Momentum Conservation
2  mA  2
m1v  (m1  m2 ) v ...(A)
 m2 
2 n A g   B  mB  v 22  m1v 
m  v    velocity after collision of
 A  
 m1  m2 

2 (m1 + m2)
m  g
v 22  2  A 
  m
 mB  1  A 
 ...(3) v
 v  ...(B)
 mB  2
and velocity of m3 = 0 just after impact.

mAv2  v  v2 
2 (b) Now just after impact the total energy
T  T  mA  1 
    1 1
E (m1  m2 ) ( v )2  mv 2 ...(1)
2 4
m 
T  2g  A  (m A  mB )
 The K.E. of m3 will max. (K.E. of m2 will
 mB  minimum) when spring is in normal position
and after some time of impact at this time
if velocity are v2 and v3 then momentum
6. 2mv2/3l
conservation
y
v2 v3
m x y m1 m2 m3

 (m1  m2 ) v  (m1  m2 ) v 2  m3 v 3

2m v
or m1v  (m1  m2 ) 2  m3 v 3

from (A) v  2v 2  v 3 ...(2)


2m (  )  m (o) 2
y cm   ...(1)
(2m  m) 3  Total energy after impact remain
same for all time
2mv  m (o) 2v so E f  Ei
v cm   ...(2)
2m  m 3

the velocity of 2m w.r.t. C.M. 1 1 1 1


 2m v 22  m v 32  k(o)2  mv 2
2 2 2 4
cm
v  v cm  v / 3
T /3 v 2  4 v 22  2 v 32 ...(3)
2
2m ( v / 3)
Then T from (2) and (3) element v2
/3 2m v – vcm = v/3

2
 v  v3 
2 v 2 2
2m ( v / 3 ) v2  4  2
  2 v3
T m  /3  2 
3 

v 2  v 2  v 32  2 v v 3  2 v 23
7. (a) v/2, v/2, 0; (b) 2mv2/9,
2v
v3  ...(4)
(c) mv2/72, (d) x  m / 6k v 3

394-Rajeev Gandhi Nagar, Kota


Centre of Mass - 169

9. B N
1 2
K.E max  mv 23  mv 2 mVf cos 30° = 1.5m
2 9
Vf
Vf cos 30° = 1.5
(c) from (4) and (2)

3
si
n
30
vf = 3 m / s 3 cos 30°

°
=
2v v

1.
3 m/sec

5
v  2v 2   v2 
3 6
10. (a) 50 m, (b) 10 m, (c) 30 m
2
1 v 1
so K.E min  m   mv 2
2 6 12 t1
e
U cos  Collide

eu cos 
t2
(d) When their velocity will same from U=30m/s

mome. con. 45º


O
50m
x
m1v  (m1  m 2  m3 ) v

v  v / 3 y component of velocity does not change

Ene. constant so y direction total time if t


t = t1 + t2 ...(1)
1 1 1
mv 2  3m (v )2  k ( x m )2
4 2 2 then 0  U sin  t  1/ 2 gt 2

at the time of col lision if ball have y


6 mv 2
2 component of velocity vy then
mv   2 K ( xm )2
9
v y  v1  g t1 (here t1 = time for collide)
m
 x cm  .v Ans. v y  10 sin 37º10 t1 ...(2)
6k

relative dis tan ce


for time t1 
[relative velocity ]in x direction
8. 1.5 m/s, 3.6 Ns
u
(0.7 – 0.25)m = 0.45 m T
m2
10
So speed of A after height T 
U  v2
= 0.45 m m1 u

Let then, 10
t1  ...(3)
v2 = 02 + 2g (0.45) U8

v = 9 = 3m/sec from (2) and (3)

Now, tension will give impulsive force and 10  10


v y  10 sin 37 º  ...(4)
both block move with same velocity. U8
So I = m1 (–u) – m1 (–v) .... (1)
 Ball again come into his hands so
I = m2 u .... (2) total time t then
from (1) and (2)
0  10 sin 37º t 1/ 2 gt 2 in y-direction
m2u = –m1u + m1v
2(10 sin 37º )
m1v 23 6 t  1.2 sec
u =m m = = g
1 2 23 5

here t1  t 2  t ...(6)
6
and I = m2u = 3 × = 3.6 If returning time is t2 then
5

www.motion.ac.in
170 - Solution Physics (XI) Topic

10  ut1 13
t2   u m / sec
v 2 u 3

t2
10  u t1 t1 2 u sin  2  30  1/ 2
 or t  sec
v2  2u g 9.8

from eq. (1) t = 4.33 sec ...(2)


again for time t1
10  u t 1
so t2  ...(7)
8  2u 50 50
t1   sec
u cos  30  1/ 2
so from eq (3), (5), (6) and (7)
putting values of t1, t2 and t t1  2.36 sec ...(3)

so from eq. (1), (2) and (3)


 10 
10  U  
10  v  8   12 t 2  t  t1  4.33  2.36
 
U8 8  2u 10
t 2  1.97 sec

 3 u2  114  104  0 so x  50  ( eu cos ) t 2

x  50  e (41.8)
 11  (11)2  4(3) (104 )
so u
23 (a) e = 0 then x = 50 m
(b) e = 1 then x = 8.2 m
26 (c) e = 1/2 then x = 2.91 m
u (from + sign)
6

394-Rajeev Gandhi Nagar, Kota


Centre of Mass - 171

1. B  
m1v1  m2v2
Since, the acceleration of centre of mass in 0
m1  m2
both the cases is same equal to g, so the
centre of mass of the bodies B and C taken  
where v1 and v2 are velocities of particles 1
together does not shift compared to that of
and 2 respectively.
body A.
   
dr1 dr   dr1  dr2 
 m1  m1 2  0  v  & v 
dt 
1 2
2. C dt dt  dt
In x - direction  
1 2
 m1d r1  m2d r2  0
mu1 + 0 = mvx before collision
 
 mv = mvx v/ 3
d r1 and d r2 repre sent the change i n
1 2 vy
 vx = v displacement of particles.
In y - direction Let 2nd particle has been displaced by dis-
vx
after collision tance x.
 v 
0  0  m   mv y  m1(d)  m2 (x)  0
 3
m1d
or x
v m2
vy 
3

Velocity of second mass after collision 5. B

2
Here momentum of the system is remaining
 v  4 2 conserved as no external force is acting on
v'     v2  v
 3 3 the bomb(system).

2
or v'  v
3 16 kg

3. C
-1
v 4 ms
This is the question based on impulse
momentum theorem. 4 kg 12 kg

F.t  change in momentum


Initial momentum (before explosion) = Final
 F  0.1Pf  Pi momentum (after explosion)
Let velocity of 4 kg mass is v ms-1. From
As the ball will stop after catching ;
momentum conservation we can say that
Pi = mvi = 0.15 × 20 = 3, Pf=0 its direction is opposite to velocity of 12 kg
 F × 0.1 = 3 mass.
 F = 30N  
From Pi  Pf  0  12  4  4  v or v=12 m/s

4. C 4  (12)2
KE of 4 kg mass   288 J
To keep the CM at the same position, velocity 2
of CM is zero, so

www.motion.ac.in
172 - Solution Physics (XI) Topic

6. C 10. C
This is the question based on impulse 0.5×2+1×0 = 1.5 × v
momentum theorem. [assumed that 2nd body is at rest]
F.t  change in momentum
2
 v  K  K f  Ki
3
 F  0.1Pf  Pi

As the ball will stop after catching ; 2


2
Pi = mvi = 0.15 × 20 = 3, Pf=0 1.5    2
  3   (0.5)  2   2 J = -0.67 J
 F × 0.1 = 3 2 2 3
 F = 30N
So, energy lost is 0.67 J.

7. A
11. A
In this question distance of centre of mass
P = mv = 3.513 × 5.00 ~
~ 17.6
of new disc from the centre of mass
of remaining disc is R.
Mass of remaining disc 12. C
R
O
M 3M O2 O1 1 2
 M  h= gt ,
4 4 2
v = –gt and after the collision, v = gt.
3M M 1
  R  R  0  
4 4 3
Note : In this question the given distance
must be R for real opproach to the solu-
tion.

8. C
(parabolic) (straight line)
9. A collision is perfectly elastic then ball reaches
to same height again and again with same
x=0 x=L velocity.
XCM 
 xdm
 dm dx 13. B
If n = 0, From the graph, it is a straight line so, uni-
form motion. Because of impulse direction
L of velocity changes as can be seen from the
then XCM 
2 slope of the graph.
As n increases, the centre of mass shift away
2
Initial velocity = = 1 m/s
L 2
from x  which only option (a) is satisfy-
2
ing. Alternately, you can use basic concept. 2
Final velocity = – = – 1 m/s
2

x
L
n 
Pi = 0.4 N – s

0
k    xdx
L 
XCM  n  1 
n  L 
L
x n  2  Pji = – 0.4 N – s
0 
k   dx
L 

394-Rajeev Gandhi Nagar, Kota


Centre of Mass - 173

   16. D
J  Pf  Pi = – 0.4 – 0.4
 3h
= –0.8 N – s (J  impulse) z0  h
4 z0

 (From class theory)


J = 0.8 N-s

17. A
14. A before collision
If it is a completely inelastic collision then  m 2v
Px = 2mv î
m1v1 + m2v2 =m1v + m2v v

m1 m2 Py = 2mv ĵ 2m
v1 v2
After collision

m1v1  m2v 2 Px = 3mv'cos V'
v
m1  m2

Py = 3 v' sin  3m
p2 p2
 KE  1  2 By momentum
2m1 2m2
conservation ;
  in horizontal  2mv = 3mv' cos  ...(i)
As P1 and P2 both simultaneously cannot be
in vertical  2mv = 3mv' sin  ...(ii)
zero therefore total KE cannot be lost.
from (i) and (ii) tan  = 1;  = 45°

15. B 2 2v
final speed v' =
Statement–I  (False) 3
For max energy loss, e = 0 initial K.E. ;  1/2 (m) (2v)2 + 1/2 (2m) (v)2
= 3mv2
mv
V=
mM 2
 2 2v 
final K.E. ;  1/2 (3m)   = 4/3 mv2

 3 
2
1 1  mv 
loss = mv2 – (m + M)  
2 2 m  M
(KE)i  (KE)f
% loss  (KE)i × 100%
1 1 m2 v2
= mv2 –
2 2 mM
= 55.55 ~
 56%
2 2 2
1 mv m  M  m v 1 mMv2
=2 =
m  M  2 mM

Statement–II  (True)

www.motion.ac.in
174 - Solution Physics (XI) Topic

1. B v2 = component of v1 along BC ....


In case of elastic collision, coefficient of  V2 = cos 30° = 45 m/s
restitution e = 1. Magnitude of relative
velocity of approach = Magnitude of relative
5. B
velocity of separation. But Relative speed
Height fallen by the block from B to C
of approach  Relative speed of separation.
h2 = 3 3 tan 30° = 3m
2. D let the required speed V3

3. A, D  V3 = V22  2gh2
Initial Momentum of the system
  = 45  2  10  3 = 105 m/s
P1  P2  0
  6. C
Final momentum P '1  P '2 shovia also be
v2
zero B
  C
B
Option A : P '1  P '2  0 30º
60º v1
30º 30º
C1 component of K̂ will not be Zero
Just Before Just after
Option (B) Elastic collision
 
P'1  P'2  0 If C1 = –C2  0
3
Option (C) V11 = v1 cos 30° = 60   45 m/s
2
 
P1'  P2' = 0
1
If a1 = –a2  0 ; b1 = –b2  0 v1 = v1 sin 30° = 60   15 m/s
2
Option D
Now, vertical component
 
P1'  P2'  0 b1 ĵ will not be zero V = V1 cos 30° – V11 cos 60°

M 7. A
4. B A
v KE & PE can change (e.g blocks at ends of
Between A and B 60° B spring) L can change (if resultant is net
h1 = tan 60° × 3 torque)
30° C
= 3  3 = 3m
3 3m 8. a/10
Speed of block just 3m
2m
before slriking the second collision m
a m
6m (–a,a)
(a,a)

V1 = 2gh1  60 m/s m (0,0) 7m


m
–a (0,–a)

Inelastic collision m

Comp. of V1  BC is Zero 2m( a)  6m(0)  m(0)  m(a)


Ycom =
Comp. of V 1 paral l el to BC is remai n 2m  6m  m  m
unchanged
ma a
= 
10m 10
B 30º
30º
30
º
v1 9. C
Collision is elastic and mass is same.
30º
C So after collision, velocity of particles will
change

394-Rajeev Gandhi Nagar, Kota


Centre of Mass - 175

v2  v1 vBBall  20 m / sec ...(2)


e = u u Bullet
1 2 for Bullet
At time t, Particles collide
 = vt ....(1) 2H
XBullet = VBullet ×
2 –  = 2vt ....(2) g
Now equaltion (1) and (2)  100 = vBall × 1 100 m
 vt 2 vBall = 100 m/s ...(3)
  2 = 2 –    =  120º
2   2vt 3 from (1) (2) & (3)
After two collision they will reach at point A. we get v = 500 m/s

10. 4 13. A
Collision between A & B
m × 9 + 2 m × 0 = mVA + 2mVB
vA + 2vB = 9 .....(1)
VB  VA 2m
e = u u m m
A B u0
A B C
 e=1
uA = 9 uB = 0
vB – vA = 9 ....(2) final kinetic energy of both the particles is
Now, equation (1) & (2) same because the vertical displacement is
 vB = 6 m/s also same.
Collision between B & C
2m × 6 + m × 0 = (2m + m) vC 1
 mgh = mv2
2
12m
vC = = 4 m/s (for both the particles)
3m
 The final angle which the composite
11. A,C system makes with the horizontal is 45º.

u
14. B
1kg 5kg Before collision
Energy given to particle
2 m/s v1 = 100 × 10–9 × 30 × 10–3
1kg 5kg After collision = 3000 × 10–12 J

E 3000×10–12
v 2 P= =
Now form e = 2 =1 C 3×108
u
u = v2 + 2 .......(1) = 1 × 10–17 kg m s–1
from M.C. u(1) = –2 (1) + 5 v2 ......(2)
After solu equa (1) & (2) 15. B
v2 = 1 m/s & u = 5 m/s V = gt

1
12. D  KE = m (gt)2
Applying momentum conservation 2
0.01×v+0.2×0  Relation is parabolic
= 0.01 VfBullet  0.2  vBall ...(1)
for Ball

2H
XB  vB 
g
20 = vB × 1
20 m

www.motion.ac.in
176 - Solution Physics (XI) Topic

1. D 4. A
Moment of inertia of a body depends upon
mr 2
mass and distribution of mass about the axis. Moment of inertia of disc = = 0.5 mr2
2

2. B 2
Moment of inertia solid sphere = mr2
5

5. D
Moment of inertia of disc

mr 2
about diameter I = = 2,
4
I I1 I
mr2 = 8
Moment of inertia about the
Moment of inertia about diameter of sphere I I1
axis through a point on rim.
2
I= mr2
5 mr 2
I1   mr 2 = 10
4
Moment of inertia about t ange nt at
their common point
6. C
2  14 As velocity of point A & B are different so it
I1   mr 2  mr 2   2  mr2
5  5 means the body is performing combined
rotational & translation motion. Consider the
I1 = 7I
situation as shown in figure.
X C
   
3. D
vA  vC    CA
 
a
Consider an element of width dx at a distance  
vB  vC    CB
of x from the end A as shown in figure.

ˆ v  10j
A B
a
Mass of this element is, dm = dx = Kxdx where v A  5j, ˆ,
B
Y
Moment of inertia of this element about the  
required axis is, CA  aiˆ  aj,
ˆ CB  aj
ˆ

dI = kx3dx Solving above equations we get,


Dx 
L 4 A ˆ  10j
vC  5j ˆ,
kL
I   kx3dx  x
0
4
7. C
Mass of solid complete cylinder,
let M is the mass of rod, then M1= (R2) (3R) = 3R2
L
kL2 2
M   dm   kxdx  Mass of each hemisphere, M2 = R2
0
2 3
Moment of inertia of given structure,
ML2 KL4 I = Icylinder –21hemisphere
I= 
2 4

394-Rajeev Gandhi Nagar, Kota


Rotational - 177

14. B
M1R 2 2  29
I= – 2   M2R 2   R 5
2 5  30 mr 2
 = I  
2
= 0.25rad/sec2
8. A

2 15. B
Moment of inertia of solid sphere I1  mr12
5  = I
 = constant   = increases
2 2
Moment of inertia of hollowsphere I2  mr2
3 16. D
 = 0 + t
2 2 r1 5 100 = 10+(15)   = 6 rad/sec2
mr12  mr22  
m 3 r2 3  = I  60 Nm

17. D
9. B
W = 
Moment of inertia about axis of rotation 100HP = 746 × 100W
I = mr2 = 4kgm2
= 74600 W
 = 1800 rad/min = 30 rad/sec.
10. C
dW d    d
P  
dt dt dt
11. C
P = 
= constant, = 0, = 0
74600 = (360)
Torque along horizontal axis is zero.
74600
  2487 W  S
30
12. D

L 18. D
Velocity of centre of mass =
2
1 2
I  1000
L / 2
2


  10rad / sec
N
(CoM) 2f = 10

force exerted by the clamp on rod 5 300


 f  rad / sec  rad / min
 
2
M  L  ML2
N=   , N=
L/2  2  2 19. A
 
P   ·  = 20 watt
13. B
From energy conservation 20. D
L
 = I
 ml 2  2 = I × 2  I = 1 kgm2
L 1 2, 
I 
Mg = I  3  I = MR2
2 2
1
1 = M(2)2 ; M kg
3Mg 3g 4
= =
ML2 L

www.motion.ac.in
178 - Solution Physics (XI) Topic

21. C The angle rotated by pulley in 2s is


Let us consider that just after releasing the
1 2
angular acceleration of rod is  & linear  = 0 × 2 + t
2
acceleration of block is a, then from constraint
theory a = l, where l is the length of rod.
20 1 10 100
= 2    22 = rad
9 2 3 9

 50
Number of revolutions made, n = 
m 2 9
Displacement of load A = 0.6 × 

100 20
m =  0.6  m
a 9 3

1 ml2 24. A
For rod, T × l – mg× = 
2 3 Let 5 kg block is sliding down the inclined
plane and 1 kg block is moving up and pulley
For block, mg – T = ma
is rotation in anticlockwise direction. FBD of
Solving above equations we get,
all three components is shown is figure for
g 4a 5 kg block

2 3 5g sin 370 – 0.2 × 5g cos370 – T1 = 5a
 22–T1 = 5a
3g
a= which is also the acceleration of
8 Ry

free end on the rod.



s3
co
5g

22. A Rx
x

T1
2
0.

23. B
By using constraint theory, Mg T2
°
37

T2
0.90 = 2 & 0.9 = 3
a
n
si
5g

where 0 &  are initial angular velocity and a


acceleration of pulley respectively. 1kg

For pulley
(T1–T2) R = 5 T1–T2 = 10
For 1 kg block
T2–g = a  T2 – 10 = a
From constraint equation, a = Ra


A  a=
2
2m/s Solving above equations, we get
3m/s2 B

394-Rajeev Gandhi Nagar, Kota


Rotational - 179

y
31. B
6 12
a= m / s2 ;   rad / s2 Angular momentum v
13 13
= mvr sin  d r

= mvd (constant) 
x
256 136 0
T1 = N; T2  N
13 13
32. C
I = Constant
25. A
 = I 33. B
 = constant Angular momentum
Its angular velocity increases I 11 = I22
But force on hinge is constant
L2
 K.E. = I  K.E.
2
26. C
 W.D. = + ve
1A  1B

So block A falls & block 34. D


B rises with difference
  
accelerations L =m(r × V)
B A

27. B = 2 × 2 [( î + ĵ ) × ( î – ĵ + k̂ )]

= 4 (– k̂ – ĵ – k̂ + î ) = 4( î – ĵ – 2 k̂ )
28. D
mg – T = ma ....(1) 
L = Angular Momentum along z-axis is the
l 2
ml  ml  2 compoent of angular momentum along z-
Tl  mg   ....(2)
2 3 3 l axis.
i.e. = – 8 kg-m2/sec
3g
solving (1) & (b)  a
8
35. C

L 10
29. B avg = = =5N-m
T 2
R = h
  = h/R
36. C
When cylinder turns through angle , length
of the string that unfold is R = downward 1 2
I  10
displacement of block (h). 2

5 2
30. A T    10    2rad / sec
2
 = I
r
Angular Momentum
 T T L = I = 5 × 2 = 10 joule-sec.
  r 
I I Cr 3

m    r2  I  r4

www.motion.ac.in
180 - Solution Physics (XI) Topic

37. B The acceleratoion of any part of the rod at a


I11 = I22 distance r from its foot is given by,

4R12 1  MR 22 2 a= (2r)2  (r)2

From above expression we can check the


R1 2 3
  correctness of different option.
R2 1 1

41. B
38. B The maximus height the lower end can rise
From angular momentum conservation is 2/even for large value of Vo. But for a value
of Vo less then a particular value the rod won't
Li = Lf
be able to complete circular motion & hence
mR 2 MR 2 mx the lower end can't reach to height 2l.
x x ' ; n' 
4 2 2M Let for a value V0 = v, the rod is just able
complete the circular motion, then

39. C 2
y ml2 v 1
 = V/r 0– ×   × =–mg × l
3 l 2
3m/s (15,8)
 = 3cos/r (0, 8)
A B v= 6gl
3 8 24   V / r
=   2 r
r r r So for v0 > 6gl maximum height is 2l &
O x
In OAB
r2=(15)2+(8)2 = 289 for v0 < 6gl maximum height can be found
by using energy conservation.
24
= rad/s
289 2
ml2  v0   1  –mgh
0– × 
3  l  2
40. B
Let M is the mass of rod & L is its length. v20
h=
Then at any moment when the rod turn by 6g
an angle  let the angular acceleration & where h is the height to which centre of mass
angular vel ocity of the rod are  &  of rod rises, so lower end rises to heigth,
respectively.
v20
Using    I 2h =
3g

L ML2
 Mg sin = 
2 3 42. B

3g sin  7/2 Mg
= 43. B
2L
Using work energy theorem,
Using energy conservation,
L ML2 2
12 L Mg (1–cos37) =
– 0  Mg (1 – cos ) 2 3 2
2 2

3g MgL 1 ML2 2 3g
 =2
(L–cos)    
L 2 5 63 5L

394-Rajeev Gandhi Nagar, Kota


Rotational - 181

44. B 47. B
From consercation of mechanical energy at Angular momentum conservation about A
initial & final position. Considering horizontal
A
line OB as the reference for PE.

x=l/2
l/2
O B
mv0

l l
mv0  I  mv1 ....(1)
2 2
A U=0
B
l
2
 v1
l
– 0  3mg
1 e 2 1
2 2 v0

17 2 2 l
 ml   3mgl   v1  v 0
12 2

17 2 36g l
 l  3g  =  v1   v0 ....(2)
12 17l 2
Solving (1) & (2)
3gl
vA =  × OA = vA = 3
17 v0 3m  4M
v1   v0
3m  4M

45. B  KE of P decreases

12mv0
Change in momentum = 2mV cos =  F.dt 
l 3m  4M
 Change in angular momentum On solving linear momentum of P1R system
decreases.

= Fd.dt = 2m Vdcos

48. D
46. B A.M. about A
J = MVCOM J A
m
J L/2
 VCOM =
M 
VCOM x=l/2

L ML2 mv0
J =  L/2
2 12
L
V= VCOM 
2
J 6J L mv0l = I + mv1l ....(1)
V= 
M ML 2
l  v1
I ....(2)
v0
MV
J=
2

www.motion.ac.in
182 - Solution Physics (XI) Topic

Solving (1) & (2)


 Ml2 
mv0l = I + mv1l ....(1) I  
6mv0 v0 3m  M  3 
 , v1   v0
l 3m  M 3m  M
l = v1 ....(2)
 KE decrease Solving,
On solving linear momentum increases.
3mv0 3mv0
 , v1   v0
M  3m l M  3m
49. D
 KE decrease
A.M. about A
 Linear momentum increases

A
51. C
2
1 v2
x=l/2 f
4m
m
mv0

R 2R

v1 f
l l
mv0  I  mv1 ....(1)
2 2 after slipping between the cylinders stops
then
 Ml2 
I   v1 = v2  f1R 1  w f2 R 2
 3 

1
l f
 v1  0 ....(2)
2  f1
R
Solving,

6mv0 3mv0
 , v1   v0 v1
l 3m  4M 3m  4M
 KE decrease
 f1  f2
 Linear momentum of P1R system
decreases for Ist cylinder +ve  – ve

50. D   f.Rdt  I  1 f1  I11 ...(1)


A.M. about A
for IInd cylinder

A  f.2Rdt  –I  2 f2  I2 2 ...(1)

2
x=l/2
 f2
mv0

394-Rajeev Gandhi Nagar, Kota


Rotational - 183

For pure rolling


I2 2 – I2 f2
 f.R dt  2
...(2) a = R F
F × R = I
from eq. (1) & (2) M
FR R
=
I2 2 – I2  f2  R
I1f1  I11  a
2 Smooth
F FR.R Surface
=
m 
I2 2 – I2  f2
2I1f2  I11  I = MR2
2
MR2 is the moment of inertia of chin pipe.
2 2 2
2MR MR 4M4R
.f2   100   200 – f 
2 2 2 2  2 
56. A
 is conserved about 0
f2  50  800 – 4f2
0 – mVR = 0 0 Circular dise
 5 f2 = 750  0 = mVR V0
O

f2  150 rad / sec , f1  300 rad / sec MR 2 f O


0 = mV0R
2

52. B 2V0 V0 1
0 =  R =
R 0 2
53. C 2V
57. B
V2 ( 2V )2 V2
= Centripetal force is necessary for a circular
R R R
path.
R = 4R
58. B
54. C 
P =  .   P=
1 1 Spherical shell
mgh = Iw2 + mV2 P= t
2 2
h
Constant
2
Ishell = MR2 90º
3 59. B
The FBD of different objects are as shown in
1 2 V2 1 figure.
mgh = × MR2 . 2 + MV2
2 3 R 2 For pulley
T × R1 = l11
1 1
mgh = MV2 + mV2 For cylinder
3 2
Mg – T = Ma & T × R = I
5 6gh
mgh = mV2  V2 = N
6 5 T

For pulley 
55. A 1

 F For cylinder
F = Ma  a= T
M M1g a Mg

www.motion.ac.in
184 - Solution Physics (XI) Topic

From constraint equation a = R + R11.


mg
Solving above equation we get the value of T   mg
3
T as,
For block to remain at rest
g T = f & f  mg
T
I I1  = 1/3
M 
R2 R12

62. D

60. D The portion of the strings between the ceiling


and the cylinder is at rest. Hence the points
As F > Mg, so the friction is kinetic in nature
of the cylinder where the strings leave it
and FBD of rod would be like as shown in
are at rest. The cylinder is thus rolling
figure.
without slipping on the strings. Suppose the
F – Mg = Ma c entre of t he c yl i nde r fal l s wi th an

2 a acceleration a. The angular acceleration of
L L ML the cylinder about its axis is  = a/R, as the
F  Mg   
3 2 12 F
cylinder does not slip over the strings.
Mg
The 2D view of the
F
Solving above equations, we get a   g cylinder is as shown in 2T
M
figure.
4F 6g The eq uati on of
and a   
ML L motion of the centre of
mg
Ini t i al ac ce l e rati on of poi nt A is, mass of the cylinder is, a
mg – 2T = ma
L F
aA  a   2g  For the rotational motion abou the centre of
2 M
mass,
Think about the importance of word initial.
1  1
2Tr   mr2   mra
2  2
61. A
The FBD of two would be like as shown in
ma
figure.  2T 
2
Solving above equation, we get
T
2 mg
a g and T=
T  3 6
F
f a As the centre of the cylinder starts moving
from rest, the velocity after is has fallen
through a distance h is given by
mg

I2 mv2
  mgh
For cylinder : mg – T = ma 2 2

mr2 a mv2 mv2


Tr      mgh
2 r 4 2

2g 4  4gh
Solving above equations, we get a  &  v2   g  h or v
3 3  3

394-Rajeev Gandhi Nagar, Kota


Rotational - 185

63. C  
 aCG  aCP  R 
5.625 cm right on the rod from the point
or a = R ....(3)
where 5 N force is acting.
Use (3) in (2)

64. B  MR  a Ma
T   
from figure  2 2 R
1R = a + 2 R Put this in (1),
mr 2 Ma
T1R = . 1 Mg   Ma
2 2
1R
2
mr 3Ma 2g
– T1R + T2R = . 2 Mg   a
2 T1 2 3
mg – (T1 + T2) = ma
T2
a
2 = 66. B
R
r
from the above equation
a = 2g/7 67. B
a   2R a
then v2 = 2as 2
1 2 1  R  v2
KE  I   
2 2  2  R 2
2.2g 3
v2 =  1.2 = 4 m / sec
7 7
1
Mv2
4
65. C
T
The spool undergoes 1 2 1
Total KE = I  mv 2
translation as well as 2 2
a
rotation.
1 1 3
For translation, mg Mv2  m2  mv2
4 2 4
Mg – T = Ma ....(1)
For rotation, about C 1
T Ratio =
3
MR 2
IC  
2 R
68. A
=T·R
As  = I 1 3 1
Mg
Disk = I  mv12
2 2
MR 2
T·R= · ....(2)
2 1 MR 2 v12 1 3
  mv12  mv12
Constraint equation 2 2 R2 2 4
 G
aPC  R  1 2 1 1 v2 1
Ring = I  mv22  mr2  22  mv22
 2 2 2 R 2
aCP  R 
    = mv22
aCG  aCP  aPG P C
R
 1
aPG = 0 mv12  mv22
4
(As point P and G are two points on an
inextensible thread). v1  4 
1 /2


v2  3 
www.motion.ac.in
186 - Solution Physics (XI) Topic

69. D 75. D
 =0
76. B
2h
a = gsin, t =
g sin 
77. C

78. B
70. C V0

////////////////////////
IAOR
 Vnet = 2V (0,5) (4,5)

Pure rolling

71. D
(a) M is instantaneous axis of Rotation
(I.A.R.) ///////////////////////////

(4,0)

(b) 10m/s
79. B
r=1m

10 5
Magnitude is same but direction is different 
2x x 5m/s
2x = 2 +x 
72. C x
x=2m
   = c Instantaneous point of rest

dw   d  cd
80. A
1 2 A vA 4m 4m B
dw  cd  w c
2
5m/s

3m
73. D
fr

fr = Mg sin 
= Mg cos  x
N

6m C
a
fr . = N.x = N
2
M
gs

5m
in

a

Mg sin   N
2
O

5 1
N   rad / sec
74. A f 10 2
O
To Balance torque
1
N shifts Downwards  vA = 6  = 6   3 m/sec
sin s 2
co
Mg M
g
Mg

394-Rajeev Gandhi Nagar, Kota


Rotational - 187

1. B
L ML2
2 r= cos 30º I1 + I2 =
MR 2 6
Cylinder = ,
2
L 3 ML2
MR 2 =  I1 =
Square lamina = , 2 2 12
6
2
ML2 M.L .3 2ML2  9ML2
2  I = + =
Solid sphere = MR2 12 8 24
5

11ML2
2. C I =
y
24
2 2
ML ML I2 I1
Ix = ; Iy = 6. C
12 12 y
L
Ix + Iy = Iz = I1 + I2 x 4ML2 M
Iz =
46
2.M.L2
= 2I1
12
ML2 x
=
ML2 6
 I1 =
12
7. A
P Q
3. C L/2
In case PQR 45º
r is larger. 2y
L/2
dy

4. A S R y

I1 x

2 M
I2 =
ML
2

dI = .2y.dy.y 
area
2

L/2
ML2  y4 
I1 =  L4 4 M L4
12 I = 2  4  = × = 2 ×
  0 2 16 L 2 16
l
l = R  R =  I1 < I2
 ML2
I=
8
5. B
I1 8. C
 = 4M/L2
y
is
ax

15º I
L
I1 y
30º 2
L r 15º dy L 
2  y
2 
x
L
I2

www.motion.ac.in
188 - Solution Physics (XI) Topic

L/2 Mg a Mg a
L  . + . = N. x
.2  y  . dy . y 2
I=  2 
3 2 3 2
0
Mga a
= mg x  x=
3 3
L / 2 L/2 
 L 2 3 
=  . 2 2.y dy  y

dy

 0 0  12. B
N1 N2
x=0
L 1  L  3 4 x
 L 
= . 2  2 . 3  2    2  4  
     
A B
Mg
1m 4m 1m
4M  L4 L4  ML2  1 1 ML2 N1 + N2 = (M + m) . g
= 2 .2   = .  =
L  3  16 16  4  2  3 
4 24 B = 0
N1 . 4 = Mg × 2+ m (5 – x)
9. C
Ix = I1x + I2x + I3x + I4x 13. C
y
  
 1x   3 x   = r × F = (– b î – c k̂ ) × a ĵ
= 2[I1x + I2x]    
 2x 4x  1
2 = (– b k̂ – c(– î )) = –b k̂ + c î
4mL 2
2ML 2 4 x
  2I4 3
24 24 14. D

ML2 
– T.dt = m.v – m × 5 ...(1)
I4 
24

mr 2
10. A  T.dt .r=
2
. ...(2)

Moment of inertia due to 1 about YY'


v
PQ2 3 = ...(3)
I1   PQ2  PQ2 r
2 2
Moment of inertia due to 2 about YY' mv
 T.dt =
2
PQ2 3
I2   PQ2  PQ2
2 2 mv 3v 10 m
5m – mv = ,5= ,v=
Moment of inertia due to 3 about YY' 2 2 3 sec

PQ2
I3  15. D
2

3 3 PQ2 7PQ2
I PQ2  PQ2  
2 2 2 2

N
11. B
Mg/3
1
fmax = Mg Mg/3 Balancing torque about B
2
f = Mg/3  N1 × 1.5 = 250 g × 0.9 + 250 a × 2
Torque Balance Mg N1 = 5500/3 N

394-Rajeev Gandhi Nagar, Kota


Rotational - 189

16. C 21. A

4 v

fr = ma
1
If N2 = 0  N1 = 2500 N mg xm = kx 2
Balancing torque about COM 2 m
2500 × 0.6 = 250 × a × 1 m
 a = 6 m/s2 2mg 2  4  9.8
xm = = = 0.98 m
k 80
17. B
1 2 1 1
N = 2500 mg × 0.2 = kx  mv 2  I2
ma = N
2 2 2
250 × 6 =  × 2500 = 0.6
22. C

18. C 2  600
L/ = –   80
4 60

3g L/4
=
L 2 
L/2 = = rad/s
By Energy Conservation 8 4

2
2
1 M L  3g  2  600  
×  × 0=   – 2 × ×
2 2 3 2 L  60  4

Mg L 
= (1 – cos ) q = 4 2 ×100 = 
2 4 2
 = 800 × rad
ML2 MgL
×g= (1 – cos )
4L 2 800
No. of ratution n =  400
2
1
cos  =   = 60º
2
23. A
19. C Let if pulley rotate by Pulley ’A’ moves up by ‘x’
String required = 2R – R = R
v1  r1 v1 r1
 = String released = 2x
v2  r2 v2 r2
2R
v1 r 1
= =
v2 2r 2
v2 = 2v1
y = 2x x

20. A

www.motion.ac.in
190 - Solution Physics (XI) Topic

 2x = R  2v = R
Ma
 T 2 – T1 = … (iii)
R 2
 v=
2
2g
From (i), (ii) & (iii) a =
7
24. B
Composite pulley
26. B
2R
R 

1 1
K.E. = 2 = (I) 
T 2 2
T
T Angular momentum is constant
T
L  constant   K.E. 

Mg
27. B
Inew = Iold + 2mR2
MR 2 = MR2 + 2mR2
T.2R – T.R = .
4
By angular momentum conservation
MR 2 2a Iold = Inew new
 T.R = .
4 R  MR2  = (M + 2m) R2 new

Ma T T
 T= …. (i) M
2 new 
M  2m
Pulley A :
Mg – 2T = Ma …. (ii)
From (i) & (ii) 28. C

g Mg L 3A 0
 a= Zav = =
2 t 4

25. A
Composite pulley – 29. B
By wall energy theorem
2R
R 

1 1 mL2 2
my =  
2 2 3
T2 T1

MR 2 2a 3g
T1 . 2R – T2R = . = ]
4 R L
Ma
 2T1 – T2 = … (i)
2 30. C
T2 T1
Pulley A : Angular momentum conservation
Mg – (T1 + T2) = Ma … (ii)
V
 a MVR = (MR2 + MR2) .  =
Mr 2 a 2R
(T2 – T1)r = .
2 r
r = radius of pulley
Mg

394-Rajeev Gandhi Nagar, Kota


Rotational - 191

31. A L.M.C. 2mV–mV=2mV


V = 0.5 V m
 . dt = I –0
x=0.5Vt+0.5b sin t y
t 0.5b cos t
y = 0.5 cost 0.5Vt
2  (1)2
10 × 1 = × 0.5 b sin t
3 3V m
where = x
b
 15 rad/sec
 = 15 rad/sec
36. B
1 2
2  (1) L.M.C. mV0 = MVCM
K.E. = × × (15)2 = 75 Joule
2 3
A.M.C. (A)

mV0
32. D ML2 C
mv0x =  x
12
(I + mR2) .  =  + mvR M
A 
L
(   mR 2 ).  mVR = VCM
 = 2 L
 VCM
2
A
L
 x
6
33. B 20º

L  to r and V 37. D
Direction of L J = M.VCOM
90º
vector is  to O
J
r and V. L  VCOM =
M

A
34. B J.L ML2 
= L

2 12 J VCOM +
L 6
Direction of L is continuously changing but
6
not Magnitude 6J VCOM
 = 
 Torque is present ML

 If there is a torque along the axis


then it will increase  B
  is constant  In H-direction
t
J
VCOM 
M

35. C
Angular Momentum conservation about
J/M
C.O.M.

b b  b2 
2m.v. + mv
v =  2m. . 
2 2  4  +0

6J ML  2J
3mvb mb 2
3V t = . =  V
 = .  ML 12J 2 M
2 2 b

www.motion.ac.in
192 - Solution Physics (XI) Topic

38. A 42. B m v

by conservation 60°
 ( V1  V2 ) 
e   , V0 of linear m
 u1  u2 
m momentum v
60° 60°
m v
L
0
2 L m
I=  =V
V 2
43. D
mVL ML2
= .
2 3
30º= 

mVL ML2 2V 1m
 = × 2m
2 3 L

M 3 
 =
m 4 line of impact

 1 m 2 2
39. C mg × sin 30º = × 
2 2 3
M M
COM 30º
MV 3g 3g 3g
= = =
2 4 2

  cos 30º 3g
 Ldt  change in angular momentum e=
'  cos 30 º
’ =  =
2

L 2ML2 V

MV sin30º =
2 4 2L 44. C

40. B Initially angular momentum = I (2n1)


J Am
Final angular momentum = (I + mx2) (2n2)
L
L
VCOM + L is conserved as ext = 0
J. = I 2
2 L/2
 I n1 = (I + mx2)n2

L ML2 VCOM
J. =2 . I n
2 4 2
 2
L/2 I  mx n1
m
J
 = B I n2  n2mx2 
ML 
2
n1  n2  I =n  n 
mx  1 2 
J = 2M VCOM

J
VCOM = 45. C
2M
When F1 is applied :
J J L J
Now, VA = + . = For pure rolling VP = 0.
2M ML 2 M
Thus friction is in backward direction. As
R2>R1, friction required for pure rolling will
41 B be less hence net translation force is in right
L = mvr which remain costant] direction. Hence COM moves towards right.

394-Rajeev Gandhi Nagar, Kota


Rotational - 193

46. A 50. B

L
f
2

f F L/4
L
8

47. C
L L
8 4

 There is no force in Horizontal direction


2m
C.O.M. will remain constant
v

L L
4 8

O 3L O
L
2 8 8
mR2 + mvR + mvR = mvR
vR 4
3 3 Quarter circle
It is not circle
L
Water is at rest w.r.t centre. with Radius
4

51. C
48. B IAR
/2
V 2v
g = = 60º /2
M  sin  
I


N 60º

in vc 2v
gs = , vC = v
30º
M  
2
Mg sin – Mg cos = Ma
a = g [sin –  cos ] ...(1)
52. A
(N)R Mgcos R
= = ...(2)
I  53. A
a is same for all N
 is maximum for hollow sphere.
So kinetic energy is more for hollow sphere.
F
b
3b/4
49. A f
A
g
sin

Mg

fmax = N ....(1)
a = g tan  a
f=F ....(2)
g cos 
A = 0 ....(3)
g
si


n 

O O
3b
 along inclined in Pseudo frame F. = Mg.b/2
4
acceleration = 0.It is a case of pure rolling.
f = F = 2 Mg/3
 f > N
 2Mg/3 > . Mg

2
>
3
www.motion.ac.in
194 - Solution Physics (XI) Topic

54. A
 r 
OA  L cos  sin1  sin     r cos 
a 3  F  L 
  a
F  2   mg 2
  The points O,A and lC from a right-angled
a a triangle.
3a
mg 2 Therefore we can use trigonometry to solve
 F 
3 a for rA and rB.

55. C rA
 tan 
Set up the solution oas drawn below, with OA
new variables introduced as shown. The line
rA  OA tan  (6)
rA is drawn as perpendicular to velocity vA.
The line rB is drawn as an extension of the
OA
line passing through point O and point B. rB  r 
cos 
Point B traces a circles during the motion of
the crankshaft. By geometry, the velocity vB OA
is tangent to this circle at point B. Also by rB  r (7)
cos 
geometry, the line rB is this circle at point B.
With vA known, substitute equations (6) and
Also by geometry, the line rB is perpendicular
(7) into equation (3), to solve for the velocity
to the velocity vB.
of point B(VB).
IC Therefore,

OA  r cos 
w vB  v A
OA sin 

rB rA

56. A
vB The distance between points A and B is d.
B
L vA
By similar triangles :
r

 vB  v A v v
A  B  A
d rB rA

The velocity of point A is given by


By geometry, the distance OA is given by vA = wrA
OA = L cos  + r cos  (4) The velocity of point B is given by
Also, vB = wrB
L sin  - r sin  Combining equations (8) – (10), it follows
that the angular velocity is
r 
  sin1  sin   (5)
L  vB  v A

d
Substitute equation (5) into equation (4) and
we get

394-Rajeev Gandhi Nagar, Kota


Rotational - 195

1. A,B,C,D
I1 + I3 = I  2I1 = 2I3 = I
I2 + I4 = I, 2I2 = 2I4 = I
I1 = I2 = I3 = I4 = I

NA NB
2. B,C A r
x d–x
N A + NB = W
W(d – x) = NA . d A w B
d 6. A,B,C,D
M = 2kg
3. B,C,D By FBD of particle
Body is in equilibrium mg – T = ma
T
So net = 0 or Fnet = 0 10–T = a ... (i) a
T
By FBD of disc
4. A,B,C m = 1kg
a MR 2 a
y-axis TR = I = L  T =
R 2 R2

T = Ma/2 = a …. (ii)
b By eq. (i) and (ii)
(A) a = 5 m/s2 and T = 5N and  = a/R =
x-axis
5 rad/s2
M M
(B) For angular displacement of disc :
m = t + 1/2 t2
a
(C) Work done by torque

1
(A) KE =
2
2  
= d   d  5  40  200 J

I depends on m (A) K.E. = w = 200 J

 KE depends on m K2 – K1 = 200 J

(B)  yaxis = 2Ma2


7. A,B,C,D

1
K.E. = × 2Ma22 = Ma22
2

 zaxis = 2(Ma2 + mb2) T1


m1
1
K.E. = I2 = (Ma2  mb2 ) 2
2
T2 m1g
m2
5. A,C,D
 is constant(mgr)
m2g
m2 g – T 2
= m2 (r2 )

www.motion.ac.in
196 - Solution Physics (XI) Topic

T1 – m1 g = m1 (r1 ) 12. B,C


T2 r2 – T1 r1 = I  ] 
No external torque about point A hence L
is conserved. Collision is elastic hence KE is
8. A,B,C,D conserved. Hing exert external impulse
Since pulley rotates cw, hence linear moment is not conserved.
T1 > T2
T2 T1
13. A,C,D
1
Kp = × I2 7kg
2 4kg

1
= × (6R2) / 2 2 = 1.5v2
2

1 If the ring is rolling then R = u


kblock = × 4 × v2 = 2v2
2 If CP is at 30° with horizontal
acm 
 7g + 4g + 6g – Fhinge = 17acm > 0
 Fhinge < 17g
After striking ground  is still cw but 4kg is
decelerating  string on LHS will betaut.

The resultant valocity is u.


9. ACD

14. B,C,D
10. C
After B there is no friction
1  Fnet  or acceleration 
LO = mv × 3R + mR2
2 F – f= ma

1 mR2 a
LA = mv × 0 + mR2 f.R= .
2 R
2

11. A,B,C,D , ma
f= acceleration became doulble
2
Angular impact 

1 2
J = L = m  vL 15. B,C v
3 Velocity of COM
is zero
3J
= I
m v v
=
R
1 1 2 9J 2 3J 2
K.E. = · m · 2 2 
23 m 2m 16. A,C

 3J  3J 17. B,C
VL =  =   ]
2 m 2 m If F < mg sin friction increase

394-Rajeev Gandhi Nagar, Kota


Rotational - 197

If F > mg sin  friction decrease


Direction of I
(C) About centre Inward
f F f

(D) No, Due to inertia

m 22. C
gs
 in

(A) Due to fricti on force i n horizontal
direction momentum to not conserved
(B) Torque by the firction forces about C.O.M.
18. A,B,C,D is not zero. So angular momentum is not
The force on centre of mass is mgsin. Thus conserved
for rolling friction opposes the motion of its (C) About P Torque is zero
O
centre of mass. Friction is responsible for
So, We can conserved f
rolling and provides necessary torque to P
angular momentum
rotate the body. For rolling the point of
(D) Mechanical Energy is not conserved
contact must remain stationary.

23. B,C,D
19. B,D
(A) Center of Mass wil also more with speed
Friction acting of ring may be more as
V.
moment of Inertia of ring is more than that
of cylinder. (B) Point of contact become stationary
Bcoz there is no slipping
F = ma = – N
20. A,B,C
ma = – mg
a a = – g
mg sin  – f = ma, f R = C mR2
R
So after time t
mg sin – C ma = ma f V = V0 – gt
g sin   Rotional Motion
a= g sin net = I
(C  1) m
f . R = 
For Ring C = 1
mgR = CmR2 (For Ring C = 1)
1 g = CR
For cylinder C =
2
g
=
R
21. A,B,C
From f = i + t

(A) gt
 = t   =
f R

v
= at Pure Rolling
R
(B) f v> r
So v = gt
initially
v0
gt = V0 – gt t =
2g

www.motion.ac.in
198 - Solution Physics (XI) Topic

V = V0 – gt
MV02  2M20
=
V0 V0 4
V = V0 – g . t=
2g 2g
MV02
 –
V0 4
V = V0 –
2
(C) Loss in K.E. = Kf – Ki
V0
V= MV02
2 =–
4

(D) Gain in Rotaional K.E.


24. A,C,D
Moves Linearly Vf2 = u12 + 2as 1
= 2
2
2
 V0 
  = (V0)2 + 2(– g) S
 2  1 V02 MV02
= MR2 2 
2 4R 8
V02 3 V02
– V02 = – 2gS S=
4 8g
25. A,B,D
Work done by Friction (A) Change in Angular Mom.
(W.D.)f = kf – ki

1 1
kf = mVf2 + 2
2 2
V0
1 v2 1 V2
= m 0 + MR2 . 02 O
2 4 2 4R

= Lf –Li = ( – mV0R) – ( + mV0R)


mV02 mv 20 2mv 20 mv 20
= + = = = – 2mV0R
8 8 8 4
(B) Impulse

MV02 1 = Change in momentum


(W.D.)f = – MV02
4 2 = – 2mV0R

394-Rajeev Gandhi Nagar, Kota


Rotational - 199

2  (2 î  ĵ  k̂ )  ( 2 î  5 ĵ  6k̂ )
MR 2  4R 
1. – M 
2  3 
+ ( î + ĵ + k̂ ) × (– î +2 ĵ – k̂ )
2
 4R   ĵ
ICOM =I–M    = – 14 î +10 – 9 k̂
 3 

4R
2
2  4R  3
MR mg
= – M  I 6. P= cot  A
2  3  2
A = 0
B
2 
14mr mg cos – P sin = 0
2. 2 mg
5

I = 2 [ICOM + MR2] mg
p= cot 
2
2 2 2 14
= 2  5 MR  MR  = MR2
  5
7. (i) 10/13 m/s2, (ii) 5000/26, (iii) 480/
13 N

3. 2r 1
4g – T1 = 4a  =  (10)2
2
MK2 = MR2 + MR2
T2 – 2g = 2a
K= 2R
40r 2 a
(T1 – T2) r = .
2 r
4. w  sin , when the bob
is at the lowest point 
10
 =  sin  2g = 26a, a= m/s2
13
=0
When  = 0º W
8. 1

5. –14 i  10 j – 9k


r1 = [1 – (–1)] î + (1 – 0) ĵ + (0 – 1) k̂
mg – T = ma  T = Ma
= 2 î + ĵ – k̂
mg Mmg
a=  T = Ma =
 Mm Mm
r2 = [0 – (–1)] î + (1 – 0) ĵ + (2 – 1) k̂

h h Mmg h h
T  Mg   Mg
= î + ĵ + k̂ 2 4 Mm2 4

    M M
So 
 = r1 × f1 + r2 × f2 2=1+   1 Ans.
m m

www.motion.ac.in
200 - Solution Physics (XI) Topic

 2gh  12. 2k kg m2 / s


9. M = 2m 2 2 – 1
R  

mg – T = m . a r = î + ĵ

 mg – T = m . a   
L =m(r × V)
mR 2
T.R= . = 2 [( î + ĵ ) × (2 î + 3 ĵ )] = 2 k̂ kg m2/s
2

ma
 T= , 13. 0.5 kg – m2/s, 75 J
2
ma2 2  100 50
2m.g Id = = = kg cm
a= 12 12 3
(2m  M)

50  104
= kg m2
3
2g a
10.    a/r
5 T1
m m 50  104
0.10 = . =  = 60 Rad/sec2
mg – T2 = ma ...(1) 3
T2

mr 2 a  =  × t = 60 × 5 = 300 Rad/sec
m a
(T2 – T1)r =  = .
2 r So L = 
mg

ma 50  104 m2
T 2 – T1 = ...(2) = × 300 = 0.5 kg
2 3 sec

T1 = ma ...(3)
1 1  0.5  102 
E= 2 =   (9 × 104) = 75 J
ma 2 2  3 
mg – ma – ma =
2

ma 14. 19.7 rad/s


mg = 2 ma +
2 Inew = Iold + mpR2
= 0.5 + (0.2) (0.2)2

11. 16 kg m2/s y
= 0.5 + 0.008 = 0.508
So  = new new
3
tan  = 8m
4 /s 0.5
new = × 20 = 19.7 rad/sec
0.508
  = 37º r

L = mvr 
x 15. 60° east of south, 30° south of east.
5 
5  ,0 
= 2 × 8 × sin  3  V  velocity of man w.r.t. ground
3
2R
5 3 t=
= 2 × 8 × × = 16 kg m2/s v
3 5
0 = mvR – 

 22mR 2 

mvR =   mR 2  
2 
 

394-Rajeev Gandhi Nagar, Kota


Rotational - 201

mvR  2 v 3g  1
= 2 = N = mg – m . .
24mR 12R 4 2 2
V
 O
13
 =   = t R N= mg
t 16

v 2R    3mg  3
 = × f r = m  2  sin60º = . .
12R v   4 2 2

 3 3
= = 30º fr = mg
6 16

16. (1/2)KE0
v0 2v
18. ( ), 0 ()
1 MR 2 3 3 m 2m
1
K.Ei = 02, = 02 A V0 B V2
2 2 2 mV0 = 2mV2 – mV1
V0 = 2V2 – V1 ...(1)
I10 = I2 l
l 
1
MR 2 2MR 2 mV02
I1 = , I2 = 2 B A
2 2 V1
0 2m M
1 1
2 = . 2mV22 + mV12
MR 2  2 2
0 = 2MR  = 0
2 2
2 V02 = 2V22 + V12...(2)
From (1) & (2)
1 1 2MR 2 20
K.E.f = 2 = . .
2 2 2 4 2
 V0  V1 
V02 = 2  + V12
 2 
1 MR 2 02 1
= , K.E.f = K.E.i 2V02 = (V0 + V1)2 + 2V12
2 4 2
2V02 = V02 + V12 + 2V1V0 + 2V12
3V12 + 2V1V0 – V02 = 0
3g  
17. (a) (cw ) (b) N  13mg , F   3 3  mg   2V0  4 V02  4  3  (– V02 )
4L 16  
 16  V1 =
23

3 3 2V0  4 V0 V 2V0
(c) V1 = = 0 , –V
V0, V2 =
16 6 3 3


(a)  = mg cos 60º
2 7
19. mv 2
10
m 2 mg
= .  = v/R
3 N
60º 1 2 1
3g So K.E. =  + mV2
 = (clock wise) fr 2 2
4
2
1  2 mR 2  V 1 7
   =   + mV2 = mV2
(b) mg – N = m  2  cos 60º [In vertical] 2 5  R 2 2 10
 

www.motion.ac.in
202 - Solution Physics (XI) Topic

20. 50m/s
Ft 6Ft 2F 2 t 2 Ft
2V = 50 m/s 24. (a) (b) (c) (d)
m m m 2

(a) Ft = mVC
10gh
21. Ft
7 VC =
m

1 1 V
mgh = 2 + mV2,  =  F
2 2 R (b) t = F. t = IC 
2

1 2 V2 1 Ft
mgh = . mR2 . 2 + mV2, = =  C
2 5 R 2 2

7 10 Ft 6Ft
= mV2, V = gh = = m
10 7 m 2
2
12

10
22. gsin 25. Topple first
7
l
7 Cube
mg sin  = mV2
10 //
//
//
//
//
//
//
//
10  mg sin  //
g sin 
//
V= mg cos 
//
//
7 //
//
 //

17 Toppling
23. mg
7 a a
mg cos   mg sin 
2 2
7
mgR = mV2 tan  = 1
10
N  = 45°
10 Slipping
 mV2 = mgR
7
tan   
mV 2
mV 2 mg   = 60°
 N = mg + R
R So, block will topple first.

10 17
N=mg+ mg = mg
7 7

394-Rajeev Gandhi Nagar, Kota


Rotational - 203

Ml 2 after solving
1.
2 m 2 4 2
T . 
dx  9
x Put the value of 
l
m 3g 4 2 4
T= . . = mg (due to circular)
  9 3

l Total jension at point A is

2M
Ttotal = T + g (due to weight)
3

M 4 m
 = , dI = xdx x2  mg  g = 2mg
1 2 3 3

2

 3Ft 2
3  4 2M  4 M2 4.
0

=  x dx , =
4
, = 2 .
 4
=
2
2m

L
=F. = 
4
2. 2mR2 2MR
2

A R FL FL 3F
2MR 2  2MR 2 = = =
4 2 ML
2 4ML
12

3. 2mg
1 1 3F 2
From energy conservation = t2 = . t
2 2 ML
L 1 m 2
Mg   2 L/3
2 2 3
A  b – a 
5. a  g
3g  3b  a 
2  2L/3

Mg – T + 2f = ma ...(1)

m f
Now dT = .dx 2 ( – x ) f T
 N

for Te ns i on at A we i nt eg rate about


experssion with time 0 to 2/3

T 2 / 3
m N a/2 Mg/2
  dT   dx2 ( – x ) Mg
0 0

Nb Nb Ta
  ...(2)
L/3 2L/3 2 2 2

T + dT T f = N ...(3)
A
mg
l–x dx x T–  ma' ...(4)
2
www.motion.ac.in
204 - Solution Physics (XI) Topic

6. 1.63 N, 1.224 m
mr 2
(a) when block x moves upward then T1R = . 1
2
mg sin  – T = ma
mr 2
1 – T1R + T2R = . 2 1R
T  (g / 2 – a) ...(1) 2
2
mg – (T1 + T2) = ma T1
Due to motion of Y
a
T×r=I 2 =
R T2
1 from the above equation
 = a/r and I  (2) r 2
2 a = 2g/7 r

then v = 2as
2
1 a a
 T.r = (2) r 2 . a   2R
2 r 2.2g 3
v2 =  1.2 = 4 m / sec
T=a ...(2) 7 7
from (1) & (2) T = 1.63 N
(b) from energy conservation 8. 3
Given Mrod = 0.75 kg Mring = 1 kg
1 2 1 2
mgh = mv  I  L = 40 cm
2 2
from angular momentum conservation
y
 Mr 2 2
  Mr 2 2

 2 mr (30)   
2kg
 12     12  2x  
T    
a
 0.9 = (0.01 + 2x2) 
g
5k
0.

0 .9
 
30° 0.01  2x 2

h = l sin 30° = l/2 Now 2x = a

vdv 0  0 .9 
  2
x
dx  0.01  2x 
u
l 0.1 2
h  0.9 
30°
G.P.E = 0
 vdv  
0.2

 0. 01  2 x 2
 xdx

v 2 18
from v = r    v = 3 m/sec
2 4

 1 1 1 2 2
 (0.5) g = × (0.5) (r)2 + . 2r 
2 2 2 2  9g 4mg
9. (a)  (b) 
7 7
 = 1.22 m
2
M2  7m 2
IC  ICM  Ma 2  IC =  M  
12 4 40
3 200
7. (a) 4 m / s, (b) N
7 7
 7M 2 12g
from figure Mg   =
4 48 7
1R = a + 2 R

394-Rajeev Gandhi Nagar, Kota


Rotational - 205

(b) Centre of mass


C
A B
 x
x
 dm.x xdx   x1  a dx
0
l/4
 dx   x 

12g 3 9g
0  dm 32 a 0
3
2
a 0
at = . R = . 
7 4 7
a
Now mg – N = ma = m (.R)  x 2 x3 
   a 2 a2
2 3a  2
m.12 g  2mg  x 
 0
 2 3  5a  2 = 5a
mg – N = . = 3 3 6  3a 9
7 4 7 a0 a
2 2
4mg
N=
7
×
(c)
x dx

10. Axis of Rotation
3 a
 x 2
 MR 2 
  0  a 
dI  .dx.x2 = 0 1   x .dx
MR 2  2
0 =  2  MR  
2   O P2 a
 2 x3 
= 0  x  a  dx

0 3  0 
= ,  = 0
2 2 3 P1 S
a
 x3 x 4   a3 a 3  7a 3  0
 0     0    =
 3 49  0 3 4 12
3 0 a 5a 7a 3  0
11. (a) (b) , (c) , (d) Angular Impulse = change in angular mo-
2 9 12
mentum
18P M 7a 3  0 12
(d) , (e) 70ag P.a = I   Pa =   
7Ma 9 12 7a 2 0

 x
Given    0 1  
 a

(a) dm  d 5a/9

(e)
× G.P.E = 0
A × B A 5a/9 COM B

x dx
from energy conservation
a
1 2 5a
a  I  Mg.
 x 2 9
 
dm  0 1   dx
0  a
1 7 3 3 5a 20g
 a 0 2  a0 g. 0  2 
2 124 2 9 7a
a
 x2   a 3
= 0  x  2a   0  a  2   2 a0 I
 0   Now Impulse =
a

www.motion.ac.in
206 - Solution Physics (XI) Topic

12. 2F/M
M
From =I Mf = (R / 2)2  =
4

F M  2 6F Initial P.E. = MgR


    A F
2 12 M
Mg R MgR
at point B. aB = at — a final P.E. = 
4 2 8
a = F/m
 F Change in potartial energy = increase in
= . –
2 m rotational K.E.

3F F 2F B mgR
= –  aB =  mgR –  K T  KR
m m m 8

7 1 M 1
13. 2m/s  mgR   v 2  I2
8 2 4  2
from v = r   3 = (0.3) 
2
 7 1 m 2 1 1  M  R  R
mgR  v     . 
8 2 a 2 2  4  2  2

7 3
 mgR  mv 2
A 8 16
v v’
r'
B String
r v gR
 v  14
f 3

 = 10 rad/sec
Both point A & B are together when 15. 1/2 ma
v – r = v  3 – (0.1) 10 = v from  = I 
v = 2 m/sec
mR 2 f m
f .R  . gs
2 in

14gR 2f
14. v=  
3
mR
Let  is the mass density of the material Now a =  R
then
M = (R2l)  2f ma
a= .R  f
mR 2
when radius R/2 then

394-Rajeev Gandhi Nagar, Kota


Rotational - 207

1. C
For pure translatory motion, net torque M

about centre of mass should be zero.


 m.R
Thus, F is applied at centre of mass of
m.R M
system.
l ie, I1 = I2'
(0,0)
X
 mR2  (mR2  2MR2 )'
P(0,2l/3)

Y
 m 
 '   
 m  2M 
(0,2l)
C
4. C
m  0  m2l
OP  1 The situation is shown in figure.
m1  m2

where m1 and m2 are masses of horizontal Q


and vertical section of the object. Assuming
object is uniform.
A B
m2 = 2m1
P
2l 4l
OP = , PC =
3 3

D C
2. D IXX' = m × DP2 + m × BQ2 + m × CA2
The mass of complete (circular) disc is
2
The moment of inertia of disc about the given  2l 
 m 2     m  ( 2l)2 = 3ml2
axis  2 
 
2Mr 2
is I   Mr 2
2
O 5. B
Let The moment of inertia As the amplitude is increased, the maximum
of semicircular disc is I1. acceleration of the platform (along with coin
The disc may be assumed as long as they does not get separated)
as combination of two semicircular parts. increases.
Thus, I1 = I-I1
Coin
2 Platform
I Mr
or I1  
2 2

Equilibrium
position
3. D
As no external torque is acting on the sys-
tem, angular momentum of system remains
Performing SHM
conserved.

www.motion.ac.in
208 - Solution Physics (XI) Topic

If we draw the free body diagram of coin at


2Mgh 2gh
one of the extreme positions as shown, then or v2  
M  I / R 2 1  I / MR 2
from Newton's law,
If s be the distance covered along the plane,
then
2A
N h = s sin 
mg
C
s
2gs sin 
mg – N = m A 2 v2 
1  I / MR2
For loosing contact with the platform, h
Now, V2 = 2as
N=0
2gs sin  C
g 2as 
So, A 1  I / MR 2 v
2

g sin 
or a
6. C 1  I / MR 2
The torque about the given posi tion,
=r×F
9. C
y
Let the each side of square lamina is d.
F
So, IEF = IGH (due to symmetry)

O x
and IAC = IBD (due to symmetry)
r Now, according to theorem of perpendicular
z
axes,
(1,–1)
IAC + IBD = IO  D F C
Here, r = i – j 2IAC = IO ...(i)
and F = –Fk H G
and IEF + IGH = IO O
 = (i – j) × (–Fk)
 2IEF = IO ...(ii)
= F[(–i × k) + (j × k)] A B
From Eqs. (i) and (ii), E
= F(j + i) = F(i + j)
we get
IAC = IEF
7. C
According to the principle of conservation md2
IAD  IEF 
of angular momentum, in the absence of 4
e xt ernal torq ue , the total angul ar
momentum of the system is constant. md2 md2  md2 
   as IEF  
12 4  12 

8. A
Assuming that no energy is used up against md2
So, IAD   4IEF
friction, the loss in potential energy is equal 3
to the total gain in the kinetic energy.
10. A
1  v2  1
ie, Mgh  I   Mv2 In this question, distance of centre of mass
2  R 2  2
of new disc from the centre of mass of re-
maining disc is R.
1 2 1 
or v  M  2   Mgh
2  R 

394-Rajeev Gandhi Nagar, Kota


Rotational - 209

12. D
T.Ei = T.Er

1 2
R I  mgh
O
2
O2 O1 n
1 1 2 2
R  ml   mgh
2 3

1 l22
Mass of original disc = M  h
6 g
M M
Mass of disc removed = 2
 R 2 

 2R  4
13. A
F = 20 t - 5t2
M 3M
Mass of remaining disc = M  
4 4 FR
  4t  t 2
I
3M M
   R+ R 0
4 4 d
  4t  t 2
dt
1
  
t
3 2
  d   (4t  t )dt
0 0
Note : In this question, the given distance
must be R for real approach to the solution.
t3
   2t 2 
3
11. D
When direction is reversed,
2
ma  = 0, i.e., t = 0, 6s
Moment of inertia of square plate xx' is ,
6 Now, d = dt
therefore moment of inertia about zz' can
be computed using parallel axes theoram  6  2 t3 
d   2t   dt
0  0
 3
z'
x'
6
 2t 3 t 4 
    
 3 12  0
z
 144 - 108 = 36 rad

 36
 Number of rotations, n   6
2 2
a  correct choice : (a)
x

2
 a  14. C
Izz'  Ixx'  m 
 2 1
MI  MR 2  mx2
2
ma2 ma2 2ma2
  = Where m = mass of insect,
6 2 3
and x = distance of insect from centre.

www.motion.ac.in
210 - Solution Physics (XI) Topic

Clearly, as the insect moves along the 19. A


diameter of the disc, MI first decreases, then
increases. Mx 2
I=
6
By conservation of angular momentum,
angul ar sp ee d fi rs t i ncrease s, t he n edge length : (x)
decreases.
2R = 3x

15. A 2R
C.O.A.M. (about bottom) x=
3

v cm Now, mass of cube :


mr20  2mr2
r
M 3
 2R  R
r 4
Vcm  0 m =  R 3 
  
2 3   3
R

16. A  3M   8R 3 
     
 L 3 3 3 
L  4R   
L vector rotates in
direction but magni-
2M
tude remains con- m=
stant. 3

1  2M   4R 2  4MR 2
17. D I=     =
3  3   3  9 3
mg – T = ma .....(1)
TR = I 
20. A or C
a
TR = mR2 By right hand thumb rule
R

T = ma ......(2) T
21. D
mg – ma = ma
m The radius towards the left side of O is
a = g/2
smaller. Thus system turns towards left.

18. C

x3
y=
6

dy 3x2 x2 f
= =
dx 6 2

f =  mg cos  = mg sin 
 = tan 
m
2
x
0.5 = x=1
2

x3 1
y= = m
6 6

394-Rajeev Gandhi Nagar, Kota


Rotational - 211

1. C about O is conserved
Block held fixed f = mg f Li = Lf
O
normal force N = F F 2
 2 mL  L
normal may produce 2a mg mvL = mL  3   M
 
torque because
m
it may shift F will not produce torque due to 3mv
passing C.O.M.  = L(3m  M) v

2. B
2g sin 
2
5. aaxis 
9mR 3
I0 =
2
g sin 
Moment of inertia of From Notes ; a =
1c
R/3
R/3 disc about is 2R/3
1
For cylinder ; c =
2 O 2
 2R  R
I = Icom + M  3 
  9m

cot 
m(R / 3)
2
4R 2 6. f  (M  m) g
=  M 2
2 9
System is in equilibrium A

MR 2 S0 2N1 = (12M + m)g ..(1)


I=
2 Torque about A mg
N1 N1
1
9MR 2 MR 2 Mg cos+fLsin=N1Lcos mg mg
remaining disc =  = 4MR2 2
2 2 f f
Mg cos 
fsin = N1cos  –
2
3. B
Put the value of N1 from equation (1), we get

(M  m)
f= g cot 
2

u
O m 7. A M Sphere
R
2 2
I= MR
5

Ma2 a
Now, I=  Ma2 I
 2
L is not constant due decreasing in speed,
only direction of L will constant. 2 Ma2
 MR 2   Ma2
5 2

3 mv 2R
4.  a=
(3m  M) L 15

Angular momentum

www.motion.ac.in
212 - Solution Physics (XI) Topic

8. C,D equation (1) / (2)


If friction is sufficient for pure ralling
x1
 2
mg sin  x2
then f =
1
1
C
12. A
1
For cylinder C = From angular momentum conservation
2
(Friction is internal force)
mg sin  mg sin 
f= , f= 2 (I) + (2I) = (2I + I) '
3 3

A  f  4I 4
' =  
3I 3
In pure rolling friction opposes translation
motion and support Rotational
  t = change is angular momentum
(Provided Net Tarque) friction

9. A,B 4 2I
(friction)t = I  2I  friction =
3 2t
At B

1 1 1 1
mv2  I2  At C mv12  I2 13. B
2 2 2 2
1 1
Initial K.E0 = I(42 )  .2I2 = 3Iw2
10. 10m/s 2 2
force due to ball
A 1 162 8 2
Final K.Ef = (3I)  I
2 9 3

B 8 2 I2
Loss in K.E. = – I  3I2 =
3 3
mg
Now if plate is in equilibrium then
AB = 0 14. D
From
1 1  3v2 
mv2  I2  mg  4g 
 .dt  chagnein A.M. 2 2  

b 3b v
Mg = n. m. v(A) Put  =
2 2 R

3  10  ab  1 I 3 1
v = 100  0.01  3  2   m m mR 2
2 R2 4 2  I= (disc)
2

ab 2
Area =  1m  v = 10 m/sec
2 15. D
Force at centre increase linear velocity
11. C

1 2 1 2
Kx  I(2) .......(1) F
2 1 2

1 2 1
Kx2  2I()2 .......(2)
2 2

394-Rajeev Gandhi Nagar, Kota


Rotational - 213

16. D 2kx – f = MaCM


P
1
For solid cylinder C = MR 2  aCM 
2 fR =
h 2  R 
For hellow cylinder C = 1

sin  Q MaCM
a = g f=
1 c 2
aHC < asc So tHC > tSC for slipping to start f should have its maxi-
Kinetic energy of both cylinder is same mum value i.e., static friction
(mgh)
MaCM
f= = Mg
17. D 2
For translational motion f = 2gM  aCM = 2g

4kx
we know aCM =
3m

4kx
 = 2g
3m

6Mg
x=
2kx – F = Ma .....(i) 4k
for rotational motion
Hence at this x slipping will start the veloc-
MR 2 a ity required to attain this displacement is
FR = l =   ....(ii)
2 R 
1 1 1
2× kx2 = kv2 + l2
2 2 2
ma
from (i) and (ii) F =
2 2
 6Mg  MR 2 v 2
K  = kv2 +
4kx  4k  2 R2
a=–
3m

4kx 3M
Force = Ma = – (D) v = g
3m k

18. D 20. A
From the above question it is evident that
Fexternal = 0
net restoring force
P = consereved
4kx 4k
F=–  =
3M M C
21. B,C 2v

VA = 0, VB = V B v
19. C
and VC = 2V
A
22. B
for sliding
tan  =   = 60º
N shifted downwards to stop the toppling
for limiting conditions

www.motion.ac.in
214 - Solution Physics (XI) Topic

26. B
N cm
15 f dL d
= = (I)
dt dt
10cm
mgsin
dI d
 =  = (Irod  Im )
dt dt

N . (5) = f . (7.5) wd
mg cos (5) = mg sin  (7.5)
as Irod = com  =
dt

I in sec t 
5  10 2  dr 
tan     33.69º d
75 3 = (mr 2 ) = m  2r  = 2m rv
dt  dt 

= 2m(vt)v   t
23. 0004
stick
2–f2 =2a=0.6  f2=1.4
27. C
a L0 remains cons. in magnitude and direction
=I  (f2–f1)R=MR2 R
2N but LP changes its direction continously
hence LP is variable
1.4 – f1 = Ma = 0.6
f1
P
f1=0.8 = (2)= ×2
10 f2 LP (varies direction)
L0
P=4 vx

24. 9
4 cm
5 28. 0003
r= cm
2
M/4
4 cm 4 cm
m = 0.5 kg 22
MI of solid sphere
0 M
2 4 cm P
Ic = mr2
5

I = [2(Ic) + 2 [Ic + m(2 2 )2]] × 10–4 kgm2 Let be the density off disc.
= [4 Ic + 16 m] × 10 –4

M M
  
8  5   (2R)2 4R 2
=  5 m 4   16m × 10–4 = 18 m × 10–4
    Here M  Mass of disc without cavity
y

= 9 × 10–4 kgm2  Mass of cavity =  x R2


N=9 = M/4Rx R= M/4
Io = MI of disc with non cavity

25. C - MI of cavity (About O)

During collision friction is impulsive and im-


1  1  M  M 
mediately after collision the ring will have a Io   M(2R)2     R 2  R 2 
2  2  4  4 
clockwise angular velocity hence friction will
be towards left.

394-Rajeev Gandhi Nagar, Kota


Rotational - 215

4MR 2 1 M
Io   MR 2  R 2
2 8 4

16MR 2 3MR 2 5R
= 
8 8 R
0 2R
2
13MR
= 31. D
8
Given  is same.
Now,
Ip = M.I. of Disc without 32. A,B
cavity about P - M.I. of cavity(aboutP) For point P

1 1  M  2 M 2 Vp=3R î –R/4 î
2 2
= 2 M(2R)  M(2R)   2  4 R  4 5R 
    
 
 P
3 ˆ
+ Rk
MR 2 4 30°
O
IP  37
8
3 ˆ
= 11/4 R î + Rk
IP 37 4
   2.8  3
Io 13
33. D
IP > IQ
29. C y R
45°
At 45° P & Q both land x g sin 
Q
in unshaded region. a=
O 1  I / MR 2
P
Hence ap < a0
30. A
tp > tQ
Consider case (a)
Vp < vQ
And as  = v/R
Q
So P < Q
(inside B A (out of A
B
paper) paper)
34. 0008
Angular momentum
P
at t = 0 at t = T/4 conservation
I11 = I22
at t = T/2 at t = 3T/4
MR2
1  MR 2  2(mr2  mr2 ) 2
2
Q
2
50  0.4 
A(inside) A
= × 10
(outside) B B 2

50  0.4 2 
P
=  2
 
 2 6.25  0.22  0.22  2
 
Hence axis is vertical.
40 = [4 + 1] 2  2 = 8 rad/s
For case (b)

www.motion.ac.in
216 - Solution Physics (XI) Topic

35. C,D
3 27
N1 = 2N2 = m×3 2
+ 300 m = + 30
4 4
N2 = mg + 1N1
By torque balance: 3 147
v2= ; v2 = 7
4 2 4
l
mg cos  = (1N1 cos  + N1 sin ) l
2
39. 6
mg
cos  = 1N1 cos  + N1 sin  2
2 dI  dm·x2
3
1 = 0
dm =  × 4x2 · dx
mg
cos  = N1 sin  2  x
2 dIA  k· 4x 4dx
3  R 
mg
 N1 tan  = For B
2
R
2 = 0 2 kx5 2k
dIB   4x 4dx  ·4 x5dx
 N1 = 0 3 R5 3R 0

36. 0004 8k 9 8k R 6 4kR 5


 x ·dx .
3R 5 
 
Li = Lf 3R 6 9
0 = I – 2mvr
8k R10 8k R 5 9R
I = 2mvr IB  5
  · 
3R 10 3 10 4kR 5
2mvr 2  0.09  0.9 n=6
= = ×2=4
I 0.45  0.5  0.5

40. A,C
37. D
Li = Lf 41. ABD
8 r (t) = t3ˆi  t2ˆj
mR2 =  (I1 + I2 + I2 + Iring)
9
Given,  = 10/3 m/s3,  = 5 m/s2, m = 0.1
kg; at t = 1 s
 m  3 2 m 2 
8  2

mR  =
2
  8  5 R   8 x  mR  v =  (3t ) î + 2 t ĵ
2
9    

on solving we get v  (10 î  10 ĵ)

4R 
x= dv 
5 = a = 6t î + 2 ĵ
dt
 
38. 7 F = m a = 0.1 [6t î  2ĵ] = 2î  ĵ
 
m (r  v)
1 1 MR 2 v2
mv
v2 + ×
2 2 2 R2
 5
l =  3 k̂ Nms
3 3
m × 32 + m × 30 × 20 = m.v2    =
4 4   r F (t3 î  t 2 ĵ) × m (6t î  2t ĵ)

3 3 20
mu22 + m × 27 × 10 = m2 = m (2 t 3k̂  6 t3k̂) = k̂ Nm
4 4 3

394-Rajeev Gandhi Nagar, Kota


Simple Harmonic Motion - 217

1. A The standard equation is


F = ma y = Asin (t+) ....(2)
 F   x (for SHM) comparing equation (1) & (2)
A = 5m, =
 a  x
2
T   2 sec .
2. C 
The distance moved in T/4 time is A.
Hence in one time period distance travelled is 10. A
4 (distance in T/4 time) = 4A We know that
x = A sin  t
3. C a
 
 asin  t
y = A(1 + cos2ωt) y = 2Asin  t +  2
3

y = A + Acos2  t Vmax = 2A t 
6
Vmax = A × 2 Ratio = 1:1 
t
6
4. C Now v = a cost
y = 0.45 sin 2t  3 a2  a 3
a cos  
6 2 T T
7.5
 0.45 sin2t
100 11. A
sin 2t = 0.167 x = A sin  t if t = 1 is t=0
speed = acos2t
v  A cos t
 0.45  2 1  sin2 2t
2  
0.5  0.25  A  cos  (t  1)
6  3 
= 0.9 × 0.98 = 0.87 = m/s
3
At t = 2 sec.
5. B
amax = 2 A 2 1  A
3
 0.25  A  
(1.57)2 = 2 (1) 6 2 2
 = 1.57 rad/sec2 12. C
2   t
 1.57 3
T
T = 4 sec.
A
2
6. C
3
4d 2 y 9 3
 9y  0  2 = =
dt 2 4 2

7. B
Slope of F-x curve gives K A
2
13.5 T
slope  F  Kx  K  9 t
1.5 6
K A 2 3A
2  9  Vavg  
  3 , T  2 T 6 T
m 3
8. B 13. B
A particle appears only once at one of the
A 1sec B 1sec
extreme position in entire oscillation.
2sec 2sec
9. A
T
y  5 sin (t  4) Hence = 2 sec. T = 8 sec.
4
 y  5 sin(t  4) ....(1)

www.motion.ac.in
218 - Solution Physics (XI) Topic

14. C [COMBINATION OF SPRINGS]


Let particle A be the particle shown with initial 23. B
phase 135° and B be the particle at extreme. 1 1 1 1 1 1
   
Hence the phase difference between them is k eq k1 k 2 ; k eq k 2k
135°.
2
k eq  k
3
0
135 m 3m
T  2  2
k eq 2k
24. C
Both the springs are parallel.
A
2
1 k1  k2
15. D f
2 m

2
25. A
300 1
m
900 600 T  2
1 600
K
m m
T1  2 T2  2
2 K1 K2
As kl = constant
A A
2 Hence klll = k2l2
2 T k1L = k2 2L
t   t T
3 2 6 k1 T1 1 1 1
2 ;   
16. C k2 T2 2 2 2

2 0
1 [ANGULAR SHM & SIMPLE PENDULUM]
30
30
0
26. B
10cm 10cm 
Time period of both A and B T = 2
g
After first collision, B acquires amplitude of
A and after second collision it acquires its
Particle 1 and 2 are as shown and their phase own amplitude in this process time taken is
difference is 60°. T T T T 
= + + + = T = 2 g
4 4 4 4
[TWO BLOCK SYSTEM]
27. B
17. B
18. A
3° 6°
19. C
20. C A
C
When after come back they collide & after B
collision the blocks comes to rest as moving Time taken by pendulum in going from A to
with equal velocities in opposite directions, B
so each mass complete half the oscillation
T 
so =where T = 2
4 g
2 M / K M Time taken by pendulum in going from B to
T= =
2 K C

21. C T
=
12
22. D
 Time period of pendulum
T T 
= 2  
 4 12 

394-Rajeev Gandhi Nagar, Kota


Simple Harmonic Motion - 219

33. D
2T 2  2
= = . = sec Initially the COM of spere and water lies
3 3 5 15 at centre of sphere .As water flows out
Altier : the COM shi fts down and l ength of
240 pendulum increases hence time period
T = .T increases but when water level becomes
360
30° half of the sphere the COM again starts
2 shifting up and hence as length decreases
= T time period also decreases.
3
28. D 34. D
1
29. C K i net i c e ne rg y T m2 (a 2  x 2 ) and
2
m  1
Tp  2 Ts  2 potential energy, V  m2 x 2
K ga 2
Tp = Remain same Ts decreases T a2  x2
 
30. A V x2
When the lift is going down with constant
velocity the acceleration is zero. 35. C
When there is a retardation of 'a' the geffective K.E. = P.E.
is g + a. 1 1 A
m2 (A 2  x 2 )  m2 x 2  x = 
V = constant  a  0 2 2 2
So there is no effect. 2 2  A
So v =  A  x  {A = x0}
 2
T1  T  2 geff  g  a
g 36. A
T = 2 sec.
I
a move T  2
mgd
Time period of a second’s pendulum is two
seconds.

T2  2
ga second
pendulum
T1 > T2
31. C
mv 2
Tmax  mg 

v  A 2mR2
T 2  2 R =.5m
 g mgR
g
v 2  A2 v
 37. C
Tmax = mg[1 + (A/)2]
mg 38. D

32. A 39. D
5T 1
LCM of T &  5T mg (h + x) = kx2
4 2
So, Pendulum of time period T will complete  2mg  2 mgh
5 o.scillations.  x2–   x– = 0
 k  k
1/ 2
mg mg 1  2hk 
 x= + 
k k  mg 

www.motion.ac.in
220 - Solution Physics (XI) Topic

40. C 42. C
Let x0 be the extention in equilibrium 
a1 = 1, a2 = 1 
3

 a a12  a22  2a1 a2 cos 




B  a 3
A S
43. B
mg = kx0
1
After slightly displacing E 2
KAeq Aeq  A2  A2
mg – k (x + x0) = IA 2
1
 
2
m rod  2 E  m 2 2 A  2A
IA = m +
2
= 1.02 kgm2 2
3
E  m 2 A 2
– k
Solving,  = x 44. D
IA
E1 = E2
k
= 2I A 1 1
 m 2 A 2 = m 2 A 2
2 1 1 1 2 2 2 2
but m1 = m2
41. A
x = 2 sin t  12 × 16 = 22 × 25
   100 × 16 = 2 × 25
y  2 sin t  
 4  = 8 units

from Lissajous figures if   then the path
4
of particle is an ellipse.

394-Rajeev Gandhi Nagar, Kota


Simple Harmonic Motion - 221

1. C 5. D
2
2Ai   A f 
i
2
f
Total Accn = 0
Aii = Aff Total change in Velocity in one T.P.
aav . 
A Time Period
If f  2i and A f  i
2 0
The condition will be satisfied. aav  0
T
2. C 6. A
v v/2 A particle has same velocity between 0 &
Vmax and 0 and -Vmax twice in its motion.
A B C D E Only Vmax is a velocity which a particle attains
x
once in its one oscillation.

v  R x 2 2 R   R 2  x2
2 2

R2 3
 R2  x 2 x R
4 2
Distance  2 x  3R -A Vmax A

7. A
3. C 1 1
2 KA2  KA2
2 2
m
 A '  2 A  T  2
K
8. B
10 
y  2 sin  t    B
E.P.  3 2 
A 2A
9. B
x  A  A sin  t v2 = 108 - 9x2  v2 = 9[12-x2]
2.5 dv
t  2v  2 x  9
 dx
2 0.5
t   a  9 x  2 = 9
 
= 5A
[TIME PERIOD AND ANGULAR FREQUENCY IN
SHM]
4. A
10. D
Velocity is maximum at mean. To come back
 = t
t o me an t he p arti cl e has to m ov e
2 T
   2  ·
 3  3 . T 8
4
  

4
60
0  x
cos 
4 A
xA / 2 T/8

2 1
Hence t   sec .
3.2 3

www.motion.ac.in
222 - Solution Physics (XI) Topic

11. D 15. B
Total max displacement 135   3
Max. Average velocity  
Total time 180 4
T T
8 8
1

A A 450
2 450
Mean 2
2
Position 45
0

Maxm displacement will be close to


2
1
M.P.  2 A
4 2 3
Vavg  A t  T 
3T
T
4  2 8
12. D 16. C
Time period = 8 sec.
2
In 1st second = ×1
8 45
0
0
A 0 60
45
Displacement =
2
2 
In 1st second = 1
8

Displacement = A =
2
t=1
[TWO BLOCK SYSTEM]
A. 2
  2 1  17. A
A 2  2 1  y  sin  t  3 cos  t

13. D 
y  2 sin  t  
3  A 2  g

A cos=1.2 60
0

A 
t
   A sin 

A

A
g  2
A cos   1.2 -- (1)    t
2 6 g
A sin   1.6 -- (2)
18. C
tan   4
3    53
m2 m
 
3 2m 2
 A.  1.2  A  2m / sec.
5 It can be considered as a two block system
T
14. C for time
2
A
A = 2cm.  x  1cm  T m 2
2 t   t  1s.
2 2K 2. 2
 v   A2  x 2   3  a  w 2 .1

3 M M
  3  2 n  n 
2 M M

394-Rajeev Gandhi Nagar, Kota


Simple Harmonic Motion - 223

19. A m 2 2E
Elastic Collision  T  T1  T2  T   
K g m
20. A 25. D
2 2 2 26. B
t   t
3 T 3 27. C
T 2 m
t  t
3 3 K [SIMPLE PENDULUM + ANGULAR SHM]
28. C
Time Period


T = 2 [geff = 0 when the bor falls

3
g eff
freely  T =  ]
 Particle will move in circular path ]
29. D
2e e 30. C
21. A y = kr2
fs = kx
d2y
(where fs is frictional force on 20 kg block = 2k
and xis instantaneous elongation or compres-
dt 2
or ay = m/s2 (as k = 1m/s2)
sion in spring)
f s = k (A cos t) l l
T1 = 2  and T2 = 2 
 | fs | = kA |cos t| g ga

[COMBINATION OF SPRINGS] T12 g  a y 10  2 6


   
T22 g 10 5
22. B
 correct answer is (A)
m m m 31. A
T1  2 , T2  2 T  2
K1 K2 Keq
32. A
K1K2 2m T
Keq (for series) t12  , T1  2   T  T2  2  
K1  K2 K1 g 4g 2
2m 1 1 
t22  and T 2  2m   
Teq 
T1 T2

T T
 
3T

K2  K1 K2  2 2

2 4 4
 T 2  t12  t22 33. A
2
23. C 
TA KB 3
K Aeq  K , K B  3K   
3 TB KA 1
mg
24. C
for x < 0 perform SHM Simple Pendulum T = 2 /g
1   m 2 
mU 2  E  
2  = mg sin  = – I = –  3  
2  
m Gravity  3g 
T1   =–   
k SHM  2 
2
Time period = 2 <T
3g
2E 2 2E
U  T2 
m g m

www.motion.ac.in
224 - Solution Physics (XI) Topic

34. B 38. C
mR2
T  2
m Ix   mR2 Ix
----> not dependent on qeff. 2
K
3 X
the velocity of particle  mR2
2 R
at M.P. = V0  V0  A0 Ring
3 mR2
T  2
2 mgR
v0
A 3R 3
0 0 = 2 
V 2g 2
Initial phase is zero.
39. C
35. C
The moment of inertia of the hoop about
  the supporting knife edge is
T1  2 T2  2
g g  a2
2 I = MR2 + MR2 = 2MR2
1
g2  a2  T 
4 4
R
  1  g2  g2  a2
T
 1  
T2 g T 
2
O
 4  0.01
4

 g 2     g2  a2 
 4 
 g 2  1  0.01 g2  a2   a2  0.01g2
mg
 a  0.1g  a  g /10
Refering to figure, we have the equation of
[COMPOUND PENDULUM/PHYSICAL PENDULUM, motion
TORSIONAL PENDULUM]

36. B I  = – MgRsin
I
T  2 Or I  = – MgR


mgd
0 For small oscillation. Hence the frequency
60
is
 3 
2

m     1 MgR 1 g
 2  = = =
 2 2 2 I 2 2R
3
mg  40. A
2 Let I1 and I2 be the moments of inertia of
the two torsion pendula respectively. If A
3L is the restoring coefficient of each wire,
 2
2g
then the equations of motion are I1  + 

37. B
A = 0,
I
T  2 I2  + A = 0. Hence the angular


mgd
frequencies of oscillation of the torsion
2md 2
I
0
45
3 pending a are 1 A / I1 and 2 = A / I 2 .
For the first pendulum, I1 = MR/2, and for
2md 2 2 2
T  2 COM the second,
3 2m gd  M  R 2 2
MR 2  3R  
I2 = 
+2 2 4    M    = 27
2 2d 2     4   16
T  2
3g MR2 .
Hence the ratio of the periods is
3/2
T1 2 I1 2
= = =   Ans.
T2 1 I2 3

394-Rajeev Gandhi Nagar, Kota


Simple Harmonic Motion - 225

[COMBINATION OF TWO OR MORE SHMs]

41. B
AQ;BP;CR;DS

0 t

(a) KE

t

(b) PE

t
t

(c) TE t

(d) velocity t

www.motion.ac.in
226 - Solution Physics (XI) Topic

1. B,C,D 8. A,B,C
10 10
mg  Kxo xo   2cm v = A &   4
500 2.5
2 
500 rad (a) T    1.57
w   10 5  2
1 sec (b) a   2 A  40
 Maximum velocity = A = 3×10–2× 10 5
(c) v   A2  x 2  2 21
 30 5 cm/sec.
 Maxm Accn =A2 = 15 m/s2
9. A,C
900
2. B,C,D v 
3
22  12   10 3  3 =30 m/s
du
u=5x2 - 20x  F   10(x  2) M.C. 3 x 30= 9 x v  v= 10 m/s
dx
900
M.P. at x = 2m 10 
9
A
2
 12   A  2m
3. A,B,C
10. B,C
1 1
mv 2  m 2  A2  x 2 
2 2 11. A,D
1
 m A 0.64
2 2

2 12. C,D
1 1 1 x  a sin t .....(1)
m 2 A2  m 2 x 2  m 2 A2 0.64
2 2 2 y  a  a cos  t ....(2)
Put A = 10 cm.  x = 6 cm.
 y  a  x 2  a2
2
(1)2  (2)2
4. A,B,C

x  5 sin 4t  tan1 4
3 13. B,C
5. A,B 14. A,C
2
 R sin 60 a
2
 v .dt
2

Vrms 
 dt
60 0

15. ABCD

6. B,D
x  3 sin100t  4 1  cos100t 


x  5 sin 100t  tan1 4
3 4
M.P. is at 4 with A = 5

7. B,C,D
v 2   2  A2  x 2  F =-Kx
2
v
 x 2  A2 ---- Ellipse
2
v 2   A2   2 x 2  , a=-2x, a2= 4x2
 a2 
v 2   A2  2   v 2  a  A2 ..Ellipse
2

   2

394-Rajeev Gandhi Nagar, Kota


Simple Harmonic Motion - 227

[EQUATION OF SHM, VELOCITY AND [TIME PERIOD AND ANGULAR FREQUENCY IN


ACCELERATION IN SHM, ENERGY OF SHM]
SHM]
1. Amplitude = 5 m ; Initial Phase = /6
11  11
Maximum speed = 5m/sec 5. (a) m (b) sec (c) x = 0.2 – cos t
5 5 5
 
x  (5m) sin  s  1  t   F=-10x + 2 = -10(x – 0.2)
 6  x = 0.2 is M.P.
 x  A sin t   (a) Amplitude = 2 + 0.2 = 2.2 m
 2 0.1 
 A  5m,   ,T   2 sec. and (b) T  2  s
6  10 5
Vmax  A  5m / s (c) x = 0.2 - 2.2 cos t

2. (a) 2.0 cm, /50 sec, 100 N/m 6. ( i) x 0 = 2m (ii) T = 2 sec (iii) 2 3
(b) 1 cm, 3 m / sec , 100 m/sec–1 U=(x2 -4x + 3)
 
x = 2.0 sin 1 00 s  t  6 
1 du
F   2 x  4  F = -2(x-2)
  dx
A=0.2 cm, w=100, M=10gm
(a) For equillibrium F=0  x  2m
 
v  2.0  100 cm/s cos 100 s  1  t   2 2
 6  (b)    2  T   2
1 2
   
a  2  104 cm / s 2  sin 100 s  1  t    1 1
 6   (c) KA  mv  A  2 3
2 2
  2 2
 a  104 x  F  Kx and F=100 N/m
1 7. 25 2 N
 K  104  M  104   100N / m
100 Fmax = KA = 50
2 2  A KA
T    sec . x  F  Kx 
w 100 50
2 2

a at t=0  a  200 m / s sin Fmax
2

6 F   50 2 F  25 2 N
2
 100 m / s 2

   8. X = 10 sin (t + /6)


3. (a) sec. , (b) sec. (c) sec.
120 30 30 General equation
= 20 2
 x  A sin t   A=10,   
(a)    t T
6 At t = 0, x = 5  5=10 sin 
 /3

t     
120   x  10 sin t  
  2  6  6
(b)    
2 6 3
2  [TWO BLOCK SYSTEM]
t  
3  20 30 5
(c) Same as (b) 9. Hz , 5 cm
2
4. 2m/sec Given A=0.1m, K=100 N/m
2 m1 = 1Kg, m2 = 3Kg
T   0.4 sec .
5 Energy of system is
Total distance 0.2  7 not conserved.
Av. Speed =  = 2m/s
Total time 0.7 1 1
KA2  m1v12
2 2
KA2
 v1 
m1

www.motion.ac.in
228 - Solution Physics (XI) Topic

 F  Keq A

F  K2  K3  F
3Kg  A 
1Kg v
K eq K1K2  K2 K3  K3 K1
v1
1
1 Keq 1 K K  K K  K K  2

f    1 2 2 3 3 1
 v1  1 m/s 2 M 2  M K  K  
 2 3 
1
m1v1  m1  m2  v v 
m1  m2  [ANGULAR SHM + SIMPLE PENDULUM]
 v  0.25 m/s 13. 1m
1 K 1 100 5 
 freqn    H2 T  2  2    1m
2 m 2 4 2 g
Now amplitude
14. (i) 2T0 , (ii) 3 g upwards
0.25  4
2
1 1
KA '2  m1  m2  v 2  A '2 
2 2 100  3g g
T  2  geff  g  
 A '  5cm. geff 4 4

10. (a) sec , (b) 4 cm, (c) 2.40 kg m/sec 4 T0
10 (i) T  2  2T0 if T ' 
g 2
3 6 k
  2 Kg.  =  20
36   
(ii) 2 g   g  geff  4g
 
(a) T  2  eff

K 10
(b) A1 + A2 = 6 g  a  4g  a  3g 
m2
 m1 A1  m2 A2  A1  A [COMPOUND PENDULUM/PHYSICAL PENDULUM,
m1  m2
TORSIONAL PENDULUM]
A1 = 4 cm, A2 = 2 cm.
(c) m2Vmax = 6.A2 = 2.4   
15. (a) 2 a  g , (b) 2 g – a , (c) 2
0 0 g
[COMBINATION OF SPRINGS]

11.
 (a) geff  g  a0  T  2 g  a
7 0

Mg = K x 20 - K x 10 
(b) geff  g  a0  T  2 g  a
mg 0
K N/m
10 /100 
(c) geff  g  T  2
M g
T  2
K
17L
16. 2
1  18 g
T  2  sec
49  4 7 m 2 m 2 17
I   m 2  m 2
3 12 12
F(K 2  K 3 ) 1 K 1K 2  K 2K 3  K 3K 1 3
12. , d
K 1K 2  K 2K 3  K 3K 1 2 M (K 2  K 3 ) 4
M
17m2 17
K1 T  2
3  T = 2 18g
12.2mg. 
4
K2 K3 F
Keq  K1  K2 K3
K2  K3

394-Rajeev Gandhi Nagar, Kota


Simple Harmonic Motion - 229

[COMBINATION OF TWO OR MORE SHMs]


3
17. sec
2 3 5 –1  1 
T =2s 18. A , tan  
8 2
T
T1  tan  
1
2 2 3A/8

 T 3A 
2
3A 
2
T2  2  Anet      
4g 2  4   8 
2
T2 T T 3 3A 1 
T '  T1   T    s. 1   
'
 
2 2 4 2 4  2  3A/4

3 5
 A
8

www.motion.ac.in
230 - Solution Physics (XI) Topic

[ EQUATI ON OF SH M, V ELOC ITY AN D 4. 1.8 a


ACCELERATION IN SHM, ENERGY OF SHM] 
1. y = 0.1 sin (4t +/4) d  a sin
2 d
K.E.  8  10 3 J Max distance = 2d

1 
m2 A 2  8  10 3 J 2d  2a sin
2 2
0.1 2 = 2a x 0.9 = 1.8 a
 (. 1) 2  8  10  3
2
= 4 1
  45º   / 4 5.
3
x  (.1) sin ( 4t   / 4) K11 =K22 = K
K1  3 T1 K2 1
  2    
2. 65 m / s K2 1 1 T2 K1 3
8m/s 7m/s 4m/s
1 2 1
x 1m 1m 6. 10 6 cm , sin –1 – sec
M.P P Q R  3 6
As V  2 ( A 2  x 2 )
2
x  30 sin t   6
2 2 2
For P, 64   ( A  x ) ...(1) 1 1 
Kx 2  2  m 2  A2  x 2 
2 2 2 2  2 
For Q, 49   [ A  ( x  1) ] ...(2)
x2
For, R, 16  2 [ A 2  ( x  2) 2 ] ...(3)   0.09  x 2 
2
(1) – (2) 6
2
15   ( 2 x  1) ; (2) – (3)  x 2  0.06  x  m.
10
15 (2x  1)
33  2 (2 x  3 ) ; 
33 (2x  3) [TWO BLOCK SYSTEM]
1
x
3 7. 3cm, x = 10 – 3 sin 5t; E = 0.135 J
Putting the value in equation above
30 cm/s 30 cm/sec
= 3
65 m M
A , Max. Speed = A   65
3
2 k
1  we know that  
3. m
2  k = m2 = (1) (10)2 = 100 N/m
At t = 0 block of mass m is at mean position
F=-Kx x = 10 cm.
a
K dx
a x velocity of block m  v m   30 cos 10 t
m dt
at t = 0 vm = 30 cm/sec.
K 
 tan   from momentum conservation
m  x (M + m) v = M(30) – m(30)
  2 f v = 15 cm/sec
1  1 1
f  Now (M  m)v 2  kA 2
2  2 2
on solving A = 3 cm
(b) New  of the system having mass (M + m)
K 100
[TIME PERIOD AND ANGULAR FREQUENCY IN     5 rad / s
M m 4
SHM]

394-Rajeev Gandhi Nagar, Kota


Simple Harmonic Motion - 231

x  10  3 sin 5t (A) from figure (i) b = A + x ...(1)


(c) Losse of energy during collision = Energy from figure (ii) A = a + x ...(2)
before collision – Energy after collision from eq. (1) & (2)
b = a + 2x  2x = b – a
1 1 1
 m(0.3)2  M(0.3)2  (M  m) (0.15)2 and x = mg/k
2 2 2 2mg
 0.135 Jule  K=
b–a
1 K
8. 0.8t + 0.12 sin 10t (B) Oscillation frequency = 2 m
total
3kg 2kg
1 2mg
2m/sec =
B A 2 (b – a)(M  m)
(C) By energy conservation.
Vcom = 4/5
In frame of chita :-
3kg 2kg 1
10. f = ; E = 42 × 10–5 J; v =2 × 10–2 m/s
2-4/5 = 6/5 
4/5
B A (a) Both the spring have same force so.
x1
It is parallel equivalent of spring
x2
Let us assume that elongation in spring is x k eq  k 1  k 2  0.2 N / m
then Now the problem change in two block system in
x1 + x2 = x ....(1) which reduced mass is
2x1 = 3x2 ....(2)
m1m 2 0.1 0.1
(Centre of mass is at rest) m   0.05 kg
from (1) & (2) m1  m 2 0.1  0.1
2x 1 K eq 1 0.20 1
x1  1  x ...(3) f   H2
3 2 m 2 0.05 
from energy conservation (b) Balls are at rest in position A & B so
2 2 Total energy is in potential energy for
1 6 1 4 1
 2      3     kx 2 1 1
2 5 2 5 2 E  k 1x 12  k 2 x 22
x = 0.2 ...(3) 2 2
from (2) & (3)
x1 = 0.12 m A B
Maximum velocity = A = 6/5 x1 x2
x1 = 6/5 /6 /6
(0.12)  = 6/5 P Q
 = 10
then equation of block A
4 1 2 1 2  x1  x 2
x    t  0.12 sin10 t  kx  kx 
5 2 2  R1  R 2
E  kx 2
[COMBINATION OF SPRINGS] x  x1  x 2  R /6 R / 6
2mg  0.02  m
9. (a) K= ;
b–a Now E  kx2
 M  m  ab 1 2mg  (0.1) (0.02)2  4 2  10 5 J
(c)  m  b – a , (c) At P & Q no stretch in spring so complete energy
2 (b – a)(M  m)
is in the kinetic form
1 1
a  m1v 12  m 2 v 22  E
2 2
y A m1  m 2  0.1 kg
b x x
v1  v 2  v  0.1v 2  42  10 5
m.p.
A v  2  10 2 m / sec

(ii)
(i)

www.motion.ac.in
232 - Solution Physics (XI) Topic

[ANGULAR SHM + SIMPLE PENDULUM] [COMPOUND PENDULUM/PHYSICAL PENDULUM,


TORSIONAL PENDULUM]
2
11. (a) 2 sec. (b) T = sec.
51/ 4 1  (P0  mg / A )A
14. f
I 10 2 mh
(a) T  2  2  2s
K 10 2 mg
P  P0 
A
 Bulk modulus B = p
(b) T  2 g  P0
eff  P
 p x
v
m
geff  g 2  a2  5 5 v0
A
h

0.5  2 p V mg
T  2 P 
5
14
V0
5 5
 A p  v
Fnet  A  P 
12. 1m v0
 xA  v  xA 
 sin  s1  t   AP  
90 hA  v 0  hA 
   x
2   A P
T   2 sec. h
  mg  x 2  mg  1
 Fnet   A P0    k  mw  A  P0  
T  2  2    1m  A h  A h
g
A  mg  1 A  mg 
2   P0   f   P0  
13. 0.06 m mh  A  2 mh  A 
1 25
Kx 2  0.4   2 
2 
[COMBINATION OF TWO OR MORE SHMs]
0.8
x 
2
,  =50, K = 500
K
1 1 15. a 7 / 4
m 2  A2  x 2   0.5  A2  x 2 
2 500
A =0.06 m
a

7
 cos  
4

a 7
 a cos  
4

394-Rajeev Gandhi Nagar, Kota


Simple Harmonic Motion - 233

1. B 4. A
2
dx
  x ...(i)
dt 2
We know that l l  l
d2x
a  2  2x ...(ii) G
dt G G'
From Eqs. (i) and (ii), we have
Spherical hollow ball Spherical hollow ball
2   filled with water half filled with water
2 2
or      or T
T  l
T  2
g
2. A As water is coming out, CG of the system
  goes down, effective length increase.
Given, y1  0.1 sin100t  3 
  l  l
dy1   T1  2 , T1  T2
  v1  0.1  100 cos100 t   g
dt  3
So, time period also increases but upto half
   empty. As water is coming out after half
or v1  10 sin100 t   
 3 2 empty, CG goes up.
 5 
or v1  10 sin100t  
 6 
and y2 = 0.1 cost l
dy 2
  v 2  0.1 sin(t  )
dt G
Hence, phase difference
 = 1-2 Spherical hollow ball
filled with water
 5  Effective length decreases and for complete
100t    (t  )
 6  hollow sphere, CG is also at the initial position,
so effective length as well as time period
5 
   (at t=0) decreases and same as initial at last.
6 6
l  l
3. B T2  2
g
Here, y = sin2t
and T1 < T2
y Hence, time period first increases and then
decreases to the original value.
O
 2 3 4 5. A
    KE of a body undergoing SHM is given by
1
KE  m2 A 2 cos2 t
2
dy
 2 sin t cos t   sin2t m2 A2
dt and KEmax 
2
d2y [symbols represent standard quantities]
 22 cos 2t
dt 2 From given information
d2y 75
For SHM,  y KE  (KEmax ) 
dt 2 100
Hence, function is not SHM, but periodic, From m2 A2 m2 A 2 3
 cos2 t  
the y-t graph, time period is 2 2 4
 3 
T or cos t   or t 
 2 6

www.motion.ac.in
234 - Solution Physics (XI) Topic

2  T 1 11. B
or t  or t  s
T 6 12 6 aT 2xT 42 4 2
  2 T   cons tan t
x x T T
6. A
The maximum velocity of a particle performing
SHM is given by v = A, where A is the 12. C
amplitude and  is the angular frequency of At mean position Fnet = 0
oscillation.  By conservation of linear momentum
 4.4  (7  103 )  2 / T Mv1 = (M+m)v2.
7  10 3 2  22 M1A1 = (M+m)2A2
or T    0.01 s
4.4 7 k
But 1  ,
M
7. A
K
x = (2×10-2)cost and 2 
Here, a = 2 ×10-2m = 2cm mM
at t = 0, x = 2 cm, ie, the object is at positive A1 mM
On solving  .
extreme, so to acquire maximum speed (ie, to A2 M
1
reach mean postion) it takes th of time period. 13. A
4
T Let x1 = Asin (t+1) and x2 = Asin(wt+2) x2
 Re quired time  - x1 = A[sin(t+2)-sin(t-1)]
4
2  2t  1  2     1 
where    = 2A cos  sin 2 
T  2   2 
 T 2s The resultant motion can be treated as a
T 2 simple harmonic motion with amplitude
So, required time    0.5 s
4 4    1 
2A sin 2 
 2 
8. A
Given, maximum distance between the par-
Average kinetic energy of particle
ticles = X0 + A
1  Amplitude of resultant SHM
 ma22
4 = X0 + A - X0 = A
1
 ma2 (2v)2    1 
4  2A sin  2 A  2  1   / 3
 2 
 2 v2ma2

9. D 14. A
1
1 k1  k 2 For spring k 
f l
2 m
kA l
 
1 k  k2 k lA
and f'  .2 1  2f
2 m lA  lB 5
kA  kA  k
10. D lA 2
 
x  x o cos  t  
 4 
d2 x
Acceleration, a 
dt 2
 
 2x o cos t  
 4
 3 
 2x o cos t  
 4 
So, A= 2x0
3
and  
4

394-Rajeev Gandhi Nagar, Kota


Simple Harmonic Motion - 235

15. B
k
Let at any instant, cube be at a depth x 1 = 20  2 where 0 =
m
from the equilibrium position, then
Net force acting ont he cube = Upthrust on b
=+
the portion of length x 2
2
So, average life =
b

l x 18. C
Amplitude of damped oscillator is given by
bt

l A  A0 e 2m

b 5

After 5s, 0.9A 0  A 0e 2m
 F= –l2
×g
= –l2g × F  –x (SHM) ....(i) b 5
 0.9  e

2m ....(i)
Negative sign shows that force is opposite
15
to x. Hence, equation of SHM b
After, 10s, A  A0 e 2m
F = –kx
3
Comparing Eqs. (i) and (ii), we get
A  A0  e  5b

 ....(ii)
2m
k = l2g
From Eqs. (i) and (ii), we get
m l3 d ld A = 0.729 A0
 T  2  2  2 Hence, = 0.729
k l2g g

19. B
16. A
As no relation between k1 and k2 is given in
the question, that is why, nothing can be
predicted about statement 1. But as in state- a 3a
ment 2, k1<k2
Then, for same force
F F2 1
W  F.x  F.   W i.e., W1  W2
k k k
But for same displacement
A – A cos () = a ....(1)
1 1 A – A cos (2) = 3a
W  F.x  kx.x  kx2
2 2 2A (1 – cos2 ) = 3a ...(2)
Divide (2) By (1)
 W  k, i.e., W1  W2
2 (1 + cos ) = 3
Thus, in the light of statement 2, statement cos  = 1/2 ; = /3
1 is false.
2
 = /3 ; T = 6
T
17. C
d2x dx 20. C
m = –kx – b
dt2 dt  '
T = 2 ; TM = 2
dx2
dx g g
m +b + kx = 0 here b is demping Mg / A
dt2 dt
=
coefficient this has solution of type  / 
x = e1 substituting this ' Mg ' Mg
 = = =1+
m2 + v + k = 0  A  A
Also:
 b  b2  4mk
=
2m TM '
on solving for x, we get =
T 
b
x = e 2m t a cos (1 t – )

www.motion.ac.in
236 - Solution Physics (XI) Topic

 Mg 
1 /2 22. C
 TM = T 1 
 A  v= A 2  x2 ;
2
TM2 Mg T M
 Mg 4A2
 1 = A2 
2 = 1 + A  v=
  2
T  T  A 9
2
1 A  TM    5A
 =    1 =
 Mg  T   3
New SHM
21. D 3v =  A2N  xN2 ;
KEmax at mean position.
3 5A 4A2
=  A2N 
3 9
U=0 4A 2
KE=0 U=0 5A2 = AN2 –
9
KE=0
49A2
Extreme Extreme A2N =
9
MP
V=max 7A
AN =
=0 3
PEmin at mean position

394-Rajeev Gandhi Nagar, Kota


Simple Harmonic Motion - 237

1. C 6. D
d2 y
y = kt2  = 2m/s2 7. C
dt 2  = 
  L L 2
 T1 = 2
g
; T2 = 2
(g  2) k ×   + kx   = ML 
2 2 12
 T1 
2
g2 12 6 ML2 2
kxL =  L
  =  g  = = 12 x 
 T2    10 5 2
L x
x=
2
12  L L
mg g 
2. y* =  2 <a = 2  2
k  ML  2
6k
V2 = w2 (a2 – y2) =
m
k V2
V2 = (a2 – y2), Now H = +y 1 6k
m 2g f=
2 m
2 2
k (a  y )
H= +y
m 2g
8. A,D
dx 2yk m mg
=– + 1 = 0 ,y= × 2g = A IB
dy 2gm 2k k   A  B
B IA
3. A,B,D

4. C 9. C
1 2 1 y 1
kx = × 4(y)2 ; =
2 2 x 2 10. B
S2 S1
2 k 1
B k 11. D
x 4k S1, k

12. B

13. A
2u sin 
T=1S=
4 2 g
Vavg = A
T
Maximum displacement will be close to 10
g
4 2 u= = 2  1  5 2  50
M.P. = 2 A ; vavg = A 2 sin  2
T

5. D 14. D
y = A sin t
2  
= = ; y = sin t
8 4 4 15. C
4 3
at t = sec  y <
3 2
16. B
2 3 3 2
a = – 2y = – × =–
16 2 32

www.motion.ac.in
238 - Solution Physics (XI) Topic

17. A,D 18. B,D


v= A 2  x2
Does not
travel
MP
or v = 2 (A2 – x2)
2

u0 or v2 = 2A2 – 2 x2
m or v2 + 2x2 = 2A2
v2 x2
v = u0/2

or + =1
2 A2 A2
u0/2 as A = a
x = A sin t x=A
MP m1 a = b x=0
v = A  cos t T/4
u0
A a 1
= A  cos t m T/6
= m = n2 (given)
2 b 1
 a
t = T/4 T/6
3 R =n (given)
T
Time to reach the wall = 1
6 E1 = m 22 A22
2
T T
Time to return to mean position = + 1
6 6 E2 = m 22 R2
2
T 2 m
= = b 1
3 3 k m1A1 = b  m1a = b  1 = 
am mn2
T T
Time for maximum compression = + 1
3 4 m2A2 = R  m2R = R  2 =
m
7T m 1 1 1
= = 14
12 k 2 = mn2 × m = n2
T T 5T
II time = + = E1 E2
3 6
2 1 = 2
5T 5 m
Equilibrium position = =
6 3 k
19. A,B,D

(a) M

MA = (m + M) 'A'
K k
M A  m  M  A'
M Mm

A' M

A mM

M  m 
(b) T  2
k
Time period remain same in both case. Af-
ter m mass is pla ced on both block.

394-Rajeev Gandhi Nagar, Kota


WAVES - 239

1. B 5. D
y = A sin (t – kx + )
Given vpmax = 4v ... (i)
at t = 0 and x = 0 y = – 0.5
vpmax = A = y0
1 
  sin   = 
  2 6
V= =
R 2
4 
from equation (i) y0 = 6. A
2

y0 10  x 
y sin  200  t  
=   17 
2
comparing with
2. C
y = A sin (t-kx)
The length 0.25 m corresponds to 2.5 
2
0.25  0.25    2000   T = 10-3 sec
T
 = 0.1 m
Maximum velocity = A
2
k 10
0.1   2000  102 = 200 m/s

V=f
330 = 0.1 f
7. D
 f = 3300
  = 2  × 3300 T
V1 =
y = A sin (t - kx) 

 2  2T V1 1
 0.25 sin  3300  2 t  x
 0.1  V2 = ,V =
 2 2

 0.25 sin2  3300 t  10x 


8. D

3. B
T 10
v=  = 100m/s
A = 0.50 m  1  103
=1m
f = 2 Hz 1
 t= = 0.01 s
Now travel in negative x direction. 100

So, t and x have same sign.


y(x, t) = 0.5 cos (2x + 4t) 9. C
Wavelength at lower end is 0.08 m
4. A
T
Velocity of wave on string = .
y vP y 
vP =  v  
x v x

www.motion.ac.in
240 - Solution Physics (XI) Topic

14. C
15
  1.5 Resultant amplitude depends on 
10
Tension at lower end  T = 50 N A= A12  A22  2A1 A2 cos 
Tension at upper end  T = 200 N
Frequency of the resulting wave is same.
50
vlower 
1.5 15. D

16. D
200
vupper 
1.5
17. C

v1 v2 18. A
frequency is constant hence   
1 2

19. A
50 200

0.08 

   2  0.08 = 0.16 m.

10. C

.x
v=


 v2 = .x

11. C

T
100 =
102

 T = 100 N
T = mg sin 30º 20. D
= 100
m = 20 kg

12. B

13. D
Resultant wave amplitude  depends on
phase difference
If p.d = 2n  Amax = 2A
p.d = (2n + 1)  Amax = 0

394-Rajeev Gandhi Nagar, Kota


WAVES - 241

21. C 25. B
v1 v2
V2 < V1 y = 10 sin 2 (100t–0.02x)+10sin 2 (100t
 ’ <  + 0.02x)
= 2×10 sin 200 t cos 2 (0.02)x

22. B 200  = 

Let equation of reflected wave be 2


2 × 0.002 =
y = A sin(– kx – t + ) 
Phase difference between incident wave and
reflected wave must be ‘’ at x = /k. 
loop length =
2
 π
 – 2k   +  = 
k
26. A
  y1 = 5 sin (t – kx)
 Equation of reflected wave y2 = 5 sin [– (t – kx – 150)]
= A sin (kx + t – 2) y2 = 5 sin (t – kx + 120º)
 Resultant amplitude
23. D
= 52  52  2.52 cos120º = 5.
For Ist wave
A1 = 5, n1 = 10
For 2nd wave A2 = 10, n2 = 20 27. B


I2 A 22n 22 (10) 2 ( 20) 2 y = 5 sin x cos 40 t
= = = 16 3
I1 A12n12 (5) 2 (10) 2
2 
k= =  = 6 cm
 3
24. D

y d= = 3 cm
2
0.5 cm/s 0.5 cm/s
2
28. D
1

x (cm) 29. A
–2 –1 0 1 2 3

x1 = 0.5 × t = 0.5 × 2 = 1cm


30. B
x2 = 0.5 × t = 0.5 × 2 = 1cm
at t = 2 sec
31. D
y
The waves will pass through each other
4 without any change in their shape.
3
2 32. C

1  = 9×10-3 kg
T = 360 N
x
0.5 1 2
nv
at x = 0.5 cm, y = 3 cm frequency 
2L

www.motion.ac.in
242 - Solution Physics (XI) Topic

33. C
nv
Hence  210
2L
n T
f1 = 350 =
2 
n  1 v  280
2L
(n  1) T
f2 = 420 =
n 21 2 
  28n  21 x  21
n  1 28
f2 – f1 = 70 Hz
3n = 21
n=3 34. A
In first case y = a cos kx sin t
2L compare it with

3 y = 2A cos kx sin t

3 a
L A= (Amplitude of each wave)
2 2


A loop is formed in length 35. A
2
Rate of transfer of energy i.e.
Hence no. of loops are 3
Power = 22A2f2v

394-Rajeev Gandhi Nagar, Kota


WAVES - 243

1. C
0.2 2  16  10
T= × (40)2 =
4 4
2. A
T = 80 N

3. D
8. C

4. B
9. C

5. C
4
for x = 4 m  = /2 4 5
 
 10 7 3
y = 2 sin (8 – 100 t + )
3

 10. C
= –2 sin (100  t – )
3

x = S2D – S1D = 5 – 4 = 1 =
 4
 0 = –2 sin (100  t – )
3
4m
S1 D
1
t= s. 3m
300

S2
6. D

  2  
x    = . = , IR = 2I0
y = A sin 2 ft     4 2
  

 v = f.
11. B
vmax. = A.  = A. 2f = 4 f
Particle will be at mean position so purely
A K.E
=
2

7. A 12. B
4 trips means 32 m
A2 = a12  a22  2a1a2 cos
v
Here, =1–2, A = a1 = a2 = a
4m Substituting these values in Eq. (1),
we get
d d 32
t= v= = = 40 m/s
v t 0.8
1 2
cos  = – or 
2 3
T
v=  T = v2

www.motion.ac.in
244 - Solution Physics (XI) Topic

13. C
2 1
K2 = 
v T / v2 v1 / 2 = 2K1
1
or v 1  RP,2  PQ,3  QS  2v2  2v1 / 2
 2
At =  v  v  Ai  v  v / 2 Ai  3 Ai
 1 2  1 1
Here  is mass per unit length.
Then equation of transmitted wave
0.1
1 = =0.05 kg/m 2
2 Ai cos(2k1x  1t)
yt =
3
0.2
2 = =0.067 kg/m
3
17. C
0.15 v1 v2
and 3 = =0.0375 kg/m
4
3 < 1 <2 y = 6 sin (5t + 40x)
 v3 > v1 > v2
Between 1 and 2, 2 is denser. Therefore T
v1 =
wave-2 will suffer a phase change of . 
Between 2 and 3,2 is denser. Therefore,
wave 4 will not suffer any phase change. T v1  v1 / 2  v1  6
v2 = = =  3v / 2  = 2 mm.
4 2  1 
14. B
At t = 2s, the pulses will super-impose to  2v2 
make the string straight. The particles of At =  v  v  = 4 mm
 1 2 
the string shall occupy their respective mean
positions. The particles shall not have any Reflected there will be phase difference of
potential energy. So, the energy shall be .
purely kinetic. y = – 2 sin (5t – 40x)
Thus, (b) is the right choice.
18. B
15. C P = 22A2f2v

Resultant amplitude is a2  a2 i.e. 2 a.


Pi = 22(6)2fv1=
16. C K1
Pr = 22(2)2f2.v1
T
Given v = , T2 = T1 and 4v1
 Pt = 22(4)2f2. = K1 × 32
2
2 = 41
K1  32
 %=  100 ~
~ 89%
v1 K1  36
then v2 =
2
Frequency doesn’t change that is 19. A
1 = 2 Pavg = 22A2f2v
Initially P0 = 22A2f02v ... (1)
Then as frequency depends only on source. So, it

394-Rajeev Gandhi Nagar, Kota


WAVES - 245

is not change then 23. C

P0  20 
= 22A2f02v ... (2) y = A sin  x  cos(1000t)
2  3 

 20  A
A20 A sin  x  
divide (1)/(2)  2 =  3  2
A2
20  5 7 11
x  , , ,
A0 3 6 6 6 6
A=
2 1 5 7
x= , ,
40 40 40
20. B 1
x= = 5 cm.
20
T/A T
y= T
 = y..t
 A

24. B
Given y1 = a cos (kx – t)
if x = 0 is a node there amplitude of standing
1 T 1 y.T.A
f=  f= wave be have
2  2 A
A = 2A sin kx
y2 = – a cos (kx + t)
1 y.T
f= y1y2 = a[cos (kx – t) – cos (kx + t)
2 
= – a cos(kx + t)

y
f 25. A

(n  1) v n 1
f= v  = v
21. A
2   2 
Energy per unit time (i.e, P) at junction will 2  
remain constant Not the energy per unit area =  d= =
(n  1) 4 2(n  1)
per unit time (i.e., intensity) Because in
transmission area of medium may change.  2d(n + 1) = 

26. D
22. B
In reflected and incident wave all other fac-
tor are common, So, it is only amplitude
which can change the power. 75% is trans-
mitted. Hence, 25% or 1/4th will be reflected
back. 3 2L
L=  =
2 3
Ai
 Ar  (as P  A2)
2 2V.3 3v
 = 2f = 
2L L
Ai
or 2 x = 0 is node
Ar
3x
v1  v2  amplitude = A sin kx = A sin
2 L
or v2  v1
 3x   3v 
v1 1 y(x, t) = A sin  L  cos  L 
or     
v2 3

www.motion.ac.in
246 - Solution Physics (XI) Topic

27. C 29. D

 100
y = 0.02 cos (10 x). cos (50 t + ) v= = 100 m/s
2 0.01
Now,
 = 50  = 2 f n
f= v
2
2
f = 25 Hz and10  = f1 = 50 Hz 100 Hz 150 Hz

f1 f2 f3
1 (2n  1)
 = m for open f = v
5 4
For antinode f1 = 25 H2, 75 H2, 125 H2
cos (10  x) = 1 n1 n2 n3
For Node cos (10  x) = 0
30. B
28. D
1
= 2 cm  1 = 4 cm
v 2
f=  T , f =K T in water

2
= 1.6 cm  2 = 3.2 cm
f 2
 K T1
2
n vw 1 n
T = mg f1 = vw =     2
2 1 1
T1 = T/4
mg – vwg = mg/4
n  1 vw

1

n  1
f2 = vw =
3mg 2 2 2 2
v.w.g = ... (1)
4
1 1 1
  
2 2 1 2
v =  300 = 200 m/s
3
1   2
f/3 = K T2   =    8 cm
1 2
T2 = T/9
mg –v.g = mg/9
31. C
8mg
vg = ... (2)
9
x 2 3
 8  4 32

(2)/(1)   9  3  27
w

394-Rajeev Gandhi Nagar, Kota


WAVES - 247

1. A,B,C,D 8. B,C
Given that = Ax + B
y = 10 –4
sin (60t + 2x) at
compare it with y = A sin (t + kx) 2g
x = 0,  = 0
 60  B=0
(a) v = 
k 2 So,  = Ax x
= 30 m/s –ve direction [at x = L]

2 2 0 = AL
(b)  =  =
k 2 0
 A=
L
 30
(c) f = = Hz
2  0
 = x
(d) A = 10 –4
m L

2. B,C,D 0 x2
.
Given y = A sin (t – x) L 2g x  vdv 3g 
v= 0
 a   
2g3  dx 4
 2 2 .x
f= , =  L
2 k 

v= x
 0
T= 
0
L
x dx 3g
3. B,C

0 x2
4. C,D T=
L 2g  3
y y
= – vw
t x
vw = Negative 9. A,D

 2v 2 
5. A,D At =   A
 v1  v2  i

6. C The phase of transmitted wave is same as


that of incident wave.
7. B,D
 v2  v1 
T Ar =   A
v=  v1  v2  i

Phase of reflected wave depends upon v1
v= xg and v2.

vdv 1 g g
a = dx  xg   2 
x  2

= constant

www.motion.ac.in
248 - Solution Physics (XI) Topic

10. A,B 12. A,C


y = 2 sin (x) sin (100  t) L
1 2

y
 2 .dx 2A sin kx 
0
vp = = 2 × 1000  sin (x) cos (100 t) A2 2 L
t 

(a) Max. Displacement   A2 (2)2 f2 L


 n2
 1
= 2 sin    
 6

1 13. C,D
=2× = 1 mm
2
1 Anti node Anti node
(b) Velocity of the particle at x = cm ;
6 Node Node Node Node
Anti node
1
t= sec.
600
  1 
v = 2 × 1000  sin cos 100   14. C,D
6  600 
Amin = |3A – A| = 2A
= 157 3 cm / sec
Amax = A + 3A = 4 A
(c) k =    = 2 cm Distance between maximum and adjacent
5  1 v
5 loop = = 5 cm. minima =   
2 4 4f

11. A,C
(a)In one time Period all the particles are
simultanesusly at rest (two times)
(c) If there is only one loop.

394-Rajeev Gandhi Nagar, Kota


WAVES - 249

1. (a) amplitude A = 5 mm wave is travelling in +ve x dirction


(b) wave number k = 1 cm –1
{  t & kx have opposite sign}

2    
(c) wavelength  = = 2 cm (b) y = (0.35)  sin  3  (100)  4  
k  

 60 = –05.48 cm
(d) frequency v = = Hz
2 2
2 2
(c)  =  = 0.66 m
1  R 3
(e) time period T = = s
v 30

(f) wave velocity u = n = 60 cm/s f= = 0.5 × 10 = 5 Hz
2
Given
y = (5 mm) sin [(1 cm–1) x – (cos–12)]  
(d) vp = (0.35) (10) cos 10t  3x  
compare the above equation with  4

y = A sin (kx – t)


(a) A = 5 mm 3. (a) 10  rad/s (b) /2 rad/m (c)
(b) Wave no. R = 1 cm–1 y = (0.120m) sin (1.57x – 31.4 t)
(d) 1.2  m/s (e) 118 m/s2
2
(c) Wave length = = 2 cm y
R

 60 x=0
(d) Frequency f =  Hz
2 2 (a) Given f = 5 Hz
  = 2 (5) = 10 
2 1
(e) Time period T = 
60 f A = 12 cm so v = 20.0 m/s

 2 10  
 (b) wave number k =  
(f) Wave velocity v = 20 v 2
T 2 / 60
(c) Expression of wave
= 60 cm/s
y = A sin (  t – kx)

  
10  y = (12cm ) sin 10t  2 x 
2. (a) i m / s (b) –5.48 cm (c) 0.667 m,  
3
5.00 Hz (d) 11.0 m/s
 y 
Given (d) vpmax =  t 
 max

y = (0.350 m) sin (10 t – 3x + ) = (0.12) (10) = 1.2 m/s
4
compare it with y = A sin (  t – kx + )   2y 
 
(e) apmax =  t 2  = (0.12) (10)2
  max
then  = 10 , k = 3 ,  =
4
= 118.2 m/s2
 10 10
(a) v =   m/s
R 3 3

www.motion.ac.in
250 - Solution Physics (XI) Topic

1 2 4.5
4. Ar = – cm, At = cm = = 2 × 103 kg/m
3 3 1000  2.25
Tension in the string T = 20 N
 v1 
 v2  v1  1  v2  T 20
Ar    i 
A   Ai v=  = 100 m/s
v1  2  10  3
 v1  v2   
1  v2 
  Time to reach the wave from the floor to

2
 2v2  the pulley t = = 0.2 sec.
100
At =  v  v  . Ai
 1 2 

 2  v1 2
=  Ai  7. (a) 0.52 m ; (b) 40 m/s ; (c) 0.40 m
v , v2 1 = 2
1  1  Given
 v 
 2 
y 1 = 0.30 sin 5x  200t 
1  2  1
 Ar = 1  2   1  3  
  y2 = 0.30 sin 5x  200  3 
 
2 Now, y = y1 + y2
and At = +
3
y = 2(0.30)

5. 0.2 cm    
 10x  400t   
y = 0.3 – 0.1 (x – 5t)2 (y  0) sin 3   cos   

  2   6
On comparing with general equation    
f (x – vt)
v1 = 5 m/s ; v2 = 2.5 m/s.
 
and Ai = 0.3 cm (At x = 0 and t = 0) y = 0.52 sin 5x  200t  6 
 
2v2 2  2.5
At = v  v Ai  2.5  5 × 0.3 = 0.2 cm 200
1 2 A = 0.52, wave speed = = 40 m/s
5

6. 0.02 s 2 2
=  = 0.40 m
k 5
Total length of string
= 2.25 m
8. 50 Hz, 4.0 cm, 2.0 m/s
Given

T
= 5 ms ; T = 20 ms
4
2.25m

2kg 2m 
= 2 cm   = 4 cm
2

 4
wave speed v = 
T 20  103  100

394-Rajeev Gandhi Nagar, Kota


WAVES - 251

= 2 m/s 12. (a) 144 m/s ; (b) 60.0 cm ; (c) 241 Hz

v 2 Given
Frequency f =  = 50 Hz
 0.04 T = 150 N

 = 7.20 × 10–3 kg/m

9. (a) 0.47 W, (b) 9.4 mJ


T 150
(a) Avg. power transmitted v=  = 144.33 m/s
 7.20  10 3
= 22 A2 f2 v
= 2 2 (.001)2 × (200)2 ×
3
= 90 cm (from fig.)
60 2
× 0.006 = 0.47 w
0.006

v 144.33
dE f=  = 240.55 Hz
(b) = 2 A2 cos2 (kx – t)  0.6
dx

2
2 2
cos2 (kx  t)dx 13. (a) 0.25 cm (b) 1.2 × 102 cm/s;
E=   A
0
(c) 3.0 cm; (d) 0

2
2A2  
E= 2  1  cos2(kx  t)dx
0
y = (0.50) sin  3 x  cos (40 t)
 

compare it with y = 2A sin kx cos t


2 A 2
= [2 + 0] 2A = 0.50  A = 0.25 cm
2

=  2 A2 = 9.47 mJ 
 = 40  ; k =
3

 40  3
10. 2 A, 8 A v=  = 1.2 × 102 cm/s
k 
N od e where am pl i t ud e mi ni mum
= 5 – 3 = 2A 2 
Now,   = 6 cm
Antinode where amplitude maximum  3
=5+3=8A

Distance between nodes = = 3 cm
2
2 
11. y = 0.8 a sin ( vt  x  ) 9
 2 At x = 1.5 and t =
8
Amplitude of Reflected ware = 0.8 a
y = 0.5 (at max.)
Phase difference = 
So, v=0
Reflected wave equation

 2   
y = 0.8 a sin   vt  x  2  
  

www.motion.ac.in
252 - Solution Physics (XI) Topic

14. (a) 100 Hz (b) 700 Hz 15. 25 kg

v 51 2
Fundamental frequency f =  = 2  1 =
2 5
Given Now,
4 loops  2 = 
5 9g
f= ....(1)
 2v 2 
= ; f’ = = 400
2 
3 mg
f' 400 f= ....(2)
f=  = 100 Hz 2l 
4 4

3 2 9 5 = m × 3
7 loopse  =
2 3
m =5  m = 25 kg
7v
f=
2

394-Rajeev Gandhi Nagar, Kota


WAVES - 253

1. ( a) n egat i v e x ; ( b) y = 4 × 10 –3 sin
 2 2
Tx  [ – x 2 ]
 1  2
100  3 t  0.5 x  400  (x, y in meter) ;

T 
(c) 144  × 10–5 J  vw   (2 – x 2 )
 2
– ve direction :
(a) A = 4 × 10–3  t
dx 
at t = 0, x = 0 , y = 2 2 × 10 –3 
0  –x
2 


0
2
dt

1 
 sin  = =  –1  x 

2 4
 sin  
    0
 y = 4 × 10 –3
[sin (t + kx + /4)]

y y 20  10 –2  0 
 vw  v   6 m / sec   t  t=
t x tan  2 2 2

 v   f0 
 6 = f. (4 × 10–2) vp
 1.5 × 102 = f 19.2 m/s
 y = 4 × 10–3 sin 3. 1/48 sec1/24 sec t
–19.2m/s
2  1.5  10 2 
[2  1.5  1012 t  x ]
6 4

1 y y
 4  10 – 3 sin 100 [3t  0.5 x  ]  –v w
400 t x

Power : 22A f2 v
t=0
Energy : 22 A2f2 v.T = 22A2. f v 24m/sec
= 2 ×(3.14)2 ×(4 × 10–3)2 × (1.5 × 102) 0.4m
×(50+10–3) × 6
x
= 144 2 × 10–5 Joule. 1m


2.
2 y  0 .4 
 –(24)    19.2 m / s
t  0 .5 
dT = – dm 2x
dT = –  dx 2x T  T  = 1m
T
T x 2
dT  – 2 xdx x dx k=  2

0

l

 = vw k = 24 × 2  = 48 
[ – x ]
2 2
 0 – Tx = –  2 2 1
2 T= = sec .
48  24
www.motion.ac.in
254 - Solution Physics (XI) Topic

Ai  Ar
V  1.5 Pt = Pi – Pr
Ai – Ar
19.2m/sec
1/24 sec
5 Ar = Ai
1/48 sec
t 2
–19.2m/sec Pt  Pr  A 
 1 –   1 –  r 
Pi  Pi   Ai 

Ar 1 1 24
4. 0.12 m  =1– =
Ai 5 25 25
Frequency depends on
TB = 80 N
source only. 6kg
Pt
%  96%
Pi
so, at point A and B
2kg TA = 20 N

TA TB
VA = , VB = 8. (a) y = (7.50 cm) sin (4.19 x – 314 t)
 
(b) 625 W

vB 180
A = 0.06 m  B = v A = = 0.03 kg/m
A 1000  6
f = 50 Hz
80
B = × 0.06 = 0.12 m 3
20 Given 4  = 6 m   = m
2

5. (a) 105 Hz ; (b) 158 m/s 15


2A = 15 cm  A = m
Given 2  100

Resonant frequency 315, 420 Wave function


And it is a multiple of 105 Hz = (7.50) sin [4.19 x – 3.14 t]
so fundamental frequency = 105 Hz
 3
P = 22 A2 f2 v v  50  2 
v  
 = 105
2
P = 625 W
2  75
v = 105 × = 157.5 m/s
100
5 10
9. (a) Hz ; (b) 5 10 Hz ;
2
6. 300 Hz
5 10
v (c) Hz
f’ = 2

 = 10 m
v 3 m = 100 g
f’ =  f’ = 3 × 100 = 300 Hz
r
100
= = 10–2 kg/m
1000  100
7. 96%
T = 250 N
A1  A 2
 1.5 Pi = Pr + Pt
A1 – A 2

394-Rajeev Gandhi Nagar, Kota


WAVES - 255

12. a. 1.72 × 10–3 m.


T 250
v=  = 50 10 b. 272 m/s.
 10 2
a) The displacement of the string at any
Lowest frequency point is y(x, t) = (ASW sin kx)sin t. For the
fundamental mode  = 2L, so at the mid-
point of the string
sin kx = sin(2/)(L/2) = 1, and
y = ASW sin t. Taking derivatives gives vy

 =   = 2 y
2
= = ASW cos t, with maximum value
t
v 50 10 5
f=   10 Hz v y
2 20 2 vy max = ASW, and ay = = – 2ASW sin t,
t
2v with maximum value ay max = 2ASW.
(b) Second lowest =  50 10 Hz
2 Dividing these gives
 = ay max /vy max = (8.40 × 103 m/s2)/(3.80
3v 15
(c) Third lowest =  10 Hz m/s) = 2.21 × 103 rad/s, and then
2 2
ASW = vy max
/ = (3.80 m/s)/(2.21 × 103
rad/s)
10. 36 N = 1.72 × 10–3 m.
4 loops  2 =  b) v = f = (2L)(/2) = L/

 = (0.386 m)(2.21 × 103 rad/s)/


=
2 = 272 m/s.

2v 2 T
f= 
l l 

2 T
60 =  0.9
0.90 0.044

T = 35.64 N

11. ( a) C = 400 m s –1
(b) stress =1.28 × 109 Nm–2
(c) a = 0.02/42

www.motion.ac.in
256 - Solution Physics (XI) Topic

1. A 4. A
For string fixed at both the ends, resonant 2
 bt2 2 ab xt)
y(x, t)  e (ax
nv
frquency are gi ve n by f  , whe re ax  bt )2
2L  e(
symbols have their usual meanings. It is It is function of type y = f(t +kx)
given that 315 Hz and 420 Hz are two
 y (x,t) represents wave travelling along
consecutive resonant frequencies, let these
are nth and (n+1)th harmonics. x direction.

nv  b b
315  ...(i) Speed of wave =  
2L k a a

(n  1)v
420  ...(ii)
2L 5. A

 Eq. (i)  Eq. (ii) y = y1 + y2 = A sin (t-kx) + A sin (t+kx)


y = 2A sin t cos kx
315 n
  or n = 3 Clearly it is equation of standing wave for
420 n  1
position of nodes y = 0
From Eq. (i), lowest resonant frequency

ie, x  (2n  1)
v 315 4
fo    105 Hz
2L 3
 1 
 n   , n  0, 1, 2, 3
 22
2. A
y (x,t) = 0.005 cos (ax-t)
6. B
2 2
   and 
 T 1 2 2
I  A v
2
2
So,    25m1 Here,  = density of medium,
0.08
A = amplitude,
2  = angular frequency and
and    s 1
2 v = velocity of wave.
 Inte nsi ty dep end upon am pl i tud e,
3. D frequency as well as velocity of wave.
Also I1 = I2
2

T  v2  
k2 7. B
(2 / 0.004)2
 0.04  6.25N
(2 / 0.50)2 
t =2 = 2 2 second
g

394-Rajeev Gandhi Nagar, Kota


WAVES - 257

3 nv n  10
1. y = (10 cm) sin (30 t ± x + f) Now f=  100 =
2 2 2  0.20
n=4
Aw = 3
n
A2 = 90 (A = 0.1 m)  
2
  = 30 rad/s
4 20 1
3    m  10 cm
  2 100 10
y = (10 cm) sin 30 t  x  
 2  
separation between successive node =
2
2. A = 5 cm.
T1 + T2 = mg 5. 5

v1 1 T1 6. BC
 C
fAB = A
2 2  2
T1 T2 K=   = 10m

D P
B
2 T2
fCD  L
2  x
m

5
 T1  2 T2  T1 = 4T2 L= = 25 cm  A = 0.01 m
2

4 mg
T1 = mg , T2 = 7. A,C,D
5 5
V = 100 m/s
Torque balance at T1x = T2( – x) L = 3m
At x = 0  node
4 mg
 mgx  (  x ) At x = 3 antinode
5 5
Satisfying the condition in the equations on
 putting x = 3, sinkx should be equal to as it
 x=
5 is an antinode.

3. A x  50 6 
sin    
 = 0.5 m A = 10 cm 6  3 

5x  250 6 
v p   102  5 2  vp  5 3  sin   3  5
6  
v 10 2 5x  250 
  2 = 2 = sin   5  2
 50 5 2  
vp = 2 3  cm / s
8. 3
3
vo = ĵ
50

4. 5
1 10 2 0.5
 = 20 cm,    kg / m
10 3  20 100
T = 0.5 N
I  I1  I2  2 I1I2 cos 
T 0.5  100
v  10 m / s = 2I0 + 2I0 cos 60° = 3I0
 0.5
n=3
www.motion.ac.in
258 - Solution Physics (XI) Topic

Q 7. B
1. C
OQ = 8 × 330
= 2640 m 8. C
PQ = OQ sin 60° 60°
O P
3 9. A
PQ = 2640 × = 2286 m
2
10. D
2. A
frequency is same
 RT  energy remains conserved
=
Μ
 Redistribution is stable with time.
for monoatomic = 1.67
for triatomic = 1.3
11. D
1 M2
 1.67  1.8  10 – 2
= , = = 1.067 
1  2 1.3  2.02  10 – 2 x = 12 = , = 24 cm

 330
f= = = 1375 Hz
3. D  24  10–2
 d d 1 1  u 
   u  = t, d    u  = t  d =  u  v  t
      12. D
x = 2 m
4. B for constructive interference
I1 2 = n
L1 – L2 = 10 log I
2 
for Distructive 2 = 2 (2n + 1)
I1 
3.0103 = 10 log I
2

I1 I1 13. B
0.30103 = log  2 =
I2 I2 I1 = I I2 = 4 I
log2 = 0.30103  I1 = 2 × 10–4 wm–2 IA = I1 + I2 = 5I
IB = 9I
5. D I = (IA – IB) = 4I
I1
(90 – 40) = 10 log
I2 14. C
I1 I1 E
5 = log I  I = 105 =
2 2 A
I' = 2I
6. B
A'2 = 2A (I A2)
I1 4
= =4
I2 1 A = 2 A – A = A ( 2 – 1)
2 2
Imax =  4  1  = 9, Imax =  4  1  =1 A
Imax 9 A
%  
2  1  100 = 41%
L = 10 log I , = 10 log = 20 log 3,
min 1

394-Rajeev Gandhi Nagar, Kota


SOUND WAVES - 259

15. A 23. C
330 1
= =
600 2
 1
 =
4 8
0.125 m 
f1 =
2

16. B


17. D f2 =
2(  x)
Interference is phenomena of more than one  
wave reaching at same point in space |f1 – f2| = –
2 2(  x)
simultaneously.
x x x
= = x =
18. B 2(  x) 22 (1  ) 22

24. D
19. B 350
1750 =

1
= = 20 cm
5

should be raised by = 10 cm.
at the middle of the pipe 2

20. A 25. B
Pr node 7
=

 
=
4 7
displacement
= zero (node) Amplitude = a

21. D 26. B
For displacement Phase change  at close
1 3 / 4 3
end = =
2 2 8
For pressure  No phase change at close
end 27. B
11th minima from the current minima
22. B x = 10
25
= = 1 cm
10

at the middle of the pipe

www.motion.ac.in
260 - Solution Physics (XI) Topic

28. B 32. C
s – 2 Relative velocity
a
 t1 =   
s 2

33. C
common beat = 1.    gt 
f’ =  f
  
29. D
gt g  1000
280 ± 10 f’ = f + . t  f’ = 1000 + .t
 
 1 T t = 30 s.
fs = =
2 2  f’ = 2000 Hz   = 300 m/s.

T ,  , fs   290 Hz


34. A
f'
30. B
   at 
  f’ =  f
     
=6   =6 f
  2 1 1.02 af
6  1.02 f’ = f + .t
 t
= = 306  300 m/s.
0.02

31. C 35. A
cu
30 m/s 30 m/s 30 m/s =
f
S1 O S2

f = 100 Hz
   0 
f s1' =      100 = 100
 s 

   0 
f s2' =      100 = 100
 s 

No Beat.

394-Rajeev Gandhi Nagar, Kota


SOUND WAVES - 261

1. A 6. C
1 = 2 , f1 = f2
2
Imax  I1  I2  49 
=   A1 = A2
Imin  I  I  = 9  =
2
 1 2 
2 2 
I1  I2 Anet= A1  A 2  2A1A 2Cos = A12  A 22
7 I1 10 2
 =  =
I1  I2 3 I2 4  = f  = f11 = f22
I1 25
 I = 7. D
2 4
 = 105 cm
2. A 7
= 105
I1 4
160 – 120 = 10 log for Pressure node
I2
 105
I1 1 = = 15 cm
10 = I ,
4
I  4 7
2 r2
3
2 = 45 cm
 r2  r2 4
   
 = 100
 r1  r1
8. C
r2 = 104 m.

f1 = 4(L  0.6r )
1
3. A

f2 = (L  0.6  2 r ) , f1 = f2
2
 
4(L  0.6 r1 )
= (L  1.2 r )
2
0.8 r1 – 0.4 r2 = – L
r2 – 2r1 = 2.5 L

4. A 9. C
x = 10 – 6 = 4 = n

4
=
n
for   max
n min = 1
=4

5. B
y = A cos (ax + bt)  f1 = f2
I1 = I 31 2 n1 m 2
=  =
I2 = 0.64I 4 1 2 4 1 2 2
I Anet
2
0.64 A = Anet
2 n1 m  31 
 =    2n = 3m
4 1 2  41 
A net = I = 0.8 A
check the options  n = 9, m = 6.

www.motion.ac.in
262 - Solution Physics (XI) Topic

10. B 13. C

3x
/4 P = P0 cos sin 300 t
2

at close end Pr Antinode


at open end Pr node
Displacement Pressure (A) at x = 0
P = P0 cos0° = P0 Antinode
3  1.2
= 1.2,  = 0.4 from open at x = 0.5 m
4 4 3
end. P = P0 cos 3 = — P0 Antinode
(c) at x = 0 antinode – close end.
11. C
at x = 2m.
RT fc v c 2 / 3 11
v ,  
M fc vD 4 / 3 , 28
P0 cos 3 = antinode – close end.
(D) x = 0 antinode – close end.
12. C
n 2
Open organ pipe f= x= Antindde – close end.
2 3
f n
 = 1.25 (1)
 2
14. A
n 1
= 1.75 (2) 
2 fc = 
n2 
= 2.25 (3) 
2
f0 = 2
2.5  1 2
from (1) and (2) = 1.75
2 |fc – f0| = 4
2.5  + 1 = 3.5  = 1   1 1
    = 4 .... (1)
2 2
 1  2
Close organ pipe
   1  1
f = (2n – 1)    = n .... (2)
4 4  2 1  2 
f (2n  1)
= = 1.25 (1) from eq. (1) and (2)
 4
2n  1 n=2
= 1.75 (2)
4
2n  3 15. B
= 2.25 (3)
4 a is the +ve const.
from eq. (1) and (2) (I) 256 + a = 262 – 2a = f0  a = 2
2n–1 = 5  f0 = 258 Hz
(II) 256 + a = 262 + 2a = f0
2n+1 = 7 
Not Possible
_ _ _ (III) 256 + a = 262 + 2a = f0
___________ Not possible a have –ve value
(IV) 256 – a = 262 – 2a = f0  a = 6
–2 = –2  = 1 m.
f0 = 250 Hz

394-Rajeev Gandhi Nagar, Kota


SOUND WAVES - 263

16. D 19. C
   0  f1 9
f' =   f
f2
=
   8
   0 
f2 =   f (v = v )

   s  1 s 0

8   0
 =   17 0 = –
9 0

0 = –20 m/s  0 = 20 m/s.

  
f" =   f'
 20. B
   s 
  
   0  f1 =  n

=      f 0 = s    s  0
 s 
f2
 330  2  f1
=f
=   × 334 = 330 Hz
 330  2 
      s 
f2 =      n0  f =     
 s   s 
17. D
1  f  n0 [2 s ]
s =     f1 – f2 = 2
l f     2s

1  1  f2 
n 

f1 – f2 = 
2 0 f 

21. A

   0 
f' =      f
 t = 1 s s = 10 m/s
 s 

   0 
    f" =      f 0 = 2m/s
1 =       2 =   =  s 
 s  
 300  2 
   f' =   150 = 156
3 =       1 > 3 > 2  300  10 
 s 
 300  2 
f" =   × 150 = 144
18. D  300  10 
 f' – f" = 12 beat
  
f
=   –1= 1
   
f  s  10

 1 
 =  s =
  s 10 11
f  11 1
=  –1= –1=
f s 12  12
f
× 100 = 8.33 %
f

www.motion.ac.in
264 - Solution Physics (XI) Topic

1. B 8. A,B,C
y y x = R = n
= – w ,  = –ve
t x
y y R
= –ve  = –ve =
t x n

n=1  = R
2. C
R
n=2 =
2
3. A
R
n=3 =
3
4. A,D
R
n=4 =
4
5. C
Stationary point or Max Displacement point.
9. A,B,D
y y
= 0. = 0. 3 2 r
x t x = (2r) –
4 4

6. A,B,D 
= r = (2n + 1)
2

7. B,C 2r
=
2n  1
  T n=0  = 2r
2
  T 2 r
n=1 =
3
2 r
T n=2 =
5
 

10. B

 
Imax = 4I0 cos2 , =
2 2
n 
f= = 4I0 cos2 ( ), = 2I0
2 4

11. A
x = R = n

R
= for  max n = 1
n
 = R

394-Rajeev Gandhi Nagar, Kota


SOUND WAVES - 265

12. B
 
 fA = 2 fB = 4
x = R = (2n + 1)  2
2
2R
 fA 2 2 10
(2n  1) = = (B)
fB 1 6
for max n=0
R
 = 2R = 1st Overtone
n

 3
fA = fB =
 4 2
13. C,D
fA 4 2 20 10
31 5 2 = 3 = =
1 = 2 = fB 1 18 9
2 4
2 1 4 2
1 = 2 =
3 5 14. B,D

3 5 1 6 v RT 1 1
f ,   , f
f = 2 = 4   = 21 M 2 m
1 2 2 5
Mf 
Fundamental

www.motion.ac.in
266 - Solution Physics (XI) Topic

3
 2 T= s.
1. (a) (b) 5
2 35
200
(a) f = 100 Hz. v= × 5 = 333.33 m/s
3
v = 350 m/s
t = 2.5 × 10–3 s 4. 30 dB, 10 10 mm
x = 350 × 2.5 × 10–3
= 0.875
2
 = . x
 5. (a) p (b) I  ( IA – IB )2  (25 / 312)2
2 
= × 100 × 0.875 =
350 2 (a) P = 25p f = 850 Hz.
(b) x = 10× 10–2 m.
x = 2.6 × 2.4 = 0.2
2 2
 = × 100 × 10 × 10–2 = 340
350 35
= = 0.4
850

2. ( a) 1.7 × 10 –5 (b) 1.08 × 10–4 2


 = (0.2)
y = 6 sin (600t – 1.8x) 

2 2
(a) K = = 1.8 = × 0.2 = 
 0.4

=
10 (b) I = I A  IB  2

9
A = 6 × 10–5 m P 25 25
IA = = =
Area 4(2.4)2 23
A 6  10–5  9
 = = 1.7 × 10–5 25
 10 25
IB = =
(b)  = 600 4(2.6)2 27
 600 2
vw = =  25 25 
k 1.8 I =   = (1.04 – 0.96)2
 23 27 
(vp)max = (6.0) × 10–5 × 600 = 36 × 10–3
(v p )m ax 36  10–3 = 6.4 × 10–3 W/m2
= × 1.8 = 10.8 × 10–5
v 600
6. 83 Hz
3. 333 m/s n = 332 m/s
At point B

x = 2 =
2
=4
 332
5 f= = = 83 Hz.
freq. =  4
3
Time period of one clap to go to wall and
come to man.

394-Rajeev Gandhi Nagar, Kota


SOUND WAVES - 267

7. 420 Hz 
  =
2
2
x = 

  x 
x =  =  x= =
2 4 2 8

I = I1 + I2 + 2 I1 I2

Imax = I 1  I2 
2
n = 336 m/s 9. 20, 80 cm, 200 Hz

=  I 
2
Imin 1  I2 f=? =x
4
At position a,  = 0, 2, 4.......  = 4x
 x = 0, , 2 ........... 320
f=
At position b,  = , 3, 5,...... 4x
 3 5 (1 – x) = 1
 x = , , , .............
2 2 2 320
f=
It at position a,  = 2 (1  x)
x1=   1– x = 4x
3 1
x2 = x=
2 5

x = x2 – x1 =
2 Length of close organ pipe

= 2 (20 × 10–2)   = 80 × 10–2 1
2 = m = 20 cm.
5
 336
f= = = 420 Hz. Length of open organ pipe
 80  10–2
= 80 cm.
Fundamental tone of open organ pipe
8. l/8
v 320
I = = = 200 Hz.
I= 0 2 2  0.8
2

I = 4I cos2 10. 1:1
2
I0  First Overtone
= I0 cos2
2 2

sound source

4 1
1 =
3

vs 2

x vw 1
Imax = I0
ear

 
=
2 4

www.motion.ac.in
268 - Solution Physics (XI) Topic

3rd harmonic  = 33 cm.


2 2 l = 4’
2 =
3 v' v'
f0’ = =
vw 3v w  4 '
f2 =  = 2
2 2 v'
 = 500
4 '
vs 3vs
f1 =  = 4 v'
1 1 ’ =
4  500
given f1 = f2
’ = 13.2 cm
3vs 3v w
41 = 4 2

1 vs 2 13. 2.5 ms–1


= 2v = = 1z
2 w 2(1)
f = 680 Hz. v s = 340 m/s

11. 3 cm 10 Beats

 v  u  v  u
fB =  v  680 fA =  v  680
   
 |fB – fA| = 10

v u  v  u
 × 680 –  v  × 680 = 10
v  
680
[ v + u – v + u] = 10
v

1 = 24.1 cm 2[2u] = 10

2 = 74.1 cm 10
u= = 2.5 m/s.
4
e = 0.3 d
L1 = (24.1 + 0.3d)
14. 345, 341 or 349 Hz
L2 = (74.1 + 0.3d)
v 3v fA = 350 Hz.
L2
f0 = 4L f1 = 4L L1 =
1 2 3 |fA – fB| = 5 Hz ............. (1)
3(24.1 + 0.3d) = (74.1 + 0.3d) |fA’ – fB| = 2
72.3 + 0.9d = 74.1 + 0.3d |fB – fC| = 4 ..............(2)
0.6d = 1.8
|fA’ – fC| = 6 Hz ..............(3)
d = 3 cm.
fB = fA – 5 = 345 Hz.

12. 33 cm and 13.2 cm fA’ = 348 Hz.


345 – fC = 4
fC = 349 Hz
v Or
f0 = 500 Hz =
2
348 – fC = 6
fC = 342 Hz.

394-Rajeev Gandhi Nagar, Kota


SOUND WAVES - 269

15. (i) (V – Vw + Vs) / f (ii) (V + Vw – Vs) / f


(iii) (V–Vw–VD) / fr ;
where fr=(V+Vw+V0/v + Vw – Vs) f
(iv) (V – Vw – Vo/ V – Vw – VD) fr

(ii)

app. = (v + vw) T – vsT


(i)
v  vw  vs
=
f

app. = (v –vw) T + vsT v  vw  vD


v  vw  v s
(iii) app. =
=
f'
f v  v w  vD
f’ = f
v  vw  v s

vw
(iv) vD f'
f vs

v  v w  vD
f’ = v  v  v .f
w s
vw
vD
f'' v0 f'
v  v w  v0
f’’ = v  v  v f’
w D

www.motion.ac.in
270 - Solution Physics (XI) Topic

1. (a) 2 /a, b/2, 5. (a) 0.3 m, (b) 1320, (c) 332 m/s, (d) 0.2 m
(b) y2 = ± 0.8 A cos (ax – bt), v  v s 332  32
(c) max. = 1.8 bA, min = 0, (a) l’ =   0.3 m
f 1000
y = A cos (ax + bt)
v  v0 332  64
2 2 (b) f’= v  v × f= ×1000 = 1320 Hz
(a) a   s 332  32
 a
b v  v s 332  64
2 f = b  f = (c) ’ =  = 0.2 m
2 f 1320
(b) yr = 0.8 A cos (ax – bt + )
yr = –0.8 A cos (ax – bt)  2  10 –9
6. W/m2
 I'  4
since A’ =  I   A
  f = 250 Hz.
A’ = 0.8 A A = 1 × 10–18 m.
(c) Reflected & incidence from a wave of r= 1 kg/m3
b = 400 N/m2
Amax = A + 0.8 A
 400
= 1.8 A v= = = 20 m/s
p 1
I = 2p2 f2 A2 rv
and Amin = A – 0.8 A = 0.2 A = 2p2 × (250)2 × (10–8)2 ×1×20
2
so Vmax = 1.8 Aw & Vmin = 0.2 Aw = × 10–9 W/m2
4

7. (a) l/6 ; (b) 150 Hz; (c) 1.28


2. 310 m/s (a)

3 2
l=   = L
3. 2c/3 2 3
disp. node to pressure
4. (a) 2; (b) 9.28 m and 1.99 m node

(a)  = vf  330
200
 1.65 m  2l l
is  =
4 3 4 6
d = 4m
max. path difference = 4m = 2.42  (b) 750 = (2n + 1) f0
So minima at 1050 = [2(n + 1) + 1] f = 750 + 2f0
path difference x = 0.5  , 1.5  2f0 = 300  f0 = 150 Hz
(b)

(c) = 20cm
2
2
l  16
4m   = 0.4 m
v = f = 1100 × 0.4 = 440 m/s
l
rRT
x = l 2  16 –  = 0.5  & 1.5  440 =
M
where  = 1.65 m 440  440  16 
 r = 8.31  293   1000  = 1.28
solving l = 9.28 m & 1.99 m  

394-Rajeev Gandhi Nagar, Kota


SOUND WAVES - 271

8. 735 N 10. f
1  10–2 0.1
m= =
0.3 3
1= 0.3 × 2 = 0.6 m
1st Overtone
3
=1 vs = v0

f’ = f
4
=
3
330 330
f= = ×3
 4 11. 9.7 m
= 247.5 Hz.
velocity in wire = f 1 660 m/s
= (247.5) (0.6) = 148.5
T T 5m
vw =  v2 =
  Distance
1
T = mv2 = × (148.5)2 = 735 N
30
165
= = 9.7 m
17
v
9. (a) vn = (2n + 1); six oscillations
4
v
(b) vn = (n+1), also six oscillations ; 12. 9 sec
2
Here n = 0, 1, 2, ..... v
f’ = v  v f
v0 = 1250 Hz. s

 = 85 cm = 0.85 m v
2f0 = v  v f0  2v – 2vs = v
s
v = 340 m/s
v
 vs = = 170 m/s
v 2
(a) f0 = (2n+1)
4
n = 0 f0 = 100
n = 1 f1 = 300
n = 2 f2 = 500
n = 3 f3 = 700
n = 4 f4 = 900
n = 5 f5 = 100
v
n = 6 f6 = 1300 So, 0.75v cos  =
2
nv
(b) f0 = 1 2
2 cos = 
1.5 3
n = 1 f0 = 200
n = 2 f1 = 400 1500 1500 4500
Now, l = sin     900 5
n = 3 f2 = 600 5 /3 5
n = 4 f3 = 800
n = 5 f4 = 1000 900 5 2012
t=   5.95
v 340
n = 6 f5 = 1200 Hz

www.motion.ac.in
272 - Solution Physics (XI) Topic

1. D 4. D
v 320 Let intensity of sound be I and I'.
vo   vo   64ms1
5 5 Loudness of sound initially
When obs erve r move s toward s the  I
stationary source, then 1  10 log 
 Io 
 v  vo   320  64 
n'   n  n'   n  I' 
 v   320  Later, 2  10 log I' 
 o
 384  n' 384
or n'   n or  1  2  20
 320  n 320
Hence, percentage increase
I I
 20  10 log  or I' 
 n'n   384  320   I'  100
   100 %
 n   320 
Therefor, intesity decreases by a factor of
 64  100.
  100 %  20
 320 
5. B
2. C
l1  18cm
The frequency of fork 2
v1
 200  4  196 or 204Hz f
4l1
Since, on attaching the tape on the prong of
fork 2, its frequency decreases, but now the 3v2
f 
number of beats per second is 6 ie, the 4l2 ,
frequency difference now increase. It is where l2 = x according to given situation and
possible only when before attaching the tape, also v1<v2 as during summer temperature
the frequency of fork 2 is less than the would be higher
frequency of tuning fork 1. Hence, the
frequency of fork 2 is 196 Hz. 3v 2 v
  1
4l2 4l1
3. C v2
or l2  3l1 
Velocity of sound in air = 300 m/s v1
If a source of sound is moving towards a  x  54  (A quantity greater than 1)
stationary listener, the frequency heard by
the listener would be different from the ac- So, x > 54
t u a l
frequency of the source, this apparent 6. A
frequency is given by
Speed of sound is given by,
 v sound in air 
fapp.    RT
 v sound in air  v source  , where symbols v
  M
have their usual meanings.
In the denominator +ve sign would be taken 7 5
when source is receding away from the RT RT
v02  5 and vHe  3
listener, while -ve sign would be taken when
32 4
source is approaching the listener.
Let v be the maximum value of source veloc- v o2 73 4
ity for which the person is able to hear the  
sound, then vHe 5  32  5

 300  5  32  5
10000  fapp.     9500
 300  v  or vHe  460   1420ms1
73 4
or v = 15ms-1
No option is matching with the real solution.

394-Rajeev Gandhi Nagar, Kota


SOUND WAVES - 273

7. C 11. A
Maximum number of beats = v +1-(v-1) = 2 V 340
f= = = 100 Hz
4L 4  85  10 2
8. B 100 H, 300 Hz, 500 Hz, 700 Hz, 900 Hz, 1100 Hz = 6
Motor cycle, u = 0, a = 2ms-2
observer is in motion and source is at rest
12. D
v  v0
 n'  n
v  vs  320 
f1 = 1000  300  20  = 941.1 Hz
 
94 330  v 0
 nn
100 330  320 
f2 = 1000  300  20  = 1142 Hz
330  94  
 330  vo 
100  Change is ~
 12%
94  33 33  6
 v o  330   ms1
10 10
13. 3
2 2
v u 9  33  33 9  1089
s    98m.
2a 100 100

9. A open pipe
Initially for open organ pipe, fundamental
frequency,
v
v f=
vo  f 2
2l
But when it is half dipped in water, then it
l
becomes closed organ pipe of length . In
2
this case fundamental frequency
v v v
vc    f closed pipe
4l' l 2l
4
2 v v
f’ = = =f
4 ' 
4 
10. D 2
1
Strain =
100

1
Stress = (strain)y = × 2.2 × 1011
100
= 2.2 × 109
T
= Stress = 2.2 × 109 T = 2.2×109A
A

T T 2.2  109.A
v= = =
M P.A P.A

2.2  109 2
v= =  106
7.7  103 7

v 2 1
f1= =  106  = 178.2 Hz
2L 7 2  1.5

www.motion.ac.in
274 - Solution Physics (XI) Topic

1. D 6. B
Related to error vSA = 340 + 20 = 360 m/s
vSB = 340 – 30 = 310 m/s
2. Vs = 30 m/s
A B
v v
2200  f , 1800  f
(v  v s ) (v  v s ) 20 m/s 340 m/s 340 m/s 30 m/s

22 v  v s
 7. A
18 v  v s
For the passengers in train A, there is no
rel at i v e moti on b et we en s ourc e and
11 V – 11 Vs = 9v + 9vs obverver, as both are moving with velocity
20 m/s. Therefore, there is no change in
observed frequencies and correspondingly
20 vs = 2v there is no change in their intensities.
Therefore, the correct option is (A).

V 300
vs  = = 30 m/s 8. A
10 10
For the passengers in train B, observer is
recending with velocity 30 m/s and source is
3. A approaching with velocity 20 m/s.
In one second number of maximas is called
 340  30 
the beat frequency. Hence,  f1'  800    775 Hz
 340  20 
100 92
fb  f1  f2    4Hz  340  30 
2 2 and f2'  1120    1085 Hz
 340  20 

 Spread of frequency = f'2=f'1=310 Hz


4. A
 Correct option is (A)

Speed of wave v 
k
9. A
100 92 With increase in tension, frequency of
or v or  200 m/s
0.5 0.46 vibrating string will increase. Since number
of b eats are d ec re as i ng. T he re fore ,
5. C frequency of vibrating stri ng or third
harmonic frequency of closed pipe should be
At x = 0, y=y1 + y2 = 2A cos 96t cos 4t
less than the frequency of tuning fork by 4.
Frequency of cos (96t) function is 48 Hz
 Frequency of tuning fork
and that of cos (4t) function is 2 Hz.
= third harmonic frequency of closed pipe + 4
In one second, cos function becomes zero
at 2f times, where f is the frequency.
v  340 
Therefore, first function will become zero at  3   4  3 4
 4l   4  0.75 
96 times and the second at 4 times. But
second will not overlap with first. Hence, net = 344 Hz
y will becomes zero 100 times in 1 s.  Correct option is (A).

394-Rajeev Gandhi Nagar, Kota


SOUND WAVES - 275

10. A,C,D 13. A  PT ; B  PS ; C  QS ; D  QR


In organ pipes, longitudinal waves are
1
(A) I  formed.
r2
In string, transverse waves are formed.
(B) Prong were kept horizontally Open end of pipe is displacement antinode
in vertical plane and closed end is displacement node. In
case of string fixed end of a string is node.
(C) Amplitude is very small. Further, least distance between a node and


an antinode is and between two nodes is
 4
(D)  L  for first Resonance condition.
4

. Keeping these points in mind answer to
2
11. 7
this question is an under ; A  P,T ; B  P,S
v1 v2
; C  Q,S ; D  Q,R
to

1.2  v  v1   v  v2 
f1 – f2 = 100 fo    14. A
 v   v 

v  v1  v  v  vc 
fapp =  v – v   f
 c 
v  v2  v

 320  10 
v 1.2 198 =    8 kHz = 8.5 kHz
v1  v2     320 – 10 
2 100 100

v 1.2 198
v1  v2     7km / hr
2 100 100
15. B,D
At closed end prssure doesn't change phase
and at open end the phase is reversed.
12. B
Given, 2nd harmonic of I = Fundamental of II

v  v T /  v2 16. A,B
 2 1   2  
 2l1  4l2 l1 4l2 Observer to source

v  w  u
2 f2  f1   f2 > f1
16T l2 16  50   0.8  v  w  u
   2 22 
v2 l1 320 2  0.52
source to observer
= 0.02 kg/m
 m1 = m l1 = (0.02)(0.2)  v  w  u
f2  f1   f2 > f1
v  w  u
= 0.01 kg = 10 g

www.motion.ac.in
276 - Solution Physics (XI) Topic

17. D 18. A,C,D


v0 v0
f = 244 Hz
R

= 0.35   = 1.4 m
4
1800 m
 (RT )
V = 244 × 1.4 = 314.6 =
M

5  (RT )  M N
For Neon : V =   Q
3 M 
f=118 Hz 20 m
121 Hz

RT 10 RT 7 RT
=  . =
12 M 6 10 6
1800 m
v0
RT v0
For N2 : V = 60 km/hr
20
P
7 RT 7RT q = 221 – 218 = 3 Hz
For O2 : V = =
5 32 160

5 RT 5RT
For argon : V = =
3 36 108  v  v0   v  v0 
f1P  f1   & f2P  f2  
 v   v 
 v  v0   v  v0 
& f1Q  f1   & f2Q  f2  
 v   v 
So, p  f2P  f1P

R  f2R  f1R

P  R   f2P  f2R    f1P  f1R 

= f2 (2) – f1 (2)
= 2 (f2 – f1)
= 2Q
P + R = 2Q

394-Rajeev Gandhi Nagar, Kota


Heat-1 - 277

1. C 9. B
Volume of ice is greater than volume of water For 1 sec we can say that
for same mass. Pc  80 % = (v) s (t – 10)
80
2. B 2×103× =(1000).100×(10–2)3.4200 (t-10).
100
3. B On solving
At a temperature T t = 13.8 °C
dQ = SdT = aT3dT
10. A
2
3 a[T 4 ]12 15a (mw + wf) (1) (70 – 40) = mice Lf+mice (1) (40-

Q = a T dT = = 0)
1 4 4
(200+wf) (70–40) = 500 Lf+50×40 ..(1)
(mw +wf+mice) (40–10) = mice Lf+m’ice (1) (10–0)
4. D (200+w+50) 30 = 80 Lf + 80 × 10 ...(2)
dT from eq. (1) & (2)
1
 dQ = msdT  = 50 × 30 = 30 Lf – 30×40
dQ ms Lf = 90 cal/gm = 3.78 × 105 J/kg

vapour 11. A
We know that
Liquid
i  K(R 2 )
dT R2
, i
dx 

Solid 12. A
Consider the two sections like two resistance
R1 & R2.
5. C 1 2 1
For vapourization the total time required is RA = k A RB =
1 k1
= (30 – 20) min = 10 min A
2
Total Heat Given = 42 KtJ × 10 = 420 KJ
so mL = 420 kJ  RB 
5L = 420  L = 84 KJ/kg So  100 – 0
R A  RB 
6. D  = 80° C
Heat is required to raise temperature of
(Calorimeter + Ice to vapour) at 0 to 100°C 13. C
= (10 × 100 + {10 × 80 + 10 × 1 × 100 + i = –kA dT / dx
10 × 540}) Slope dT/dx = – i/kA is – ve but due to
= 8200 Cal. radiation loss because of not lagged, as we
move ahead current i will be less. Hence slope
7. A wil be more – ve to less – ve.
Required heat/sec = 0.1×80 cal/gm
= 8 cal/sec 14. D
Produced mass = 0.1×100 = 10 gm ice dT dT 1
or water [now Q = msT] i  kA  
dx dx K
In unit time rise of temperature will be
 i and A are same for both the layers.
T = Q/ms = 8/(10×1) = 0.8°C/s
i = – kA (dT/dx)
i and A are constant hence slope
8. C
dT/dx = –i/(kA) is – ve but
Water flow rate = 20 gm/sec
Slope  (1/k)
for 1 sec
Hence in air slope will be more – ve due to
Q = P×t = 2100×1 = 2100 J
very less conductivity.
Q = 2100 = 20 × 4.2 (t – 10)
t = 35°C

www.motion.ac.in
278 - Solution Physics (XI) Topic

15. B
K1  K 2
We know that K eq 
i = – kAdT / dx 2
And slope of the curve but dT/dx = – i/kA
i is constant (steady state), A is constant but 20. C
since k is decreasing from 2k to k, hence slope The resistance formed by two cylinders R1
is –ve but less – ve to more –ve. & R2 are in parallel as they are kept between
same temperature difference.
16. B A1 = R2 A2 = 4R2 – R2 = 3R2
R 1R 2  
x Now Req =  
R1  R 2 K1R 2 K2 3R 2
2K K
 1 1  
   =
R 2  K1 3K 2  K eq(4R 2 )
0°C 36°C
1 1 1
A B =  keq = (k 1  3k 2 )
4K eq 3k 2  k 2 4

21. C
t t Not Reflected and Not Refracted.

d A dB 22. D

dt dt Good Absorbers are good emitters.

x 0  36 – x  23. C
2KA    KA  t 
 t    dQ
x = 12°C = e A T4
dt
17. A dQ
inet = iA + icu So, A
dt
 100  20   e (nature of surface)
100  20
 K A A  –2  + K Cu A  T (temprature)
 3  10  3  10–2
But independent of mass.
80
= (209 + 385) (3 × 10–2) = 24. C
3  10  2 (A) Heat absorption is surface phenomenon
= 1.43 × 103 W hence wooden (Black surface) absorbs
more.(wrong)
18. B (B) Aft er l ong ti me b ot h wi l l hav e
 100  0  temperature of surroundings.(wrong)
dQ 10
in (a)  cal/min = AK  2a  ..(1) (C) Because metal is better conductor it feels
dt 2   hotter.
(D) Because emission depend on surface
d'  100  0 
in (b) = (2A) K   ...(2) (i.e. more for black surface)
dt  a 
d' 25. A
so 10 = .t = 2 × 10 × t By Theory
dt
t = 0.5 min 26. B
By Theory
19. B
Here the thermal resistances are in parallel 27. B
as temperature difference across all the rods By Theory
is same.
1  1 1  28. B
   3
 By Theory
R eq  R 1 R 2 
29. A
K eq(6A) k A k A 
  3 1  2  By Theory
d  d d 

394-Rajeev Gandhi Nagar, Kota


Heat-1 - 279

30. B dQa
By Theory (B) = e A T04
dt
31. B (Rate of obsorption is same always)

P' eAT 4 dT eA(T 4  T04 )


Let I  or I  (C) 
4d2 dt ms
4d2
(Due to lesser mass of hollow sphere it cools
and I Af = P (Given)
fast.) (wrong)
eA(2T)4 (D) Since hollow sphere cools fast ;
Now Pnew = Inew Af = .A f hollow will have smaller temperature at any
4(2d)2
moment.(wrong)
16  eAT 4  16
37. D
= 4  2
.A f   P
 4d  4
 dT  eA 4
  T  T04  
32. B  dt  mc
Using formula
P = eAT4 38. A
PP = P  (A) P4 and PQ = Q  A Q4 4 4
mA= 4mB ,  × rA3 = × rB3 × 4
3 3
Now PP = PQ
1/ 4 rA
 Q   rB
= 41/3 = 22/3
  Q  P
 
 p 
dQ
Rate of heat loss = =eA(T4–T
T04)
dt
33. A
E273 = eA (273 + 273)4 2
(dQ / dt)A A A  rA 
= E(Given) Ratio (dQ / dt) = A =   =24/3
E0 = eA (273 + 0)4 B B  rB 
E dT dQ / dt
E0  Rate of cooling 
16 dt ms

34. D (dT / dt)A (dQ / dt)A mB


Ratio (dT / dt) = (dQ / dt) × m
Power B B A
4
dQ b 1
P  AT 4  A  = 24/3 × = 2–2/3
dt  4

4 4 39. A
P2   1   0  256
     We know that
P1   2  3 / 4
 0

 81
1
 max 
35. C T
1 max T2
dTp  dT  
–  x – Q  2 max T1
dt  dt 
T2 3
eA p (T 4 – T04 ) xeA Q (T 4 – T04 )  
 = T1 4
mpS mQS

2 3 40. B
ApmQ
 r   3r 
 x      1
A Qmp  3r   r  Using relation max 
T
36. A TS NSmax 350
   0.69
dQ TNS  Smax 510
(A) = e A T4
dt
(Rate of emission is same initially)

www.motion.ac.in
280 - Solution Physics (XI) Topic

1. A
Using Energy conservation. The energy loss –T°C
due to potential energy goes into increasing
the temperature of ice.
x
m
(L)  mgh
5 dx t
t+dt
L
 h=
5g
KAT dx A ice
it = = . Lf
2. D x dt
From the data given 3600 4
SAA(8V) = (12V) BsB
 KAT  dt = AiceL xdx

s A 12B 3 2000 0 2
 =  =2
sB 8 A 2 1500 T = 30°C

7. C
3. C
The heat current is equal to the heat required
Ic e Change s to w at er hence v ol um e
for fusion of ice per dt time.
decreases but mass remains same hence
Vw Pw = Vice Pice dm  20  0 
i = . Lf = KA  
VicePice dt  2.35 
Vw = Pw
dm
Let volume (Vice) change to water  2.4   10  6
dt
(0.9 w Vice)L = H ...(1)

 vice ice  8. A
v = vice – vw =  vice    From the given condition as the plates are
 w 
in series so heat current is same.
= vice (1 – 0.9) = 0.1 vice = 1 cm3
vice = 10 cm3 TB  TA k A(Tc  TB )
i 1 = i 2  k 1A  2
So from eq. (1) d 2d
[0.9 × 1 × 10] × 80 = H
k1 TC  TB 1  4T  2 TA 
H = 720 cal.    A  = 1
k2 2(TB  TA) 2  2TA  TA 

4. A
Let m is the mass 9. C
mLv + msw (100 – 80)
= (1.1 + 0.02)sw(80 – 15) dT bA
 (T  TS )
m(540 + 20) = (1.12) 65 dt ms
 m = 0.130 kg
70  75 bA   70  75  
    TS 
5. C T1 ms   2  

6. C 65  70 bA   65  70  
The heat current is equal to heat released     TS 
T2 ms   2  
to formation of ice at the surface in dt time.
In the first case, where wf is watere quivalent
of flask. 60  65 bA   60  65  
    TS 
T3 ms   2  

394-Rajeev Gandhi Nagar, Kota


Heat-1 - 281

10. A 19. B

65  75 bA   65  75   1
    30  I  T4 
2 ms   2  4

20. C
45  55 bA   45  55   Radiant Energy  T4
    30 
2 ms   2   21. D
dE
11. A Area =  ydx =  d  d   dE
12. A 4
b
By Theory Area (A) = E = T4 =  

13. D
4 4
By Theory Area1   2  1  2 
   =  
Area2  1 
 9  1


14. C
1
15. D    3
2

16. B
22. B
17. C
23. C
Energy absorved Role of coding  temperature difference
absorption coefficient =
Energy incident
24. B
18. A

www.motion.ac.in
282 - Solution Physics (XI) Topic

1. B,C 13. A,C,D


No change in temperature, state change At t = 0, Rate of heat loss for two sphere is
same as surface area and temperature of
2. A,C,D the two sphere is same at t = 0. But rate of
If mate ri al e xi st i n si ng l e p hase , temperature loss
temperature always increases with heat
d dH 1 1
supplied. = × 
dt dt ms m
3. C,D Hence more for hollow sphere. Thus at t > 0
temperature of hollow sphere is less than
4. B,C that of solid sphere and hence at t > 0, rate
of heat loss is more for solid sphere.
5. A,B
14. A,C
6. A,B 1 1
T  and Intensity  2
 d
7. A,B

Q  dT  15. A,B,C,D
 kA  
t  dx  According to Wein displacement law

1
8. A,C,D m 
T
k eq AT Intensity corresponding to a particular
i wavelength is dependent on temperature

as well as distance of source.
T  100  20

9. A, B 16. B,C
conceptual Heat absorbed by black body will be more
than heat absorbed by silver body
10. A, B

11. A,B,C

12. A,C,D

394-Rajeev Gandhi Nagar, Kota


Heat-1 - 283

1. 8.6 × 10–3 °C 7. 64 J
From energy conservation
dQ dT
mgh + K = msT = KA
dt dx
(200×10–3)×10×(60×10–2 sin 37°) +0
= 200 × 10–3 × 420 × T
 90 – 10 
T = 86 × 10–3°C = 0.8 × (10 × 10–2) ×  –2
 = 64 J
 1  10 

315
2.  =  C = 28.66°C gm
11 8. 5 × 10–5
Let all are at 0°C water then sec
heat given is dT
Qice=–[(10×0.5× 10) + (10 × 80)] i = mLs = KA
dx
= –850 cal
Qwater= 10 × 1 × 20 = 200 cal 42  (0.04  10–4 )100
Qsteam= [2×540+2×1×100] + 1280 cal. m=
3.36  105  1
So,at 0°C water now have (1280+200–850)
cal. = 50 × 10–9 Kg
As the heat is extra so it will increase = 5 × 10–5 g/s
temperature to t
(10 + 10 + 2) (1) (t – 0) 9. 4.0 W
= (1280 + 200 – 850) (T-100)/2.5 + (T–0)/2.5 + (T–25)/5=0
t = 28.636°C So, T = 45°C
Hence,
3. LB > LA = LC I(CD) = V/R
A 200 J = 4 × LA LA = 50 = (45–25)/5 = 4 W
B 300 J = 5 × LB LB = 60
C 300 J = 6 × LC LC = 50 t1  t 2  t 3
10.
t1 t 2 t
4. H = 590 Kcal.   3
k1 k 2 k 3
H = Energy/heat required to change 100°C
Water in 200°C Vapour Req = R1 + R2 + R3
=1000 LV +1000×0.5×(200–100)
t1  t 2  t 3 t1 t2 t3
= 590 Kcal. =K A + K A + K A
Re q A 1 2 3

5. 136 km 11. 65°C


Let mass = m
dT
1
KA = 1 × 103 W
dx
So, mgh = mLs
4
(T – 25)
5 0.2 × 5 5 = 103
4L s

4  3.4  10 4  10–2
h= = 13.6 × 10 m 4
g 10 T = 4 × 10–2 × 103 + 25

T  65C
6. SA = SB > SC > SD
mASA[TA] = – mSBTB, C, D
 SATA = –SBTB, C, D .........(1) 12. 5°C
So,
dQ1 10
1  SA = SB  KA    5C
dt d
2  SA > SC
3  SA >> SD

www.motion.ac.in
284 - Solution Physics (XI) Topic

16. 250W
dQ2  10   
 2KA      5C In the stady state, the absorption of the
dt  d  radiant energy by the rod ceases, which
dQ 3   5 evidently implies that the heat energy
 KA    5C liberated by the heater per second is emitted
dt d
by the rod per second.
dQ 4
 2K
5 So, the stady state rate of emission of radiant
   5C
dt d heat from the rod = power of heater = 250
W.
13. ( 6 / )1/ 3
17. 1700 K
Mcube = msphere
100 = e  AsT4
1
4 a  4 3 100
 × a =  × r3 
3    T4 = eA
3 r 3  s

–dT  As = 2R
Rate of cooling = eA(T4 – T04) T = 1696.7 K
dt
1700 K
Cube ACu 6a2
Ratio : Sphere  A  4r2 18. 0.3
Sp
i = eA (T4 – T04) = 210
2 1
3 4  3
6 3 = eA ((500)4 – (300)4) = 210 ...(1)
=       Now,
2  3  
i = 700 = A [(500)4 – (300)4] ...(2)
(1)/(2)
14. 8000 kJ
210
 Q  1 = = 0.3
Using, emissive power E   700
 ,
 t  A
E = 50 kW/m2 19. 10 sec
 Emissivity power of a blck body Here
50 50.1 – 49.9
= kW/m2 = 111.11 kW/m2 = K (50 – 30°)
0.45 5
 Heat radiated Q = EAt
= 8000 kJ 0.2 1
= K × 20  K=
5 500
15. 10 min Now,
40.1 – 39.9
As per Newton’s law of cooling, = K (40 – 30)
t
80  70
 25 Rate of cooling R1
2 0.2 0.2
= (10) t = 10sec.
t 5  20
60  50
 25 Rate of cooling R2
2
R1  50; R2  30 20. 3025 K

R1 5 b
 max =
R2 3 T

1 s T
But time of cooling is proportional to   star  T = 4753  6050 K
R  st Tsum Star
9506
R1 t 5 5 5 = 3025 K
  2  (i .e., )t 2  t1 = x 6
R2 t1 3 3 3
= 10 min

394-Rajeev Gandhi Nagar, Kota


Heat-1 - 285

1. 1 : 1.26
C
Steam = 330 – 200 – 100 = 30 g a C
Let ice = xg & water = (200 – x) g T1 a b KA
T2 1 kA
a b 
loss (steam) = gain (ice + water + KB R l
calorimeter) C

30 × 2.25 × 105 = x × 3.36 × 105


+ 200 × 4.2 × 103 × 50 K||(2ab) K A (ab) KB (cb)
  
C C C
+ 100 × 0.42 × 103 × 50
 x = 70g & water = 200 – x = 130 g Ka  Kb
 K|| 
2
7
Ratios = Req = R1 + R2
13

T1
c
2. 800 cal kg –1
K , 1000 cal kg
–1 –1
K –1
a c
–10ºC  – 2ºC a b KA
a b KB
Q = msT
c
64 = 10 × SL × 8  SL = 0.8 T2

1ºC to 3ºC  Q = 900 – 880 = 20 cal


Q = msT  20 = 10 × Ss × 2 2b b b
 
keq(ac) K A (ac) KB (ac)
 Ss = 1
Now, –2ºC to + 1ºC
2K A KB
880 = 10 × 0.8 × (tm + 2) + 10 L + 10 × 1  Keq  K  K
A B
× (1 – tm)
 L = 85.4 + 0.2 t (in cal/gm) for cal/kg
m

a2 s  b  T  T1 
 L = 85400 + 200 tm 5. loge   loge  0 
2K  a  T0  T2 

3. (a) 37.8 J/s (Watts), (b) 2.005 N-m for cylinder (a to b)


(a) Q = msT
ln(b / a)
Req =
ln one second 2kl
(a) 180 × 0.1 × 0.5 = 9 cal/s = 37.8 water for conductor (R=0 to R=a)

p 37.8 dQ = msdT = s × a2 l × dT


(b) P = w   = w  6 T
1

b
K
K A  KB 2K AK B a
4. K11 > K  , K|| = , K = K K
2 A B

K ||

www.motion.ac.in
286 - Solution Physics (XI) Topic

7. The black object


dQ dT T T
= a2 sl ×  0
dt dt Re q
8. 1:1

dT T0  T
 a2sl × dt  ln(b / a) × 2 k l 9. Only utensil

T2 T
as
2
dT 1 10. 9.72°C/min
 2k
T1
 
T0  T ln(b / a)
dt 0 4 4
dT dQ / dt eA(T  T0 )
 
dt ms ms
a2s b  T0  T1 
 t = 2k ln  a  ln  T  T  4 4
 0 2 
dT K(T  T0 )
  (R = Radius)
dt R

l1 k( T1  Tm ) K
  2.8 = (4004 – 3004)
6. R
l k( T1  Tm )  ( Tm  T2 ) T2 < Tm
dT k
&   (6004 – 3004)
 dQ   dQ  solid dt 2R
 dt   
molten
 dt  solid ks ls
Tm dT
km = kks dividing  = –9.72º C/s
K m A ( T1  T m ) molten lm
dt
lm
T1 > Tm
K s A ( Tm  T2 )
11. T’’ = 4
2 × 500 = 600 K
= ls The shell of a space station is a blackened
Rate of loss initially
K(T1  Tm ) lm P = A  T4 = A  (500)4
Tm - T2 = l
s later half of radiation emmited are

fraction of moltan metal emmited back by shell but not loss must
be same. So, it radiated double P' = 2p
lm 1 A  T4 = 2 A (500)4
= l l = ls
m s 1
1
lm T  200  2 4

1 k(T1  Tm ) 12. 207 K


=
Tm  T2 K(T1  Tm )  (Tm  T2 ) P = A T4
1
k(T1  Tm )

394-Rajeev Gandhi Nagar, Kota


Heat-1 - 287

1. C [Sun i s a perfectl y
TK r
To measure the radial rate of heat flow, we black body as it emits Earth
have to go for integration technique as here radiations of all wave- R
the area of the surface through which heat lengths and so
will flow is not constant. for it e =1.] Sun
The intensity of this power at earth's sur-
face [under the assumption r>> r0] is

P
x I
dx 4r 2

  4R 2r 4 R 2 T 4
r1  
T1 4r 2 r2
The area of earth which receives this en-
r2 ergy is only one half of total surface area of
T2 earth, whose projection would be r02
 Total radiant power as received by earth
Let us consider an element (spherical shell)  r02  I
of thickness dx and radius x as shown in
figure. Let us first find the equivalent ther- r02  R 2 T 4 r 2R 2 T 4
  0 2
mal resistance between inner and outer r 2
r
sphere.

dx 3. C
Rsistance of shell = dR = Let temperature at the interface is T
K  4x2
For part AB,
 1  T1 l1 T l2 T2
R  KA 
where K  thermal conductivity 
 

r2 dx
  dR  R   r1 4kx2
A K1 B K2 C
1 1 1 r2  r1
    Q1 (T  T)K1
4K  r1 r2  4K(r1r2 )  1
t l1
Rate of heat flow = H
Q2 (T  T2 )K2
T  T2 For part BC, 
 1 t l2
R
Q1 Q2
T1  T2 r1r2 At equilibrium, 
  4K(r1r2 )  t t
r2  r1 r2  r1
(T1  T)K1 (T  T2 )K 2
 
2. B l1 l2
From Stefan's law, the rate at which energy
is radiated by sun at its surface is T1K1l2  T2K 2l1
or T
K1l2  K 2l1
P    4R 2  T 4

www.motion.ac.in
288 - Solution Physics (XI) Topic

4. B By Kirchoff law

dQ d T  100 T 0 T 0
We know that  KA + +
dt dx R1 R2 R3 = 0
In steady state flow of heat
T  100 T T
dQ 1  (0.92) + (0.26) + (0.12)
d    dx (46) 13 12
dt kA
= 0
 qH –  = k'x
  = H – k'x T  100 T T
+ + =0
Equation  = H – k'x represents a straight 50 50 100
line. 2T – 200 + 2T + T = 0
T = 40
5. A
Q = U+ W (ignoring expansion)  dQ  (100  40)
 dt  = (0.92)(4) = 4.8 cal/s
U = msT = 0.1 × 4.184 × 20 = 8.368kJ  Cu 46

6. D 9. A
According to Newton's law of cooling, rate
of fall in temperature is proportional to the U
u  T4
difference in temperature of the body with V
surrounding i.e.
1 U
P

d
 k(  0 ) 3  V 
dt
Adiabatic expansion
d TV–1 = K

   o

  k dt 
TV 4  C
In (0) = kt +C
Which is a straight line with negative slope. 
r  1 
4
7. B
3
Newton’s law of cooling 1
4
d
= – K ( – 0) 4
dt 
3
d t
  K  dt 

0 TV 4  C
1

8. A TV 3  C

1
4 3
100° 0°C T  R 3   C
3 
Cu Br
46cm 13 cm 1
k=0.92 T k=0.26 T
R

st, 12 cm
k = 0.12
0°C

394-Rajeev Gandhi Nagar, Kota


Heat-1 - 289

1. A (A)  S (B)  Q ; (C)  P ; (D)  R (Using


theory.)
m T = const.
from graph T3 > T2 > T1
P water
0°C Q1 Q2 100°C
2. C ice B
9. A l1 400°C l2
Glass of bulb heats due to filament by radiation
10 x

3. A Let the mass of water that melts or evaporates is m


Energy gained by water (in 1 s)
kA
= 1000 – 160 = 8405  Q1  (400  0)  m  80  42
1
Time required t

ms kA
2(4.2  10 3 ) (50 ) Q 2  ( 400  100)  m  540  42
= Rate by which energy  2
840

= 500 Sec. Q1  2 400 80 2 1


     
Q 2  1 300 540 1 9
4. A
 2   1 10
dQ   
 eAT 4 = 0.6 A T4 1 9
dt
9
1   10  9
10
5. B
We know that 14.5ºC to 15.5ºC at 760 mm of Hg.
10. Total Heat given Q = 420 J

6. A,D
ice
ice Q1 ice Q2 water
Using theory.
–5°C m 0°C m = 1 gm
0°C

7. 0.05 kg steam at (373 k)     


Q1 50540  27 kcal
Q1 = m × 2100 (0 – (–5) = m × 2100 × 5J
0.05 kg water at 373 k Q2 = 10–3 × 3.36 × 105 = 336 J
0.05 kg water 
Q2  50100  5 kcal
    Q = Q1 + Q2
0.05 kg water at 273 k 420 = m × 2100 ×5 + 336

0.45 kg ice at 
Q3  450 0.5 20  4.5 kcal
   84
 m = kg = 8 gm
2100  5
0.45 kg water at 273 k
Q1 + Q > Q3 but Q1 + Q2 < Q3 + Q4
whole ice will not melt 11. A,B,C,D
T = 273 k B, C, D are in parallel

1 1 1
8. (A)  S, Q ; (B)  Q ; (C)  P, Q ; (D)  D, R RB = , RC = , RD =
3kL 8kL 5kL
or

www.motion.ac.in
290 - Solution Physics (XI) Topic

1 1 1 1 200
TE = T2– 0 =
R eq = R B + R C + R D 11
TE is smallest (C)

0 1L 5L 6L 300
T1 – T2 900
heat iB = 11
A B 3K E
1 / 3kL  = 1 / 3kL  = 11 kL
1L

2K C 4K 6K 300
T1 – T2 2400
iC = 1 / 8kL  = 11 = kL
3L 1 / 8kL  11
D 5K

4L 300
100° c T1 T2 0°C
T1 – T2 1500
iD = 11
1 / 5kL  = 1 / 5kL  = 11 kL

1
 Req = 900 1500 2400
16kL iB + iD = kL + kL = kL
11 11 11

1 1  iB + iD = iC (D)
RA = RE =
8kL 24kL
In steady state
12. C
100 – T1 T1 – T2 T2 – 0
For middle plate(in unit time)
1 / 8kL  = 1 / 16kL  = 1 / 24kL 
Heat absorbed/Area
8(100 – T1) = 16(T1–T2) = 24 T2 2T T0 = ? 3T
= Heat emitted/Area
after solving  (2T)4 + (3T)4
500 200 = (T0)4×2
T1 = k T2 = k
11 11 1/4
 97 
Heat flow through A(iA) T0 =   T
 2 
100 – T1 4800
= = kL
RA 11
13. A
Heat flow through B(iB) Equivalent circuit diagram

T2 R R/2
4800
=
1 / 24kL  = 11 kL T1 T2

iA = iB (A)
R

500 600
TA =100 – =
11 11 R/2
T1 T2
TB = TC = TD = T1 – T2

500 200 300


= – = T1  T2
11 11 11 9
Q= ..... (i)
3R
2

394-Rajeev Gandhi Nagar, Kota


Heat-1 - 291

17. B
T1  T2
Q= t ..... (ii) Rate of cooling = P watts
R
3 Thermal load = 3 kW
Capacity = 120 litres.
Solving (i) & (ii)
Initially cooled to 10°C
t = 2 sec. Ans.
Max. Temp = 30°C
Time of operation = 3 hours
14. ABCD
Q = msT
dT = 120 × 4.2 × 20 × 103
= constant
dt = 10080000
d = mC.dT  A option = 10080 × 103 J
(B) C Between 0 – 100 less as compared to Heat = 3 × 103 × 3600 ×3
between 400 – 500 = 32400 × 103
(C) 400 – 500 = C is constant Heat extracted = (32400 – 10080) × 103
= 22320 × 103
dQ MCdT dQ
(D) = C  Now :
dt dt dt
P ×3 × 3600 = 22320 × 103
 P = 2067 Watts
15. A
912×R 2 = 5.7×10–18×1×[T4–(300)4] × 4
R 2 18. 9

 108 (121) = T4  P1 
 T = 330 K log2  P  = 1
 0

 P1 = 2P0
16. 2
P  T4
A TB
B = TA
R 400R P2  3040 
4

 
P1 =  760  = (4)
4

P = eA T4
PA = e. 4 (400 R)2 × TA4  P2 = P1 (4)4 = (4)4 2P0
PB = e. 4pR2 × TB4
 P2   29 P0 
(400R)2 × TA4 = 104 × TB4  log2  P  = log2  P  = 9
 0  0 
16 × 104 × TA4 = 104 × TB4

TB
TA = 2

www.motion.ac.in
292 - Solution Physics (XI) Topic

1. C 7. A
P m From the formula

3 3RT
Since m is increased by a factor of ,
2 Vrms = M0

3
therefore, P will increase by a factor of .
2 3 R To2 3 R TN 2

M o2 M N2
3
 New pressure = × 76 cm of Hg
2
= 114 cm of Hg. Mo2
To 2  . TN 2
Mn2
2. B

mN
16
Fx   Vx 2    373  426.3 K
 14

3. D 8. C
From the formula
1 2
P  PVrms
3 3RT
Vrms =
M0
4. C

RT 3 RTo 2
P= M Vrms o 
w
2
Mo2

 Mw
=
P RT 3RTo2  2 3RTo2
Vrms O  2  2V
M o2 / 2 M o2
 Mw
  = =x
 P  0 C R  273

9. B
 Mw 273
  = = x The average velocity is given as
P
 100C R  373 373

8 RT
Vav 
5. C πM
P1V1 = P2V2
Independent of other gases. Hence average
velocity of oxigen in third container will be V1
6. B only.
Pv = n RT

394-Rajeev Gandhi Nagar, Kota


Heat-2 - 293

10. D 18. A
Same as above
1  2  3......... N NN  1 (N  1)
Vavg   =
N 2N 2
19. D

12  22  ........N2 2N  1 ΔQ  ΔU Δ W
Vrms  
N 6 ΔU  ΔQ  ΔW

Vrms 2 2N  1
U = Q  Po ΔV

Vavg (N  1) 3
1 1
U = Q  Po     
 2 1 

11. B

mN 20. D
Fx   Vx 2 
 Ist Process

ΔU1  ΔQ1  ΔW1


12. C
 16  20  4 KJ
13. A IInd Process
He - monoatomic H2 - diatomic
ΔW2  ΔQ2  ΔU2
f=3 f=5
ΔU1  ΔU2  ΔT  same
3 n He  5 n H 2
f
n He  n H 2 So, ΔW2  9   4   13KJ

3 5  5  2 21. D

5  2   3.57 Ans . U = Same
W = minimum in case of AFB
14. A

1
Average rotational K. E. = KT 2  KT 22. A
2
Q = U + W
So it will be same for both the gases.

23. B
15. D
Q = U + W

16. C
24. B
7  5 4  31
 V  V2 V2
5 W   PdV  2

V1
 aV dV
V1

17. A
V2
From the formula  V3  a 3
 a   
V2  V13 
 3 V 3
3RT 1

Vrms =
M

www.motion.ac.in
294 - Solution Physics (XI) Topic

1 .03 100 4 6  103  104 60


 P2 V2  P1V1  ΔT   
3 5R R R

1 1 30. A
 nRΔT   R  T2  T1 
3 3 V/T = const. P - const.

R
ΔQ  nCp ΔT  n ΔT
25. A  1
From the graph shown.
Δ W  nRΔT
Vav  T  PV
ΔQ 

Vav1 : Vav 2 : Vav3 ΔW   1

Vo Po : Vo . 4 Po : 4 Vo Po 31. B
 U = nCV T
1:2:2
Given
6300 = Ui = nCv (150)
26. D

ΔU ab  ΔQ acp  ΔWacb 6300


So, ΔU   300
150
 200  80  120 J
ΔU  12600
ΔWadb  ΔQadb  ΔU ab
32. C
 144  120  24 J
PV = constant

27. B dP dV
V   PV 1 0
dV dV
ΔQ ba  ΔWba  ΔU ba
dP  PV 1  P
 52 J 120 J  

dV V V
 172 J
0.7  105
 1.4 
0.0049
28. D

ΔU ab  U b  U a  2 107

U b  120  40  160 J 33. C

29. A 34. A

1
mv 2  nC v dT 35. C
2

1 m 5  36. C
mv2   R  ΔT
2 .03  2 

394-Rajeev Gandhi Nagar, Kota


Heat-2 - 295

37. A 39. A

Δ Q  ΔW  3 ΔW  4 ΔW Temp. of source T1 = 100 + 273 = 373 K.


Temp of sink T2 = 0 + 273 = 273 K.
ΔW ΔW Efficiency of carnot engine
 n   0.25
Δ Q 4 ΔW
T2 273
=1–T =1–
1 373
38. C
As the volume remains constant on increasing 100
=
temperature pressure becomes double. 373
V = const. T = doubled p = 2Po
100 100 1 120
To increase  = + × =
373 373 5 373

2PoA  TS  PoA  2 Po A T2 120


TS  Po A  = 1 – =
373 373
TS
Po A 373  120
or T2 = × 373 = 253 K.
373

New sink temp  253 K.

40. B
When volume = constant
(presure)  (Temp)

www.motion.ac.in
296 - Solution Physics (XI) Topic

1. D 2 2
KV2  KV1 P V PV
  2 2 1 1
2. C 2 2
3 3
3. A  ΔU  R 2 T0  T0   RT0
2 2
4. C
ΔQ  ΔU  ΔW  2 RT0
m
PV = nRT = RT
MW 12. D
2
Avg. momentum/mol   Vx
5. A
2
T1P11  T2 P21 V x is same at NTP

1

1
4
3
K . E .avg  T
P  1 4
T2  T1  1   300    300 2
 P2  4
3
K . E . /vol. T
6. D
13. A
v T Free Expansion
T1 = 4T
ΔW  0
P T So,  ΔU  0  ΔT  0
ΔQ  0 
7. C
and P1V1  P2 2 V1 
8. D
P1
PV  Mass of gas × Temperature P2 
In this problem pressure and volume remain
2
constant,
So M1T1 = M2T2 = constant 14. A

M2 T1 (27  273) 300 12


M =T = = = 15. A
1 2 (52  273) 325 13
5 7
Q = V + W = PV  PV = PV
12 12 2 2
M2 = M1 × = 13 × g = 12 g
13 13
V 5
i.e., the mass of gas released from the flask So 
Q 7
= 13 g – 12 g = 1 g

9. C 16. A
The number of molecules in 1 mole is always From the graph shown
same for all the ideal gases. P0 V
P – 2P0 = + P0
V0
10. D
P0 V
11. A P= + 3P0
V0
V2 V2
P 2P0 V dT
ΔW   PdV   KVdV  K + 3P0 = nR
V V0 dV
V1 V1

394-Rajeev Gandhi Nagar, Kota


Heat-2 - 297

3 Isothermal ΔUT  0 _______(2)


Till V = V the change in temperature is
2 0
Adiabatic PV   K
positive hence temperature increases. After
dT TV 1  K
that <0 so temperature decreases.
dV T1 V 1 1
 21  1
T2 V1 2
17. C
From the graph shown the equation of line T1
T2  2 1 T1 
is 2

 P0  F T
  P0  ΔUAdiabatic  n R 2 1  1
2 2  2
P – P0 =    V  V0 
 2V0  V0 



  
ΔUadiabatic  ΔUp 21 _____ 3

P0
P – P0 = 2V (V – V0) 19. B
0 Average velocity will be same for same
temperature.
P0 V 3P0
P = 2V +
0 2 20. C
Now we know PV = nRT
PV   Constant
3 PV
  P0  0  V  nRT TV  1  Constant
 2 2 V0 
1
For maximum temperature VB  T1  VA
  
VC  T2  VD
dT 3 PV
 0  P0  0  0
dV 2 V0
21. C
3 V2 > V1
V V0
2 nRT2 nRT1

P P
3 P 3 3 1
Tmax   P0  0 . V0  V0 . T2 > T1
2 2 V0 2  2 nR
22. B
3 3 1 9 P0 V0
 P0 . V0 . 
4 2 R 8R 23. D
vf = v0 Wgas = RT0 n
18. B Watm= pdv = pv ( – 1) = RT0 ( – 1)
At constant temperature U = 0
f
ΔU  n R ΔT
2 24. B
P1V1 1
For Isobaric process V1  T1  P = FA = 2(Kx)A = 2  100  1
nR 2
P1  V1 / 2  T1 N
At V2  T2   = 100
nR 2 m2

nfR  T1 
 ΔU P  _______ (1)
2  2 

www.motion.ac.in
298 - Solution Physics (XI) Topic

1. A,B 5. A,C

2. A,B 6. A,B

3. A,B,C,D
Vav  T
In the equilibrium position, the net force on
the partition will be zero. 7. B
Hence pressure on both sides is same U independent of path
Hence, (A) is correct
8. B,D
P1V1 PV
n1 = RT =
1 RT 9. A,B
UAB + UBC + UCD + UDE = Uif
(2P)(2V) PV 0 + (– WBC) + 0 + (– WDE) = Uif
n2 = = 4  n2 = 4n1
RT RT – 40 J + (– (– 25)) = Uif
Moles remain conserved. – 40 + 25 = Uif
Finally pressure becomes equal in both the Uif = – 15 J
parts. For path AB : Isothermal process
Using, P1V1 = n1RT1 QAB = WAB = 50 J
P2V2 = n2RT2
P1 = P2 and T1 = T2 10. A,D
V1 n1 1 7
V2 = n2 = 4 n1  ; n2 = 11
28 44
V2 = 4V1
Also, V1 + V2 = 3V  V1 + 4V1 = 3V 1 1 1
So n 1  n 2   
3 12 4 4 2
V1 = V and V2 = V
5 5 18kg
Hence, Option (B) and (C) are correct. m0   36kg
(1 / 2)
In compartment I:
P1'V1 = n1RT1
1 5 1
 R   3R
 3V   PV  4 2 4 11
P1'  5  =  RT  R (T) C V mix   R
    1 1 4

4 4
5PV 5
P1' = = P 11 15
3V 3 CP mix  R R  R
Hence, Option (D) is also correct. 4 4

C P 15 45 47
4. B,D r   
C V 11 33 35
P2 P 2RT
=k =k
 PM
11. A,B,D
 kM 
PT =    P  1 ....(1) 12. C
 R  T

P2 P2 P 13. C
=  P' =
  /2 2
14. A, B
Hence, from eq. (1) T' = T 2
PT = constant, hence P–T curve i s a 15. A,C
hyperbola in equilibrium
F net = 0

394-Rajeev Gandhi Nagar, Kota


Heat-2 - 299

4. 2/3
Mg(n  1)
1.  Let pA, pB be the initial pressures in A and B
nR
respectively. When the gases double their
The temperature gradient is given by volumnes at constant temperature, their
dT dT d dP pA p
 . . ...... (i) pressures fail to and B
dh d dP dh 2 2

From, dP = –gdh

dP mA m
 g ...... (ii) Also, pAV = RT and pBV = B RT
dh M M

Given that p/n = c (constant)


pA mA
 P = cn  
pB mB

dP
 cnn 1 ...... (iii) mA 2
d 
 mB 3
We know that

R 5. 28.7236 × 103 sec.


cn   T
M
d d 12800  103
dT M t  
 c(n  1)n 2 ...... (iv) vavg 8 RT 8 8.314  300
d R 
 m0  32  103
Solving above
= 28.7236 × 103 sec
dT Mg(n  1)

dh nR
20012 . 428
6.  10  25 kg  m / s
2. 2P 

Momentum = mv
3P 3PV m(v rms )2
vrms   P
 m 3V 8 RT
mvavg  m
 M0
P' = 2P

3. 7h  27 8  8.314  273


= 664  10
  (4  103 )
From Boyle's law (T = constant)
P1V1 = P2V2 = 8 × 10–24 kgm/sec

4 3
 (Hdwater + h dmercury) g  r  7. 1:2
3 
As average velocity is same.
dmercury
 H = 7h d 8RTH 8RTHe
water 
 H  He
 H = 7h

www.motion.ac.in
300 - Solution Physics (XI) Topic

TH M 2 1 a 2a
 H   v  dv  3 dT .....(3)
THe M He 4 2 T2 T

(1), (2) and (3)

8. 4 atmosphere  nR 2aP  (3  2  )
dQ =   3  dT = R  T
  1 T   1

9. 14

24
14.
1 1 5
VH MH 2  14
 
VN 1 1 15. T1 > T2
M N2 28
16. P/n

10. 50 calorie P dP
  V
V dV .....(1)
Given V

f  VPn = K
70  n  1 RT
 2   dV. Pn + nPn-1VdP = 0

dP P
f   .....(2)
70  n RT  nRT dV nV
2
From 1 & 2
f
Qv  n RT = 70  2R(45  40) P P
2    V( )  
nV V
 8.314 
 70  10 
 4.2  17. 120 R

 50 cal V2
nR 2nR  V2  V1 
W 
V1
Vk
dv 
K 1
7
11.
5  2nR(T2  T1 )

v2
12. Monoatomic W   Pdv
v1

 3 – 2  2
13. RT   – 1   Pv 
   v  KT , v  K 
2

 nR 
Q = U + W and nRT3 = aP

nR nR
dT  Pdv .....(1)
P = 2nR(60) = 120(1) R = 120 R
 1 vk
Pv = nRT => PdV + VdP = nRdT .....(2)

394-Rajeev Gandhi Nagar, Kota


Heat-2 - 301

18. 1500 J 3
22. 0
Wg = PV 2

V  V 105  nRT  m m
 105   V   105   0   ...........(1)
 VA  nRT0 
2  2 2  P   
 P 

105 1 8.314  360 m m


  = – 180R J Now B   ......(2)
2 105 VB  nR(2T0 ) 
 
 3P 
= – 1496.52 J
So work done on gas = –Wg = 1496.52 J B 1 3
From (1) & (2)   2 / 3  B  2 0
0

19. 3R
23. 1.25 × 104 N/m2
Q Q 3
Q  U  U   RT V nR
2 2 2  tan 53° = 4/3
T P
 Q = 3RT
 nCPT = 3RT 3 3
P nR   2  0.0821 atm
CP = 3R 4 4
= 1.23 × 104 Pa
20. PROOF

W   Pdv 24. (i) QT = WT in cyclic process


[5960 + (–5585) + (–2980) + 3645]
v2
2 2
= [2200 – 825 – 1100 + W4]
  kvdv  kv 2  kv1  p 2 v 2  p1v1 = nRT W4 = 765 J
v1 2 2 2
Wr
(ii)    100
T R Q(only the )
ncP T  nc v T  nR  Cp  C v 
2 2

 2200  825  1100  765 


21. –  5960  3645  100
dW = PdV
dQ = dW + du 104000 208
  100
Molar specific heat capacity is zero. 9605 1921
 Q = 0
So adiabatic 25. 1600 J
PV = Constant
r

TL W TL TL  300K
Now PV- = a = 1 –  = 1 – 
TH Q1 TH TH  600 K
= -b =   b = -

800 1 1
=1– =
Q1 2 2

 Q1 = 1600 J

www.motion.ac.in
302 - Solution Physics (XI) Topic

1. 5 For oxygen
v'2 = 750 m/s
hV2
P=
V1  V2
6. 1
n=5

7. – 20 kJ
2. 2
2 N/m2
8. 460 J

283
3. x 4
383 9. f = 6 & Cv = 3R & CP = 4R & r =
3

7 B
v0 C
4. 900 m3 2

v0 A D
3RT
5. For helium, 100 
M T0

3RT1
1000 
4  103
 3RT1 = 4000 3P0
D (3T0) C
(
21
2
T0

For oxygen,

3RT2
P0
A T0
B ( 7 T
2 0
1000   3RT2 = 32000
32  103 v0 7
v
2 0
By internal energy coservation
(U1 + U2 = U'1 + U'2) 7 
work done = Area = – (3P0 – P0)  2 V0  V0 
 
f
As we know U  nRT ; = – 2 P0 × 2.5 V0 - = – 5 P0V0
2
Heat is absorbed in AB & BC
3 5 5 24  3 5 5 24 
 RT1   RT       RT 8
2 4 2 32 2  2 4 2 32  7 
UAB = nCp T = n × 2 R  2 T0  T0 
 
T1  T2
T
2 5 
= 4nR ×  2 T0  = 10 nRT0 = 10 P0V0
 
After mixing rms speed of helium,

3RT 3R (T1  T2 )  21 7 
v '1   UBC = nCV T = n × 3R  2  2  T0
mw 2 mw  

= 21 n RT0 = 21 P0V0
4000  32000 Total = Q = 31 P0V0
=
2  4  103

v'1 = 1500 2 m/s


394-Rajeev Gandhi Nagar, Kota
Heat-2 - 303

10. 8 atmosphere
9 15
Adiabatic Process 12. T1 = (207/16)T0 ; T2 = T0 , – PV
4 8 0 0

V
PatmV   Pnew  
4
P0 P0
3
 Pnew  Patm 4 2
 8Patm V0 T0 V0 T0

11. (a) WAD=88 J, (b) VC=1.223 litre, (c) WCDA =


–85J
243 243
P P
(a) W = area under AD 32 32

1
W= (PA + P0) (V0 – VA)
2
P1– T = cons
1
= (1.6 × 105) (1.1 × 10–3)
2 2 / 3
2 / 3 5/3  243  5 /3 9
P0 T0  P  T  T T
= 0.8 × 1.1 × 102 = 88  32 0  4 0
(b) WADC =WAD + WDC and PV = cons
8 > WCD = –3
243  5 / 3
1 P0 V05 / 3   P V  V=
8
V
3 = (PB + PD) (VD – VC)  32 0  27 0
2

1 8V0 46
= (0.9 × 105) (1.3 – VC) × 10–3 V1 = 2V0 –  V
2 27 27 0

6 = 90 × (1.3 – VC)
 PV    PV 
   
80 1  T r  T  f
Vc = 1.3 –  1. 3  = `1.223 l
90 15
P0 V0 243 46 207
(c) = –85
 T  32 P0  27 V0  T  T
0 16 0

P1 V1  P2 V2
W=
 1

243 8
P0 V0  P  V
= 32 0 27 0   158
2 8
3

www.motion.ac.in
304 - Solution Physics (XI) Topic

1. B 2. A,C

n1C v1  n2C v2 Statements (a) and (d) are wrong. Concept


CV  of entropy is associated with second law of
n1  n2
thermodynamics.
For helium,
3. C
16 According to the figure
n1  4
4
1 3
Q1 = T0S0 + T0S0  ToS0
5 2 2
and 1 
3
Q2 = T0(2S0-S0) =T0S0
For oxygen,
Q3 = 0
16 1
n2  
32 2 W
 
Q1
7
and 2 
5
Q1  Q2

R 3 Q1
R
C v1    R
1  1 5  1 2
3 T

2T0
R R 5 3 1
C v2    R
2  1 7 2 T0
1
5 2
S
S0 2S0
3 1 5
4 R  R
 Cv  2 2 2
1 Q2 2 1
4  1 1 
2 Q1 3 3

4. A
5
6R  R
 4  29R  2  29R The change in internal energy does not de-
9 9 4 18 pend upon path followed by the process. It
2 only depends on initial and final states.
Hence, U1 = U2
R
Now, Cv 
 1
5. C
R
  1  Here, change in internal energy of the sys-
Cv
tem is zero, ie, increase in internal energy
of one is equal to decrease in internal en-
R R
or  1  1 ergy of other.
Cv 29
R
18
Box Box
A B
CP 18 18  29 Thermal
   1  1.62 a mole contact a mole
C v 29 29
N2 He

394-Rajeev Gandhi Nagar, Kota


Heat-2 - 305

9. A
5R
UA  1  (Tf  T0 ) As no work is done and system is thermally
2
insulated from surrounding, it means sum of
3R 7 internal energy of gas in two partitions is
UB  1  (Tf  T0 )
2 3 constant ie, U = U1 + U2.
Assuming, both gases have same degree of
Now UA  UB  0 freedom, then

3 f(n1  n2 )RT
Tf  T0 U
2 2

fn1RT1 fn2RT2
6. A and U1  ; U2 
2 2
For adiabatic process,
Solving we get,
dQ = 0
So, dU = - W (p1V1  p2 V2 )T1T2
T
p1V1 T1  p2 V2 T2
 nCV dT   146  103 J

nfR 10. A
  7  146  103
2
From first law of thermodynamics,
(f  Degree of freedom) Q = U + W
For path iaf,
103  f  8.3  7
  146  103 50 = U + 20
2
U = Uf - Ui = 30 cal
f = 5.02 =5
For path ibf
So, it is a diatomic gas.
Q = U +W
or W = Q - U = 36 - 30 = 6 cal
7. A
According to Mayer's relation

R R 11. C
CP  C V  
m 28
WAB = Q – U = nCpdT –nCVT
(at constant pressure)
8. B = n(CP – CV)dT = nR dT
For carnot engine using as refrigerator = 2 × R × (500 – 300) = 400 R

T 
W  Q2  1  1
12. B
 T2 
At cons tant tem perat ure (i sothermal
1 process),
It is given that,  
10
p 
T WDA  nRT ln  1 
 1 2  p2 
T1

T2 9
or  p 
T1 10  2.303 nRT log10  1 
 p2 
So, Q2 = 90 J (as W = 10 J)

www.motion.ac.in
306 - Solution Physics (XI) Topic

U(x = ) = 0
 105 
 2.303  2R  300 log  
 2  105  dU 12a 6b 
As, F    13  7 
dx x x 
1
 2.303  600 R log   At equilibrium, F = 0
2

= 0.693 × 600 R = – 414 R 2a


 x6 
b
So, work done on the gas is +414 R.

a b b2
13. C  Uat equilibrium  2
 
 2a   2a  4a
WAB = nRT = 2R × 200 = 400 R    
 b  b 

p 
WBC  nRT log  1 
b2
 p2   D  U  x     Uat equilibrium  
4a

 2  105 
 2.303  R  500 log  
 105  16. A

= 2 × 346 R = 692 R F F F
n1kT1  n2kT2  n3kT3
WCD = nRT = 2R × 200 = 400 R 2 2 2

 105  F
WDA  2.303  R  300 log    (n1  n2  n3 )kT
 2  105  2

= –415.963 R n1 T1  n2 T2  n3 T3
T
 Net work done in a cycle n1  n2  n3
= WAB + WBC + WCD + WDA
= 400 R + 692 R + 400 R – 415.963 R
17. D
= 1076 R
T2 1 T
1  1   1  2
14. B T1 6 T1

Thermal energy corresponds to internal en-


ergy T2 5
  ...(i)
T1 6
Mass = 1 kg
density = 4kg m-3
T2  62
2  1 
mass 1 T1
 Volume   m3
density 4
1 T  62
Pressure = 8 × 104 Nm-2  1 2 ...(ii)
3 T1
5 On solving Eqs. (i) and (ii)
 Internal energy = P  V  5  104 J
2
T1 = 372 K and T2 = 310 K

15. C 18. C
a b As no heat is lost,
U x  12
 6
x x Loss of kinetic energy = gain of internal en-

394-Rajeev Gandhi Nagar, Kota


Heat-2 - 307

ergy of gas As temperature is changing from 20 K to 4 K,


work done required will be more than W1 but
1 1 2 m R less than W2.
mv2  nC V T  mv  . T
2 2 M  1

21. C
Mv 2 (  1)
 T  K
2R Tsin k
Efficiency,   1  T
source

19. A
Tsin k
Internal energy of the gas remains constant, Now, 0.4  1 
hence 500 K

T2 = T  Tsin k  0.6  500 K  300 K

P
Using p1V1 = p2V2 , p2  300 K
2 Thus, 0.6  1  T'
source

20. C 300 K
T'source   750 K
Heat required to change the temperature of 0.4
vessel by a small amount dT
-dQ = mCPdT
22. A
total heat required
Efficiency of a process is defined as the ra-
3 tio of work done to energy supplied. Here,
4  T 
Q m 32   dT

20  400  W Area under p  V diagram
 
Q QAB  QBC
4
100  10 3  32  T 4 
   Po Vo
(400)3  4  20  
nC v T1  nCP T2

 Q  0.001996 kJ
Po Vo
Work done required to maintain the tem- 
3 5
perature of sink to T2 nR(TB  TA )  nR(TC  TD )
2 2
W = Q1 -Q2

Q1  Q2 Po Vo
Q2 
= 3 5
Q2 (2Po Vo  Po Vo )  (4Po Vo  2Po Vo )
2 4

 T1   T  T2 
W   T  1Q2  W   1 Q2 1
=  15.4 %
 2   T2  6.5

For T2 = 20K
23. A
300  20
W1   0.001996 =0.028 kJ
20
For T2 = 4k

300  4
W2   0.001996 =0.148 kJ
4

www.motion.ac.in
308 - Solution Physics (XI) Topic

24. A
 8  8
FBD of piston at equilibrium 54 –  = 76 1    = 76   
   

PatmA 54 – 2 = 76  – 608


PatmA
2 + 22 – 608 = 0

22  (22)2  4  808 22  54


P0A = =
(P 0+dP)A 2 2
mg
mg
= 16 cm

FBD of pistion when piston is pushed down a


distance x 27. B

d2x
Patm + mg – (P0 + dP) A = m ....(ii) B
dt2 800
P
Process is adiabatic
600
A C
PdV 400
 PV = C  –dP =
V V

1 A 2 P0 (1) dUAB = n cv dT


Using 1,2,3 we get f =
2 MV0
5R
=1× (400)
2

25. B = 1000 R
Newton’s law of cooling (2) dUBC = n cv dT

d 5R
= – K ( – 0) =1× (–200)
dt 2

t = – 500 R
d
   0  K  dt (3) dUABCA = 0

5R
(4) dUCA = 1 × (–200)
2
26. C
= – 500 R

8cm 28. 1
mean free path

P0 1

2d2n

76 × 8 =  × pgas no. of molecules


n
volume
76  8
= pgas
 = C
vavg.  T T.V-1
76  8
+ (54 – ) = 76

394-Rajeev Gandhi Nagar, Kota


Heat-2 - 309

 V
t  v  is volume 30. 4
v avg. T
Tmax. at mid point
 1
V 2  3  3V0 
V  P0 
C pv  9  P0 V0 
T= =  2  2  =  
vr 1 nR 4  nR 
nR

 1
vq  v 2
31. 1
 1 Pvn = k
q
2
R R
C = Cv + ; C – Cv =
1n 1n
29. 4
R R
150 200 1–n = CC ; n = 1 – CC
dQ dT dT v v
(i) s1    ms   ms 
T 100
T 150
T
C  Cv  R C  C v  (Cp  C v )
n= C  Cv ;n=
 150   200  C  Cv
 ln    ln  
 100   150 
C  Cv  Cp  Cv C  Cp
n= ; n = CC
3 4 C  Cv v
 ln    ln
2
  3

s1  ln2

112.5 125
dQ dQ dQ
(ii) s2        ......
T 100
T 112.5 T

 112.5   125 
 ln    ln    .....
 100   112.5 

9  10   16 
 ln    ln    ln  
8  9   15 

 16 
 ln   = ln2
 8 

www.motion.ac.in
310 - Solution Physics (XI) Topic

1. A 5. C
P1 = P2
Q 20,000
T = = = 50°C P0 + g (L0 – H) =P ...(i)
ms 1 400

Tf = 20 + 50° = 70°C
Now apply P1V1 =P
 1 
(B) V =  V T = (9 × 10–5)   (50)
 9000 

= 5 × 10–7 m3
w = PV = (105) (5 × 10–7) = 0.05 J
(C) U = Q – W = (2000 – 0.05) J
= 19999.95 J
P0L0 = P(L0 – H)

2. A-S ; B-P,R ; C-R ; D-Q,S P0L 0


P
Process J  K L0  H
(A) V = constant P  T 
W = 0, U = – ve and Q < 0 P0L 0
P0 + g (L0 – H) = L  H
(B) Process K  L 0

P = const. V  T  W > 0  g (L0 – H)2 + P0 (L0 – H) – P0L0 = 0


U > 0 and Q > 0
(C) In process L  M 6. B
W = U > 0 and Q > 0
1
(D) Process M  J PV = mN v 2
3
V w<0
(PV)J < (PV)M  TJ < TM 2 2
PV  {N(1/ 2 m v 2 )}  { total K.E.}
U < 0 Q<0 3 3

3
3. A  K.E. =  PV
2
Since it is open from top pressure will be P0.
Statement 1 is correct

4. D Statement 2 is correct but not the correct


explanantion of statement - 1.
Let P be the pressure in equilibrium
Then PA = P0A – Mg
7. C
PA
Mg
P = P0 – nRT 2
A PT2 = constant  T = constant
V
P0(2AL) = P(AL)
T3 V–1 = constant
{using P1V1 = P2V2} P0A
Mg on differentiating

  3T2 T3
2P0L  P0  dT  2 dV  0
L'   ( 2L) V V
P  P  mg 
0
 R 2 
394-Rajeev Gandhi Nagar, Kota
Heat-2 - 311

T2 Vf
3T dT = dV WAB = nRT ln V = nRT ln4 = P0V0 ln4
V i

dV 3
we know  =  ans. C
VdT T If in BC V T

TB V T 4V0
so  B  0 
8. A TC VC TC V0
A-q ; B-p,r ; C-p,s ; D-q,s
T0
(A) Free expansion W = 0, U = 0.  TC = PV = nRT
4
(B) PV2= c, PV = nRT, Q = n C T, for
polytropic process, PVx = constant, C = at A P0V0 = nRT0

R T0 P0
CV + . at C PCV0 = nR  PC =
1 x 4 4

(C) Q = n C T, for polytropic process, PVx =


14. 4
R
constant, C = Cv + .
1x 7
PV = PV 5 = consant
PV
(D) T = , U = +ve, W = +ve. nRT 5
7
nR .V = consant
V

2
9. B,D 2 2
 V 5
5
TV = consant
5 T1 V  aT1 
 32 
f 2 f
Cp  Cv     R  (f  1) R
 2 2
2
2
aT1 V 5
T1 V 5
 a 4
 f  2  f  4
CP C v    
 2  2 

15. A
10. B,D nR(T1  T2 )
W 
In BCD W < 0  Q1 < 0  1
U < 0 TV–1 = cm
In ABC, W = Area of semicircular  0
T1V12 / 3  T2 V22 / 3
For ABCDA, W = Area within curve > 0
2 2
T1(5.6)3  T2(0.7)3
11. A-P,Q,S,T ; B-Q ; C-S ; D-S
2
T2 = T (8)3  4T
1 1
12. D
Pressure is low and temperature is high 5.6
nR(T1  T2 )  R(T1  4T1 )
W= = 22.4
 1 2
13. A,B 3
f f
U PV  nRT UA = UB
2 2 1
 R  (3T1 )
4 9
=  RT1
2 8
3

www.motion.ac.in
312 - Solution Physics (XI) Topic

16. A-P,T,R ; B-P,R ; C-Q,S ; D-R,T 21. D


(A) A  B
3
2× R (700 – T)
Volume and temperature both decrease so 2
internal energy decreases and work is done
on gas and 5
= 2R (T – 400) + R (T – 400) × 2
2
Q = U + W = – ve
Hence heat is lost. T = 490 K

(B) W = 0 ; Temperature decreases so


internal energy of the system decreases. 22. D

W=0 3
2× R (700 – T)
Q = U + W = + ve 2
Hence heat is gained.
5
(C) C  D = 2R (T – 400) + R (T – 400) × 2
2
P = constant hence Temperature increases
T = 490 K
U = + ve
Q = nCP T = + ve 23. 2
Hence heat is gained.
(D) D  A
TD = TA
U = 0
W = – ve
Q = U + W = – ve
Hence heat is lost.

17. D Qbf
Qib = ??
3RT
vrms 
M wiaf = 0 + 200 = 200
Qiaf = 500 Thus Uiaf = 300
vHe MAr 40  Uibf = 300
   10  3.16
v Ar MHe 4
But Uib = 100 Thus Ubf = 200
 Qib = Wib + Uib = 50 + 100 = 150
18. D Qbf = Wbf + Ubf = 100 + (200) = 300
Q = nCpT Qbf 300
= 2 × 5/2 × R × 5 Ratio Q = =2
ib 150
= 208 J

24. D
19. D
f = 244 Hz

20. A 
= 0.35   = 1.4 m
4

394-Rajeev Gandhi Nagar, Kota


Heat-2 - 313

 (RT ) U  nCV dT
V = 244 × 1.4 = 314.6 =
M
3 
 n  R  3T1  T1 
2 
5  (RT ) 
For Neon : V =  
3 M 
3
 nR  T1  = 3nRT1 So option B
2
RT 10 RT 7 RT
=  . = U = 3P1V1
12 M 6 10 6
P1V1 nRT1

RT P2 V2 nRT2
For N2 : V =
20
P1V1 nRT1

7 RT 7RT P2 2V1  nR 3T1 
For O2 : V = =
5 32 160

5 RT 5RT
For argon : V = =
3 36 108

25. A,B,D

3RT 5RT
1  1
Emix. = 2 2 = 2RT
11

CPmix = 3R
3
3 P2  P1
mix = 2
2

1 4  1 2 1 P
Mmix = =3 w  2V1  1  P1 2V1 
11 2 2

RT P1 V1 5
  2V1P1  P1V1
Vs = 2 2
Mw
V1 = A
Vmix mix MHe 3 3 4 6 dV = V2 – V1 = Ax = V1
= =   =
VHe He Mmx 2 5 3 5
V1
x
A
1
Vrms  F = kx = (P2 – P1) A
MV

3 
MH2
kx   P1  P1  A
VHe 2 1 2 
VH2 = MHe = 4
=
2
P1
kx  A
2
26. A,B / B
Change in internal energy 1 2
E kx
2
P1 V1  nRT1

www.motion.ac.in
314 - Solution Physics (XI) Topic

1    5 3
kx x =n R (T2 – T1) + n R (T3 – T2)
2 2 2

1  P1   V1  P1V1 5 3
  A    = nR (8T1 – T1) + nR (T3 – 8T1)
2 2  A  4 2 2

So option (A) Ans A will come only when 5 3


we have considered that pressure on spring = nR (7T1) + nRT3 – 12 nRT1
2 2
side is constant.
Now for heat supply 35 3
= nRT1 – 12 nRT1 + nRT3
2 2
9
U  P1V1
2 11 3
= nRT1 + nRT3
2 2
P1V1 1 9  29
W ; Q  P1 V1    = P1V1
3 3 2  6 11 3
(100) + (25)
2 2

27. C 1100  75 1175


= = = 588
2 2
P1
P1 = 105 P2 =
32
V1 = 10–3 V2 = 8V1
PV 5/3
=K

P
P1V1 8T1
T1

1
P2V2

 = 5/3
Q = nCPdT + n CVdT

394-Rajeev Gandhi Nagar, Kota


Elasticity & Thermal Expansion - 315

1. B 8. B
Given  = 1.4 × 1011 Pa,  = 1.7 × 10-
5
2. A °C-1
T = 30°C - 20°C = 10°C
3. B P V
=– P = – 
V / V V
4. A P = (3T) = 1.4×1011×3×1.7×10-5×10
stress = 7.14 × 107 Pa.
Y  for same strai n (stress) A >
strain
(stress)B 9. D
YA > YB Even a small stress causes large strain.
Brealomg point of B > Breaking point of A In the case of perfectly rigid body, strain is
So, A is less dectile. zero.

5. B 10. C

In ductile materials, yield point exist while Let  = coefficient of thermal expansion, Y =
in Brittle material, failure would occur with- Young modulus of the wire. If the wire werre
out yielding. free to contract, its decrease in length would
be lt, where t = decrease in temperature.
To maintain constant length, lt becomes the
6. B effective elongation

 dU 
F   
 dx  11. A
Tension must be the same in both the rods
In the region BC slop of the graph is posi-
for their junction to be in equilibrium.
tive
Y1A1t = Y2A2t
 F = negative i.e. force is attractive
in nature
12. C
7. B DC2 = l2 – (l/2)2 = [l(1 + 2t)]2 – [l/2 (1 +
1t)] 2
We know that
Neglect 2 terms.
1
U= × stress × strain × volume
2
13. A
1 Since it is property of material
= × Y (strain)2 volume
2
14. C
1
U= Y (T)2 AL because stress is no longer proportional to
2
strain
U T2
U  (T - 20)2

www.motion.ac.in
316 - Solution Physics (XI) Topic

15. A 18. B
Let the extension of wire be x for an external Given L = 20 cm, L1=0.075 cm, L2 = 0.045
force F. cm
L = L  T
F x F/A FL
Stress = , Strain = Y= = 0·075 = 20 1 (100)
A L x /L xA
0·045 = 20 2 (100)
F YA Let for third rod L1 and L2 = 20 – L1
Equivalent force constnat = k = =
x L So L3 = L1 + L2
 0·06 = L11 100 + (20 – L1) 2 100
16. B L1 = 10 cm.
For two springs in series, the equivalent force
constant is 19. C

k1k 2 Given L = 1 mm, L = 6 × 10-5 mm


YA
k = k  k . Here, k1 = k and k2 =  = 12 × 10-6 k-1
1 2 L
then
KYA L = LT
L KYA m 6 × 10–5 mm = (1mm) (12 × 10–6) T
k= YA =  T = 2
K KL  YA k T = 5°C
L

20. C
17. B I = CMR2

L dI = 2CMRdR = 2CMR [RT]


=t = – 20
L = 2IT

means read more so actual is less

394-Rajeev Gandhi Nagar, Kota


Elasticity & Thermal Expansion - 317

1. A 
  
d2
2. B
i.e., the extension is inversely proportional
with in elastic limit body regains its previous to the square of the diameter. Hence, choice
state after some time A is correct. The strain is

 4F
3. C 
 d2 Y
F  A  F  (radius)2
1
Thus, strain 
4. A d2

T M2  2 i.e., the extension is inversely proportional


Stress   L  x2  to the square of the diameter. Hence, choice
A 2LA
A is correct. The strain is

5. A  4F

 d2 Y
We know that

F  1
Y Thus, strain 
A  d2
Hence, the correct choices is A
F
  
AY
7. C
Since, the two wires are made of the same
material. Young's modulus Y is same for both. 1
Since, load F and the cross sectional area A P.E. = × stress × strain × volume
2
are the same.
8. C
   
i.e., extension is proportional to the length 
of the wire. Hence, option A is correct. The r 1   V 2 r  
strain in a wire is given by   ,  ,w  
r r   V r 
 4F r

 AY
9. C
6. A Since maximum number of affected terms
Area of cross section in this only

d2
A  10. B
4
where d i s the diameter of the wi re. L
F  A
Therefore, L(1  t)

4FL A t AEt
  F 
d2 Y (1  t) (1  t)
Since F, L and Y are the same for wires A
and B

www.motion.ac.in
318 - Solution Physics (XI) Topic

 TA = 4TB
1. A,B St re ss T /r 2 = S. W i re breaks w he n
S = breaking stress.
2. A, D For rA = rB, SA = 4SB
Statement (A) is correct because the Young's  A breaks before B.
modulus of steel is greater than that of rubber.
TB
Statement (B) is incorrect. If a spring is For rA  2rB , SB 
stretched, both the total length of the wire rB2
is the coil and the volume of the wire do not
change. Only the shape of the coils of the TA 4TB T
SA    B2  SB
wire undergoes a change. Hence, it is the rA2  2rB 
2
 rB
shear modul us that d et ermi ne s the
As the stresses are equal, either may break.
stretching of the coil.
Statement (C) is also incorrect. The bending
7. A,D
moment of the prongs of a tuning fork is
The slope of the linear portion of the curve
determined by the Young's modulus of the
gives the Young's modulus of the material.
material. Hence the restoring force on the
The slope of the linear portion OP for material
prongs depends on Young's modulus which
A is greater than that of the linear portion
determines the frequency of the fork.
OR for material B. Hence, statement A is
Statement (D) is correct. When the material
correct. The plastic region for material A
is not subjected to any stress, its atoms are
(from P to Q) is greater than that (from R to
in their normal (equilibrium) positions. When
S) for material B, which indicates that
a tensile stress is applied, the separation R
material A is more ductile. Hence, statement
between the atoms becomes greater than
B is correct. The breaking stress for material
the equilibrium separation R0. For R > R0,
B (i.e. stress corresponding to point S) is
the interatomic forces are attractive.
less than that for material A (i.e., stress
coresponding to point Q), which implies that
3. A,C,D
material B can break more easily than
material A. Thus, material B is more brittle.
Hence, choice C is also incorrect. Material A
4. B, C
is stronger than material B because it can
StrainSame
withstand a greater stress before it breaks.
F The breaking stress is the corresponding to
Stress = = constant
A point Q for material A and to point S for
F  A material B. Hence, the correct choices are A
 F  r2 and D.

1 8. A,C,D
Energy = stress × strain × volume
2
 Area
 r2 9. A, C, D
(A) % rise in area =T
5. A, C = 2(T)
The compressive force in the rod will be = 2 × 0.2 = 0.4%
same at left end therefore, compressive (C) % rise is volume = 3 T
stress will be maximum at this end. Hence, = 3 × 0.2 = 0.6%
option (b) alone is correct
0.2
(D)  = = 0.25 × 10–4/°C
6. A,B,C 80  100
Tension in B = TB = mg/3
Tension in A = TA = TB + mg = 4mg/3 10. A, C, D

394-Rajeev Gandhi Nagar, Kota


Elasticity & Thermal Expansion - 319

1. 0
1 l D2 n2
Since it is free to elongate or l  or  12 
D2 l1 D2 1
2. 12 × 107 N/m2
10. 0.375 × 105 N
stress
 12  1010
1 1 D2
 Breaking strength = Breaking stress 
10 100 4
or stress × 1000 = 12 × 1010 Breaking stress in unchanged.
or stress = 12 × 107 N/m2 D is halved. So, breaking strength becomes
one–fourth, i.e.

(l1T2  l2T1 ) 1
 1.5  105 N or 0.375 × 105 N
3. 4
T2  T1

11. 5.28 × 10 8 Pa
4. 7200
On increasing temperature of bull by 100°C
Energy 1 (from 20°C to 120°C), the thermal expan-
 stress × strain sion in its volume can be given as
volume 2
V = st VT = 3stVT
1 Here it is given that no change of volume is
= Y(Strain) 2
2 allowed. This implies that the volume in-
crement by thermal expansion is com-
pressed elastically by external pressure.
3K Thus elastic compression in the sphere must
5.
2 be equal to that given in Eq. (i). Bulk modu-
lus of a material is defined as
Fl  F  l Kl
Y     ; Kl = constant
al  l  a a increase in pressure P
B 
K × 3 = K' × 2 volume strain V / V

3K V
or K'  . P  B   B(3st T)  5.28  108 Pa
2 V

12. 2
2
6. 1 P.E. 1
 Y(Strain) 2
1 1 T = 2 1 T Vol. 2

1  2 13. 10000 N

 2 1

F = Ay = Ay 

7. W = 10–3 × 1011 × 10–6 (100 – 0°) = 10000 N
Young's modulus will remain unchange
14. 5/3
8. 0 L = L1 + L2
(3L)0  = L1  + 2L2 
9. 2 30= (1 + 22)
F 1 5
Fl 0 = ( + 42) =
Y  a  Y 
Fl  4 3 3
l al or
D2  l
l

www.motion.ac.in
320 - Solution Physics (XI) Topic

1. 4 cm
m g2  d2 m  g2  a2 l 
F  F 5. a. b. 2
 Y ,   r2 r 
A  AY
a. Relative to the accelerating frame (trolley
car), the forces acting on the bob are mg ,
2. 1:2
–ma and T . Since, the acceleration of
1 the bob is zero relative to the trolleycar, we
P.E.  Y(Strain) 2  Vol have
2
    
Fnet  mg   ma  T  marel
3. 9 × 107 N/m2 
a
Free expansion of the rod = L
= 15 × 10–6/°C × 2m × (50 – 20)°C
= 9 × 10–4 m = 0.9 mm.
T
If the expansion is fully prevented, then 
m ma

9  104 
Strain = = 4.5 × 10–4 mg
2

where anet  0
 Temperature stress = Strain × Y
= 4.5 × 10–4 × 2 × 1011   
Then T  m a  g
= 9 × 107 N/m2
Since the string it light, same tension is felt
at all points of the string.
4. As the wire is not free to contract, the thermal Putting the area of cross section of the string
stress is developed at the wire A = r2 in the formula
The change in temperature
F'
p
T = 27°C – (–39°C) = 66°C A
Let L be teh change in length of the wire.  
Where F '  T  m a  g
L = LT = (2 × 10–5) × 1.8 × 66
 
= 2.376 × 10–3 m m a  g m g2  d2
p 
r 2 r2
FL FL 4FL
As Y=  
AL (D2 / 4)L D2 L F / A pl
b. As Y  
l / l l
YD2 L
F(tension in the wire) =  
4L ag
Sub st i t ut i ng p i n t he ab ov e
11 3 2 3
r2
(0.91  10 )  3.142  (2  10 )  (2.376  10 )
= expression, we have
4  1.8
 
= 3.8 × 102 N
l 
m agl

m  g2  a2 l 
2 2
r Y r Y

394-Rajeev Gandhi Nagar, Kota


Elasticity & Thermal Expansion - 321

6. 0.076 cm
It is given that at 10°C, volume of beer is l12
h  l22
500 cm3 and the area of cross section of can 4
is 125 cm2. Thus height of bear level is
When temperature is changed by t, new
500 value of h will become
h  4cm
125
l12
Now at 80°, volue of beer becomes h '  l22 (1  2 t ) 2  (1  1t )2
4
V80°C = 500 (1 + 3.2 × 10–4 × 70)
As we require h = h'. we have
= 511.2 cm3
At 80°, area of cross section of can becomes l12 l2
l22   l22 (1  2 2 t )  1 (1  21t )
Ar 80°C = 125 [1 + 2Al × 70] 4 4
= 125 [1 + 2 × 2.3 × 10–5 × 70]
= 125.402 cm2 l12
or 2 2 l22  1
Thus, rise in level of beer is 2
h = h' – h = 4.076 – 4.0 = 0.076 cm
l1 1
or l2 
2 2
l1 1
7.
2 2
F2l
First we must know what is a compenseted 8.
6AY
pendulum. We've discused that due to
change in temperature the time period of a As discussed in Mechanics, the tension T in
pendulum clock changes. Due to this, in the string at a distance x from its free end is
pendulum clocks, to make a pendulum some given as
specific metals are used which have very low
F
coefficient of expansion so that the error T  x
l
introuduced in their time is very small. The
other alternative of minimizing the error in
T F
time is very small. The other alternative of Hence stress, p   x
A Al
minimizing the error in time measurement, is
to use compensated pendulum. This is a T
pendulum made up of two or more metals as F
x
shown in given figure.

1
Substituting (p) in the formula U  p2dV
2Y

l
1 F2 2
We have U  x dV , where dV=Adx
2Y 
0
A2l2

F2l
This gives U 
6AY

In this case the distance of centre of mass


from suspension point O is

www.motion.ac.in
322 - Solution Physics (XI) Topic

11. (i) hollow sphere > solid sphere,


 1  3 s  
9. ( a) V0d0g  1     (ii) hollow sphere = solid sphere
L

(b) (i) L < 3s (ii) L > 3s (iii) L = 3s. 12. 100
Consider an element of thickness dx. Change
10. 4.90 × 10 –6
m
T dx
Let L, L2, L3 be the extensions of rods, in the length of the element is dl  and
S Y
AB, BC and CD respectively, when the weight
of 10 kg i applied at end D'. We have x
T  F1  F1  F2 
l
Force
Stress 
Area
dx
Using Young's modulus equation x

F2 F1
MgL MgL T T
Y  2

r L AL

MgL
or L  l l F1 
F1  F2  dx
YA
l
Considering rod AB,
 dl  
0 0
SY
L1 = 0.1 m, A = 10–2 m2,
YAB = 2.5 × 1010 N/m2 F1  F2  200  1
l  
2SY 2  0.5  2  1011
10  9.8  0.1
 L1   3.92  106 m
2.5  1010  101 = 100 × 10–11 m

Considering rod BC, x = 100

L2 = 0.2 m, A = 10–4 m2,


YBC = 4.0 × 1010 N/m2

10  9.8  0.2
 L 2   4.90  106 m
2.5  1010  101

394-Rajeev Gandhi Nagar, Kota


Elasticity & Thermal Expansion - 323

1. D
F1
The elastic energy stored x1   1 ..... (i)
A
1
U=  F  DI F2
2 x2   2 ..... (ii)
3A
1
=  200  103 J = 0.1 J Here x1 = x2
2
F2 F
 2  1 1
3A A
2. D
work done by constant force in displacing 1
the object by a distance l F2  3F1   3F1  3  9F
2
1 6. C
=  stress  strain  volume
2
 F/A
1 F l Fl  .T and Y 
  /
=    A L =
2 A L 2
So F = AYt
3. B
Energy stored per unit volume, F
Thermal stress    Yt
A
1
E=  stress  strain 7. A
2
given d = 20 cm
We know that
V = V0 = (1 + t) = V0(1 + 3at)
1 stress 1 S (since =  = 3)
E=  stress  =  2
2 Y 2 Y
Change in volume = V – V0 = 3V0t

3
4. A 4  d
= 3     23  106  100
3 2
We know that the Young's modulus Y = Force
×L/A×l
3
Or rewriting we hagve Y = WL/Al 4  0.2  6
= 3    23  10  100
3  2 
 l = WL/AY
Since the length of the wire is on both sides = 28.9 cc (1 cc = 10–6 m3)
of the pulley length L/2 will be on both sides. The correct option is (A)

5. D 8. D
F = ya T = Tension
A A  1 1
A11 = A22  2 = 1 1   F = 2 Tension = 2 ya T
A2 3A 3
= 2 sy T
1 9. (4)
 2 = 3
1
Strain =
100

www.motion.ac.in
324 - Solution Physics (XI) Topic

11. 3
1
Stress = (strain)y = × 2.2 × 1011
100
 '
T = 2 ; TM = 2
= 2.2 × 109 g g

T
= Stress = 2.2 × 109 T = 2.2 × 109 A Mg / A ' Mg ' Mg
A =  = = =1+
 /   A  A

T T 2.2  109.A Also:


v= = =
M P.A P.A
1 /2
TM '  Mg 
=  TM = T 1 
T   A 
2.2  109 2
v= =  106
7.7  103 7
TM2 Mg  TM2  Mg
 = 1 +   2  1 =
A T A
2
v 2 1 T 
f1 = =  106  = 178.2 Hz
2L 7 2  1.5
2
1 A  TM  
 =    1
10. C  Mg  T  
y = 2 × 1011
lf = l0 (1 + t)
12. D
 t = 100° C
 = 1.1 × 10–5 k–1 T
 
T
lf – l0 = t
 l = 1.1 × 10–5 × 100 12 40  
=
= 1.1 × 10–3 4   20
p 3 – 60 = 40 – 
y=
1.1  103
4 = 100
p
2 × 10 = 11
 = 25° C
1.1  103
 p = 2 × 1.1 × 1011 × 10–3 1 1
T =   × T ;4=  5 × 86400
 p = 2.2 × 108 Pa 2 2

8  105 8000
= ; =
5  86400 4320

 = 1.85 × 10–5/°C

394-Rajeev Gandhi Nagar, Kota


Elasticity & Thermal Expansion - 325

1. A To region its original length, l1 = l2


1 a t = 2 s t
mgL
1 s 1 s   L  
    r 2 Y
 2 a  1   2 a   s

 r2 Y   
2. l = 2s   m 
 g 
T  Vcube   
Depth submerged in liquid remains same
Substituting the values we get, m  3 kg
Upthrust = Weight
vi L g = vi’ L’ g  Answer is 3.
 L 
(Ahi) L g = A(1 + 2s T) hi  g 7. C
 1    T  R
2R
  = 2s F F F F

3. (a) 50°C (b) 0.05 J (c) 19999.95 J


2F /   4R 
2
F
(a) From Q = msT y= y=
 2R 2 1
Q 20000
T    50C F  2
ms 1  400 y=   = 2 Ans
2
R  2 1

 1 
 9  10  50 
5
(b) V  V T=  8. C
 9000  R
2R
= 5 × 10–7 m3 F F F F

 W= P · V = (105) (5 × 10–7) = 0.05 J


(c) U = Q – W = (20000 – 0.05) J 2
F
2F /   4R 
= 19999.95 J y= y=
 2R 2 1

4. 4 F  2
y=   = 2 Ans
k 2
R  2 1

m k=0.1 (140)
2
9. A,B
The maximum stress is called the breaking
F strength (stress) or tensile strength.
0.1  140  140  x  1  10
Y A , Y  The materials of the wire which break as
L 4.9  107  x  10 soon as stress is increased beyond elastic
L limit are called brittle. While the materials
 4  109 Nm2 N=4 of the wire, which have a good plastic range
are called ductile.
plastic region
5. D
h = T cos ( + /2)
2s
Strain

h= cos ( + /2) P
bg
Q

6. 3
proportional Stress breaking
limit strength
FL mgL
l1    Increase in Length elastic
AY r 2 Y limit
l2 = L   = Decrease in length

www.motion.ac.in
326 - Solution Physics (XI) Topic

1. B 9. B
G m1m2 GMsMe
F F
r2 R2
Gm1m2 F GMsMe F
F' 2
 F' 2

2r  4 3R  9
2. B
10. A
GM
VP= – (ve)
R
m 4m

as R  GM  but due to –ve it decreases. Double star system


R 2
k 1 1 / 2I1 m
3. D   2
k 2 1 / 2I 2 m1
2
4. B
11. C
GmM GmM 4
Uf  ui   G  R3 
 R R GM 3
 R   g 2 
 5 R R2
3g
5GmM GmM GmM mgR 
    4 R G
6R R 6R 6
12. D
5. C
g g

1 mGM 4 
2
mv2  mgh   90 ....(1) h 
2 R2 1  
 Re 

 1  h
mG  M 2 1 
1 Re
mv2   10   G
2 1 h = Re
2 R  ....(2)
3
  13. B
from (1) & (2) g g 4g
g'  2
 
 h  1 9
m
GM
 90 
9 mGM
 h1  h1  100m 1  R  1  2 
   
R2 10 R 2
decrease = g - g'
6. B 4g 5g
g 
9 9
Gm1m2
F=
r2
14. B
g GMe
7. D 
4 (R e  h)2
By theory
GMe GMe
2

8. B 4R (R e  h)2
Re + h = 2Re
GmM GmM Re = h
W 
R nR  R
GMm  1  GMm  n  15. B
 1 
R  nH  R  n  1  GMe GMe'
g= 2

 n  Re (5 R e )2
 mgR  
n 1

394-Rajeev Gandhi Nagar, Kota


Gravitation - 327

4 4 20. C
 R e3  (5 R e )3  ' (a) cavity at center, field is zero
3  3
R e2 (5 R e )2 (b) Arc of ellipse
 = 5’ (c) for escape T.E. = 0
(d) Notes.
16. D
21. B
GMPm GMP
 54   3  54
RP RP GM
g= 2
GMP R
 18
RP
2GMP 22. B
ve   2  18  6 m / sec v (Move tangentially)
RP

17. A

1 1
mv '2  2  mv2
2 2
23. C
v '  2 v0
2 2  2   24  hr.
v '  ve T =   2  2  2
rel.
 so escape.
T = 12 hr.
18. C
24. C
4
2 G  R e3
2 GMe 3
v0    2 
Re Re T  r 3 / 2  
 T 
4
 2G   R e2 1
3 
r /2
4 2
v' 2G  2R e      R1 
3 /2

3  2    r 
= 2v0    

R1 1
19. A  2 /3
r 2 
4 4
G R3A G R3B r
gA = 3 ; gB = 3 2 R1  1/3
R2A RB 4
RA = 2RB
 gA = 2gB 25. C

Ves = 2gR Gm Gm
v 
r R h
(Ves)A = 2gAR A  2 2gBR B

(Ves)B = 2gBR B Gm 2
v1   v
R 3
vA R
2
vB = 2

www.motion.ac.in
328 - Solution Physics (XI) Topic

1. C 64  2Gm
 v12 
From E.C. 27R
Gm1m2 1 1 After collision
O=–  m1v12  m2 v22
d 2 2
from M.C. m1v1 – m2v2 = 0 m 8m

2
Gm1m2 1 1 m v  v'1 v'2
Now,  m1v12  m2  1 1  u1 = v1
d 2 2  m2 
u2 = –v2
2 
2Gm2
v1 = d (m1  m2 ) o  8m (v2  v1 )
v’1 = 2 4
 (v1  v 2 )
2Gm12 9m 9
v2 = d (m  m ) v  v2
1 2 v’2 = 1
Relative velocity of approach = v1 + v2 18m
Gm8m
2G (m1  m2 ) Initial K.E. =
v= 3R
d
1 1
final K.E. = mv12  8mv22
2. A 2 2
GMm 1 16 2 1 2 1
F1 = 2
= 2 m 8 1 (v 1  v 2 )  2 8m (v 2  v 1 ) (1 8)2
R
GMm m 2
F2 = 2
= g (v1  v 2 )
3R
2 GMm 2 Gm2
 Change = 3 =
R
2 3 R
3. B 4. A
After collision r is max. separation
from M.C.
8mv’2 + mv’1 = 9mv
R 2R
 v  v2  4 
8m 1   m 9 (v1  v2 )  9mv
 18   

4 4 (2R3) =8m
v=0
3 3 from E.C.
8m G8m2 1 1 2 Gm2 G8m2
from E.C.
m  8mv 2  mv 2  
r 2 2 3 R 3R
Gm8m  r = 4R
O+O=–
3R v1 v2 5. A
1 2 1 2
+ mv1  8mv2 .......(1) 2
2 2 2GM
from M.C. In (a) & (b)
a2
mv1 = 8mv2
v1 = 8v2 .......(2) 2
Put value from eq. (2) to eq. (1) 4GM
In (c) & (d)
a2
G8m2 1 1
 m64v22  8m22
3R 2 2
2Gm
v 22 
27R

394-Rajeev Gandhi Nagar, Kota


Gravitation - 329

6. B 10. C
GM G 4  Gm1m2
 2  R 3  P.E. = –
We have g= 2 r
R R 3 
Gm1m2 Gm1m2
where  is the density of the earth. T.E. = – ; K.E. = +
2r 2r
G 4 3 
For the planet g =
R'2  3 R' 2 11. A
According to the question
g = g Fnet
(A)
G 4 3  G 4 3

R2  3 R   R'2  3 R' 2 
(B) Both direction and Magnitude not
R change
R =
2 (C) Total Mechanical is constant
 (B) (D) Linear momentum changes becoz v
change as r changes but rv = constant
7. D 12. A
G M em
= P.E.
8. C r
(A  P ; B  Q ; C  Q ; D  S) G Me
v 
r
GM GM
At surface g ;V  2
2 = r T  
R R T
GM g 13. A
height  R, g '   G – 2R = g/2
(A) 2 4
 2R 2 R = g/2
 h  R/2 g v2
(B) Depth  R, g'  g  1    g 1   = g/2
 R  R  2 R
2v2 = gR
GM V
(C) height  R, V '    ves = 2gR
2R 2
2
3 GM 3 v es = 2(2v )
(D) At centre, v'    V
2 R 2 = 2v

9. C
Both D & C between
Total energy is always –ve

www.motion.ac.in
330 - Solution Physics (XI) Topic

1. A,D
3 Gm 2
F  B.E. =
m1 F m2 2 a
F F
a1 = m a2 = m 5. BC
1 2
m1< m2  a1 > a2
6. B, C
2. B,C,D Net force towards centre of earth
mgx
M = mg =
E=0 R
v = const. Normal force N = mg sin 
mgx R
Thus pressing face N =
R 2X
3. AB
mg
N= Constant and independent
 GMr 2
 R r  R of X
g=  3 tangential force F = ma = mg cos 
 GM r  R
 r 2 R2
F1 r1  X2
a = g’ cos  = gx 4
(A) r, r2 < R  F2 = r2 R X
gx
F1 r22 a= R 2  4x 2
R
(B) r, r2 > R  F2 = r12 curve is parabolic and at X = R/2, a = 0

4. BC
7. A,C
a At the instant shown, both particle are at
Distance of any mass from centre = their mean position and moving in opposite
3
direction. Phase difference = 180°
 Radius of circular path followed = a / 3 As ‘’ is same for both particle
Mass is moving in circular path of radius  GM 
a/ 3    the phase difference will
 R 3 
mv 2 3Gm 2 a C be maintained throughout and they can
 = never meet.
(a / 3 ) a a/ 3
v max = A  For particle undergoing
S.H.M.
Gm 2(a / 3 ) 2a 3
v= T= = v1 R 2
a v 3Gm
 v = =
2 R/2 1
1 2 3 Gm 2
Total K.E. = 3 mv  =
2  2 a
Gm 2
P.E. = – 3.
a
3 Gm 2
 Total energy = – .
2 a

394-Rajeev Gandhi Nagar, Kota


Gravitation - 331

8. A,B,C 10. B,D


GM e Theory
v= (M ax.)
R 11. A,C
2 3
T r r T Theory
GMm
T.E. =
2R 12. A,D
9. A,C 2
T= r3 / 2
Radius decreases  Velocity increases G Me
due to which K.E. increases
G Me
GM g = R 2 and Tmin. at rmin. = R
m.v.r  m .r   r e
r
2 R 3 / 2 R
T 2  r3 Tmin. =  2
 gRe 2 g

www.motion.ac.in
332 - Solution Physics (XI) Topic

3Gm 2 5 –1
1. – 5. h R
a 2
G m2 2G m 2 3G m 2 g  h 
TE =    2
 g1  
a a a  h   Re
1  Re 
 6Gm2
 T.E. =
a
 6Gm2 4 Gm 2 2 Gm
final = 6. (i) , (ii)
2a 3 R 3 5R
6Gm2 6Gm2 3Gm2
W.D. =  
a 2a a  G M e .m 1  G Mem
(i)  m v2 
R 2 9R
Gm 2
2.
3 L2 4 GM e
v=
3 R
m
x 2L  GM em 1  GM e m 1
dx (ii)  m v2   m v 12
R 2 5R 2
3L
m Gmdx 2 2 GMe
=
2L
 dF = 
L
x2
v1 =
3 5 R

1 1   
F = Gm  L  3L 
   
 G0R  1 3
8   2G 0R 
7. g – i
2 , g – i
Gm2 6  2
x 3
F=   x – R  
3L
  2 
4
2G +v0
- Gr 0 pR 3
3. (sin ), (–G 2) g= 3
R x2

G
E=
R2  Rd cos 

–v 0

G
E= [2 sin ]  
R
G 0R 3  1 8 
Gm GR(2) gnet =   2 
Potential =  = – G2 6  (x  R )2 x 
R R  2 

–4GM2  3 1 
4. 3    r1
L  2 3 r2

n
U U1
2
m m 4 
4 R3 
G R 30 r1 G  0 r2
gnet = 3  3 83
m m R3 R
m m 4G0      2
= r r  G0Rˆi
3  2 1  3
m m
2

8 3Gm 3Gm2 Gm2 
U    
2  L 2L 3 L

394-Rajeev Gandhi Nagar, Kota


Gravitation - 333

8. 1.6 hours if is rotating from west to east, 10. (a) –GmMe/r, (b) –2GmMe/r
24/17 hours if it is rotating from west to
G M em G M em G M em
east.    
2r 2r r
When sense of ratation of both earth and
satellite is opposite v v
By M.C. m m
2 2
T1 =   2 2 final velocity = 0
rel 
24 1.5  G M e 2m
 T.E =
When sense of ralation of both earth and r
satellite is same. Straight line.
2
T2 = 2  2  GMm  1 1
 11. t  – 
24 1.5 2C  Re r 

9. 1 × 105 J GMm
 T.E.initial
2r
P.E.
 1 1 0 5 J | T.E. |  | |
2 G M m
 T.E.fin a l
2Re
5
T.E.final =  1  1 0 J | T.E. |  | |
GMm  1 1 
T.E.initial = –2 × 105 J Ct =   
2  Re r 
So given energy = 1 × 105 J

 x2 – R 2 
1 – 4 R 2
12.  x 
 

www.motion.ac.in
334 - Solution Physics (XI) Topic

Gm 2MG 
1. 3.
2d2 L
   dx.m Considering Figure. Let us divide the rod into
dF  G  2
x x very small mass elements, each one of mag-
d nitude dm, which can be expressed as :

m rd Ld
dm =  =  ......(i)
x
cos  cos 2 
x 
dx 1 Gm 
F  Gm  
xd
x 3  F   Gm   2 
x d
–
2d2
R
 M
r = R/cos
e
2. s
= 3.31.
Note: We are using a polar coordinate system.
Let r be the distance between the sun and rd
the earth, Me and Ms be the masses and Re Hence, we have d = on the rod.
cos 
and Rs be the radii of the earth and the sun
respectively, and G be the gravitational con- Since the rod is symmetric, the components
stant. We then have of the force element parallel to the rod will
cancel each other, and the total force will be
GM e M s the sum of the force elements perpendicular
= Mer2,
r2 to the rod. So, for a mass element dm at
distance r from M, we have :
2R s 1 2 
= = rad, GM (dm)
r 2 360 360
2 MG
720R s F(dm) =  L  cos  = cos d .....(ii)
i.e. r = . L
  cos  
and the total force is :
GM s
The above gives = 2,  
(720R s / )3 MG 2 cos d
2 MG 2 cos d 
2MG
F=
L



L  0 L
GM s 3 2 2
 720   2 
or =     .
R 3s     3  10 7 
da R
4.  0 for amax  x =
dx 2
For a mass m on the earth’s surface,
GmM e aMR 2GM
a 
= mg, 3/ 2
(3)3 / 2 R 2
R e2  3R 2 
2  
 2 
g 
GM e g
Giving = =  360  100 
R 3e Re 2  R GMx
 a
(R 2  x 2 ) 3 / 2
g
= .
18  10 3 x
2
e g  720  3  2 
Hence  =    
s 18  10 3     3  10 7 
4 2Gr 2
= 3.31 M  ( r 2 ) (2r )   a
(3)3 / 2 R
Now F = ma

394-Rajeev Gandhi Nagar, Kota


Gravitation - 335

2R 3 / 2 ( 6 6 ) cos  
3
5. 8.
GM (2 2  3 3 ) 10

GM GM w.r toCOM of ship & pad


1  3/2 ; 2 
(2R) (3R)3 / 2 Vr = V0
with respect to one time taken
m 2m vx
2vx
3Vx = V0
R
R 1 2
R0=3Rm
2R

Rm
2 2R 3 / 2 ( 6 6 )
t  t
1  2 GM (2 2  3 3 )

4r 3 / 2 Vx = V0 / 3
6.
G(4M  m)
m 2m
mv 2
Attraction force Fa = (2/3 + 1)v0
r
2 2
GMm Gm mv V0(1 – 1/3) = 2/3v0
  KI + Ui = Kf + Uf
r2 4r 2 r
2
1 2   – GM(2m)  1 2  – GM(2m) 
mv2/r ( 2m) v 0      (2m)v   
r fa 2  3   3R m  2  Rm 
mI

M ...(1)
GM(m 2 ) (2m )v 20 GM GM
  v0  
mII R 02 R0 R0 3R m
...(2)
G 2r 4r 3 / 2 from (1) & (2)
[4M  m]  v 2 Now T  
4r v G( 4M  m)
40 GM
 v
27 R m
7. 6.4 km
u2
Rmax = 4 = u= 4g v
g

According to problem 4g  2gpRp



4g = 2gpRp
GMe GMp 2/3v0
(2)  .R p
Re2
Rp2 1 3
cos   & sin  = Ans.
4 3 4 3
10 10
 R e  (2)  R p  ( 2) R p
3    3 
R 2e R P2
Rp  R e  R p  6.4 Km

www.motion.ac.in
336 - Solution Physics (XI) Topic

 1  Re
GM  2 8  10. T  sin –1  
 –   3 g
9. R  3 15  A
a = –2x
Gme
a= x
R 3e P
r
R M r2
Gme
2 
v1 R 3e

From W.E.T
v1r1  v2r2 Wg = kt – ki
6GM R
 R  v2r2 Gmemr 1 1
5R   3
dr  mv2p  mv2A
from energy conservation r
Re 2 2

1 6GM GMm GMm 1 1 1 Gmem 2


m    mv22 mv2p  mv2A  (R  r2 )
2 5R R r2 2 2 2 R 3e

v v2p  v2A  2 (R2  r2 )


r2
dr
 2gR e  2 (R 2  r2 )
dt

v A  2gR e 
mv2 GMm GM  
 v
r2 r22 r2
 1  Re
t   sin1 
 3 g

394-Rajeev Gandhi Nagar, Kota


Gravitation - 337

1. D 6. C
GMm Mass of planet, MP= 10Me, where Me is mass
Ui   of earth.
r
R
6.67  1011  100  102 Radius of planet, R P  e , Where Re is radius
Ui   10
0.1 of earth.
6.67  10 11 2GM
Ui   = - 6.67 × 10-10 J Escape speed is given by, vP 
0.1 R
2G  MP
For planet, vP 
-3
m=10x10 kg
RP
R = 0.1m 100  2GMe

M= 100kg Re
= 10 × ve
= 10 × 11 = 110 Km s-1
We know that
W = U 7. A
= Uf - Ui (Uf = 0) Gm
g'  , acceleration due to gravity at
 W = -Ui = 6.67 × 10-10 J (R  h)2
height h
2. C g GM R2  R 
2

  2 .  g 
 2h  9 R (R  h)2
R  h
gh  g1   ...(i)
 R  2
1  R  R 1
 d     
gd  g1   ...(ii) 9 R  h R h 3
 h 
As per statement of the problem,  3R  R  h  2R  h
ie, gh = gd
 2h   d 8. C
g1    g1    2h  d
 R   R  Let gravitation field is zero at P as shown in
figure.
3. C Gm G(4m)
 
GM 4  x2 (r  x)2
g ; M   R 3 
R2 3  m P 4m
A B
3 x r-x
4G R r
 g 
3 R2  2
4x  (r  x) 2

 4GR 
or g  (=average density) r
 3   2x  r  x  x
3
 g   or g Gm G(4m)
 VP   
x rx
4. A 3Gm 6Gm 9Gm
According to Millikan's oil drop experiment,   
r r r
electronic charge is given by, 9. B
6r(v1  v 2 ) Gravitational force provides necessary
q
E centripetal force,
Which is independent of g. Gm2 mv 2
electronic ch arg e on the moon ie, 
So, 1 (2R )2 R
elecronic ch arg e on the earth
m R R m
5. A
Correct option is (a) you can make an
analogy with Gauss's law in electrostatics. Gm
 v
4R
www.motion.ac.in
338 - Solution Physics (XI) Topic

10. D 14. A
Potential energy on earth surface is - mgR Solid sphere is of mass M, radius R.
while in free space it is zero. So, to free the Spherical portion removed have radius R/2,
spaceship, minimum required energy is therefore its mass is M/8.
K = mgR = 103 × 10 × 6400 × 103 J
= 6.4 × 1010 J Potential at the centre of cavity
= Vsolid sphere + Vremoved part
11. C
Energy = u + KE
 GMm GMm  1  Gm 
   M 
 R 3R  2  3R  R/2
C
5 GMm
=
6 R

12. B
mv2  2

= F14 + F24 + F34 GM 3R2   R   3G (M / 8) GM
r =   + =
2R 3
  2   2 (R / 2) R
Gm2 Gm2
= + 2 (1) (2)
4r2 2r2
Gm2 1  15. C
=   2 r
2r2 2 
(3) (4) GM GM
mv2 Gm2  1  2 2  v0 = = = Rg ; h  R
= R h R
 2 
r 2r2  
Gm 2GM
v= (1  2 2) Ve = = 2gR
4r R

1 Gm ve
v= (1  2 2 )
2 r v0 = 2

13. C Therefore i ncrease i n orbi tal vel oci ty


a1: a2 = 2r1: 2r2 = r1 : r2 (is same)
=  2  1 Rg

394-Rajeev Gandhi Nagar, Kota


Gravitation - 339

1. D
7. 3
In theory TA = TB
GMP 6 GMe MPR 2e 6
Given    ....(1)
RP2 11 R 2e MeR 2p 11
2. A-P; B-Q,R; C-P; D-Q,R
2
p  e
3
3. C
MP 2 Me MPR 3e 2
for r < R  3
 3 
 ....(2)
4 / 3  Rp 3 4 / 3  R e MeR3p 3
2
GMr mv
 2GMe 2GMp
R3 r  11 ;  v esc
R Re Rp
vr
GM mv 2 11 RP Me
for r  R    V  R e Mp ....(3)
r2 r esc

1 from (1), (2), (3) Vsc = 3 km/sec


 v2 
r
8. B
4. A
GMeMs ms v2 G.Me
Astronaut feel weight lessness when only gravitational 2
   v2
r r r
field is act
Object escape when its P.E equal to its K.E.
5. A G.Mem0
 K.E. =
r
M M
  K.E. = m0v = mv2
2
( 4R )2  (3R ) 2 7R 2
dm =  2rdr 9. B, D
4R 4R
Gdm(1) 2Grdr R
Up    2 2 1/2 =
3R (16 R  r )

3R
(16 
R 2  r 2 )1/ 2
P
Q

r
P r 2r

MP = M MQ = 8M MR = 9M
r 3R Radius of R will be slightly larger than 2r,
dr
4R
2GM
V  2gR  R
R
Hence VR > VQ > VP
2GM( 4 2  5) VP 1
after solving Up = – Also V  2 .
7R Q

work done by external agent = U – Up


10. B
2GM( 4 2  5)
 From energy conservation
7R
6. 6 GMm GMm GM2 1 GM2
  = mv2 +
L L 2L 2 2L
L m v r  mB v BrB m A v A rA
 A AA = m v r 1 2GMm 1
LB mB v BrB B B B = mv2
L 2
mB
= m 1 6 GM
A v= 2
L
www.motion.ac.in
340 - Solution Physics (XI) Topic

1 13. 2
Ei = mv2 Ef = 0.
2
In one particle potential energy should not g
be considered. 
2
8 h h
h  = ;1  R = 2 =1
1  
Energy of m is not constant.  R e  4 e Re

11. B h = Re

1 GMe.m GMem
R= R 1
10 e Re + Mv 2
=
2 2R e
9  x 
 dF =  .dx. 10 1  R 

e / 10  1 GMem GMe
m2 = ;=
2 2 2R e Re
R  Re   10
e
= 10–3 × 1  50   50   × R  2
    e
ve = 2v 0
1 1 
= 10–3 × 6 × 106  50  900 
  ve = 2gR e
= 108
 ve  2v  Nv
12. 7
= gR e

N=2

Writing the net force on system :

GMm GMm GMm Gm


2
 2 2

3l  4l   l2
 3l
2m m

GM M 2M 2m
    2
9l2 16l2 9l2 l

M M
   2m
16 9

9M  16M
  2m
144

7M
  2m
144

7M
 m
288
 k=7

394-Rajeev Gandhi Nagar, Kota


Fluid - 341

1. B 5. D
Pressure = h  geff. All the forces passes through O
a = g/3  no torque.
geff= g – g/3 = 2g/3
6. B
0.15  1000  2  10
P = h  g = 2P
3

P = 1KPa 4h
5
g =
8P
5 After loweringP
to liquid.
due

2. B 8P
Given A = 2 × 10–3, h = 0.4 m,  = 900 Kg/m3  PT = + P (Atmospheric pressure)
5
F = mg = Vg = (r2h)g
= 2 × 10–3 × 0.4 × 900 × 10 13P
=
5
= 7.2 N

3. A 7. C
F = mg
F = 10 N

4. D

PA = PB
 5×4×g+x×1×g
= (40 - x) × 1 × g  x = 10
Now, h1 = x + 5 = 15 cm
h2 = 40 – x = 30 cm
h2/h1 = 2
60 wg = h g
60 × 1 × g = h × 4 h 15 cm
8. C
Given m = 12 kg, A = 800 cm2,
 = 1000 kg/m3
P = gh

mg
= gh
A

So, volume = Ah 12  10
= 1000 × 10 × h
800  104
= 1 × 35 = 35 cm3

12 1200
=h ; h= = 15 cm
80 80

www.motion.ac.in
342 - Solution Physics (XI) Topic

9. B 14. A

1-x
x

Total buoyancy
= Total Gravitation
 13 × 1 × g + 12x × 1 mg (x + 2) = v × 1 × g × x
= 1 × 0.6 × g + 1 1.15 × g
3 3
v 0.8 g (x + 2)= v × 1 × g × x
1 + x = 0.6 + 1.15 0.8 x + 1.6 = x
x = 0.75 m 0.2x = 1.6 ; x=8
1 – x = 25 cm.
15. D
10. D Equilibrium Position W = FB
At equilibrium position W = L2h Mg
(abc) (d)g = (bc) hg
W
After displacing slightly x, extra buoyancy h=
L2 Mg
force.
net = ((bc)x)g
c 16. C
xbcg xg h a

a= = b Volume where metal is present


abcd ad
9. 8
= = 1.256 × 10–3
g 7800
=
ad Buoyancy = vg = 1.5 g
v × 1000 = 1.5
11. B v = 1.5 × 10–3
1V = 22V fraction of volume
m1 = m2
1.5  10–3 – 1.256  10–3
m1g = 0.92 Vg = m2g – xVg = × 100 = 16%
1.5  10–3
x = 1.8 gm/cm 3

17. A
12. C
4 4
×  (R3 – r3)g = 1 × R3g
13. B 3 3
W – v × 1 × g = W1
1/3
W – v × x × g = W2 R   
=  
W – (W – W1) × x= W2 r    1

W – W2
x = W–W
1

394-Rajeev Gandhi Nagar, Kota


Fluid - 343

18. B 23. B

0.5
Volume = = 10–3 m3 A B
500 v1 P1
Buoyamcy = Vg = 1000 × 10 × 10 = 10 N
–3

h v2 P2
m = 1 kg
If float = 2.5 kg C D

19. B v1 = v2 (equation of continuity)

dm v12 P1 v2 p2
= Av Also; + gh + = 2 +0+
dt 2  2 

P = Favg. 1sec. P2 – P1 = hg  P2 > P1


2Av2cos60°
24. B
1
1000×6×10 ×(12) × × 2
–4
2
= 86.4 Nt.

20. B
A1V1 = A2V2
0.02 × 2 = 0.01 × V2 a
g
V2 = 4 m/sec.

1 1
P1 + V21 = P2 + V22 a  g sin 
2 2 tan=
g cos 
1
4 × 104 + × 1000 × 22
2

1
= P2 + × 1000 × 42
2
 P2 = 3.4 × 104 N/m2

21. A
Liquid can not produce shear stress therefore
it surface becomes perfendicular to g.
25. C

22. A
From APVP = AQVQ

VP AQ (2  10–2 )2
= =
VQ AP (1  10–2 )2

VP = 4VQ

www.motion.ac.in
344 - Solution Physics (XI) Topic

dP . A = A dx . 2x
C1
2
 dp    xdx P2 = Cosntant – ax  b 2
 
2 2 3g Where C1 = Constant
P = 
2 2
29. B
3g
=
 2  20  10–2  10 = 2 m/sec.

26. A 30. B
AV = constant By theory
A  V
31. D
1 2
P + gh + v = constant Force exerted by the water on the corner
2
= change in momentum in 1 sec
V P
mv
27. B = 2 mv
mv
r1 3
A1V1 = A2V2 (Given r = )
2 2 = 2 vL
2
v1 A2 r22 2 4
v2
= A1
= 2 =   = 32. B
r1 3 9

28. A

x 2gx
P

h
V
x
R
A = ax + b
Continuty equation bV = (ax + b) V2

1
2gx
2h R2
By bernaulies equation = P2 + v22 = R= x=
2 g 4h
cosntant

1
P2 = Costant - v22
2

1 b2 V2
P2 = Costant -  2
2  ax  b

394-Rajeev Gandhi Nagar, Kota


Fluid - 345

33. B 37. C
By Bernoulli's theorem
Na
10  10 50
P0 + +1000 × 10 ×
1000  104 100

a
1
= P0 + × 1000 × v2 H
2
H/2
1 v=?
6000 = × 1000 × v2
2

v= 12 = 3.4 m/s
Force = a  2gh / 2  2

34. A agh
acceleration = = g/N
Change in momentum is/sec. Na.H

2 Av = 565.7 N.
2

38. B

35. D dt = Av dt

A1V1 = A2V2 1
(1 × 10 ) × 3
–2 2 10–4 = 10–4 2gh  h=
2g

h = 0.051 m
= 100 × 
0.05  10 –2 2
× V2
4
39. C
V2 = 48 m/sec.
dV
 A 2gh
36. D dt

Inlet = outlet
40. D
dt = a 2gh dt
2h1
R= 2g(H  h1 )
2 g
h=
2ga2
2h2
= 2g(H  h2 )
1002 g
h=
2  1000 (1)
(H – h1) h1 = (H – h2) h2
H = h1+ h2

H
For max. range =
2

www.motion.ac.in
346 - Solution Physics (XI) Topic

1. D T = Ag [1.5 L – x]
Force is same pressure is different
 x
A xg cos    2  = T cos
 
2. B
Take area of projection from left  x
x     =  [1.5 L – x] x=
 2 
2gh2 3gR 2
=
2 2
5. D
3
h= .R Pressure at the bottom = 2hg
2
force at the bottom = 2hgA
At balancing condition
3. B
Downward force by vessel wall + W = F
g/2 F.W. = FD

h 6. A
Let AB = L, AC = L/2, AD = l, A = area of
h-x cross–section of the rod.
x
Weight of the rod = ALg, acting through C.
B A
2h – x x
Buoyancy force = Al0g, acting through the
midpoint of AD.
xg/2 + 2(2h-x)g/2 = PB – PA
PA = (h – x)g D
B

PB = hg + 2xg 0
L C
xg/2 + 2(2h–x)g/2 = hg + 2xg–hg + h
2
xg

4hg 2xg A
– = 2xg + xg/2
2 2
Taking torque about A,
2hg – xg = 5Pxg/2
l L
4h lA0g cos   LAg cos 
x = 2 2
7
l2 
or 
4. A L2 0

T h L
Also, sin   
l 2l

l 1 
or  
L 2 sin  0
mg

mg = A.2L × 0.75 × g 1 0
or sin  
2 
T + Axg = A.2L × 0.75 g

394-Rajeev Gandhi Nagar, Kota


Fluid - 347

7. D (3M + m)g = vg


Torque balance about B
dx
x 
mg (d-) + vg
 2

p+dp p
vg d(v  2M)
= 2 mg + (d – ) =
2 2(v  3M)
Consider an element of the liquid of width
dx and area of cross–section , at a distacne
x from the front of the tank. 11. C

Mass of the element = dm = ( dx).


Net force to the right on the element =
(p + dp) – p =  dp.
  dp = ( dx)a

C C

or  dp   a dx
A A
or pC = pA = al.

(Refer to the figure in the question to identify 4 3


A,B,C) . a =M
5
Also pB – pC = gh
(m + M) = a3
or pB – (pA + al) = gh
or pB – pA = hg + la. 5 M
m= M m=
4 4
8. D M=4m

12. A

2
gHR2
3

9. A

2T
10. B + mg = vwg
2

v v
.g .g
2 2 13. A

m,v B,M 2m,v

mg

mg 2mg

www.motion.ac.in
348 - Solution Physics (XI) Topic

17. B
3
d1AL + d2 AL = LAd
2

3d 3d
d 1 + d2 =  d1 >
2 4

14. B
geff = g + a
 t + mgeff = FB 1 2
(a-x)a2 + x a2 = a3
2 3
T = Vd(g + a) – v (g+a)
= v[(g + a) (d-)]
x 2
(a-x) + = a
2 3
15. B
Initially x 2
a = a
2 3
Wmetal = Wice = Buoyancy
Vmetal mg + Vice ice g = Vdg 2a
x=
3
v metal m V 
Vd =  ice ice
 
x acc. 2
 tan  = = a= g
finally volume displaced a g 3
V = Vm + Vw (From ice)

m Vi  i 18. B
= Vm +  = Vm +  < previous From A1V1 = A2V2
w w

1
(1) (V1) ( )V
16. B 2 2

V1 1
 V 
2 2

V2 = 2V1
Now,
V22 = V21 + 2gh

 10 
4V21 = V21 + 2(10)  
PA(1.2 × 0.7 × g + 0.8 × 1.2 g)  100 
0.8 × A0 (x + 1.2 + 0.8) g= PA.P0
x + 1.2 + 0.8 = 0.84 + 0.96 2
V1 =
3
x = 0.25 cm
Now volumetric rate of flow
= A1V1

1  10–4 60 2
= –3
 = 4.9 lit/min.
10 3

394-Rajeev Gandhi Nagar, Kota


Fluid - 349

19. A Distance through which it moves down =


By Theory h1  h2
.
2
20. B  loss in gravitational potential energy
2
 h  h2 
  1  A .
 2 
h p
The mass of the entire liquid = (h1+h2+h)A.
dh If this moves with a velocity v,
p+dp
1
it kinetic energy = h1  h2  h Av2 .
2
Consider an element of the liquid of height Equating energies, we get v.
dh and area of cross–section  at a depth h
below the surface of the liquid. Let p and p
22. A
+ dp be the liquid pressures at the upper
and lower surfaces of the element.
Mass of the liquid in the element=dm=( 23. C
dh). By Theory
Ne t up ward force on the el em ent =
[(p + dp) – p] – g dm 24. D
or  dp – g dm = a dm, By Theory
as the element moves up with an acceleration
a
25. C
or  dp = (g + a)( dh)

or  dp     g  a dh
or p = (g + a)h a0

dh
21. C
When the levels equalize,
h
the height of the liquid in each arm =
v = 2gh
h1  h2
. a1
2
We may then vi sual i ze that a l ength dm dh
 A  a0 –a1 2gh
h1  h2 h1  h2 dt dt
h1   of the liquid has been
2 2
dh
transferred from the left arm to the right 4000 = 1 × 2 – 0.5 2gh
dt
arm.
Then, dh
for t =  =0
dt
 h1  h2 
mass of the liquid =   a ,
 2  2 = 0.5 2gh
where, A = area of tube,  = density of the
h = 0.8
liquid.

www.motion.ac.in
350 - Solution Physics (XI) Topic

26. D 29. D
Initial v1 = 2gh / 2 = gh

1 for v2
ag = V2
2 h 1
gh + 2g = 2.v22
2 2
V0 = 2ga
2gh = v22

a V0 v2 = 2gh
Now V = 2 = 4
2 2
30. A
27. C
2
From A1V1 2  d
 mg =    .2gH.
Where V1  to area
2 2

2M
d=
H
60

31. D
°

(A) Slphon works when h3 > O


This will create a pressure difference
V cos 60° (B) P3 = P0 = P2 + gh3
P2 = P0 – gh3
(C) P3 = P0
V
ratio = =2 32. B
V cos 60
2(H  y)
x= 2gy
g
28. D

2H 33. D
R= 2g  10 .............. (1)
g

1 2 Y
Now gh + PO + PE = PO + V
2 A1,V1 A2,V2
X
2PE
V2 = 2gh + 

2PE 2H
R’ = (2g10)  ( ) .....(2)
 g
From A1v1 = A2v2
From (1) & (2) PE = 3 atm.
v2 > v1
From Bernoulli’s
1 1 2
Px + v21 = PY + v
2 2 2

34. A
By Theory

394-Rajeev Gandhi Nagar, Kota


Fluid - 351

1. B,C 5. A,C
Gauge pressure at C = gh
Gauge pressure at D = ax

y
2. B,C h
Balance B reads = 5 Kg + Buoyancy
A reads = 2 Kg –FB

h
3. A,C

xm
ma

a
tan  = (backward) 2(2n  y)
g
x= 2gy
mg g

dx
for xm  = 0
4. A,C dy

WB = W1 y = h
Wa = W
Buoyancy due to air = W 6. B,D
 When air inside the balloon F = av2
W = W2 P = Fv = av3
Buoyancy eliminate the effect of air inside
the balloon
 W1 = W2
So, W2 = W1 + W

www.motion.ac.in
352 - Solution Physics (XI) Topic

4. 37.5 N
2m 11
1. h ,H m
 32
h
Pressure at A & B is same
So, PA = PB

Mg A B

P0 + 2 2 H
(R  r )

= P0 + gh
F
M Pressure at point A =
A1
h =
(R  r 2 )w
2

F
Now, Pressure at point B= A +g(8+h)
1
Total water is cylinder + Total water in pipe
Pressure at point C = Pr. at B
750 750
= Kg R H + r h = 
2 2
Pressure at D = PC – gh
1000 1000
F 
3 2  1 =  A  gh
H =  4  r h  2  1 
  R 
Now at equilibrium

2. 101.8 Kgf–m F 
(600) g =  A  g8 A/2
 1 
3R
__
W = mgh (600) g =
B
COM
 F 
 –4
 750  10  8800  10– 4
 25  10 
2
= R3  × g × h 60 F
3 = + 6  F  37.5N
8 25

3. 54.4 cm
2(3   ) 3
5. h1 = = 0.26 ; h2 =  0.195
15  10
Po

3g
0.1 m

Po

PA = PB wgh = Hg g (0.04) g / (0.1  X)(0.05)2  3g  g(0.1)2 x

Hg x = 0.16
h =  (0.04) = 13.5 × 4, h = 54 cm h= 0.1 + 0.16 = 0.26 m
w

394-Rajeev Gandhi Nagar, Kota


Fluid - 353

(b) 7. 19.6 m, 4 sec


30 = (0.1) x  1000  10
2
Wnet = K = 0
 Wmg = WB = 0
30
x  0.096 m Total = 0.196 m. mg(19.6 + h) = 2 mgh
100 x 
h = 19.6 m

1 2
Y  gt = h
6. 2
y– xz
t = 2 sec
Initial depth in sea water = x0 T = 2t = 4 sec.

8. 2.79 gm/cc

V m 
m+ w =    V  w
2  
(x0+x) (x0+x-y) (x-y-z)+x0
m = mass of beaker
Water V = interior volume of beaker
WS  WC  x 0 A  Sw g ........(1) f = density of material of beaker

WS  WC  (x o  x)A  w g........(2)
9. at the water surface,
WS  (x o  x  y)A  w g........(3) They will meet at the surface
2m
WS  (xo  x  y  z)Aswg........(4)

(1)/(2) 2h
2m 2g.2h
SW xo
1 .
W x o  x , m 2g. h/2
h

   W  W x __
3 mg
xo  SW   x  xo  m
2

 W  SW  W
a = g/2

x o  x  y W mg
(3)/(4) 1
x o  x  y  z SW
10. h/2
  Just before collision
x0  x  y  z  W x o  (x  y) W
SW SW

   W  W
x0  SW   (x  y) xyz
  SW  SW

Just After collision


W 
x  (x  y) W  x  y  z
SW SW

SW y
 ( gh)2 h
W yxz  Hmax = =
2g 2

www.motion.ac.in
354 - Solution Physics (XI) Topic

11. 45° , 9600 2 (gauge) N/m2 2 x 2 dy 2 x


y= =
2g dx g

dy
= 1 at x=0.3 m
dx
2R sin 45° 2R
=R 2
10 10
 = = rad/s.
0.3 3

2
dy  10  1
   5
  2 = = Tan
dx 1 =  3  an
x m
2
3
a 10
 = tan–1 = 45°
g

Pmax = g R 2 14. 5

1 
12. 100 kg a =   1 geff
 

 17 
=  1 .12 = 5m/s2
 12 

15. 21.42 cm

P0 P1
2 1
tan= = x1
10 5

(b)2 a 111/5 1 3 a
V= = = m P1 = P0 + g .
2 2 10

m = V = 100 Kg. = 72.5 + 2.5 = 75 cm of Hg

10 5
13. = rad / s , tan = P0 P2
3 3
x2

P2 = 72.5 – 2.5 = 70 cm of Hg
P2 x2 = P1x1

75 20
 x2 = = 21.42 cm
70

394-Rajeev Gandhi Nagar, Kota


Fluid - 355

Applying Bernoulli’s theorem b/w


3a
16.  G & I we get
2g

For the given situation, liquid of density 2 1


P0 +  gh1 = P0 +  V2
should be behind that of  from right limb : 2
& b/w I & H

a 1 1
PH +  (2V)2 = P0 +  V2
2 2

3
PH = P0 –  V2 = P0 –3  gh
2

C A PH = P0 –  gh2 3  gh1 =  gh2


B 
2
h2 = 3h1

PA = Patm + gh
18. (a) 6 2 m / s , (b) 9.6 2 × 10–3 M3 /sec,
 
Pa = PA + a = Patm + gh + a (c) 4.6 × 104 N/m2
2 2


PC = Pa + (2) a
2

3
= Patm + gh + a …(1)
2
But from left limb :
PC = Patm + (2) gh …(2)
from (1) and (2)

3 1
Patm + gh + a = Patm + 2gh (a)  g(3.6) =  V2
2 2

3a V = 6 2 m/s
 h = 2g 
(b) Discharge rate of flow = AV

2
 4 
17. h 2 = 3 h1 =    10– 4  6 2

  

9.6 2  10–3 m3 / s

(c) PA = P0 –  g(5.4)
= 105 – 105 (5.4),
= 4.6 × 104 N/m2

www.motion.ac.in
356 - Solution Physics (XI) Topic

19. 20. 20 cm, 60 cm


3: 2

M N
40 cm

(a) for A 20 cm

h 1 h
g   v2 v= g
4 2 2

RA = vT

h 3h 2 h In N water level is = 60 cm.


= g . . RA = 3
2 4 g 2 In M = 20 cm.

(b) for B

h h 1
2g  g  .2v2 gh = v2
4 2 2

v= gh RB = gh

2h h

4g 2

RA 3h 3
=  2 =
RB 2h 2

394-Rajeev Gandhi Nagar, Kota


Fluid - 357

1. 6N
1
 (2  4  5)  (0.6  5.4)   2.4  4  5
2
3/5 = 40 - 12 - 24 = 40 - 36 = 4m3
2/5
a=g/2
Inital = 2 x 4 x 5 = 40m3

4
T mg %= x 100  10 %
40

 2 3g  y' 9m/sec2
3  
T+mg =  5 x10  x 1500x 2  y
    x'
3
(c)
 3   3g 
  x103  x 1000x  T=6N
 5   2  x Pressure = 0

9 y'
2. 4m/s2, 10%, 0, 45 kPa tan  =  = 0.9  y' / x'
10 x'
a
9 y y ' 3
(3-x)
tan    
10 x x
y' = 0.9x - 3
(a) x
1 1 y'
40 = (y' 3)x X x 4  y' x4x
5m 2 2 0.9
x=5
1
2  4  5  (5  x  4)   5(3  x)  4 Pressure at rear wall
2
= ax = 1000 x 9 x5 = 45 KPa
a 3x
 x 1m tan   
g 5 3. x = 1/3

a 2
  a  4 m / sec2
10 5
l-x
x

x
x l-x
3
(b)

5m
l-x x

a = 4 x 1.2= 4.8 m/sec 2


x sin 600 g + (+20) (l-x)
a x sin 600 = (+20) x sin 600g + (+0) (l-x) sin 600g
tan    x  2.4 m
g 5
x  (  2o )(l  x)  (  2 o )x  (  o )(l  x)
Water split out
x = l/3

www.motion.ac.in
358 - Solution Physics (XI) Topic

 M – m (M – m)gx  L  M  m
4. (a)  g – (b) t   
M  m (M  m)L 2 g  M – m
8a 2a 4a 2a 6a
  2a    2a
3 3 3 3 3
C
M M A B
 A
LA L
2a/3 a/3
a
Mg - T - Ax wg=Ma ...(1) PA  2ag PB  22a.g  4ago
T-mg = ma ...(2) 5a
T 3
M
Mg - mg - .w xg m
L 5a/3
a 2  25a2 
M L (PC  PA )S  s2  xdx =   a2 
= (M+m) a x
a
2  9 
A
M w
(M  m)  xg  (M  m)a 8
L   s2a2.........(1)
9

2a
 M  m  M  m  gx
a g (PB  PC )s  2s2
 M  m 

 M  m L  xdx
5a / 3

(b)
a = 0 initial x = L 2s2  2 25a2  2 2 11
 4a    s a x ..(2)
M.P. is when it is just submerged. 2  9  9

1+ 2
M m  L  x'  x'
a g1 
M  m  L   8 11 
(PB  PA )S  s2a2  
9 9 
 M  m  gx' (L-x')
 
M  m L 19
4ag  2ag  2a2x
9

M  m g M m g 19 18g
2       2g  2a 
M  m L M  m L 9 19a

M  m L T  M  m L
T  2       2
M  m g 4 2 M  m g 6. m
80

2x 2 (20)2 x2
y   20x2
18g 2g 2 x 10
5.
19a Volume = ydx 2x
1.5 = 20x2.2xdx
1.5 = 40x3dx
0.5
x
8a 1.5  40 x3dx  y
3 x
x
dx
A B
40
a a
1.5 
4

(0.5)4  x 4 
1

8a 4a x  (0.0155)4
gx  2.xg x
3 3 Area = x2 = 0.39m2.

394-Rajeev Gandhi Nagar, Kota


Fluid - 359

Ld v  4m / sec .
7. (g + a) P1
2
(ii) F  (av)v  av2
P1 = Patm
P2  900  5  104  (4)2  7.2 N
Ldg P3
P2 = Patm + (iii)
2
Fmin = 0
Lda mg  0.01 900  0.6  0.5  600  0.6  0.5 10
P3 = P2 +
2
= 45 N
Ld Fmax = 45 + 7.2 = 52.2 N.
 P3 – Patm = (g + a) (iv)
2
A1v1  A 2v 2 0.5 v1  5  104  4
8. (hg + a) 
v1  4  103 m / sec for both
9. 1.2 sec

1 S  12 14
Acceleration w.r.t vessel is a =   (g + a). 12. t
 S  15 – 14
One glass of juice = Vm3
10. h 1 = h2

A
h2 15V
u2 15V
ho h
A
h1
u1 a

u1 = 2g(h0  h1 ) u2  2gh2
-adh = a 2gh dt

2h1 2(ho  h2 ) 14V/A


u1  u2
g g
dh a
2 2
u h  u (h0  h2 )
  2gh
  A dt cm
1 1 2

2g(ho  h1 )h1  2gh2(ho  h2 )


15V
2
ho (h1  h2 )  h  h 2 A
1 2

ho  h1  h2 h1  h2  0 2  15V 14V  a
    x 12....(1)
h1  h2 2g  A A  A

11. (i) 4m/s, (ii) F = 7.2 N, 2  14V  a


  0  t....(2)
(iii) Fmin = 0, Fmax = 52.2 N, 2g  A  A
(iv) both 4 × 10–3 m/s
(2)/(1)
1 2
(i) v  (600  10  0.6)  (900  10  0.4)
2 t 14 12 14
 
12 15  14 15  14
1
900 v2  3600  3600  7200
2

www.motion.ac.in
360 - Solution Physics (XI) Topic

1. D 5. C
1 < 2 as denser liquid acquiires lowest From Bernoulli's theorem,
position of vessel.
1
3 > 1 as ball sinks in liquid 1 and 3 < 2 gh  (v22  v12 )
as ball doesn't sinks in liquid 2, so 2
1 < 3<2 2
1 2   v2  
 gh  v1    1
2. B 2   v1  
 
oil <  < water
Oil is the least dense of them so it should 2
1 2   A2  
settle at the top with water at the base.  gh  v1    1 ( A1v1  A2v 2 )
Now the ball is denser than oil but less denser 2   A1  
 
than water. So, it will sink through oil but
2
will not sink in water. So it will stay at the  A2  2hg
oil-water interface.     1  2
A
 1 v1
3. C 4
From FBD of sphere, using Lami's theorem,  D1  2hg
   1 2
D
 2 v1
F
 tan 
mg
D1 8  103
When suspended in liquid, as  remains same.  D2  1/4
 1/4
 2gh   2  10  0.2 
F 1  2  1  
 v1  (0.4)2
  tan    
 
mg 1  
 d  3.6  10 3 m.
Using Eqs. (i) and (ii), we get 
6. A
F F T
 , Mg = Kx + B
mg  
mg 1   F AL
 d Mg = Kx +  g
2
F Mg ALg
L
L
where F   mg
  2
K x= M
K 2K L A
2
F F Mg  AL 
  1
= 
mg  
mgK  1   K  2M 
 d

1 1 7. A
or K  2
  0.8 
1 1  
d  1.6 
R
Rsin

4. A 
R R
 Rcos
u2 v2
Max. Range = i.e.,, (radius of circle) R-Rsin
R–Rcos
g g
2
 v2  v 4 R – R sin  = d1 (R – R sin ) + d2 (R sin  + R cos )
Area occupied =    l = d1 (1 – sin ) = d1 (1 – cos ) + d2 (sin  + cos )
 g  g2
d1 – d1 sin  = d1 – d1 cos  + d2 (sin  + cod )
d1 (cos  – sin ) = d2 (sin  + cos )

d1 sin   cos  1  tan 


d2 = cos   sin  = 1  tan 

394-Rajeev Gandhi Nagar, Kota


Fluid - 361

1. C
2L2 2L2
From A1V1 = A2V2 .....(1) Hg = H
2 2g

3. C
h

4r
h
h

2r
H

1 2 1
Po  gh  v1  Po  0  v22 .....(2)
2 2
Area of Base of the cylinder = (2r)2 =
After solving eqn (1) & (2)
4r 2
you will get the answer.
Area of Hole = (r)2
Net force is just balanced when height is
h1 then
L2  2
2. H
2g

PA  (h1g)(4r 2 )  P0 4r 2

h dx h 
mg 
3
(2r)2hg Po r 2

B A P0 3r 2  (h  h1 )g(3r 2 )

L
x

at balancing coudition

PB  gh  Po 
P0 (4r 2 )  (h1g)(4r 2 )  (4r 2 )hg
3
PA  Po  (H  h)g ....(1)

In small element having mass dm.  Po r2  Po 3r2  (h  h1 )g(3r 2 )

AdP = (dm)2x
4
AdP = A.dx2x 4h1g  hg  3hg  3h1g
3
PA L
5
 dP  2 xdx  h1g  hg
PB 0
3

5h
2L2 h1 
(PA  PB )  3
2

www.motion.ac.in
362 - Solution Physics (XI) Topic

4. B 8. A
Statement-1 is true (by observation)
Statement-2 is also true
Po A  Po (4r2)
but Statement-2 does explain statement-1
si nce when water move s up vel oci ty
decreases and area increase while when
h2 water moves down, velocity increases and
thus stream down.

9. B
 Po r 2
.4r2hg
3 air from end 1 flows towards end 2.
(Po  h2 g)3r 2 Volume of the soap bubble at end 1
decreases

At balancing condition 10. D


Buoyant force is resultant of pressure force
 4h
.4r 2hg  h2g3r 2 h2  of liquid.
3 9

11. B
5. A
P11  T1  P21 T2
Cylinder will not move up and remains at
its original position P1  Po  lgh T1  To

4h P2  Po  lg(H  y)
Because at h2  , cylinder bend to
9
move 12. B

4h  nRT2 
upward and h2  it remains at rest.
9 Buoyancy force = lVg  lg 
 P2 

2 /5
6. C  P  lg(H  Y) 
T2  To  o 
R2P0L0 = P(L0 – H)R2 .....(i)  P  lgH 
P = P0 + g (L0 – H) .....(ii)
P2  Po  lg(H  Y )
E lim inating P we get the answer

13. 6
7. B
Pf = P0 – gHf = 0.98 ×105 . Since for air,
Force from right hand side liquid on left hand
Pi Vi = Pf Vf , 1(300 – H) = 0.98 × 300
side liquid.
 H = 6 mm
(i) Due to surface tension force = 2RT
(towards right)
14. A  P, T, B  Q, S, T, C  P, R, T
(ii) Due to liquid pressure force

x h N
f
  p0  gh 2R ·x  dx
x0
v

= (2p0Rh + Rgh )(towards left)


2

 Net force is |2p0Rh + Rgh2 – 2RT|


For (P)

394-Rajeev Gandhi Nagar, Kota


Fluid - 363

Force exerted by x on Y. F > A


(A) is correct.
= R  N2  f 2  mg
For equilibrium of B
W.D. by friction is +ve so mechanical T + VFg = VBg
energy of the system is decreasing T = Vg (B –F) .... (2)
For (Q) For T > 0, B > F
x is balancing weight of Y and Z. (B) is correct.
P.E. is increasing because height is From (1) & (2)
increasing.
mg
2F = A + B
For (R) Mg 2 (D) is correct.
P.E. is decreasing
Total Mechanical
16. A
energy is decreasing.
For c = 0.5 exactly it will be half submerged
For (S) Mg because upthrust of half solid portion of
As mass move downwards, container will balance its total weight.
Fluid move upward so potential energy For c < 0.5 more water is required and c >
of x is increase. 0.5 less water is required.
Mechanical energy is constant because
Fluid is non-viscous. 17. A,D
For (T) Hint :
V = constant so Fx = Mg (A) For upper block
In this due to viscous force 4 4
kx + R3 × = R3 × 2
Mechanical energy is not conserved. 3 3

4 R 3  
15. A,B,D x=
3 k

(D) Cannot provide upward force if partially


submerged.

18. C
A
From equation of continuity
A1V1 = A2V2
T (20)2 × 5 × t = (1)2 × v × t
v = 400 × 5 = 2000 mm/s
T
= 2 m/s

B 19. A

Let V be the volume of the sphere


for equilibrium of A
T + VAg = VFg
T = Vg (F – A) .... (1)
For T > 0

www.motion.ac.in
364 - Solution Physics (XI) Topic

Pressure at point in nozzle  P 20. C


P = P0 – 1 a V1 2
......(1)

1
P = P0 –  V 2 – lgh ......(2)
2 l 2
equation (1) and (2)

1 1
 V2=  V 2 + lgh
2 a 1 2 l 2
Neglecting the term regh

a
V2 = V1
l 2h
d = vt = 2gh = 2h which is independent
g
Rate of liquid flow = AV2
of g.
a a But when the lift falls freely no water leaks
= AV1 l Rate of liquid flow  l out of the jar as geff = 0.

394-Rajeev Gandhi Nagar, Kota


Surface Tension - 365

1. D 5. A
2T’ sin d = 2 Rd. T × 2 F = (1 – 2)l
T’ = 2RT due to film
6. C

2A 2  8  75  10
F  dyn  105 dyn
T’ T’ t 0.12

7. C
2. C
2T
P= r is small pressure is high
r
 air from higher pressure to lower
pressure

2T  2
hg = 8. B
0.1
4T
2  75  10 3  2 P1 = P0 +
r1
h × 1000 × 10 = P2
0.1  10 3
P2 > P1
P1
300 4T
h= - 0.3 m = 30 cm.
1000 P2 = P0 +
r2

r2 < r1
3. D
Initial surface Area 4T
P2 – P1 = Pexcess =
= 2 [1 × b] = 2 [10 × 0.5] = 10 cm2 R
final surface area
1 1  4T
= 2 [10 × (0.5 + 0.1)] = 12 cm2 4T  r  r   R
Increase in surface Area = 12 – 10 2 1

= 2 × 10–4 m2
r2r1
W.D. = Surface Tension × Increase in R = r r
Surface Area 1 2

W.D. = 72 × 10–3 × 2 × 10–4


= 1.44 × 10–5 Joule 9. C
W.D. = 1.44 × 10–5 Joule
4T
Pexi =
R
4. A
(Force)S.T. = S.T. × Total length 4
4 R2d = r3
F =  × (2r + 2R) = 2 (r + R) 3
(3R2d)1/3 = r
F
 =
2(r  R ) 2T
Pef =
F = 3.97 gwt = 3.97 × 10 × 9.8 N
–3 r
r = 4.25 × 10–2 m, R = 4.35 × 10–2 m 1/3
putting values R  R 
 Ratio = =  
 = 7.2 × 10–2 Nm–1 2r  24d 

www.motion.ac.in
366 - Solution Physics (XI) Topic

10. D 15. B
(Force) surface Tesion
4 4
R 3  n  r 3 or R3 = nr3
3 3 = Surface Tension × circumference
= 6 × 10–2 × circumference.
Energy evolved = (n × 4r2 – 4r2)T

Q Q (force)S.T. = (Force)weight of liquid


Now,   = (In C.G.S. system,
V S V
r = 1 and S = 1) 6 × 10–2 × C = 75 × 10–4

n  4r2  4R2  T 75  10 4
  C=
4 6  10  2
R 3
3 Surface Tension = 1.25 × 10–2 m

3T nr2  R 2  16. D

R3 Total excess pressure = weight of liquid
column of h
 nr2 R 2  1 1 
 3T  3  3   3T    4T cos 0º
 nr R  r R  = hg
r

4T
h=
11. B rg
Assuming isothermal conditions,
4  73.5  10 3
h=
 4   4 3   4   4 3  2  10 3  103  9.8
P    a    P    b 
 a 3   b 3 
h = 1.5 × 10–2 m
h = 1.5 cm
 4   4 3 
 P    c 
 c 3 
17. B
or P[a3 + b3 – c3] = 4[c2 – a2 – b2] Let P1 in the broader tube & P2 that in the
narrower tube
P  c3  a3  b3 
or  Pressure just below the meniscus in the
4  a2  b2  c2 
2T
broader tube = P1 –
r1
12. B
r1 = 1.48 × 10–3 m
13. B
2T
narrower tube = P2 – r
R h 2
cos =
R r2 = 1.44 × 10–3 m
difference of these pressure is
R  h
 = cos–1  
 R   2T   2T 
 P1     P2   =hg
 
r1    r2 

14. B
1 1
2T P1 – P2 = h  g – 2T   
= h g  r2 r1 
R
Radius of Meniscus P1 – P2 = 1860 Nm–2

394-Rajeev Gandhi Nagar, Kota


Surface Tension - 367

18. B 21. B
initial pressure in the capillary P1 = P
4 3 4
Initial volume of air in 2 r  R 3
3 3
capillary = v1 = LA
Final pressure in capillary R = 21 / 3. r

2T v  r2
= P2 = P +
r V’  R2
Final volume of air in capillary V’  (21/3 r)2
V2 = (L – ) A V’  41/3 r2
P1V1 = P2V2 V’  41/3 V

22. A
 2T 
P × LA =  P   ( L – ) A v
 r   a2 = mg sin 37º
t
 2T 
 L v 3
 r  3
= 2T  a2 = a g.
P  t 5
r

 = 0.005 m 3agt
=
= 1/2 cm 5v

19. C 23. C
v
Terminal Velocity
1.5
800 = A. x
x
24. D
v
2400 =  A 2 r 2g
x VT =
9 
  
v = 4.5 cm /sec.
2
2 0.003   10
 (1260) × 2
9 1.260
20. D
vT = 0.04 m /sec.
4 3
FR = K .r2 . v2 =  r g 0.1
3  Time = = 0.25 sec.
0.04
v r
25. C

www.motion.ac.in
368 - Solution Physics (XI) Topic

1. B
2F cos 
Pressure due to 15 cm long liquid needs to This gives m  F  F
g 
be balanced. m
where F = Tl
2. D
2Tl cos 
25 × 10–3 × 2 × 0.1 = m × 10 Then m
g mg
5  103
or m kg  0.5 g For m to be maximum, cos  = 1
10
2Tl
Hence, mmax 
3. C g

40 2  7  102 6. D
 1000  9.8 
100 R Initial pressure of air in the cylinder
P0 = 105 Nm–2
14  102  100
or R  m  = Surface tension
40  1000  9.8
4
Pinitial inside bubble = P1 = P0 +
14  1000 1 r
 mm  mm
40  1000  9.8 28
4
1 Pfinal inside bubble = P2 = P +
Diameter = 2R = mm r /2
14
4 3
Vinitial = v1 = r
4. C 3
If surface tension is neglected the condition
3
for floating gives 4 r  1
Vfinal = v2 =   v1
800 × 10–3g = (0.12x)g 3  2  8
or x = 0.08 m
By Boyle’s law
( = 1000 kg m–3 for water)
P1V1 = P2V2
Since water wets the cube, the angle of
contact is zero and force of surface tension  4   8  v1
acts vertically downwards. So it is buoyed  P0  r  v1   P  r  8
   
down by surface tension.
 800 × 10–3g + 4 × 0.1 × 0.7 24 24  0.08
= (0.12x')g P = 8P0 + = 8 × 105 +
r 2.4  10  4
0.028 P = 8.08 × 105 Nm–2
or x  0.08   0.08  2.8  10 4
98
Therefore, the additional distance = 2.8 × 7. B
10–4 m Excess pressure in bubble A

4
5. A P1 = r1
Let the mass of the needle be m. As the
liquid surface is distorted, the surface
tension forces acting on both sides of the B
needle make an angle , say, with vertical. A
Since the forces acting on the needle are F,
F and mg, resolving the forces vertically for
its equilibrium, we have

F y  F cos   F cos   mg  0
Excess pressure in bubble B
394-Rajeev Gandhi Nagar, Kota
Surface Tension - 369

12. B
4
P2 = Due to surface tension property.
r2

Excess pressure in double bubble 13. C


v = (/4)(d2/4 – r2) d v/dr = ( /
4
P= 4)(– r2) = – 2r/4
r
 =  (dv/dr) = (–2r/4) = –2r/4
P1 – P2 = P At the wall, r = d/2. Hence,
4 4 4 2(d / 2) d
  wall = = –
r1 r2 r 4 4

1 1 1
  14. A
r r1 r2
From Formula

r1r2
r = r r 15. A
2 1
By definition
r = .004 m
16. D
8. D

2T
R T T
4 T' =  T’ = = 1:1
2 T' 2
R
d  (3 3  4)iˆ

9. A
Excess pressure always on the concave side
of the surface. For angle of contact of 2/3,
the liquid should have a convex surface. So, Wall = Change in K.E.
the excess pressure should be in the upward
direction. 17. A
Assume a cylindrically symmetric, linear
10. C velocity profile for the flow of oil in the film.

h2  cos 2 1 r
 2  Cylinder
h1 2 1 cos 1
r v Oil film

Piston
1
h1 140 
2 1
or 
h2 2 70  1
To find the frictional resistance, compute
the shear stress at the piston surface.
h1 6
or h2   cm  3 cm
2 2 dv  19 
= µ = 0.020   (12) =
dr  (5 . 000  4 . 990) / 2 
11. A
912 lb/ft2
Effective length of flat part = 2r
Effective length of curved part is r.   4.990  3 
F = l F = A = 912  12  12  = 298 lb.
   
So, required ratio is 2 : .

www.motion.ac.in

f
370 - Solution Physics (XI) Topic

1. C,D the capillaries must be equal


Initial surface area = 2 × length × separation So, options (A) and (B) are correct.
= 2 × 10 cm × 0.5 cm = 10 cm 2

= 10 × 10–4 m2 5. A,C
Final surface area = 2×10 cm×(0.5+0.1) Let S = surface tension
cm = surface energy per unit area
= 2 × 10 × 0.6 cm2 = 12 × 10–4 m 2 r = radius of each small drop
The required work, R = Radius of a single drop
W = T A
4 3 4
= 72×10–3 × (12 × 10–4 – 10 × 10–4) J n r  R 3
3 3
= 72 × 10–3 × 2 × 10–4
or R = rn 1/3
= 144 × 10–7 J
Initial surface energy,
Ei = n × 4r2 × S = nE
2. A,B,C,D
Final surface energy,
i. Excess pressure inside a spherical liquid
drop = 2T/R, because there is only one free Ef = 4R2S = 4r2n2/3 S = n2/3E
surface here. Therefore, energy released = Ei – Ef
ii. In case of spherical meniscus of radius of = E(n – n2/3)
curvature R, also excess pressure = 2T/R
because again there is only one free surface.
6. C,D
iii. Excess pressure inside a cylindrical drop
The angle of contact at the free liquid surface
of radius R = T/R. Hence, for a cylindrical
inside the capillary tube will change such
bubble is air, excess pressure is 2T/R,
that the vertical component of the surface
because there exist two free surfaces in this
tension forces just balances the weight of
case.
the liquid column.
iv. For a spherical bubble in water, excess
pressure = T/R, as there is only one free
7. A,C
surface.
Viscous force is somewhat like a frictional
Hence, all the four options are correct.
force but not exactly the same because it
differs from it due to the two main following
3. A,D statements :
Theory i. Viscous force is velocity dependent while
frictional force is not.
4. A,B ii. Viscous force is temperature dependent
while frictional force is not.
T
h So, options (A) and (C) are correct.
Rdg

Given that h, T, d and g are fixed, hence R 8. B,C,D


must be same. As weight of the liquid is
Logic & Theory
balanced by force due to surface tension (T
cos  and R are fixed for a given liquid, given
mate ri al of capi l l ary at a c onst ant
temperature) hence, weights of liquids in both

394-Rajeev Gandhi Nagar, Kota


Surface Tension - 371

1. 6 × 10–3 J and B are 4T/RA and 4T/RB, respectively.


Original total surface area = 2 × 4r1 2

= 2 × 4  × (0.1)2m2
(as bubble has two surfaces) RA
PA PB RB
Final total surface area = 2 × 4r2 2

= 2 × 4 × (0.2)2m2
Therefore, extension in area
= 2 × 4[(0.2)2–(0.1)2] = 0.24 m2 4T 4T
 pA  and pB 
Now, work done W = surface tension × RA RB
extension in area
If R is radius of common interface, then we
= 25 × 10–3 × 0.24 = 6 × 10–3 J
must have
4T
2. 0.6 g p A  pB 
R
10 4T 4T 4T
m × 10 = 2 × 3 × 10–2 × or  
100 R A RB R
or m = 6 × 10–4kg This gives
= 6 × 10–4 × 103g = 0.6 g

R AR B 0.002  0.004
R    0.004m
1 1  RB  R A 0.004  0.002
3. 4R3T   
r R
As the excess pressure is always towards
concave surface and pressure in smaller
bubble is greater than that in larger bubble,
4. 1.274 × 105 N/m3
the common surface is concave towards the
The excess pressure for air bubble is
centre of the smaller bubble.
2T
p
r 6. 4.76 mm

2  72  103 2T
or p = 1440 N/m3 Pressure at A = p0  r
0.1  103 2

Since the bubble is just below the water (Since, pressure inside a curved surface is
surface, the external pressure on it is equal greater than that outside)
to the atmospheric pressure P. Hence the
pressure inside the bubble is 2T
Pressure at B = p0 
r1
P + p = 1.013 × 105 + 1440
= 1.013 + 105 + 0.0144 × 105
1 1
= 1.274 × 105 N/m3 Therefore, pressure difference = 2T   
 r1 r2 

5. 0.0004 m Let this pressure difference correspond to h


units of the liquid.
Let R be the radius of curvature of common
surface when bubbles A and B of radii RA Then,
and RB coalesce. The excess pressure in A

www.motion.ac.in
372 - Solution Physics (XI) Topic

9. 20 cm
1 1
2T     gh In the condition of weightlessness, water
 r1 r2  rises to the whole of the available length.

2T 1 1
 h=    10. 20.4 m
g  r1 r2 
2 r 2 (p  )
vT = g
2  0.07  1 1  9 
 h  3
 3 
1000  9.8  1  10 1.5  10 

= 6 mm vT2 – 0 = 2 g × h

4 2
vT 2 4 r     g
Velocity v0

h=  2
2g 81 2

7. v1 h = 20.4 m

Dist. moved H
Fnet = FB + 6  r v – mg 11. 4.5 m
Let its initial volume be v0
FB 6rv 2
a=  g V0
m m Vp =
3

8. 2.5 cm
PV = const.
2T
Pressure due to S.T. = 2
r (9 m )V0 = (9 + h) m v0
3
Pressure difference due S. T. = gh
2
9 = (9 + h)
3
 2 2 
 2T   3   = 3600×7×10–2
10 10  2 
h = 4.5 m

2  18  7 12. 3
h= cm.
100 V g + F = V(4)g

or F  3V g = 3mg
h = 2.52 cm
F
or 3
mg

394-Rajeev Gandhi Nagar, Kota


Surface Tension - 373

Weight of glass plate = 8.2 g = 8.2 × 10–3


2T 2
 1. U  kg
g
Total length of plate in contact in with water
In the equilibrium position ( = 0° for water surface is
and glass) l = 2(10 + 0.2)cm = 20.4 × 10–2 m
2rT cos 0° = r hg 2
Therefore, downward pull upon the plate due
to surface tension
2T
or h TL = 7.×10–2 × 20.4 × 10–2
gr
= 14.892 × 10–3 N
Work done by surface tension
14.892  103
4T 2   1.5196  103 kg wt
= (2rT) × h  9.8
g
Volume of plate immersed in water is
The potential energy of water in the tube, U
= (r 2 h)gh/2; it i s mul tipli ed by h/2
because the centre of gravity of the water 1
v  10  1.54   0.2  1.54  106 m3
and the capillary tube is at a height h/2. 2
Threrefore, upthrust = vg
2T 2
 U = 1.54 × 10–6 × 103 × 9.8 N
g

Thus, it is seen that the mechanical energy 1.54  103  9.8


 kg wt
is not conserved. 9.8
Therefore, mechani cal energy l oss = = 1.54 × 10–3 kg wt
4T 2 2T 2 2 T 2 Therefore, apparent weight of plate is water
 
g g g = Weight of plate in air + Force of surface
tension – Upthrust
This energy is converted into heat.
= 8.2 × 10–3 + 1.5196 × 10–3 – 1.54 × 10–
3

2. 8.1796 kg wt = 8.1796 × 10–6 kg wt


The force acting on the plate are = 8.1796 kg wt
(i) weight of plate (mg) acting vertically
downward.
3. 6 cm
(ii) force of surface tension (TL) acting
vertically downward. 2T
If R is radius of meniscus, then  hg
(iii) upthrust. R

B r2  r1
Here R 
B cos 

 being angle of contact, r1 = radius of glass


rod, r2 = radius of capillary.
Force due mg F
to 2T cos 
surface surface tension  hg
tension r2  r1
mg

www.motion.ac.in
374 - Solution Physics (XI) Topic

Alternative method :
2T cos 
or h Force upward = 2lTcos  = 2lT ( =0°)
 2  r1  g
r
Gravitational pull = (Volume × Density)g
d2 = lhdg
Here r1 = d1/2, r2 = .
2  2lT  Ihdg

4T cos  2T
 h  h
 d2 d1  g
gd
Substituting given values and  = 0° for
water–glass interface, we have
5. 10.9 m/s
4  73  103 cos 0 The terminal velocity of the spheri cal
h raindrop of radius r is given by
2.0  1.5  103  103  9.8

= 60 × 10–3m = 6cm 2r2 g


vt  where  is the density of water
9
and  the viscocity of air.
2T
 4. h Substituting r = 0.3 mm = 0.3 × 10–3m
gd
 = 103 kg/m3
The meni s cus b etween two pl ates i s
g = 9.8 m/s2
cylindrical in shape. Pressure at A (the
lowest point of the meniscus) and  = 1.8 × 10–5 Ns/m2

pA = p0 – T/r We get

Pressure at B = Pressure at C = p0 = Pressure 2


2   0.3  103  9.8
at A + gh vt   10.9 m/s
9  1.8  105
T
 p0  p0   gh
r 6. 1.52 × 103 poise

T 2T
 
gr gd

394-Rajeev Gandhi Nagar, Kota


Surface Tension - 375

1. D v2   02 1
  
v1   01 2
Water fills the tube entirely in gravity less
7.8  1.2 8.5  104  10
condition.  
7. 8  1 13.2
2. C  6.25  10 4 cms1

Soap solution has lower surface tension, T


7. D
as compared to pure water and capillary rise
Let R be the radius of the bigger drop, then
2T cos  Volume of bigger drop = 2 × volume of small
h , so h is less for soap solution.
rg drop

4 4
3. A R 3  2  r 3
3 3
The forces acting on the ball are gravity
force, buoyancy force and viscous force. R  22/3 r
When ball acquires terminal speed, Surface energy of bigger drop,
it is in dy nami c
E  4R 2 T  4  22 / 3 r2 T  28 / 3 2 T
equilibrium, let terminal
speed of ball is vT. So,
8. D
V2g  kv2T  V1g At equilibrium, weight of the given block is
Kv 2T
balanced by force due to surface tension,
V(1  2 )g i.e.,
or VT 
k 2 L.S = W

4. B W 1.5  102 N
S   0.025 Nm1
2L 2  0.3m
Soap solution has lower surface tension, T
as compared to pure water and capillary rise
2T cos  9. B
h , so h is less for soap solution.
rg
 
s 4(r  dr)2  4r 2  (dm)L
5. C
Work done = change in surface energy  dm  4r2dr

2T r
 W  2T  4(R 22  R12 ) r
L

 2  0.03  4[(5)2  (3)2 ]  104 J


10. The bubble will detach, if
= 0.4 mJ
Buoyant force Surface tension force
6. B
4
v
  o i.e., R 3 w g   T  dl sin 
 3

www.motion.ac.in
376 - Solution Physics (XI) Topic

11. C

R
8cm

r r
 P0
T  dl  T 2rsin 
76 × 8 =  × pgas
4
 w   
R 3  g   T  2r  sin  76  8
3  = pgas

r 76  8
sin   + (54 – ) = 76
R 
 8  8
54 –  = 76 1   = 76  
    
2wR 4g 2wg 54 – 2 = 76  – 608
Solving, r  R2
3T 3T 2 + 22 – 608 = 0
22  (22)2  4  808 22  54
No option matches with the correct answer. = =
2 2
= 16 cm

394-Rajeev Gandhi Nagar, Kota


Surface Tension - 377

(JEE PROBLEMS)

1. A,D
1/ 4
2T
 6 Tr2 
Pin = P0 = R=  
r  4fg 
If jump is const then

P1V1 = P2V2 7. B
Surface Energy = T (4 × R2)
as bubble move upward = P  v  = 0.11 × 4 × (1.4 × 10–3)2 = 2.7 × 10–6 J

2. B 8. 0003

3. 6
Although not given in the question, but we
will have to assume that temperatures of A
and B are same.
nB p V / RT p V B 2Ta
 B B  B B
nA pA VA / RT p A VA A p
–2
p = 8 Nm
3 45° 45°
p  4S / rA  4 / 3 rA  kq
2 2
 3
kq2 kq
2
kq
p  4S / rB   4 / 3 rB  2a
2
a
2
a
2
2a
2

(S = surface tension)
Substituting the values, we get
nB 2kq2  kq2 
6 2Ta = + 2  2 cos 45  
2  2a 
nA a  

4. D kq2  1 
Ta = 1  
 
q = mg ....(i) a2  2 2
mg = 6  rv ....(ii)
1
4 3 1 /3
and m = r g ...(iii)  q2   1  3
3 a =   1  k 
from eq. (i), (ii), (iii) find q  T   2 2  

So N= 3
5. C
Net vertical force due to surface tension is
9. D
F = T sin  (2r)
1 2 3RT
P = ρvrms where vrms =
3 M
r
 F= T 2r 
R
1  3RT 
 P= 
2r T2 3  M 
F=
R
 4 ρ1 3
P  3 = ρ ×2
6. A M 2

 2T  2 4
  
 r  R 3  g 1
=
8
Ans.
 R  3 2 9

www.motion.ac.in
378 - Solution Physics (XI) Topic

10. D
h = T cos ( + /2)

2s
h= cos ( + /2)
bg

VP   1
 2
VQ 1  2
11. B,C
(dm) g = dP. dA 2 2    g
VT  r
9 
r(4G)
(.dA) dr. = dp. dA
3  = density of fluid
 = density of object
0
dP =
4G2 R
rdr
p 3 r VP    2  1  1
 1
VQ 2  2  1  2
R
4G2  2 
p= . 2  2  1  2  1
3  r

p  (R2 – r2)
13. 3
12. A,D P = 8 g/cm3 = Q
RP = 1 cm ; RQ = 0.5 cm
4 4
1 R 3g  T  1 R 3g
3 3

4
  1  1  R g  T
3
0.8g / cm3  R 1
P

  3 poise

l  1.6g / cm3
2
P

4 4
2 R 3g  T  2 R 3g   2 poise
3 3

4 Applying :
  2  2  R g  T
3
2 r2
VT   0    g
9 

2 103  103 
VT1   8  0.8   103  10
9 3

2 103  103  0.25


VT2    8  1.6  103  10
1 – 1 = 2 – 2 9 2
2 – 1 = 2 – 1
1 – 2 = 1 – 2 7.2
VT1
4 4
2 R 3g  1 R 3g  2RvP  3 3
3 3 VT2 6.4
8
394-Rajeev Gandhi Nagar, Kota
Errors - 379

1. B 9. A
x(90) = 9(10)
1
 E= mv2 x = 1
2
 Percentage error in K.E. Initially X 9
= Percentage error in mass +2 × Percentage
error in velocity = 2 + 2 × 3 = 8%
G

2. B 10  90
In second condition
3. C

4. D
The number of significant figures in all of the G
given number is 4.

5. C Now x ( 1+ 1 T)(90-) = 9(1+2 T) (10+))

(1  1T ) 9(10  )
6. B
(1  2 T) = (90  )
Here, S = (13.8 ± 0.2) m
and t = (4.0 ± 0.3) sec
    
Expressing it in percentage error, we have, 1  (1  2 )T = 1  10  1  90 
  

0.2
S = 13.8 ± ×100% = 13.8±1.4% 9(1  2 )T  
13.8

0.3 10. A
and t = 4.0 ± ×100 % = 4 ± 7.5%
4 High Resistivity for low current flow & low
Heating And low temp coefficient, so that
s 13.8  1.4
 V=  = (3.45±0.3) m/s Resistance doesn’t vary with temprature
t 4  7.5

11. B
7. C
% error in velocity = % error in L + % error in
deflection
0.2 0.3
t   100   100
13.8 4
= 1.44 + 7.5 = 8.94%
5 X
8. D 8 326
For best results amplitude of oscillation should 320 R
-3
be as small as possible and more oscillation 6
should be taken.

www.motion.ac.in
380 - Solution Physics (XI) Topic

15. B
4
tan  = 8/6 = H = I2Rt
3
 = 53° H  2I R t 
  100      100
 tan  = 5/x  x = 3.75 H  I R t 
So R = 320 + x = 323.75 = 3.24  = (2×3+4+6) % = 16%

12. D 16. B
Ensures zero Magnetic field Given length (l) = 3.124 m and breadth (b) =
3.002 m. We know that area of the sheet
(A) = l × b = 3.124 × 3.002 = 9.378248 m2.
13. A
Since, both length and breadth have four
Weight in air = (5.00 ± 0.05) N
significant figures, therefore area of the sheet
Weight in water = (4.00 ± 0.05) N after rounding off to four significant figures
Loss of weight in water = (1.00 ± 0.1)N is 9.378 m2.

Weight in air
Now, relative density = 17. C
Weight loss in water
Volume of cylinder V = r2l
500  0.05 Percentage error in volume
i.e., R.D. =
1.00  0.1
V 2r l
Now, relative density with max permissible  100   100   100
V r l
error
 0.01 0.1 
5.00  0.05 0.1   2   100   100  = (1+2)% = 3%
    100  5.0  (1  10)% 2.0 5.0
1.00  5.00 1.00   

= 5.0 ± 11%
18. C

4MgL
Y= , so maximum permissible error in Y
D2l
14. B
Average value Y  M g L 2D l 
  100         100
Y  M g L D l 
2.63  2.56  2.42  2.71  2.80

5
 1 1 1 1 1 
   2    100
= 2.62 sec  300 981 2820 41 87 
Now, |T1| = 2.63 – 2.62 = 0.01
= 0.065 × 100 = 6.5%
|T2| = 2.62 – 2.56 = 0.60
|T3| = 2.62 – 2.42 = 0.20
19. C
|T4| = 2.71 – 2.62 = 0.09
M.S.D. = 1 mm
|T5| = 2.80 – 2.62 = 0.18
Mean absolute error L.C. = 0.02 cm = 0.2 mm
L.C. = M.S.D. - V.S.D.

| T1 |  | T2 |  | T3 |  | T4 |  | T5 | m


T  = 1 mm - = 0.02
5 n

0.54 m
= = 0.108 = 0.11 sec   0.08
5 n

394-Rajeev Gandhi Nagar, Kota


Errors - 381

20. A 25. A
R8 = 15  ± 2%, Rp = 3.3  ± 3%
21. A (i) series

av. R  5  10  15
22. A
R  R 1  R 2  0.3
Since percentage increase in length = 2%
Here, percentage increase in area of square So
sheet = 2 × 2% = 4% R  15  0.3

0.3
23. B OR x 100  2%  R  15  2%
15
Least count = 1 S.D–1 V.D
R 1R 2
 8  2 (ii) av. R = R  R
= 1   mm = 0.02 cm.
10  10
1 2

10
So R =  3.3
3

24. A
1 1 1 R R1 R2
Now R  R  R   
1 2 R2 R12 R22

So R = 0.1

R 10
In % age x100 = =3%
R 3.3

v = bh

v  b h
  
v  b h

0 .1 0 .1 0 .1 0 .5
=   =  5%
10 5 5 10

www.motion.ac.in
382 - Solution Physics (XI) Topic

1. C 6. B
Since, for 50.14 cm, significant number = In 23.023 number of significant figures will
4 and for 0.00025, significant number = 2 be 5 because all the zero’s between non
zero digits are si gni ficant. In 0.0003,
2. D number of singificant figures will be one
a = b  c  / d e  because all the zero’s before and after
So, maximum error in a is given by decimal point are insignificant if the number
is less decimal point are insignificant fi the
   b c number i s less than one. In 2.1×10 -3
   100   .
b
 100  .
c
 100
 max number of significant figure are because
power of 10 is not considered as significant
d e figuere.
.  100  .  100
d e
= (b1 + c1 + d1 + e1 )% 7. D
1 2 1 2
 L1   T1   m   sec 
n2 = n1      10    
3. E  L 2   T2   km   hr 
Time period of a simple pendulum is
1 2
L  m   sec 
n2 = 10  3     129600
T = 2 10 m   3600 sec 
g

4 L 8. S = (1.20  0.18)cm
 g
T2

g  L T  9. C
  100   2  100
g  L T 
42l
T = 2 l / g  T2 = 42/l / g =
= 1 % + 2 × 2 % = 5% T2

1mm
4. C Here percentage error in l  ×100
100cm
Quantity C has maximum power. So, it
brings maximum error in P. 0.1
 ×100 = 0.1%
100
5. C
and percentage error in T
1 2
From h = ut + gt 0.1
2  ×100 = 0.05%
2  100
1  Percentage error in g
h0  9.8  (2)2  19.4m
2 = percentage error in l + 2
(percentage error in T)
h t
 2 = 0.1 + 2 × 0.05 = 0.2%
h t
[ a = g = Constant]
10. C
0.1 1 Mean time period T = 2.00 sec
=±2 
2 10 and mean absolute error = T = 0.05 sec
h 19.6 To express maximum estimate of error, the
 h = ±   1.96m time period should be written as (2.00 ±
10 10
0.05) sec.

394-Rajeev Gandhi Nagar, Kota


Errors - 383

11. A 16. D
Percentage error in X = a + b + c 3.07 cm
L.C. = M.S.D. - V.S.D.
12. D
9
Percentage error in A = 1mm - mm = 0.1 mm
10
 1 
  2  1  3  3  1  2   2  %  14% ZERO ERROR
 2 
Z.E.  0.7mm

13. C Reading 
(M.S.R. + n x L.C.) - Z.E.
R1R 2 = (31mm + 4 x 0.1) - 0.7
Rparallel = (R  R )
1 2 = 31.4 - 0.7 = 30.7

 RP R1 R 2 (R1  R 2 )
 R  R  R  R R 17. B
P 1 2 1 2
(i) x = – 0.7 msd, (ii) 6, 1
R P 0.3 0.2 (0.3  0.2)
 R  6  10  10  6 -1
P
0 1 2 3
= 0.05 + 0.02 + 0.03125 = 0.10125 (i)
0
1 2 3
 RP
 R P ×100 = 10.125 or 10.125% M.S.R. + V.S.R. X L.C.
-1 + 3 x 0.1 = -0.7
OR
14. A
Here, d 2mm

Mass of body, M = 5.00 ± 0.05 kg


Volume of a body, 3 x 0.9
V = 1.00 ± 0.05 m3
= 2.7 mm d = 2.7 - 2
M = 0.7 mm
Density,  =
V (ii)
ACTUAL READING
 M V
Relative error in density is,   = READING - Z.E.
 M V
= 5.4 - (-0.7)
Percentage error in density is
= 6,1
 M V
 100   100   100
 M V
Main On vernier
Scale Scale
 0.05   0.05 
  100     100 
 5   1 
= 1% + 5% = 6% 18. D

P
15. D
By substituting the dimensions of mass [M],
Vernier Scale
length [L] and coefficient of rigidity [ML-1T-

M Q
2
], we get T = 2  is the right formula
L
for time period of oscillations. main Scale

www.motion.ac.in
384 - Solution Physics (XI) Topic

21. A

I.M.S.D. =
cos 
22. B
I.V.S.D. = 

  A  Sm 
sin 
L.C. = -  +  2  sin 45
cos   ,   2
sin(A ) sin 30
2
( cos   1)

cos 
 A  m  A
ln   lnsin    lnsin 
19. A  2  2

1mm
L.C.   0.02mm  A  m 
50div cos  

  2     A  m 
Z.E. = L.S.R. + L.C. x VSR   A  m 
 2 
sin  
= -1 + 0.02 x 44 
 2 
= -1 + 0.88
= -0.12
 A  m  1
Now Reading = tan  2   2   m
 
= L.S.R. + L.C. x V.S.R. -Z.E.
= 3+ 0.02 x 26 + 0.12 1  5
= 3.64 mm  tan 45   rad  100%  %
2 180 18

20. C
23. B
Ist data
Let M.S.D. = 1mm Å

21.0 – 10.9 = 10.1+0.1


Main Scale
O I
10cm L
N+2
10cm 21.0
X=1.1 10.9
Rod

10.9 - 1.1 = 9.8


error = –0.2 Vernier Scale
O u L v I
Now
11.4 22.5
X=0.6 (N+2) x 1 mm = l (Rod)+2 x 0.9
U = (11.4 - 0.6) + 0.2 = 11 cm  N  2  l  1.8
V=22.5 - 11.4 - 0.1 = 11 cm Now charge for (N+2) mm on
1 1 1 M.S. is equal to change of
   f  5.5 cm
f v u 1.8 mm on V.S.
 1 (N  2)T  2 (1.8)T
 V V 
2
f  f  2  2 
V V  1 1.8
 
2 N  2
 0.1 0.1 
= 5.5 x 5.5  2  2  = 0.05 cm
 11 11 

394-Rajeev Gandhi Nagar, Kota


Errors - 385

1. A,C 2. A,D

t 40 0.5mm
Since, t = nT. So, T =  or T = 2 sec. L.C. =
n 20 50 division

t T = 0.01 mm
Now, t = n T and 
t T Reading = L.S.R.
+ L.C. x V.S.R.
1 T
So,   T = 0.05 = 5 x 0.5 + 34 x 0.01
40 2
= 2.5 + 0.34

l = 2.84 mm
Time period, T = 2
g

T 1 g g T
So,  or  2
T 2 g g T

g
So, percentage error in g =  100
g

T 0.05
 2  100  2   100  5%
T 2

www.motion.ac.in
386 - Solution Physics (XI) Topic

1. 7%
g L T
Given : Voltage V =- (100±5) V  100   100  2  100
Current I = (10±0.2)A g L T
According to Ohm’s law, V = IR or R = V/I Actual percentage error in
Taking log of both sides, log R = log V – log I L T
Differentiating, we get; g=  100  2  100
L T
R V I = + 2% – 2 ×1% = 0%
 
R V I
7. (2.00±0.05)s
R V I
For maximum error,   True value = measured value ± error
R V I = (2.00 ± 0.05) s.
Multiplying both sides by 100 for taking
percentage, we get ; 8. 6%
R V l Heat produced H is given by :
 100   100   100
R V l I2Rt
Percentage error in resistance R H
J
V l H I R t J
  100   100  2   
V l H I R t J
5 0.2
  100   100  7% H  l  R t J
100 10 100  2 100  100  100  100
H I  R t J

2. 0.002 cm For maximum percentage error,


H l R t J
3. 1.7×10-6m3 100  2 100  100  100  100
H I R t J
= 2 × 2% + 1% + 1% + 0% = 6%
4. A

abc2 9. ± 0.2 cm2


P
d3e1 / 3  l b 
A    A
 l b 
P 1 a 1 b c d 1 e
100        3     100
P 2 a 2 b c d 3 e  0.1 0.01 
    ×(10.0×1.00)cm2
1 1 1   10.0 1.00 
   2%   3%  2%  3  1%   6% = ±0.02×10=±0.2cm2
2 2 3 
= [1% + 1.5% + 2% + 3% + 2%] 10. 3%
The minimum amount of error is contributed Percentage error
by the measurement of a.
V  L 2d 
5. 19% = V  100   L  d   100
 
X = [M-1 L3 T-2]
X M L T  0.1 2  0.01 
   100  3%
 3 2  5.0 2.00 
X M L T
= 2 + 3 × 3 + 2 × 4 = 19%
11. 1.00 cm
6. 0% Length measured with Vernier callipers
= reading before the zero of vernier scale
L 2 2 L + number of Vernier divisions coinciding
T  2 or T  4
g g with any main scale division × least count
= 10 mm + 0 × 0.1 mm = 10 mm = 1.00 cm.
L g L T
g  42 ;  2
T2 g L T 12. 2.5%

394-Rajeev Gandhi Nagar, Kota


Errors - 387

4. 8%
1
1. x Maximum percentage error in the measure-
4
ment of pressure,
When number of observations is made n
times, the random error reduces to 1/n P F r
 100   100  2   100
times. P F r

4 2
2. (22.1±0.1) cm   100  2   100  8%
100 100

 X Y  5. 1s
3. Q    n m Q
 X Y 
t 1 1010
 , t = = 1 s.
Here, maximum fractional error is : t 1010 1010

Q  X Y 
  n m
Q  x Y  6. 1.0 mm

 Absolute error in Q, i.e. : pitch


Least count =
number of divisions
 X Y 
Q    n m Q Pitch = least count × number of divisions
 X Y 
= 0.005 × 200 = 1 mm.

www.motion.ac.in
388 - Solution Physics (XI) Topic

1. D 4. C
V = f = 2f(2-1)
0.5mm
V = 2f (2+1) L.C. =
50 division
= 0.01 mm
= 2 x 512 (0.1+0.1)
Z.E. = 0 + 5 x 0.01 = 0.05
= 512 x 0.4
Reading = 2 x 0.5 + 25 x 0.01- Z.E.
= 204.8 cm/s
= 1 + 0.25 - 0.05 = 1.20

2. B
5. D
L.C. = 0.1 mm
Reading = 10 + 1 x 0.1 g l 2T
 
= 10.1 mm g l T

m 2.736 l is least in (D) (i.e 1mm)


  2.736 gm

v (10.1)3 1.03cm3 and

= 2.6563 g/cm3 = 2.66g/cm3 T 0.1


 is also least in D
T 50

3. C So least error is in D.
31 We can also see that amplitude is also less
so it is more closes to S.H.M.
0.1cm
30
6. B

10 7. B

0.1cm F l
Y
A L

FL mgL
Y 
31 30 -10 Al Al

1 1 1 1 x 9.8 x 2
  Y
f v u  x d2
 0.4 x 10 3
4
1 1 1
=  
10 10 5 = 2 x 1011 N/m2

f = 5 cm Y l d 0.05 0.01
 2 =  2x
L.C. 0.1 cm Y l d 0.8 0.4

f V u 0.09 9
 2  2  
f2 V u 0.8 80

 0.1 0.1  9Y 9x2


 f  25    0.05 Y   x1011  0.225x1011
 100 100  80 80

 Y  (2  0.2)x1011

394-Rajeev Gandhi Nagar, Kota


Errors - 389

8. D
1 dg 1 1
Diameter = M.S.R. + C.S.R × L.C. + Z.E. = 3  
2 g 400 90
+ 35 × (0.5/50) + 0.03 = 3.38 mm

dg  490   490 
9. A g

400  90   2   200  90  = 0.20272
   
Least Count =
= dg/g × 100  2.72%  3%
Value of main scale division
No. of divisions on vernier scale 16. D

1 1 1 1 90  91  95  92
= MSD    = 1 minute Tmean = = 92
30 30 2 60 4
1 = 90 – 92 = 2
10. A 2 = 91 – 92 = 1
3 = 95 – 92 = 3
11. 4 = 92 – 92 = 0

12. A 2 13 0
mean = = 1.5
Reading by vernier scale 4
 T = 92 ± 2 s (Using significant Figure)
 29 
= 0.5 1 –  = 0.15
 30 
17. A
Reading = 58.65
0.5
LC = = 0.01 mm
13. C 50
zero error = 0.50 – 0.45 = –0.05
V R V I
R=    = 3% + 3% = 6% Thickness = (0.5 + 25 × 0.01) + 0.05
I R V I
Hence (2) is correct = 0.5 + 0.25 + 0.05
= 0.8 mm
14. D
18. B
0.1cm
The vernier calliper has least count
10 42l
g
= 0.01 cm T2
Also 3.50 cm has smallest reading 0.01 cm
g l 2T
E  
g l T
15. D

dT 1 dL 1 dg 0.1 2x0.1
  EI  
T 2 L 2 g 64.0 128

90 1 0.1 0.1
 EII   2x
100 100 64.0 64

1 dg 1 dL dT
  0.1 0.1
2 g 2 L T EIII   2x
20 36

1 0.1 1 /100 1 1 Clearly E1 <E2 < E3.


   
2 20 90 /100 400 90

www.motion.ac.in
390 - Solution Physics (XI) Topic

19. CD 22. C

0.5
20. A,C Least count of screw gauge =
50
40 s = 0.01 mm = r
T   2 s.
20
0.5
Further, t = nT = 20T or t = 20 t Diameter r = 2.5 mm + 20   2.70 mm
50
t T
  r 0.01 r 1
t T  or  100 
r 2.70 r 2.7

T  2  
or T  .t    1  0.05 s
t  40  Now, density d  m  m
3
V 4 r 
 
3 2
Further T  2 l or T  g-1/2
g Here, r is the diameter.

T 1 g d  m  r  
  100     100   100    3     100
T 2 g d m  r 

or % error in determination of g is
m  r 
  100  3     100
g T m  r 
 100  200 
g T
1
 2%  3   3.11%
200  0.05 2.7
  5%
2
 Correct options are (A) and (B) 23. A

4MgL
21. D Y 
d2
Least count of vernier calipers
LC = 1 MSD – 1 VSD Y M g L  2d
    
Y M g L  d
Smallest division on main scale

Number of divisions on vernier scale   d (same instrment)

20 divisions of vernier scale = 16 divisions of  2d


main scale so  hence both contribute same in
 d
16 error
 1 VSD = mm  0.8 mm
20

 LC = 1 MSD – 1 VSD 24. B


= 1 mm – 0.8 mm = 0.2 mm 2.45 
 –3
 Correct option is (D). L.C.=  0.05–  cm=10 cm.
 50 

Reading = 5.10 cm + 24 × 10–3 cm.


= 5.124 cm.

394-Rajeev Gandhi Nagar, Kota


Errors - 391

1. B 5. B
for Ist resonance LC1 = 1 MSD – 1 VSD LC2 = 1 MSD – VSD
/4 =x + 0.6r ....(1) 10 VSD  9 MSD 11 VSD  10 MSD
 v = f
9 10
336 = 5/2 ×    = 336/5/2 1 MSD 1 VSD  MSD
10 11
 /4 16.4 cm
16.4 = x + 0.6 × 2 9  10 
LC1 = 1 MS – MS LC2 = 1  11  MS
x = 16.4 – 1.2 10  
= 15.2 cm
1 1 1
= MSD =  × cm
10 11 10
2. D
2dsin =  1 1
= cm =– cm
100 110

d=
2 sin  1 1
C1 = 2.8 + 7 × C2 = 2.9 – 7 ×
100 110
 cos ec cos  d
dd = = 2.87 = 2.83
2

dd  d. cot  d ( d  constant)

6. A,B,D
dd
 
d r 1
(a)  100   100  10%
r 10
dd = 

dd 7 R  r 
 fraction (b) T  2
d 5g

dd = absolute error R = 60 ± 1
r = 10 ± 1
3. C
0.52  0.56  0.57  0.54  0.59
Tm 
5
4. 4
= 0.556
F/A T1 = 0.556 – 0.52 = 0.036
Y  , l=25×10–50m
l T2 = 0.556 – 0.56 = 0.004
l
T3 = 0.556 – 0.57 = 0.014

F l T4 = 0.556 – 0.54 = 0.016


Y .
A l T5 = 0.556 – 0.59 = 0.036

0.036  0.016  0.014  0.004  0.036


Y 105 Tn 
 100   100  4% 5
Y 25  105

T 0.020
  100 = 3.57%
T 0.556

www.motion.ac.in
392 - Solution Physics (XI) Topic

8. 4
42 7R  r 
(d) g
5T 2  A2et   0.2

lnE  2 ln A  t
g  R  r  T
 2
g R r T dE dA
2  dt
E A
2 2  0.02  2 42 
      100 = (2×1.25×10–2) + (0.2 × 7.5 × 10–2)
50 0.556  50 55 
= 2.5 × 10–2 + w1.5 × 10–2
= 4 + 7.5 = 11.7 %
dE
 4  102
E
7. B,C
VERNIER
1cm = 8MSD  1MSD = 1/8 cm
5 VSD = 4 MSD  L.C. = 1 MSD – 1 VSD

1 1
= MSD = cm = 0.25 mm
5 40
SCREW GAUGE
1 pitch = 100 div. = 2 LSD = 2 Linear scale
div.
If 1 pitch = 2 (L.C. of Ver.) = 0.5 mm

1 pitch
 L.C. of screw = = 0.005 mm
100
1 LSD = 0.5 mm

1 pitch 1mm
L.C. of screw = = = 0.01 mm
100 100

394-Rajeev Gandhi Nagar, Kota

You might also like